You are on page 1of 339

PRELIM BITS

UPSC/ CSE Ancient History PYQs


From 1979 - 2022

P. Roman
12/15/2022

Telegram Link : https://t.me/prelimbits

Persnal Contact For any Quary : https://t.me/EN123upsc

Do NOT SHARE THIS PDF (All the content is Protected)


1

CSE Ancient History PYQ [1979 – 2022]


1. The worship of idols started in India in

A. Pre-Aryan period

B. Gupta period

C. Mauryan period

D. Kushan period

Ans : B

Note : The language of this question may be ambiguous.

 If we talk about the Archaeological evidence of deity worship, then we find traces of Puja
rituals and worship of idols in temples from Gupta Empire era (i.e. 4th century AD).

Religion During Gupta Period

 The period 300–600 CE is often considered as the phase of consolidation of


Brahmanical ideology as temple-based sectarian cults became popular and Sanskrit was
firmly established as the language of royal inscriptions. [So Sanskrit was official
Language, Common people Such as woman & Sudra Speak Prakrit]
 Sectarian symbols such as the linga, trishula, bull, Gaja-lakshmi, chakra, and shankha (conch
shell) appeared on the royal prashastis, coins, and seals. Hinduism acquired its present
shape Gupta age only.
 The Gupta rulers gave patronage to Bhagvatism, which was centred around the worship
of Bhagvat or Vishnu and his incarnations.
o So Bhagvatism was also identified with Vaishnavism. It put emphasis on bhakti
(loving devotion) and ahimsa (non-killing of animals, which is why it stresses more
on vegetarianism) rather than Vedic rituals and sacrifices. This new religion→
o Was quite liberal, and assimilated the lower classes in its fold.
o By the end of the Gupta period, Mahayana Buddhism was overshadowed by
Bhagavatism and Vishnu became a member of the trinity of gods along with Shiva
and Brahma.
o Idol worship became a common feature and the idols of different incarnations of
Vishnu were housed in the temples constructed in the Gupta period.
 There was also an emergence of worship of composite deities such as Hari-hara, who is a
part Vishnu and part Shiva, and religious syncretism (for instance, the incorporation of
the Buddha as one of the ten Vishnu avatars).
 This phase also saw the competitive relationship between various cults, & myriad of
local cults, beliefs, and practices.

https://t.me/prelimbits
2

 The Gupta kings were devout Hindus, but they followed a policy of tolerance towards
other religious sects.
 The Chinese pilgrims Fa-Hien and Hiuen Tsang, who came to India during the reign of
Chandragupta II and Harsha respectively, clearly give the impression that Buddhism
was also flourishing. Though it is also true that Buddhism no longer received royal
patronage. However→
o Some stupas and viharas were constructed and Nalanda developed as a great
centre of education for Mahayana Buddhism during his time. Students from other
countries also came to study in this university. According to Hiuen Tsang, the
revenues of one hundred villages supported it.

2. The sculpture with the three faces of Brahma, Vishnu and Mahesh known as “Trimurti”
appears (CSE 1979]

A. Aanta caves

B. Ellora caves

C. Kalva caves

D. Elephanta caves

Ans : D

Elephanta caves

 Cave temples predominantly dedicated to the Hindu god Shiva.


 They are on Elephanta Island, or Gharapuri (literally "the city of caves"),[4] in Mumbai
Harbour
 It consists of five Hindu caves, a few Buddhist stupa mounds that date back to the 2nd
century BCE, and two Buddhist caves with water tanks.
 The Elephanta Caves contain rock cut stone sculptures, mostly in high relief, that show
syncretism of Hindu and Buddhist ideas and iconography. The caves are hewn from solid
basalt rock
 They are most commonly placed between the 5th and 7th centuries. Many scholars
consider them to have been completed by about 550 CE
Important Figures

Faced-Image of Shiva (Asked in CSE)

 One of the master-piece of this cave is the three faced-image of Shiva (Trimurti)
 It is also known as Sadashiva and Maheshmurti. The image, 5.45
m (17.9 ft) in height, depicts a three-headed Shiva, representing
Panchamukha Shiva.
 The three-headed Shiva are his creator, preserver and destroyer

https://t.me/prelimbits
3

aspects in Shaivism. They are equivalently symbolism for Shiva, Vishnu, and Brahma, they
being equivalent of the three aspects found in Shaivism.
Gangadhara

 The Gangadhara image to the right of the Trimurti shows Shiva and Parvati standing. Shiva
brings the River Ganges down from the heavens to serve man.
Ardhanarishvara (Half Shiva & Half Parvati)

 It represents the ancient Hindu concept of essential interdependence of the feminine and
the masculine aspects in the universe, for its creation, its sustenance and its destruction
Nataraja: Lord of Dance

 It shows Shiva as the Nataraja, "cosmic dancer" and "the lord of dancers". It is also called
the Nrittamurti.
Other figures

 Ravana shaking Kailasa, marriage of Shiva and Parvati,

3. The name Pahlavi Dynasty in Iran is derived from the [CSE 1979]

A. Language

B. King

C. Religion

D. River

Ans: B

4. Which of the following throws light on Harappan Culture? [CSE 1979]

A. Archaeological excavations

B. The script on copper sheets

C. Rock edicts

D. All of the above

Ans: A

5. Who is regarded as the greatest law giver of ancient India? [1980]

A. Megasthanese

B. Panini

C. Manu

D. Kautilya

https://t.me/prelimbits
4

Ans : C

 Manusmriti, a book on Indian law is attributed to the legendary first man and lawgiver,
Manu. Manu-smriti, traditionally the most authoritative of the books of the Hindu code
(Dharmashastra) in India

 It was one of the first Sanskrit texts to be translated into English in 1776, by British
philologist Sir William Jones, and was used to construct the Hindu law code, for the East
India Company administered.

6. At which place Gautama Buddha delivered his first sermon? 1980

A. Sarnath

B. Lumbini

C. Bodh Gaya

D. Vaishali

Ans : A

 At the age of 35, Buddha ultimately attained Nirvana/Enlightenment (the cessation of


desire and hence the end of suffering)
at Gaya, Magadha (Bihar) under a Peepal tree at
Uruvela, on the banks of river Niranjana, and came to be known as the Buddha, the
Enlightened One.
 Peepal tree was known as the famous Bodhi tree. It is believed that tree was destroyed by
Asoka‘s queen Tissarakkha, Pushyamitra Shunga and by King Shashanka. [But again planted
by Buddhist]

 Delivered his First Sermon with his five former companions at deer park at Sarnath.
Event is known as ‟DhammaChakka-Pavattana‘ meaning „turning the wheel of
dharma’. These five disciples became Arhats

o Arhats : one who has gained insight into the true nature of existence and has
achieved nirvana BUT but who may not have reached full Buddhahood

7. Neolithic period of age is not characterised by 1980

A. Agriculture

B. Use of copper

C. Domestication of animals

D. Fishing

Ans : B

Neolithic Age [8000 BC – 4000 BC]

https://t.me/prelimbits
5

Evolution  Lived in huts, reared cattle, Developed Agriculture (wheat, barley,


cotton, rice etc.), Used earthenware (both handmade and wheel-made).

Agriculture  The people of the Neolithic age cultivated land and grew fruits and corn
like ragi and horse gram (kulati). They also domesticated cattle, sheep
and goats.

Housing and  The people of Neolithic age lived in rectangular or circular


settled life houses which were made of mud and reeds. Neolithic men also knew
how to make boats and could spin cotton, wool and weave cloth.
 The people of the Neolithic age led a more settled life and paved the
way for the beginning of civilization.

Tools  Sharper, symmetrical and polished stone tools for not only hunting but
agriculture also.

Examples (tools)  Daggers, digging sticks, celts, grinding stones, sickle, saw, sling-
stones etc. (continuous rubbing to smoothen)

Pottery  The pottery first appeared in the Neolithic Age. The pottery of the
period was classified under Grey Ware, Black-Burnished Ware, and mat-
impressed ware.

Sites  Koldihwa and Mahagara (lying south of Allahabad)

 Mehrgarh (Balochistan, Pakistan) The earliest Neolithic site, where


people lived in houses built of sun-dried bricks and cultivated crops like
cotton and wheat.

 Burzahom (Kashmir) [Pit dwelling house]

 Gufkral (Kashmir)

 Chirand (Bihar)

 Piklihal, Brahmagiri, Maski, Takkalakota, Hallur (Karnataka) –

 Belan Valley (which is located on the northern spurs of the Vindhyas


and middle part of Narmada valley) – All the three phases i.e.,
palaeolithic, Mesolithic and neolithic ages are found in sequence.

Note : Copper was used during CHALCOLITHIC AGE [3000 BC – 500 BC]

8. Name the God who lost his importance in the beginning of Christian era? 1980

A. Indra

B. Brahma

C. Vishnu

https://t.me/prelimbits
6

D. Mahesh

Ans : A

 Indra was the chief god of the fire worshiping steppe Caucasians who migrated to India
about 3500 years back.

 As years and generations rolled on and as they mixed and intermarried more with the local
Dravidians their religious outlook also changed.

 The Dravidian gods gained more importance and the Aryan gods were gradually
relegated to the background. Mahavishnu, Shiva, Devi etc were given more
importance.

9. We can know about early Vedic period from (1980)

A. Archaeological excavations

B. Contemporary cultures

C. Rigveda

D. Jatak Katha

Ans : C

10. Upanishads are [1980]

A. Religious books of Hindus

B. Books dealing with ancient Hindu laws

C. Books on social behavior of man

D. Prayers to God

Ans : A

Upanishads

 The upanishada indicates knowledge acquired by sitting close to the teacher.


 The Upaniads are ancient Indian literature that were written orally in Sanskrit between 700
BCE and 300 BCE.
 There are thirteen main Upaniads, several of which are thought to have been written by
different writers and are written in a variety of styles.
 The Upaniads were written in a ritual context as part of a larger group of texts known as
the Vedas, but they mark the beginning of a reasoned inquiry into a number of
perennial philosophical questions concerning the nature of being, the nature of the self, the
foundation of life, what happens to the self at the time of death, the good life, and ways of
interacting with others.

https://t.me/prelimbits
7

 As a result, the Upaniads are sometimes regarded as the cradle of India's later rich and
diversified intellectual heritage.
 The Upaniads contain some of the earliest discussions about key philosophical terms
like atman (the self), brahman (ultimate reality), karma, and yoga, as well as sasra
(worldly existence), moksha (enlightenment), purua (person), and prakti (nature), all
of which would remain central to the philosophical vocabulary of later traditions.
 They are also known as Vedantas [end of the Veda], because they embody the
Veda's fundamental objective and include the highest and final goal of the Veda,
which is Moksha or Supreme Bliss.

Important Upanishads
 Aitareya Upanishad: Talks about the creation of Atman (Soul) and Consciousness.
 Chandogya Upanishad: Deals with rhythm and chanting of Mantras.
 Katha Upanishad: Tells the story of Nachiketa and Yama. Their conversation evolves into
discussion of Man, Atman (soul), Knowledge, and Moksha (liberation).
 Brihadaranyaka Upanishad: Talks about the transmigration of Atman; passages on
metaphysics and ethics.
 Mundaka Upanishad: Contains the mantra ―Satyameva Jayate‖ (truth alone triumphs)
which is borrowed in the National Emblem of India.

11. The Indus Valley people had contacts with (1980)

A. Egyptians

B. Sumerians

C. Chinese

D. Mesopotamians

Ans : D

Harappan Economy

 The inferences about Harappan Economy have been derived from its flourishing trade
relations with its contemporary Mesopotamian and Persian civilisations. The
Mesopotamians called the Indus Region ‗Meluhha‟.

 The Mesopotamian texts speak of three intermediate trading stations called Dilmun
(probably Bahrain on Persian Gulf), Makan (probably the Makran coast, Oman) and,
Meluhha.

 Seals hold a special significance in the Harappan context. Every merchant probably had a
seal bearing an emblem, often of a religions character and a name / brief description on
one side.

o The standard Harappa seal was a square / oblong plaque made of steatite stone.
https://t.me/prelimbits
8

o Though its primary purpose is inferred to mark the ownership of property, they
may also have served as amulets.

 The key aspects of Harappan trade networks and economy are as follows:

o They carried out internal and external trade.

o There was no metallic money in circulation and trade was conducted by means of
barter.

o Inland transport primarily employed bullock carts. Harappans seals have been
found in Mesopotamian cities like Susa and Ur. In Nippur, a seal has been found
bearing Harappan script and a unicorn.

o Recently, some ancient sites in the Persian Gulf like Failaka and Bahrain have also
yielded Harappan seals. In Mohenjo-Daro, three cylinder seals of the
Mesopotamian type have been found, which underlines their trading relations. In
Lothal, a button seal has been found.

o There were bead-making factories at Chanhudaro and Lothal. The products of


these factories were items of export.

o A dockyard has been exacavated in Lothal and sea ports have been found at
Rangpur, Somnath and Balakot.

o A remarkable aspect of the Harappan Culture was its standardisation and accuracy in
the context of weights and measures.

o Weights followed a binary system in the lower denominations — 1, 2, 8, 16, 32 to


64, and then in decimal multiples of 160 like 160, 320, 640, 1600, 3200, and so on.
Weights were made of chert, limestone, and steatite and were generally cubical in
shape. Measures of length were based on the foot (37.6 cm) the cubit (51.8 to 53.3
cm).

o A shell scale has been found at Mohenjo- Daro, a shell object probably used to
measure angles has been found at Saurashtra, and an ivory scale has been
discovered at Lothal.

12. Which of the following is not related to the Gandhara School of Art? (1980)

A. Ellora

B. Ajanta

C. Khajuraho

D. Elephanta

Ans : C

 All the Ellora caves ,Ajanta caves and Elephanta caves has the influence of Gnadhara school

https://t.me/prelimbits
9

of art but the Khajuraho temple does not relate to this art form.

Gandhara school of Art

 The Gandhara school of Art arose in modern-day Peshawar and Afghanistan on Punjab's
western boundaries.

 The Greek invaders brought the traditions of Greek and Roman sculptors with them,
which affected the region's native traditions ==> Greco-Indian School of Art

 Between 50 B.C. and 500 A.D., the Gandhara School flourished in two periods. While the
former school's sculptures were made of bluish-grey sandstone, the latter school's were
made of mud and plaster.

 The Buddha and Bodhisattvas iconography was based on the Greco-Roman pantheon
and resembled Apollo's.

 Major Centres :

o Jalalabad, Hadda, Bamaran, Begram & Taxila were the main centers where art pieces
of Gandhara School have been found.

 Major Features :

o The Gandhara sculptures have been discovered in the Taxila ruins as well as other
ancient sites in Afghanistan and Pakistan.

o They generally consist of Buddha images and relief sculptures depicting scenes
from Buddhist literature.

o Several Bodhisattva figures were cut out of the rock. The first preaching in the
deer park and the Buddha's death is depicted in a Gandhara figure.

o The predominant focus of this type of painting was Lord Buddha


and Bodhisattvas, as it was intimately tied with Mahayana Buddhism.

o The Bamiyan Buddha sculptures are an example of Gandhara style art.

o It thrived primarily in Afghanistan and present-day North-Western India.


Taxila, Peshawar, Begram, and Bamiyan were among the most prominent sites.

o From the first century BCE through the fourth century CE, the Gandhara School of
art flourished.

 Themes

o The Buddhist themes were largely represented in Gandhara art. The Buddha's
mother resembles an Anthenian matron.

o A Buddhist scene was created with an Apollo-like face. The image of Athena of
Rome in Lahore is perhaps one of the most beautiful Gandhara sculptures depicting
a western topic.

https://t.me/prelimbits
10

o It was created in order to express Buddhist ideas and practices.

o The specimens do not include any Greek art motif, with a few exceptions. The hand
of the Gandhara artist was Greek, but his heart was Indian.

 Patrons

o Gandhara School was fostered by the Shakas and the Kushanas. The first
sculptural portrayals of the Buddha in human form are attributed to
the Gandhara school. [The first image of the Buddha-was carved out during
the reign of famous Kushan king Kanishka I.]

 Features of sculpture

o The human body is sculpted in a realistic manner in these sculptures, with


considerable attention to realism and physical details, particularly in the depiction
of muscles, mustaches, and other facial features.

o The portrayal of the broad bold fold lines is a unique trait that sets it apart from what
has been discovered elsewhere in India.

 The Various Mudras of Buddha in Gandhara Art

o In all the Buddha depicted in the Gandhara Art is shown making four types of hand
gestures and this is a remarkable feature in this art.

 Greek influences [Hellenistic influence] on Gandhara School of art

o Greek god as protector: In many images of Buddha in Gandhara; he is seen under


the protection of Greek god Hercules.

o Vajrapani: Vajrapani found in the right hand of future Buddha is told as a


transformed symbol of Hercules who is seen as the protector of Buddha.

o Greek architectural influence: Some images of Buddha in Gandhara are presented


in the Greek architectural environment bearing the affinity of Corinthian.

o Artistic beauty: The Apollo-like face of Buddha; natural realism; wavy hair as
seen in images of Buddha in Gandhara resembles Hellenistic tradition.

https://t.me/prelimbits
11

o Intellectual affinity: The hello and bun of Gandhara Buddha signify intellectual
imbibitions of Buddha from Greek.

o Despite the fact that Gandharan sculpture's iconography was predominantly Indian, it
also included elements and methods from Classical Roman art. Vine scrolls,
cherubs with garlands, tritons, and centaurs are examples of Classical Roman art
found in Gandharan sculptures.

o Additionally, the Gandharan artists drew on the Roman religion's anthropomorphic


traditions.

o Gandharan art depictions of Buddha are evocative of statues of a youthful Apollo.

o The drapery on Buddha's garments was also similar to that on Roman imperial
monuments.

13. Which of the following is the most important cause for the decline of Buddhism after
Ashoka?

A. Non-patronage by the kings

B. Condemnation of animal sacrifices

C. Growth of licentious practices in Buddhist centres

D. Allegiance to the middle path

Ans : A

Reasons for the Decline of Buddhism

 The birthplace of Buddhism is India. Though Buddhism is still practised in many parts of
the world, it has diminished in its birthplace.

 One of the fundamental issues with the collapse of Buddhism in India is that it is extremely
difficult, if not impossible, to provide a continuous narrative of the decline's history.

 In fact, the deterioration is nearly imperceptible.

 Even though there is a lot of archaeological and epigraphical evidence accessible, it is still
insufficient.

Corruptions in Buddhist Sanghas

https://t.me/prelimbits
12

 The Buddhist 'Sangha' became corrupted throughout time. The monks and their followers
were enamoured with luxury and pleasure → became hungry and materialistic → gold
and silver

 They had devolved into a life of indiscipline.

Reform in Hinduism

 Buddhism had dealt a heavy blow to Brahminical faith. Threatened with extinction,
Hinduism started to re-organize itself. Attempts were now made to give up the complex
system of rites and rituals and make Hinduism simple and attractive.

 The Hindus even came to accept the Buddha as a Hindu incarnation and accepted the
principle of non-violence.

 This helped revive Hinduism and made it popular again. The fragrance of Buddhism was
taken away as a result of this. Buddhism's demise became unavoidable.

Division among the Buddhists

 Buddhism has seen its share of schisms.

 Buddhism's uniqueness was lost due to its division into several breakaway groups such as
the "Hinayana," "Mahayana," "Vajrayana," "Tantrayana," and "Sahajayana."

 Also the influence of tantricism made people hate it.

 The simplicity of Buddhism was lost and it was becoming complex. This was enough for
the people to keep away from it.

Use of Sanskrit Language

 Pali and Prakrit, the spoken languages of the majority of Indians, were used to propagate
the Buddhist message.

 During Kaniska's rule, however, Sanskrit superseded these in the Fourth Buddhist
Council.

 Sanskrit was a difficult language that few people could understand →People rejected
what they could not understand

Patronage of Brahmanism

 With the passage of time, the Brahminical faith rose to prominence once more.

 Pushyamitra Sunga, the final Maurya emperor Vrihadratha's Brahmin commander,


killed him and established the Sunga dynasty in his place.

 He was the one who performed the Asvamedha sacrifice. It gave the Brahminical beliefs a
boost.

 The essential premise of Buddhism, nonviolence, was abandoned. He demolished a


number of stupas and monasteries.

https://t.me/prelimbits
13

 A large number of Buddhist monks were executed. Buddhism's expansion was stifled as a
result of this.

 The imperial Guptas' sponsorship of the Brahminical creed once again paved the way for
Buddhism's demise.

Role of Hindu Preachers

 Harsavardhan expelled the Brahmins from the Kanauj religious council.

 Under Kumarila Bhatta's leadership, these Brahmins escaped to the Deccan. Brahmanism
made a comeback under Bhatta's leadership.

 In addition, Adi Sankaracharya revitalised and deepened Hinduism.

 During his journey of India, he vanquished Buddhist experts in religious debates held in a
variety of locations. As a result, Hinduism's dominance over Buddhism was established.

 This tradition was perpetuated by Ramanuja, Nimbarka, Ramananda, and others.

 Hinduism reclaimed its former greatness, stature, and appeal. It happened at the cost of
Buddhism.

Rifts in Buddhist Order

 The Buddhist order's internal schisms and conflicts rendered the rise of a new apostle
impossible.

 Ananda, Sariputta, and Maudgalayana's earlier instances became extremely rare.

 Buddhism's passion and missionary zeal were forever lost.

 As a result of the lack of strong preachers and reformers, Buddhism began to wane.

Worship of the Buddha

 Mahayana Buddhists were the first to introduce image worship into Buddhism. They
began to venerate the Buddha's image.

 This mode of worship was a violation of the Buddhist principles of opposing complex
rites and rituals of Brahminical worship.

 This paradox led the people to believe that Buddhism is tending towards the fold of
Hinduism. Buddhism‘s importance decreased thereby.

Lose of Royal Patronage

 With the passage of time, Buddhism's royal sponsorship dwindled.

 After Asoka, Kaniska, and Harsavardhan, no notable kings stepped up to support


Buddhism.

 In the end, the lack of such sponsorship for Buddhism paved the road for its demise.

Emergence of Rajputs

https://t.me/prelimbits
14

 The rise of the Rajputs was a major factor in the demise of Buddhism.

 Bundela, Chahamana, Chauhan, Rathore, and other dynasties were violent monarchs who
enjoyed fighting. They couldn't stand the Buddhists because of their nonviolent
teaching.

 The Buddhists fled India because they were afraid of being persecuted by the Rajput
kings. Buddhism weakened and fell out of favour.

Hun and Muslim Invasion

 Buddhism was shaken by the 'Huna' invasion. Huna chiefs like Toamana and
Mihirakula were adamantly opposed to nonviolence.

 They assassinated Buddhists in India's northwestern provinces.

 During those times, no one ventured to share the Buddha's teachings. As a result,
Buddhism has become impoverished and feeble.

 Invasion of India by Muslims nearly wiped off Buddhism. From 712 A.D. onwards, their
invasions of India became more frequent and recurrent.

 As a result of these invasions, Buddhist monks have sought refuge in Nepal and Tibet.

14. Saka era commenced from (1980)

A. 78 AD

B. 120 AD

C. 1000 AD

D. 1953 AD

Ans : A

Saka Era 78 CE/AD


 Founded by King Shalivanhana of Shatavahana dynasty
 365 days and 12 months
 India uses this calendar along with the Gregorian Calendar.

15. Which of the following was the most important characteristic of Kanishka's rule?

A. Expansion of Buddhism outside India

B. Re-emergence of Jainism

C. Fourth Buddhist Council at Srinagar

D. Gandhara School of Art

Ans : C

https://t.me/prelimbits
15

Kushanas

 The best known of the Kushana kings was Kanishka, who is thought to have ruled from
78 CE till 101 or 102 CE. BUT →
 Ironically, 78 CE is held to be the beginning of the “Saka era” in the Indian calendar.
Historians are, hence, divided about Kanishka‘s period. His rule is said to have started
anywhere between 78 and 144 CE.
o However, the latest research by Henry Falk indicated that Kanishka ascended the
throne in 127 CE. [so update ur notes]

 Kanishka was an ardent follower of Buddhism and hosted the fourth Buddhist
mahasangha or council (the third council had been held in Pataliputra during Ashoka‘s
reign). By now Mahayana Buddhism had become the dominant sect, and Kanishka
supported the missions sent to China to preach Buddhism
o Kushan rulers worshipped both Shiva and the Buddha, and the images of these
two gods appeared on the Kushan coins.
o Seveal Kushan rulers were worshippers of Vishnu, as was certainly the case with the
Kushan ruler Vasudeva, whose very name is a synonym for Krishna, an incarnation of
Vishnu.
 Kushana coins were of the highest quality [No. of Gold coins Increased during Kushana
period)
 Kanishka also patronised the Gandhara and Mathura schools of sculpture. At Mathura, a
headless statue of Kanishka depicting him as a warrior been found.
 He controlled the famous Silk Route in Central Asia.

 Another striking aspect of Kanishka‘s personality was his attitude of


religious tolerance as the king‟s coinage depicts motifs drawn from a
great variety of religious traditions: Indian (the motifs of the Shiva and the
Buddha), Greek & West Asian.
Towards the end of his long reign, it appears that he led an unsuccessful military expedition into
Central Asia against the Chinese and was defeated by General Pan-chao and was forced to pay
tribute to Chinese emperor Ho-ti.

16. Megasthenes visited the court of (1981)

A. Ashoka

B. Harsha

C. Chandragupta Maurya

D. Vikramaditya

Ans : C

Megasthenes‟s Indica
https://t.me/prelimbits
16

 Indica is an account left by a Greek ambassador, Megasthenes, sent by Seleucus Nikator


to the court of Chandragupta Maurya who lived in the Mauryan capital of Pataliputra.
 Indica describes the subcontinent in terms of its size and shape (India is a quadrilateral-
shaped country, bound by the ocean on the southern and the eastern side), soil, climate,
rivers, plants, animals (contains lengthy descriptions of elephants, horses, and monkeys as
it appears that Greeks were specially captivated by Indian animals), produce,
administration, society, legends, and folklore.
 The Greeks referred to the Indians‟ worship of Lord Krishna and addressed him as
Dionysus and Herakles/Hercules.
 They portrayed Indians as honest (since theft was rare) and having great character and
moral values.
 Megasthenes was so impressed by the contentment and prosperity of the people that he
made the sweeping remark that, ―famine has never visited India and that there has never
been a general scarcity in the supply of nourishing food and even in times of war the
Indians would never ravage the cultivated soil as was done by other nations but would fight
their battles, away from agricultural fields allowing the tillers of the soil to carry on
cultivation even when battles were raging.‖

The major faults of Megasthenes‘ work were mistakes in details, the uncritical acceptance of Indian
folklore, and a tendency to idealise Indian culture by the standards of Greek philosophy.

However, Megasthenes did identify two of the most important aspects of the Indian caste system:
Endogamy and Hereditary Occupation

17. Earliest Indian coins belonged to which period? (1981)

A. 5th century B.C.

B. Ashoka's reign

C. 1st century A.D.

D. 3rd century A.D.

Ans : A

 The first documented coinage is deemed to start with 'Punch Marked' coins issued
between the 7th -6th century BC and 1st century AD.

 These coins are called 'punch-marked' coins because of their manufacturing technique.
Mostly made of silver, these bear symbols, each of which was punched on the coin with a
separate punch.

https://t.me/prelimbits
17

Punch Marked Coin, Silver Bentbar

 Issued initially by merchant Guilds and later by States, the coins represented a trade
currency belonging to a period of intensive trade activity and urban development.

 They are broadly classified into two periods : the first period (attributed to the Janapadas
or small local states) and the second period (attributed to the Imperial Mauryan period).

 The motifs found on these coins were mostly drawn from nature like the sun, various animal
motifs, trees, hills etc. and some were geometrical symbols.

18. Out of the following the oldest dynasty is [82]

A. Mauryas

B. Nandas

C. Pallavas

D. Guptas

Ans : B

Popular Dynasty During Ancient India

 Haryanka Dynasty [544 BCE]

o The Haryanka dynasty was the third ruling dynasty of Magadha, an empire of ancient
India, which succeeded the Pradyota dynasty and Barhadratha dynasty.

 Shishunaga Dynasty [lasted from 413 BCE to 345 BCE]

 Nanda Dynasty [First non-Kshatriya dynasty]

 Maurya

 Sunga [185 BCE−73 BCE]

 Kanvas [73 BCE to 28 BCE]

 Indo–Greeks/ Bactrian Greeks [190 BCE]

 SAKAS [100 BC – 150 CE]

https://t.me/prelimbits
18

 Parthians

 Kushanas

 Satavahanas [2nd century BCE and lasted until the early 3rd century CE]

 Pallava dynasty existed from 275 CE to 897 CE

 GUPTA PERIOD [300 CE- 600 CE]

19. Lothal is [1982]

A. A closed water system

B. A poisonous gas

C. A place in Gujarat where excavations have established a link with the Indus Valley civilisation

D. Legal action taken against defector

Ans : C

 Lothal is located along the Bhogava River, a tributary of river Sabarmati, in the Gulf of
Khambat.

 It was one of the southernmost sites and only port-town of the Indus Valley civilization.

 It was a trade centre for beads, gems and ornaments in ancient times that traded with
West Asia and Africa.

 Lothal (Loth + thal) means ‗the mound of the dead‟ in Gujarati.

o Mohenjo-Daro, an Indus Valley Civilisation site, (now in Pakistan) also means ‗the
mound of the dead‘ in Sindhi.

 Lothal was a major trading centre of the Indus Valley Civilisation, and also a symbol of
India‟s maritime power and prosperity.

 According to the Archaeological Survey of India (ASI), Lothal had the world‟s earliest
known dock, connecting the city to an ancient course of the Sabarmati
River.
 The excavations shows Lothal had the Upper Town (citadel), the Lower Town, a bead-
making factory, a ware house and a tidal dockyard.

 Lothal is in the tentative list of UNESCO World Heritage Site.

National Maritime Heritage Complex

 National Maritime heritage complex (NMHC) is being developed to showcase India‟s rich
and diverse maritime heritage.

 The construction of the National Maritime Heritage Complex is being developed by the
Ministry of ports, shipping and waterways.

https://t.me/prelimbits
19

 The Heritage complex comprises of –

o Lothal mini recreation to recreate Harappan architecture and lifestyle,

o Memorial theme park,

o Maritime and Navy theme park,

o Climate theme park and

o Adventure and amusement theme park.

o World‘s tallest lighthouse museum,

o 14 galleries highlighting India‘s maritime heritage

20. Saka era was introduced by

A. Ashoka

B. Kanishka

C. Chandragupta Maurya

D. Harsha Vardhana

E. None of above [not was in official paper]

Ans : E

Shaka era

 The Shaka era is a historical Hindu calendar era (year numbering), the epoch (its year zero)
of which corresponds to Julian year 78.

 Its was started by the king called Shalivahana of Shatavahana dynasty

 The beginning of the Shaka era is now widely equated to the ascension of king
Chashtana in 78 CE.

 A previously more common view was that the beginning of the Shaka era corresponds to
the ascension of Kanishka I in 78 CE

o However, the latest research by Henry Falk indicated that Kanishka ascended the
throne in 127 CE

 According to historian Dineshchandra Sircar, the historically inaccurate notion of


"Shalivahana era" appears to be based on the victory of the Satavahana ruler
Gautamiputra Satakarni over some Shaka (Western Kshatrapa) kings.

 An account claims that the emperor Shalivahana, grandson of legendary emperor


Vikramaditya defeated the Shakas in 78 CE, and the Shaka era marks the day of
this conquest.
o This legend has been mentioned in the writings of Brahmagupta (7th century CE), Al-
https://t.me/prelimbits
20

Biruni (973–1048 CE), and others.

o Over time, the word "Shaka" became generic, and came to be mean "an era"; the era
thus came to be known as "Shalivahana Shaka"

21. The script of Indus Valley civilisation was [1982]

A. Persian

B. Dravidian

C. Sanskrit

D. Undeciphered

Ans. D

Script of IVC
 Oldest script in Indian sub-continent.
 Pictographic script (yet to be deciphered). [So main source of knowledge about
Harappan culture is based only on the archaeological excavations ]
 Writing was boustrophedon -writing in right to left in one line & then left to right in the
next line.
 Most inscriptions are short
 Too many sign.

22. The philosophical systems of the Hindus were propounded and properly codified in the
[1982]

A. Vedic age

B. Maurya age

C. Kanishka age

D. Gupta age

Ans. A

SCHOOLS OF INDIAN PHILOSOPHY

 Philosophy arose in India as an enquiry into the mystery of life and existence
 Indian Philosophy refers to several traditions of philosophical thought that originated in the
Indian subcontinent.
 Over centuries, India‘s intellectual exploration of truth has come to be represented by six
systems of philosophy. These are known as Vaishesika, Nyaya, Samkhya, Yoga, Purva
Mimansa and Vedanta or Uttara Mimansa.

https://t.me/prelimbits
21

o These six systems of philosophy are said to have been founded by sages Konada,
Gotama, Kapila, Patanjali, Jaimini and Vyasa, respectively. These philosophies still
guide scholarly discourse in the country.
Orthodox Schools of Indian Philosophy

 Orthodox (astika) schools, originally called sanatana dharma, are collectively referred to as
Hinduism in modern times. The ancient Vedas are their source and scriptural authority.
Hinduism consists of six systems of philosophy & theology.
Samkhya (Kapila)

 Samkhya is the oldest of the orthodox philosophical systems, and it postulates that
everything in reality stems from purusha (self, soul or mind) and prakriti (matter,
creative agency, energy).
 Purush cannot be modified or changed while prakriti brings change in all objects.
 Emphasizes the attainment of knowledge of self through meditation and concentration.
 School does not believe in the existence of God.
At the same point, it does not describe what happens after moksha and does not
mention anything about Ishwara or God, because after liberation there is no essential
distinction of individual and universal puruṣa.
o So what happens after Moksha is the irrelevant thing for this school.
o But at the same time, though godless, the Sankhya believes in the doctrine of
Karma and of transmigration of souls.
 This philosophy adversely affected the Tantra sadhana a lot.
 Sankhya accepts the theory of rebirth or transmigration of the soul
Yoga (Pata njali)

 Yoga school introduces the methods of the discipline of body and mind. Sage Patanjali is
the founder of Yoga.
 Emancipation of Purusha from Prakriti by self-awareness through the discipline of
body and mind is conceptualized by Yoga.
 It is believed that practising Ashtanga Yoga is the way to relieve oneself from past sins in
order to make way for liberation.
Nyaya (Gautama Muni)

 The term ‗Nyaya‘ means “justice”, “rules”, “method” or “judgment”


 It approaches philosophical questions in a scientific and rational approach.
 According to Nyaya, there were four valid sources of knowledge — perception, inference,
comparison, and testimony
 Four sources of invalid knowledge: memory, doubt, error (false certainty),
and hypothetical argument (―If there was no fire, there wouldn‘t be smoke, but there is
https://t.me/prelimbits
22

smoke, so there must be fire‖).


 This school believes attaining knowledge through the five senses is the sole way of
attaining liberation from the cycle of birth and death.
Vaisheshika (Kanada)

 Vaisheshika school of Hinduism, like Buddhism, accepted only two reliable means to
knowledge: perception and inference
 Vaisheshika School is known for its insights in naturalism. It is a form of atomism in
natural philosophy.
 It postulated that all objects in the physical universe are reducible to paramāṇu(atoms),
and one‘s experiences are derived from the interplay of substance, quality, activity,
commonness, particularity and inherence
Purva Mimamsa (Jaimini)

 It places emphasis on the power of yajnas and mantras in sustaining the activities of the
universe.
 This school of thought believes in complete authority of Vedas.
 This school of thought states that a human being can attain salvation only by acting in
conformity with the principles of Vedas.
 Two of their most ardent supporters, Sabar Swami and Kumarila Bhatta, made further
inroads into the doctrine
Uttara Mimamsa (Vedanta)

 Vedanta school is a monistic school of philosophy that believes that the world is unreal
and the only reality is Brahman.
 The school separated into six sub-schools,
o Advaita (Adi Shankara): It states that both the individual self (Atman) and Brahman
are the same, and knowing this difference causes liberation.
o Visishtadvaita (Ramanuja): It believes that all diversity is subsumed to a unified
whole.
o Dvaita (Madhvacharya): It considers Brahman and Atman as two different entities,
and Bhakti as the route to eternal salvation.
o Dvaitadvaita (Nimbarka): It states that the Brahman is the highest reality, the
controller of all.
o Shuddhadvaita (Vallabhacharya): It states that both God and the individual self are
the same, and not different.
o Achintya Bheda Abheda (Chaitanya Mahaprabhu): It emphasizes that the
individual self (Jīvatman) is both different and not different from Brahman.

https://t.me/prelimbits
23

23. Which of the following statements is not correct about Nalanda University? [1983]

A. It was an ancient Buddhist learning centre

B. A Chinese Hiuen Tsang studied here

C. We came to know about it only through the writings of Chinese pilgrims

D. It was mainly responsible for spreading Buddhism to China, Japan and South East Asia

Ans. C

Nalanda

 Nalanda was a renowned mahavihara (Buddhist monastic university) in ancient


Magadha (modern-day Bihar)

 Considered by historians to be the world's first residential university and among the
greatest centers of learning in the ancient world, it was located near the city of
Rajagriha (now Rajgir) and about 90 kilometres (56 mi) southeast of Pataliputra (now
Patna).

 Operating from 427 until 1197 CE, Nalanda played a vital role in promoting the
patronage of arts and academics during the 5th and 6th century CE, a period that has
since been described as the "Golden Age of India" by scholars.

 Nalanda was established during the Gupta Empire era, and was supported by numerous
Indian and Javanese patrons – both Buddhists and non-Buddhists

 Over some 750 years, its faculty included some of the most revered scholars of Mahayana
Buddhism.

 Nalanda mahavihara taught six major Buddhist schools and philosophies such as
Yogachara and Sarvastivada as well as subjects such as grammar, medicine, logic and
mathematics.

 The university was also a major source of the 657 Sanskrit texts carried by pilgrim
Xuanzang and the 400 Sanskrit texts carried by Yijing to China in the 7th century, which
influenced East Asian Buddhism.

 Many of the texts composed at Nalanda played an important role in the development of
Mahayana and Vajrayana Buddhism including the Mahavairocana Tantra and the
Bodhisattvacaryāvatāra of Shantideva.

 It was sacked and destroyed by the troops of Muhammad Bakhtiyar Khalji, partly
restored thereafter, and continued to exist till about 1400 CE.

 Today, it is a UNESCO World Heritage Site.

Faxian visit (399–412 CE)

 When Faxian, a Chinese Buddhist pilgrim monk, visited the city of Nalanda, there probably

https://t.me/prelimbits
24

was no university yet.

 Faxian had come to India to acquire Buddhist texts, and spent 10 years in India in the
early fifth century, visiting major Buddhist pilgrimage sites including the Nalanda area.

 He also wrote a travelogue, which inspired other Chinese and Korean Buddhists to visit India
over the centuries; in it he mentions many Buddhist monasteries and monuments across
India.

 However, he makes no mention of any monastery or university at Nalanda even though


he was looking for Sanskrit texts and took a large number of them from other parts of India
back to China. This silence in Faxian's memoir suggests that Nalanda monastery-
university did not exist around 400 CE.

Foundation (5th century)

 Nalanda's dateable history begins in the 5th century. A seal discovered at the site
identifies a monarch named Shakraditya (Śakrāditya) as its founder and attributes the
foundation of a sangharama (monastery) at the site to him.

 In the Indian tradition and texts, kings were called by many epithets and names. Scholars
such as Andrea Pinkney and Hartmut Scharfe conclude that Shakraditya is same as
Kumaragupta I. → He was one of the kings in the Hindu dynasty of the Guptas.
 Further, numismatic evidence discovered at Nalanda corroborate that Kumaragupta I
was the founder patron of the Nalanda monastery-university.
Post-Gupta dynasty (550–600 CE)

 After the decline of the Guptas, the most notable patron of the Nalanda Mahavihara was
Harsha (known as Śīlāditya in some Buddhist records).

 He was a seventh-century emperor with a capital at Kannauj (Kanyakubja).

 According to Xuanzang, Harsha was a third generation Hindu king from the Vaishya
caste, who built majestic Buddhist viharas, as well as three temples – Buddha, Surya and
Shiva, all of the same size.

 He states (c. 637 CE), "a long succession of kings" had built up Nalanda till "the whole is
truly marvellous to behold"

Xuanzang's visit (630–643 CE)

 Xuanzang travelled around India between 630 and 643 CE, visiting Nalanda in 637 and
642, spending a total of around two years at the monastery

 He was warmly welcomed in Nalanda where he received the Indian name of Mokshadeva
and studied under the guidance of Shilabhadra, the venerable head of the institution at
the time. He learnt from him Yogachara, a school of thought that had then only partially
been transmitted to China.
https://t.me/prelimbits
25

 Besides Buddhist studies, the monk also attended courses in grammar, logic, and Sanskrit,
and later also lectured at the Mahavihara.

 Xuanzang returned to China with 657 Sanskrit texts and 150 relics carried by 20 horses in
520 cases. He translated 74 of the texts himself

Yijing's visit (673–700 CE)

 Unlike Faxian and Xuanzang, Yijing followed the sea route around Southeast Asia and Sri
Lanka.

 He arrived in 673 CE, and stayed in India for fourteen years, ten of which he spent at
the Nalanda Mahavihara.

 When he returned to China in 695, he had with him 400 Sanskrit texts and 300 grains of
Buddha relics which were subsequently translated in China

 Unlike Xuanzang, who also described the geography and culture of seventh-century India,
Yijing's account primarily concentrates on the practice of Buddhism in India and detailed
descriptions of the customs, rules, and regulations of the monks at the monastery. In his
chronicle, Yijing notes that revenues from 200 villages (as opposed to 100 in Xuanzang's
time) had been assigned toward the maintenance of Nalanda. He described there being
eight vihara with as many as 300 cells. According to him, Nalanda monastery has numerous
daily Nikaya procedures and rules for the monks.

Korean and Tibetan pilgrims (650–1400 CE)

Just remember the names for Mcq purpose

 Monks such as Kyom-ik began visiting Indian monasteries by the mid-6th century.

 In the mid-7th century, the Silla monk Hyon-jo visited and stayed at several Indian
monasteries, including three years at Nalanda, his visit corroborated by Yingji.

 In and after the 7th century, Tibetan monks such as Thonmi Sambhota came to Nalanda
and other Indian monasteries to study, not only Buddhism, but Sanskrit language, grammar
and other subjects.

Pala dynasty (750–1200 CE)

 The Palas established themselves in eastern regions of India in mid-8th century and
reigned until the last quarter of the 12th century.

 They were a Buddhist dynasty. However, under the Palas, the traditional Mahayana
Buddhism of Nalanda that inspired East Asian pilgrims such as Xuanzang was superseded
by the then newly emerging Vajrayana tradition, a Tantra-imbibed, eros- and deity-
inclusive esoteric version of Buddhism.

 Nalanda continued to get support from the Palas, but they subscribed to Vajrayana
Buddhism and they were prolific builders of new monasteries on Vajrayana mandala
ideas such as those at Jagaddala, Odantapura, Somapura, and Vikramashila.
https://t.me/prelimbits
26

 Odantapura was founded by Gopala, the progenitor of the royal line, only 9.7 kilometres
(6 mi) from Nalanda.

 Inscriptions, literary evidence, seals, and ruined artwork excavated at the Nalanda site
suggest that Nalanda remained active and continued to thrive under the Palas.

 Kings Dharmapala and Devapala were active patrons.

 A number of 9th-century metallic statues containing references to Devapala have been


found in its ruins as well as two notable inscriptions.

 The first, a copper plate inscription unearthed at Nalanda, details an endowment by the
Shailendra King, Balaputradeva of Suvarnadvipa (Sumatra in modern-day Indonesia).

 This Srivijayan king, "attracted by the manifold excellences of Nalanda" had built a
monastery there and had requested Devapala to grant the revenue of five
villages for its upkeep, a request which was granted.
 The Ghosrawan inscription is the other inscription from Devapala's time and it mentions
that he received and patronised a learned Vedic scholar named Viradeva who was later
elected the head of Nalanda.

 While the Palas continued to patronize Nalanda liberally, the fame and influence of Nalanda
helped the Palas. The Srivijaya kingdom of southeast Asia maintained a direct
contact with Nalanda and the Palas, thus influencing the 9th to 12th century art in
Sumatra, Java, southern Thailand and regions that actively traded with the Srivijaya
kingdom.

 The influence extended to the Indonesian Shailendra dynasty. The Indonesian bronzes and
votive tablets from this period show the creativity of its people

 Monks from Indonesia, Myanmar and other parts of southeast Asia came to Nalanda
during the Pala rule.

Destruction under Bakhtiyar Khalji (c. 1200 CE)

 The troops of the Ghurid dynasty general Muhammad Bakhtiyar Khalji destroyed and
began the demise of Nalanda and other monasteries near it, such as the Odantapura Vihar
(now called Bihar Sharif.

24. Ancient Indian art and architecture reached its zenith during the period of [1983]

A. Rajputs

B. Mauryas

C. Guptas

D. None of these

https://t.me/prelimbits
27

Ans. C

Gupta Age : The Golden Era [300 CE- 600 CE]

 Around c. 230 CE, the Kushans reign ultimately came to an end in northern India. →
 Around the last decades of the 3rd century CE (around 275 CE), a new empire arose on
the ruins of the Kushan empire and established its sway over a good part of the former
dominions of both the Kushans and the Satavahanas. →

 This was the GUPTA EMPIRE, which kept northern India politically united for
more than a century. The Guptas are believed to be feudatories of the Kushans.
 Founder of Dynasty was Sri Gupta.

 Historians often consider the Gupta period as the „Golden Age‟ of India‘s past as it is
often remembered for certain key classical features, such as the →
o Political Unification of a large part of the subcontinent under a mostly centralised
government.
o EXCEPTIONAL SANSKRIT LITERARY work
o Sophisticated Stone Sculpture and Architecture, and
o Innovative Developments in the field of science and technology.

25. The following were famous scholars in ancient India

1. Ashvaghosh

2. Nagarjuna

3. Vasumitra

4. Buddhaghosa

Which of them was/were contemporary of Kanishka?

A. 1,2, 3, 4

B. 1 3 and 4

C. 1 and 2 only

D. 1, 2 and 3

Ans. D

Kushanas

 The best known of the Kushana kings was Kanishka


 Kanishka was an ardent follower of Buddhism and hosted the fourth Buddhist
mahasangha or council
He patronised great scholars and eminent personalities of that era such as:

https://t.me/prelimbits
28

 Ashvagosha : A Buddhist scholar who wrote the hagiographic Buddhacharita (the sacred
biography of the Buddha) and composed the Saundarananda (a Sanskrit kavya).
He is also author of the first Sanskrit play, Sariputraprakarana (Source: Tamil Nadu
state board.)
 Charaka : He is known as the father of Ayurveda who wrote a book on medicine called
Charaksamhita and also wrote the Sasruta.
 Vasumitra : An eminent philosopher who authored the encyclopaedia of Buddhist
philosophy called Mahavibhasa.
 Nagarjuna: He is often termed an Indian Einstein who proposed the Theory of Relativity in
his time in the form of a Sutra, the Prajna Parimata Sutra. He was also a great exponent of
the Mahayana doctrine and propounded the Madhyamaka (also known as Sunyavada
School) which focuses on sunyata or emptiness
 Mathara: He was a minister who was noted for his unusual Intelligence.
 Agesilaus: A Greek engineer under whose guidance, it is believed, the great stupa of
Purushapura was built.

26. Rigveda, the oldest of the Vedas, chiefly contains [1983]

A. Collection of hymns to gods

B. Methods of Vedic yagnas

C. Early Aryan culture

D. Origin and works of Hindu gods

Ans. A

VEDIC LITERATURE

Rig Veda

 The Rig Veda is a collection of 1,028 hymns, divided into 10 Mandals (books).

 They are the earliest compositions and hence, they depict the life of the Early Vedic
people in India.

 Recently, the Rig Veda has been included by the UNESCO in the list of literature signifying
World Human Heritage.

o Earliest Mandals, i.e., II to VII are called family books as they are ascribed to
particular families of seers/rishis.

o Mandal VIII – Mostly relate to Kanva‟s family.

o Mandal IX – Compilation of Soma hymns.

o Mandal I and X – Are later additions and contains the Purusashukta which
explains the four varnas. They also deal with philosophical questions and also talk
https://t.me/prelimbits
29

about various virtues including a charity in the society

o The priests related to the Rig Veda are Kotri or Motri, and the Upveda of the Rig
Veda is the Ayurveda.

o Only surviving recension of the Rig Veda is the Shakala shakha.

 Rigvedic books 1 and 10 deal with philosophical questions and also talk about various
virtues including a charity in the society

 The meters used to form hymns are Gayatri, Anushtubh, Trishtubh and Jagati
(Trishtubh and Gayatri are most important)

 The Rigvedic hymns are dedicated to various deities, chief of whom are Indra, a heroic
god praised for having slain his enemy.

 Upaveds : Ayurveda (Medicine), associated with the Ṛig Veda


 Rigveda Brahmanas : Aitareya-brahmana and the Kaushitaki- (or Sankhayana-)
brahmana

 Aranyakas of the Rigveda:


o Aitareya Aranyaka

o Kaushitaki/ Shankhayana Aranyaka

 Upanishads of the Rigveda :


o Aitareya Upanishad,

o Kaushitaki Upanishad

Sama Veda

 The Sama Veda is the collection of verses mostly taken from the Rig Veda, but arranged
in a poetic form to facilitate singing

 Samaveda is categorised into two parts – Part-I includes melodies called Gana & Part-II
includes three verses book called Archika.

 It is a collection of 1,810 melodies, and also contains the famous Dhrupada raga, later
sung by Tansen.

 The Upveda of the Sama Veda is the Gandharva Veda

 There are two Upanishads embedded in Samaveda

o Chandogya Upanishad and

o Kena Upanishad

 Aranyakas of the Samaveda:


o Talavakara or Jaiminiya-Upanshad Aranyaka
https://t.me/prelimbits
30

o Chandogya- Aranyaka

 Brahmana
o Panchavimsha / Tandya Brahmana

o Samavidhana Brahmana

o Daivata Brahmana

o Samhitopanishad Brahmana

o Arsheya Brahmana

o Vamsha Brahmana

o Jaiminiya Brahmana

 Recensions (Shakhas) of the Sama Veda are

o Kauthuma,

o Jaiminiya (Talavakara), and

o Ranayaniya.

Yajur Veda

 The Yajur Veda deals with the procedure for the performance of sacrifices.

 The texts are further divided into:

o Shukla Yajur Veda/White Yajur Veda/Vajasaneya (contains only the Mantras). It


contains the Madhyandina and Kanva recensions. Vajasaneyi Samhita is the
Samhita in the Shukla Yajurveda

o Krishna Yajur Veda/Black Yajur Veda (contains both mantras and prose
explanations/ commentary). It contains the Kathaka, Maitrayani, Taittiriya, and
Kapishthala recensions.

 The Upveda of the Yajur Veda is the Dhanur Veda.

 Yajurveda consists of various Upanishads

o Brihadaranyaka Upanishad, the Isha Upanishad, the Taittiriya Upanishad, the


Katha Upanishad, the Shvetashvatara Upanishad, the Maitri Upanishad

 Aranyaka of Shukla Yajurveda: Brihadaranyaka


 Brahmana
o Shatapatha Brahmana

o Taittiriya Brahmana

Atharva Veda

https://t.me/prelimbits
31

 The Atharva Veda is a collection of magic spells and charms to ward off the evil spirits
and diseases.

 It is the last Veda and is considered a non-Aryan work divided into 20 kandas (books),
with 711 hymns. Unlike Samaveda where hymns are borrowed from Rigveda, hymns of
Atharvaveda are unique except a few

 Brahmana : Gopatha Brahmana.


 Recensions : Shaunaka and Paippalada
 Upveda : Shilpa Veda
 Upanishads of the Atharvaveda:
o Mundaka Upanishad,

o Mandukya Upanishad,

o Prashna Upanishad.

 No Aranyaka which belongs to the Atharvaveda.

27. Major difference between 'Varna' and 'Jati', is that [1984]

A. Jatis are limited whereas Varnas are unlimited

B. Jatis are only four whereas Varnas are many

C. Varnas are only four whereas Jatis are many

D. Jati is decided by birth but not Varna

Ans. C

Verna system

 Varṇa in the context of Hinduism, refers to a social class within a hierarchical caste
system.

 The ideology is epitomized in texts like Manusmriti, which describes and ranks FOUR
VARNAS, and prescribes their occupations and duties, or dharma

o Brahmins: vedic scholars or priests or teachers.

o Kshatriyas: administrators or rulers or warriors.

o Vaishyas: agriculturalists or farmers or merchants.

o Shudras: artisans or laborers or service providers.

 Communities which belong to one of the four varnas or classes are called savarna Hindus.
The Dalits and tribals who do not belong to any varna were called avarna.

https://t.me/prelimbits
32

 This quadruple division is a form of social stratification, quite different from the more
nuanced system Jātis which correspond to the European term "caste".

 The varna system is discussed in Hindu texts, and understood as idealised human callings.
The concept is generally traced to the PURUSHA SUKTA verse of the Rig Veda.

During Early Vedic Period

 The 4-fold Varna order and rigid caste- system was not yet completely developed

 The term Varna used in Rigveda with refrence to only Aryans & Dasa having respectively
fairs & dark complexion

 Rig Vedic society was patrilineal and birth of a son was preferred. Though the women had
important positions in the society since they were educated and had access to the
assembly. They Composed Hymns and received Upanayana. Women attended the
meetings of the Vidatha. Girls were free to choose their life partners

In later Vedic Period

 Visible 4-fold varna system and appearance of several ‗jaatis‟ or castes made the social
system complex.

 Untouchability appeared; women‟s position degraded as they no longer got formal


education.
 Gotra was the place where cattle resided together with ‗janas‘ and later developed into an
identity for the ‗janas‘.
 Nishad, Chandala and Shabar were the untouchables mentioned. Guests were called
‗Goghna‟ (cow-killer).
 Niyoga too was considered a negative activity.
 Male members of upper three varnas were called dvija or twice born. Only these were
entitled to Upanayan i.e. wearing the sacred thread.
 Women like Gargi Gotra & practice of gotra exogamy and Maitreyi accomplished in the
knowledge arena; Gargi outwitted Yajnavalkya in a philosophical discourse.

 Sati and Child Marriage were still largely absent. But system of Sati emerged in the
shape of a formal custom during the later Vedic period.

 4-fold „ashram‟ (stages) for 4 „purusharthas‟ (goals)


o Brahmacharya (Celibate Student) for knowledge i.e. Dharma.

o Grihastha (Householder) for wealth and progeny i.e. „artha‟ and „kama‟.

o Vanaprastha (hermit in retreat) for spiritual wisdom.

o Sanyasa (Renunciation) for liberation i.e. Mukti/Moksha.

https://t.me/prelimbits
33

28. Ajanta paintings depict [1984]

A. Ramayana

B. Mahabharata

C. Jatakas

D. Panchatantra

Ans. C

Ajanta Caves (2nd C. – 5th C.)

 Approx. 30 rock-cut Caves → Almost exclusively Buddhist.


 Location : Aurangabad district of Maharashtra (UNESCO World Heritage Site)
 The walls of the caves have both Murals and Fresco Paintings(painted on wet plaster) They
use tempera style, i.e. use of pigments.
 The paintings portray human values and social fabric along with styles, costumes and
ornaments of that period. The emotions are expressed through hand gestures.
 The unique feature of the paintings is that each female figure has a unique hairstyle. Even
animals and birds are shown with emotions.
 The Common Themes of these paintings range from Jataka stories to life of Buddha to
elaborate decorative patterns of flora and fauna.
 The outline of the figures is in red ochre, with contours of brown, black or deep red.
Some important paintings at Ajanta are:
 Scenes from the Jataka stories → Buddha‟s former lives as a Bodhisattva, the life of
Gautam Buddha, etc.
 Paintings of various Bodhisattvas in tribhanga pose in Cave No.
o Vajrapani (protector and guide, a symbol of Buddha‘s power)
o Manjusri (manifestation of Buddha‘s wisdom) and
o Padmapani (Avalokitesvara) (symbol of Buddha‘s compassion).
 The Dying Princess in Cave No. 16.
 Scene of Shibi Jataka, where King Shibi offered his own flesh to save the pigeon.
 Painting of Maya Devi, the mother of the Buddha
 Cave No. XVII representing Buddha‟s visit to the door of Yashodhara‟s abode in the city
of Kapilavastu
History
 The Ajanta Caves are generally agreed to have been made in two distinct phases
o 1st Phase → Satavahana period

https://t.me/prelimbits
34

o 2nd Phase (5th century) → During the reign of Hindu Emperor Harishena of the
Vākāṭaka dynasty. The second phase is attributed to the theistic Mahāyāna, or
Greater Vehicle tradition of Buddhist

29. Which of the following presents the most significant features of Indus Valley
Civilisation?

A. Buildings with perfect arches


B. Use of burnt mud bricks
C. Use of sun dried mud bricks
D. None of these
Ans. B
Harappan Civilization (Indus Valley Civilization)
 It flourished around 2,500 BC, in the western part of South Asia, in contemporary
Pakistan and Western India.

 Since Harappa was the first site → Harappan Civilisation.


 Harappan culture is divided into various phases
o Early Harappan 3000–2600 BC
o Mature Harappan 2600–1900 BC
o Late Harappan 1900–1700 BC
 It was first site excavated in 1921 by Dayaram Sahni. It was larger than ancient Egypt and
Mesopotamia civilisations.
 Northern-most site Manda (Jammu-Kashmir),
 Southern-most site Daimabad (Maharashtra),
 Eastern-most site Alamgirpur (Uttar Pradesh),
 Western-most site Sutkagendor (Pakistan-Iran border).
Important features
 Systematic town-planning on the lines of grid system (Non Non- Aryan civilisation)

https://t.me/prelimbits
35

 Use of burnt bricks in construction.


 Underground drainage system.
 Own characteristic pottery, seals and script

30. What was the main feature of Maurya dynasty in India? [1984]

A. Education was widespread

B. Terrorists were driven out of Sind and Punjab

C. Removal of land tax for the first time

D. De-centralization of administration

Ans. D*

 Decentralized government was the key feature of the Mauryan government.

 All the powers were concentrated in the hands of the emperor. Though he was the
supreme authority of judiciary, civil and military administration, he wasn‟t a complete
autocrat and his authority was under some restraints.

 The most important feature of the Mauryan administration was the presence of vast,
numerous powers of the bureaucracy.

 Mauryan administration was highly organised. The detailed work of administration was
divided among various departments, and managed by a systematic and highly
organised bureaucracy.

 Mauryan bureaucracy was hierarchical and pyramidal and the highest functionaries was
the mantrin, purohita, senāpati and yuvarāja.

o Though it was a monarchy form of government which Kautilya insisted on, he stood
against royal absolutism. He advocated on idea that, king should run the
administration with assistance from council of ministers

Though Feudal System was not present which were seen during gupta . In many books and site it
is written that Mauryan Empire was Centralized one.

So this is not given in option. So it is like india where we say we are federal but at the same time
we are a Union Nation.

31. Indus Valley Civilization was spread over [1984]

A. Baluchistan, Sind, Punjab and Rajasthan

B. Sind, Punjab and Kashmir

C. Afghanistan, Sind, Punjab, Rajasthan, Gujarat and Kashmir

D. None of these

https://t.me/prelimbits
36

Ans. C

HARAPPAN CIVILIZATION (INDUS VALLEY CIVILIZATION)

 IVC also known a Bronze Age civilisation in the northwestern regions of South Asia, lasting
from 3300 BCE to 1300 BCE, and in its mature form 2600 BCE to 1900 BCE
 Together with ancient Egypt and Mesopotamia, it was one of three early civilisations of
the Near East and South Asia, and of the three, the most widespread.

 Its sites spanned an area from northeast Afghanistan and much of


Pakistan to western and northwestern India.
 The civilisation flourished both in the alluvial plain of the Indus River, which flows
through the length of Pakistan, and along a system of perennial monsoon-fed rivers that
once coursed in the vicinity of the Ghaggar-Hakra, a seasonal river in northwest India and
eastern Pakistan.
 The cities of the ancient Indus were noted for their
o Urban planning,
o Baked brick houses,
o Elaborate drainage systems,
o Water supply systems,
o Clusters of large non-residential buildings, and
o Techniques of handicraft and metallurgy.

https://t.me/prelimbits
37

 Mohenjo-daro and Harappa very likely grew to contain between 30,000 and 60,000
individuals, and the civilisation may have contained between one and five million
individuals during its florescence.
 The Indus civilisation is also known as the Harappan civilisation, after Harappa was first
to be excavated early in the 20th century.
 The discovery of Harappa and soon afterwards Mohenjo-daro was the culmination of work
that had begun after the founding of the Archaeological Survey of India in the British Raj
in 1861.
 There were earlier and later cultures called Early Harappan and Late Harappan in the
same area.
o The early Harappan cultures were populated from Neolithic cultures, the earliest
and best-known of which is Mehrgarh, in Balochistan, Pakistan.
o Harappan civilisation is sometimes called MATURE HARAPPAN to distinguish it
from the earlier cultures.
 There are five major urban centres:
o Mohenjo-daro in the lower Indus Valley (declared a UNESCO World Heritage Site in
https://t.me/prelimbits
38

1980 as "Archaeological Ruins at Moenjodaro"),


o Harappa in the western Punjab region, Ganeriwala in the Cholistan Desert,
o Dholavira in western Gujarat (declared a UNESCO World Heritage Site in 2021 as
"Dholavira: A Harappan City"), and
o Rakhigarhi in Haryana.
 The Harappan language is not directly attested, and its affiliation uncertain as the Indus
script has remained undeciphered. A relationship with the Dravidian or Elamo-Dravidian
language family is favoured by a section of scholars
Harappan culture is divided into various phases

 Early Harappan 3000–2600 BC


 Mature Harappan 2600–1900 BC [Harappan civilisation is sometimes called Mature
Harappan to distinguish it from the earlier cultures.]
 Late Harappan 1900–1700 BC

Discovery and History of Excavation

Harappa

 The first modern accounts of the ruins of the Indus civilisation are those of Charles
Masson, a deserter from the East India Company's army.

 Later Company contracted Alexander Burnes to sail up the Indus (Harappa) to assess the
feasibility of water travel for its army. He also noted there ancient brick wall.

 In 1861, three years after the dissolution of the East India Company and the establishment
of Crown rule in India, archaeology on the subcontinent became more formally
organised with the founding of the Archaeological Survey of India (ASI). Alexander
Cunningham, the Survey's first director-general

o According to him Harappa was Buddhist city mentioned in the seventh century CE
travels of the Chinese visitor, Xuanzang (Aka. Hiuen Tsang)

 Archaeological work in Harappa kicked off after Lord Curzon became viceroy of India.

 He est. Ancient Monuments Preservation Act 1904, and appointed John Marshall to
lead the ASI.

 Several years later, Hiranand Sastri, who had been assigned by Marshall to survey

https://t.me/prelimbits
39

Harappa, reported it to be of non-Buddhist origin, and by implication more ancient.

 Expropriating Harappa for the ASI under the Act, Marshall directed ASI archaeologist
Daya Ram Sahni to excavate the site's two mounds

Mohenjo-daro

 Farther south, along the main stem of the Indus Marshall deputed a succession of ASI
officers to survey the site. These included D. R. Bhandarkar (1911), R. D. Banerji (1919,
1922–1923), and M. S. Vats (1924).
 In 1923 Banerji wrote to Marshall about the site, postulating an origin in "remote
antiquity," and noting a congruence of some of its artifacts with those of Harappa
 Systematic excavations began in Mohenjo-daro in 1924–25 with that of K. N. Dikshit,
continuing with those of H. Hargreaves (1925–1926), and Ernest J. H. Mackay (1927–1931).
 By 1931, much of Mohenjo-daro had been excavated, but occasional excavations
continued, such as the one led by Mortimer Wheeler, a new director-general of the ASI
appointed in 1944, and including Ahmad Hasan Dani.
After the partition of India in 1947, when most excavated sites of the Indus Valley Civilisation lay
in territory awarded to Pakistan.

Chronology

 The mature phase of the Harappan civilisation lasted from c. 2600–1900 BCE. With the
inclusion of the predecessor and successor cultures – Early Harappan and Late Harappan,
respectively – the entire Indus Valley Civilisation may be taken to have lasted from the 33rd
to the 14th centuries BCE.

 It is part of the Indus Valley Tradition, which also includes the pre-Harappan occupation
of Mehrgarh, the earliest farming site of the Indus Valley.

 Several periodisations are employed for the IVC. The most commonly used classifies the
Indus Valley Civilisation into Early, Mature and Late Harappan Phase.

 An alternative approach by Shaffer divides the broader Indus Valley Tradition into four
eras,

o the pre-Harappan "Early Food Producing Era"

o Regionalisation

o Integration, and

o Localisation eras

Pre-Harappan era: Mehrgarh

 Mehrgarh is a Neolithic (7000 BCE to c. 2500 BCE) mountain site in the Balochistan
province of Pakistan,

 Mehrgarh is one of the EARLIEST SITES WITH EVIDENCE OF FARMING and herding in

https://t.me/prelimbits
40

South Asia.

 Mehrgarh was influenced by the Near Eastern Neolithic, with similarities between
"domesticated wheat varieties, early phases of farming, pottery, other archaeological
artefacts, some domesticated plants and herd animals.

Early Harappan [Early Harappan Ravi Phase]

 Lasted from c. 3300 BCE until 2800 BCE. It started when farmers from the mountains
gradually moved between their mountain homes and the lowland river valleys, and is
related to the Hakra Phase, identified in the Ghaggar-Hakra River Valley to the west, and
predates the Kot Diji Phase (2800–2600 BCE, Harappan 2), named after a site in northern
Sindh, Pakistan, near Mohenjo-daro.

 The earliest examples of the Indus script date to the 3rd millennium BCE.

 The mature phase of earlier village cultures is represented by Rehman Dheri and Amri in
Pakistan.

 Kot Diji represents the phase leading up to Mature Harappan, with the citadel
representing centralised authority and an increasingly urban quality of life. Another
town of this stage was found at Kalibangan in India on the Hakra River

 Trade networks linked this culture with related regional cultures and distant sources of
raw materials, including lapis lazuli and other materials for bead-making.

 By this time, villagers had domesticated numerous crops, including peas, sesame seeds,
dates, and cotton, as well as animals, including the water buffalo.
https://t.me/prelimbits
41

 Early Harappan communities turned to large urban centres by 2600 BCE, from where the
mature Harappan phase started. The latest research shows that Indus Valley people
migrated from villages to cities.

 The final stages of the Early Harappan period are characterised by the building of large
walled settlements, the expansion of trade networks, and the increasing integration of
regional communities into a "relatively uniform" material culture in terms of pottery
styles, ornaments, and stamp seals with Indus script, leading into the transition to the
Mature Harappan phase.

Mature Harappan

 Due to good monsoon Flood-Supported Farming led to large agricultural surpluses,


which in turn supported the development of cities.

 The IVC residents did not develop irrigation capabilities, relying mainly on the seasonal
monsoons leading to summer floods

 According to J.G. Shaffer and D.A. Lichtenstein, the Mature Harappan civilisation was "a
fusion of the Bagor, Hakra, and Kot Diji traditions or 'ethnic groups' in the Ghaggar-
Hakra valley on the borders of India and Pakistan".

 By 2600 BCE, the Early Harappan communities turned into large urban centres. Such
urban centres include Harappa, Ganeriwala, Mohenjo-daro in modern-day Pakistan, and
Dholavira, Kalibangan, Rakhigarhi, Rupar, and Lothal in modern-day India.[

32. Main preaching‟s of Buddha were in regard to [1984]

A. Right faith (thought) and conduct (action)

B. Love of God

C. Practice of rituals

D. Idol worship

Ans. A

Astangika-Marga (Eight Fold Paths)

 The Noble Eightfold Path is one of the principal teachings of the Buddha, who described it
as the way leading to the cessation of suffering (dukkha) and the achievement of self-
awakening. It is used to develop insight into the true nature of phenomena (or reality) and
to eradicate greed, hatred, and delusion.
 In Buddhist symbolism, the Noble Eightfold Path is often represented by means of the
dharma wheel (dharmachakra), whose eight spokes represent the eight elements of the
path.
 These are :

https://t.me/prelimbits
42

The path taught by the Buddha is often referred to as the MIDDLE PATH (the one between
extreme indulgence and extreme asceticism.) It should be noted that ‗Right‘ here signifies ‗proper‘,
‗whole‘, ‗thorough‘, ‗integral‘, ‗complete‘, and ‗perfect‘.
On the other hand Jainism advocated extreme penance and asceticism.

33. Gautam Buddha attained Nirvana at [1984]

A. Bodh Gaya

B. Sarnath

C. Kushi Nagar

D. Lumbini

Ans. A

LIFE OF THE BUDDHA

 Was born as Siddhartha at Lumbini (near Kapilavastu, Nepal) to Suddhodana (chief of


republican Sakya clan & mother was Mahamaya
 At the age of 29, become a wanderer & left his palace with Channa, the charioteer, and
his favourite horse, Kanthaka, in search of truth. This event is known as
Mahabhinishkramana/Great Renunciation
 Initially Meditated with Alara Kalama and then with Uddaka Ramaputta.

 Later accompanied by five wandering ascetics - Kondanna, Bhaddiya, Vappa,


Mahanama, and Assaji → he practiced severe austerities
 Later he abandoned them then moved towards the village of Senani (slightly east of Bodh
Gaya), where he was offered a bowl of milk-rice by a low-caste village girl, Sujata.

 Then took a seat under a Peepal tree facing east. Here he resolved not to rise again
until enlightenment was attained

 In deep meditation, Mara, the Lord of Illusions, rushed to distract him. Gautama
https://t.me/prelimbits
43

touched the earth, calling it to bear witness the countless lifetimes of virtue
that had led him to this place of enlightenment. Mara then unleashed his army of
demons. Gautama‟s wisdom transformed the demons‟ weapons into flowers. Mara and
all his forces fled in disarray.

 At the age of 35, he ultimately attained Nirvana/Enlightenment (the cessation of desire


and hence the end of suffering)
at Gaya, Magadha (Bihar) under a Peepal tree at Uruvela, on
the banks of river Niranjana, and came to be known as the Buddha, the Enlightened One.

 Peepal tree was known as the famous Bodhi tree. It is believed that tree was destroyed by
Asoka‘s queen Tissarakkha, Pushyamitra Shunga and by King Shashanka. [But again planted
by Buddhist]

 Delivered his First Sermon with his five former companions at deer park at Sarnath.
Event is known as ‟DhammaChakka-Pavattana‘ meaning „turning the wheel of
dharma’. These five disciples became Arhats

o Arhats : one who has gained insight into the true nature of existence and has
achieved nirvana BUT but who may not have reached full Buddhahood

 Wandered for over four decades, and established an order of monks and nuns known
as the Sangha.

 Attained Parinibbana at Kusinara (lying in Mallas).

o Parinibbana : nirvana-after-death, which occurs upon the death of someone


who has attained nirvana during his or her lifetime. It implies a release from
the Saṃsāra, karma and rebirth.
 The core of Buddha‟s doctrine is expressed in the Ariya-Sachchani (Four Noble Truths),
Ashtangika-Marga (Eight-Fold Path), Middle Pad, Social Code of Conduct, and Attainment
of Nibbana /Nirvana.

34. Which of the following can be regarded as the chief contribution of Buddhism to India?
[1985]

A. Art and literature

B. Architecture

C. Ahimsa and self-purification

D. Rituals

Ans. C

 Buddhism laid too much emphasis on the principle of Ahimsa, which greatly affected the
character of the people.

 Buddhism laid emphasis on non-violence and the sanctity of animal life.

https://t.me/prelimbits
44

 The Hindus were originally meat eaters but due to the influence of Buddhism became
vegetarian.

 Thus, Buddhism exercised a tremendous influence on India culture. It enriched religion, art,
sculpture, language and literature of India.

Important concepts

5 Teachings of Buddha [Panchshila]


 No killing Respect for life.
 Abstention from theft.
 Abstention from sexual misconduct.
 Abstention from falsehood.
 Abstention from intoxication
Five Precepts or Panchashil
 Buddha also established code of conduct both for the monastic order and the laymen to
follow. These are:
1. Do not commit violence
2. Do not covet the property of others.
3. Do not indulge in corrupt practices or sensual behaviour.
4. Do not speak a lie.
5. Do not use intoxicants

35. Which of the following statements is incorrect? [1985]

A. Chandragupta Vikramaditya killed the Saka

B. Samudragupta is known as Indian Napolean

C. Skandagupta defeated Huns

D. Kumaragupta restored the embankment of Sundarsana Lake

Ans. D

1st statement

Chandragupta II (c. 376 – 413/15 CE)

 The peak of the territorial expansion of the Gupta empire was reached during
Chandragupta II‟s reign (also known as Chandragupta Vikramaditya).
o First Gupta ruler to assume the title of 'Param Bhagawata'
o He fought many wars and thus he was awarded the title of 'Prithivyah Pratham
Veer' which means the first powerful man of the world.

https://t.me/prelimbits
45

 Extended the limits of his empire by conquests & marital alliances with other royal
dynasties of the period.
 He married to the Naga princess, and had a daughter. His daughter was married to
Rudrasena II of the ruling Vakataka dynasty in Deccan.
 Chandragupta thus exercised indirect control over the Vakataka kingdom, which proved
very beneficial to him.
 He conquered western Malwa & Gujarat from the Shaka satraps. This conquest took the
expanse of the Gupta empire to the western sea coast, which was an important region for
trade and commerce. (aka. Saka Conqueror)
 This contributed to the prosperity of Malwa and its chief city Ujjain, which was also
Chandragupta II‘s second capital (1st Pataliputra).
 An iron pillar inscription at Mehrauli in Delhi indicates that his empire included even
north-western India and Bengal.
2nd Statement

Samudragupta (c.335/336–375 CE)

 Followed a policy of war and conquest and enormously enlarged his kingdom.
 Due to this policy he is called Napoleon of India.
 His achievements are recorded in a long inscription (an eulogy) engraved on the
Allahabad pillar (Prayag prashasti), whose surface also contains inscriptions of Ashoka
and later, the Mughal emperor Jahangir.
o Unlike the Ashokan inscription which stresses on peace and nonviolence,
Samudragupta‟s inscription extolling his deeds and virtues stresses on violence
and military conquest, and gives us information about his different conquests.
o It was composed in pure Sanskrit by Harisena, who was not only his court poet but
was also a kumaramatya, that is, a high ranking official who described himself by
different titles such as sandhivigrahika (minister for peace and war) and
mahadandanayaka (judicial/military officer).
o Inscriptions called him ―Dharma Prachar Bandhu”.
 He celebrated his conquests by performing an ashvamedha (horse sacrifice) and by
issuing ashvamedha coins (portraying the scene of the sacrifice)
 He was called Anukampavan (full of compassion) as he granted permission to the
Buddhist king of Ceylon, Meghavarman, to build a monastery and rest house for
pilgrims from Ceylon at Bodh Gaya.
 He was a great patron of art and adopted the title of Kaviraja (king among poets).
 His love for music is attested by his coins that represent him playing on the veena (lyre)
on occasion.
 Though Hindu by practice, he extended his patronage to other religions, and one of his

https://t.me/prelimbits
46

ministers was the great Buddhist philosopher Vasubandhu. Samudragupta is


remembered for his patronage of art and literature.
3rd & 4th Statement

Skandagupta (c. 455 – 467 CE)

 Faced and repulsed the Huns successfully.


 Junagarh/Girnar inscription of his reign tells that his governor Parnadatta repaired the
Sudarshan lake
 Bhitari pillar inscription dates to his reign gives the chronology of Guptas and his
conflict with Pushyamitra and Huns
 The Gupta empire continued to exist but central control weakened and local governors
became feudatory kings with hereditary rights.
 Later Gupta kings included Purugupta, Kumaragupta II, Budhagupta, Narasimhagupta,
Kumaragupta III, and Vishnugupta.
 The Gupta empire declined due to a variety of factors.

36. Which of the following was not known to Indus Valley people? [1985]

A. Silver

B. Copper

C. Iron

D. Bronze

Ans. C

IVC : TECHNOLOGY AND CRAFT


 This is known as the first urbanization in India
 Along with stone, they were well acquainted with bronze (occasionally mixed arsenic with
copper instead of tin). As neither tin nor copper was easily available, bronze tools do not
abound in the region.

 Harappan culture is recognized for advanced metallurgical techniques. Archaeologists


have discovered usage of copper, bronze, tin, silver and gold. It was the first
time that silver was extracted and used in Indus valley
 Iron was not known to the people.
 Important crafts: spinning (Spindle whorls), bricklaying, boat-making, seal making,
terracotta manufacturing (potter‘s wheel), goldsmiths, bead making.
 They were aware of the use of the wheel.

https://t.me/prelimbits
47

37. Match the columns : [1985]

A. Upanishads : 1 World‘s oldest scriptures

B. Samaveda : 2. Magical chants

C. Rigveda : 3. Mysterious doctrine

D. Atharvaveda : 4. Ritual chants / Musical Chanting

Select the correct answer from the codes given below :

A B C D

a) 3 4 1 2

b) 3 4 2 1

c) 4 3 1 2

d) 3 4 2 1

Ans. A

Discussed earlier (Q.26)

38. Match the columns : [1985]

A. Amamath : 1. Buddhist caves

B. Rock cut temples : 2. Jain saint

C. Ajanta : 3. Hindu shrine

D. Bahubali: 4. Mahabalipuram

Select the correct answer from the codes given below

A B C D

a) 3 4 1 2

b) 3 4 2 1

c) 4 3 1 2

d) 3 4 2 1

Ans. A

Group of Monuments at Mahabalipuram

 Collection of 7th- and 8th-century CE religious monuments in Tamil Nadu.


 The monuments were built during the Pallava dynasty.

https://t.me/prelimbits
48

 The group contains several categories of monuments:


 Ratha temples
o They are with monolithic temples carved in the shape of chariots.
o The best-known are the five monolithic structures known as the Five Rathas or the
Pandava Rathas: Dharmaraja Ratha, Bhima Ratha, Arjuna Ratha, Nakula-Sahadeva
Ratha and Draupadi Rathas.
o Other ratha monuments at Mahabalipuram include the late-7th-century Ganesha
Ratha. It is three storeyed and of better workmanship, it resembles the Bhima-ratha
in roof form.
 Structural temples
o The structural (free-standing) temples at Mamallapuram have been built with cut
stones as building blocks.
o Some of the major temples are:
 Shore Temple: It consists of a large temple, two smaller temples and many
minor shrines, open halls, gateways, and other elements, much of which is
buried by sand.
 Olakkanesvara temple: It was built in the early 8th century from grey granite
by King Rajasimha. The walls of the temple depict the Ravananugraha legend
from the Ramayana and a relief of Dakshinamurti (Shiva as a yoga teacher)
 Mukundanayanar temple: It has ratha-like architecture.
 Rock reliefs
o The best-known rock relief is the Descent of the Ganges also known as Arjuna's
Penance or Bhagiratha's Penance. (carved on two monolithic rock boulders)
o It has two primary interpretations: the effort needed to bring the Ganges from
the heavens to earth, and the Kirātārjunīya legend and the chapter from the
Mahabharata about Arjuna's efforts to gain the weapon he needed to help good
triumph over evil.

 Cave temples
https://t.me/prelimbits
49

o Varaha cave
o Kotikal cave
o Dharmaraja cave
o Ramanuja Cave
Chola : Art & Architecture

 The art of sculpture attained high standard during this period as evident in the giant
statue of Gomateswar at Sravan Belgola.
o The Gommateshwara statue is a 57-foot (17 m) high monolithic statue on
Vindhyagiri Hill in the town of Shravanbelagola in state of Karnataka. Carved of a
single block of granite, it is the tallest monolithic statue in India and is visible
from 30 kilometres (19 mi) away.
o The Gommateshwara statue is dedicated to the Jain figure Bahubali and
symbolises the Jain precepts of peace, non-violence, sacrifice of worldly affairs, and
simple living.
Ajanta caves [Q. 28]

Amarnath Temple

 Amarnath Temple is a Hindu shrine located in Anantnag district of the Jammu and
Kashmir

 The cave is situated at an altitude of 3,888 m (12,756 ft), about 168 km from Anantnag city,
the district headquarter, 141 km (88 mi) from Srinagar, the summer capital of Jammu and
Kashmir

 The Amarnath cave, abode of the Mahamaya Shakti Peetha, is one of the 51 Shakti
Peethas, temples throughout Indian Subcontinent that commemorate the location of fallen
body parts of the Hindu deity Sati

39. Lineage is [1985]

A. A straight line joining any two points on the globe

B. The line of descendants of a particular ancestor

C. The Paleolithic age

D. The linking of genes

Ans. B

 The members of a person's family who are directly related to that person and who lived a
long time before him or her [NO FURTHER PHD]

https://t.me/prelimbits
50

40. Which of the following was the principal article of food of the Indus Valley people?

A. Fish

B. Palm-date

C. Eggs

D. Wheat

Ans. D

IVC : AGRICULTURE
 Main crops: Two types of Wheat and Barley. Evidence of cultivation of rice in Lothal and
Rangpur (Gujarat) only. Other crops: Dates, mustard, sesamum, cotton, rai, peas etc.

 First to produce cotton in the world so Greeks called them Sindon.


 Used animal drawn wooden plough, and stone sickles.
 Gabarbands or nalas enclosed by dams found but channel or canal irrigation was
probably not practised
 Produced sufficient food grains and cereals were received as taxes from peasants and
stored in Granaries for wages and emergencies same as Mesopotamia.

Granaries
Granaries are storehouses that are used to store grain or animal feed. They are usually built
above ground to keep the food stored away from mice and other animals as well as protect
foodstuffs from natural occurrences such as floods.
Granaries in the Indus Valley Civilization were found in the cities of Harappa and Mohenjo Daro.

41. Which of the following is a characteristic feature of the Indus Valley Civilisation? [1986]

A. Their chief deities were Mother Goddess and Shiva

B. They worshipped bulls

C. They had all amenities of a developed city life

D. They did not know of iron

Ans. C

IVC was noted for their

 Urban planning,
 Baked brick houses,
 Elaborate drainage systems,
 Water supply systems,
 Clusters of large non-residential buildings, and
https://t.me/prelimbits
51

 Techniques of handicraft and metallurgy.

42. Gandhara School of Art was developed during the reign of

A. Ashoka

B. Kanishka

C. Chandragupta

D. Bindusara

Ans. B

GANDHARA SCHOOL OF ART

 The Gandhara school of Art arose in modern-day Peshawar and Afghanistan on Punjab's
western boundaries.

 The Greek invaders brought the traditions of Greek and Roman sculptors with them,
which affected the region's native traditions ==> Greco-Indian School of Art

 Between 50 B.C. and 500 A.D., the Gandhara School flourished in two periods. While the
former school's sculptures were made of bluish-grey sandstone, the latter school's were
made of mud and plaster.

 The Buddha and Bodhisattvas iconography was based on the Greco-Roman pantheon
and resembled Apollo's.

 Major Centres :

o Jalalabad, Hadda, Bamaran, Begram & Taxila were the main centers where art pieces
of Gandhara School have been found.

 Major Features :

o The Gandhara sculptures have been discovered in the Taxila ruins as well as other
ancient sites in Afghanistan and Pakistan.

o They generally consist of Buddha images and relief sculptures depicting scenes
from Buddhist literature.

o Several Bodhisattva figures were cut out of the rock. The first preaching in the
deer park and the Buddha's death is depicted in a Gandhara figure.

o The predominant focus of this type of painting was Lord Buddha


and Bodhisattvas, as it was intimately tied with Mahayana Buddhism.

o The Bamiyan Buddha sculptures are an example of Gandhara style art.

o It thrived primarily in Afghanistan and present-day North-Western India.


Taxila, Peshawar, Begram, and Bamiyan were among the most prominent sites.

o From the first century BCE through the fourth century CE, the Gandhara School of
https://t.me/prelimbits
52

art flourished.

 Themes

o The Buddhist themes were largely represented in Gandhara art. The Buddha's
mother resembles an Anthenian matron.

o A Buddhist scene was created with an Apollo-like face. The image of Athena of
Rome in Lahore is perhaps one of the most beautiful Gandhara sculptures depicting
a western topic.

o It was created in order to express Buddhist ideas and practices.

o The specimens do not include any Greek art motif, with a few exceptions. The hand
of the Gandhara artist was Greek, but his heart was Indian.

 Patrons

o Gandhara School was fostered by the Shakas and the Kushanas. The first
sculptural portrayals of the Buddha in human form are attributed to
the Gandhara school. [The first image of the Buddha-was carved out during
the reign of famous Kushan king Kanishka I.]

 Features of sculpture

o The human body is sculpted in a realistic manner in these sculptures, with


considerable attention to realism and physical details, particularly in the depiction
of muscles, mustaches, and other facial features.

o The portrayal of the broad bold fold lines is a unique trait that sets it apart from what
has been discovered elsewhere in India.

43. Which of the following was the main reason of the downfall of Mauryan empire?

A. Weak successors of Ashoka who could not control the empire properly

B. Religious policy of Ashoka

C. Ashoka maintained friendly relations with his neighbours

D. None of the above

Ans. A

DECLINE OF THE MAURYAN EMPIRE

 Over-centralisation, cumbersome bureaucracy, super heavy taxation (on almost every


activity) and foreign invasions were main factors of decline.
 Neglect of North-West frontier and construction of great wall of China.
 Financial Crisis- Ashoka incurred huge expenditure for promotion of his ‗Dhamma Mission‘.
 Over-tolerant, non-violent attitude killed the fighting spirit of the army.
https://t.me/prelimbits
53

 After Ashoka‟s death in 232 BC Mauryan Emperors were incompetent and oppressive.
Pushyamitra Shunga, a Mauryan army commander, killed the last Mauryan emperor Brihadrath and
founded Shunga Dynasty in 187 BC.

44.Arrange the following in proper sequence : [1986]

1. Vinayapitaka

2. Gita Govinda

3. Raghuvamsha

4. Atharvaveda

The correct sequence is

A. 3, 2,1 ,4

B. 4, 3 ,2,1

C. 4,1,3, 2

D. 2 ,4 ,3,1

Ans. C

 Atharvaveda = 900 BCE [wiki]

 Vinayapitaka = According to tradition, the Tripiṭaka was compiled at the First Council
shortly after the Buddha's death → Budtha death = 483 BCE or 400 BCE

 Raghuvamsa = 5th century CE [exact date of writing is uncertain]

 Gita Govinda = 12th century CE by Hindu poet, Jayadeva.

Jayadev

 12th century Sanskrit poet

 Patron: -
 Work: Geet Govinda (Krishna and Radha)
o Concentrates on Krishna's love with the Radha.
o Considered an important text in the Bhakti movement of Hinduism.
Kalidasa

 4th–5th century CE) ancient India's greatest poet and playwright.


 Patron : Chandragupta II
 Work:
o Kumarasambhava
o Raghuvamsa

https://t.me/prelimbits
54

o Meghdoota
o Ritusambara.
o Abhijnana Shakuntalam,
o Vikramorvashi
o Malvikaganimithram.
Vinaya Pitaka

 The Vinaya Pitaka (Basket of Discipline') is a Buddhist scripture, one of the three parts
that make up the Tipiṭaka (lit. Three Baskets).

 Its primary subject matter is the monastic rules of conduct for monks and nuns of the
Sangha.

Other 2 Pitaka

 Sutta (Conventional teaching)


o Buddha‟s discourses on various doctrinal issues in dialogue form
o Accepted by all Buddhist schools
o divided into five Nikayas or collections:
1. Digha Nikaya
2. Majjhima Nikaya
3. Samyutta Nikaya
4. Anguttara Nikaya
5. Khuddaka Nikaya
 Abhidhamma (moral psychology / Higher Teachings).
o Detailed scholastic analysis and summary of the Buddha's teachings in the Suttas.

45. The ancient Indian coins were introduced during the [1986]

A. 3rd century AD

B. 3rd century BC

C. Kanishka's period

D. Ashoka's reign

Ans. B

Repeated Q. Also covered in Day -2 Q.7

 The first documented coinage is deemed to start with 'Punch Marked' coins issued
between the 7th -6th century BC and 1st century AD.

https://t.me/prelimbits
55

 These coins are called 'punch-marked' coins because of their manufacturing technique.
Mostly made of silver, these bear symbols, each of which was
punched on the coin with a separate punch.

 Issued initially by merchant Guilds and later by States, the


coins represented a trade currency belonging to a period of
intensive trade activity and urban development.

 They are broadly classified into two periods : the first period
(attributed to the Janapadas or small local states) and the
second period (attributed to the Imperial Mauryan period).

 The motifs found on these coins were mostly drawn from nature like the sun, various animal
motifs, trees, hills etc. and some were geometrical symbols.

46. Arrange the following Buddhist councils in a chronological order [1986]

1. Vaishali

2. Rajagraha

3. Patliputra

4. Sri Nagar

The correct order is

A. 1, 4,3,2

B. 3, 2,1,4

C. 4,1,2,3

D. 2,1,3,4

Ans. D

BUDDHIST COUNCILS

Number Venue, Patronage & Outcome

First Buddhist  At Satparni Rajagriha (483 BC)


Council  Under the patronage of King Ajatshatru with the monk Mhakasyapa
Upali presiding, at Rajgriha, in the Sattapani Caves
 The idea was to preserve Buddha‟s teachings (Sutta) and rules for
disciples (Vinaya). Ananda was one of the great disciples of Buddha
recited Suttas and Upali, another recited Vinaya. Abhidhamma Pitaka was
also included.

Second Buddhist  At Vaishali (383 BC)


Council  Under the patronage of King Kalasoka and the presidency of

https://t.me/prelimbits
56

Sabakami. The ruler at time was Kalasoka (Shishunag Dynasty)


 This council settled down the disputes on the Vinaya Pitaka and code
of discipline.

Third Buddhist  At Pataliputra (250 BC)


Council  Under the patronage of King Asoka and the presidency of
Moggaliputta Tissa
 The teachings of Buddha which were under two baskets were now
classified in 3 baskets as Abhidhamma Pitaka was established in this
council and they were known as „Tripitaka‟. It also tried to settle all the
disputes of Vinaya Pitaka. Establishment of Sthaviravada School as an
orthodox school.

Fourth Buddhist  At Kundalvana, Kashmir (72 AD)


Council  Under the patronage of Kushan king Kanishka and the president of this
councilwas Vasumitra, with Avaghosa /Ashvaghosha as his deputy.
 This council distinctly divided the Buddhism into 2 sects Mahayana and
Hinayana.

47. Metals known to Indus Valley Civilization were [1986]

A. Gold, copper, silver but not iron

B. Silver, iron, bronze but not gold

C. Iron, copper, gold but not silver

D. Zinc, silver, iron but not gold

Ans. A

Repeated Q

IVC : TECHNOLOGY AND CRAFT

 This is known as the first urbanization in India


 Along with stone, they were well acquainted with bronze (occasionally mixed arsenic with
copper instead of tin). As neither tin nor copper was easily available, bronze tools do not
abound in the region.
 Harappan culture is recognized for advanced metallurgical techniques. Archaeologists
have discovered usage of copper, bronze, tin, silver and gold. It was the first time that
silver was extracted and used in Indus valley
 Iron was not known to the people.
 Important crafts: spinning (Spindle whorls), bricklaying, boat-making, seal making,
terracotta manufacturing (potter‘s wheel), goldsmiths, bead making.
 They were aware of the use of the wheel.
https://t.me/prelimbits
57

48. Match the columns : [1987]

A. Roaper 1. Informs about Maury an Empire

B. Kumarahar 2. Treaty of alliance between Bentinck and Ranjit Singh

C. Aihole 3. Statue of Lord Bahubali

D. Shravan Belgola 4. Sites of Chalukyan architecture

Select the correct answer using the codes given below:

Select the correct answer from the codes given below

A B C D

a) 3 4 1 2

b) 2 1 4 3

c) 4 3 1 2

d) 3 4 2 1

Ans. B

RUPNAGAR TREATY OF 1831 [aka. TEATY OF ROPAR]

 In October 1831, the small town of Rupnagar (Ropar) on the banks of Sutlej witnessed a
historic meeting between Maharaja Ranjeet Singh and Lord William Bentnick, the
Governor General of Indian territory under the East India Company.

 At the conclusion, a document was presented by Lord Bentnick to the Maharaja – often
erroneously referred to as the ―Treaty” of 1831, it was in fact, more of a British
proclamation of good intention and its desire for good relations with the Maharaja.

 Main aim was to protect British from Russian Menace

 By this time the British empire had extended up to the Eastern banks of the Satluj, while
territory beyond the river right up to Afghanistan was firmly under the control of Ranjeet
Singh, a strong ruler in his own right. It was only after his death and subsequent intrigues
that the British conspired to extend their rule beyond Satluj to encompass his territories.

KUMHRAR OR KUMRAHAR

 Kumhrar or Kumrahar is the area of Patna where remains of the ancient city of
Pataliputra were excavated by the Archaeological Survey of India starting from 1913.

 Archaeological remains of the Mauryan period (322–185 BCE) have been discovered here,
this include the ruins of a hypostyle 80-pillared hall

 The excavation finding here dates back to 600 BCE, and marks the ancient capital of
Ajātasattu, Chandragupta and Ashoka, and collectively the relics range from four

https://t.me/prelimbits
58

continuous periods from 600 BCE to 600 CE.

AIHOLE INSCRIPTIONS [634 AD]

 Aihole in Karnataka was the Chalukyas' first capital.


 Many inscriptions were discovered at Aihole, but the inscription discovered at Meguti
Temple, also known as the Aihole inscription, witnessed many historical events during the
Chalukya period.
 The inscription is in Kannada script and written in Sanskrit.
 The defeat of Harshavardhana by Pulakeshin II and the victory of the Chalukyas over
the Pallavas are mentioned.
o Pulakeshin II defeated Harsha on the banks of Narmada in the winter of 618-619
CE. Pulakeshin entered into a treaty with Harsha, with the NARMADA RIVER
DESIGNATED AS THE BORDER BETWEEN THE CHALUKYA EMPIRE AND THAT OF
HARSHAVARDHANA.
 It also mentions the capital's relocation from Aihole to Badami.
 Bharavi, who is associated with the Pallavas of Kanchi along with Kalidasa is mentioned
in the Aihole Inscription.
 Ravikirti, the court poet of Pulakeshin II, who reigned from 610 to 642 CE, wrote them.
GOMATESHWARA TEMPLE [KARNATAKA]

 It is also known as Bahubali temple.


 The tallest monolithic statue (Of Gomateshwara son of lord
Rishabhanatha) in the world carved out of a single block of granite.
 It is a Jain temple that is 57 feet high.
 Bahubali statue is one of the largest free-standing statues in the
world depicting the prolonged meditation of Bahubali.
 The construction of the statue was commissioned by the Ganga
dynasty minister and commander,
 The Mahamastakabhisheka Mahotsava is an anointing [Abhisheka]
ceremony of the statue of Lord Bahubali. Only once in 12 years

49. Man passed from the food gathering stage to the food producing stage in the [1987]

A. Neolithic Age

B. Mesolithic Age

C. Chalcolithic Age

D. Palaeolithic Age

Ans. A

https://t.me/prelimbits
59

 The Neolithic period, or New Stone Age, is an Old World archaeological period and the final
division of the Stone Age.

 It saw the Neolithic Revolution, a wide-ranging set of developments that appear to have
arisen independently in several parts of the world.

 This "Neolithic package" included the introduction of farming, domestication of animals,


and change from a hunter-gatherer lifestyle to one of settlement.

Neolithic Age [8000 BC – 4000 BC]

EVOLUTION  Lived in huts, reared cattle, Developed Agriculture (wheat, barley,


cotton, rice etc.), Used earthenware (both handmade and wheel-made).

Agriculture  The people of the Neolithic age cultivated land and grew fruits and corn
like ragi and horse gram (kulati). They also domesticated cattle, sheep
and goats.

Housing and  The people of Neolithic age lived in rectangular or circular


Settled Life houses which were made of mud and reeds. Neolithic men also knew
how to make boats and could spin cotton, wool and weave cloth.
 The people of the Neolithic age led a more settled life and paved the
way for the beginning of civilization.

TOOLS  Sharper, symmetrical and polished stone tools for not only hunting but
agriculture also.

EXAMPLES  Daggers, digging sticks, celts, grinding stones, sickle, saw, sling-
(TOOLS) stones etc. (continuous rubbing to smoothen)

POTTERY  The pottery first appeared in the Neolithic Age. The pottery of the
period was classified under Grey Ware, Black-Burnished Ware, and mat-
impressed ware.

SITES  Koldihwa and Mahagara (lying south of Allahabad)


 Mehrgarh (Balochistan, Pakistan) The earliest Neolithic site, where
people lived in houses built of sun-dried bricks and cultivated crops like
cotton and wheat.
 Burzahom (Kashmir) [Pit dwelling house]
 Gufkral (Kashmir)
 Chirand (Bihar)
 Piklihal, Brahmagiri, Maski, Takkalakota, Hallur (Karnataka) –
 Belan Valley (which is located on the northern spurs of the Vindhyas
and middle part of Narmada valley) – All the three phases i.e.,
palaeolithic, Mesolithic and neolithic ages are found in sequence.

https://t.me/prelimbits
60

50. The oldest Hindu epic is

A. Mahabhashya

B. Ramayana

C. Ashtadhyayi

D. Mahabharata

Ans. B

 The Hindu epic Ramayana is the oldest and one of the largest epics in the world. It is
written by Maharishi Valmiki at about 7th centuries (700) BCE in India. It narrates the life of
Rama, prince of Ayodhya city in the kingdom of Kosala. [NO further PHD]

51. Sculptures of the Mathura School of Art, which flourished in the early centuries of the
Christian Era, are made out of [1988]

A. Marble

B. Slate stone

C. Granite

D. Red sandstone

Ans. D

MATHURA SCHOOL OF ART

 The Mathura School of Art flourished mostly during the reign of


Kushana ruler Kanishka in the first century AD. Mathura School grew up
on its own.

 Mathura was the traditional centre of output for this school, with Sarnath and
Kosambi also playing key roles.

 Spotted red sandstone was utilised in the construction of this school.


During the Gupta period, in the 6th or 7th century, this art achieved its
pinnacle.

 The representations of Buddha, Bodhisattvas, Vishnu, Shiva, Yakshas,


Yakshinis, Jinas, and others found in the Mathura school depict the city's
life and assimilation character as a result of Brahmanism, Jainism, and Buddhism's
religious fervour.

 Mathura art was notable for its Jina image and indigenous style of Buddha's image.

Salient features

 The Buddha image at Mathura is modeled on the lines of earlier Yaksha images whereas
in Gandhara it has Hellenistic features.
https://t.me/prelimbits
61

 It may be noted that the images of Vishnu and Shiva are represented by their ayudhas
(weapons).

 There is boldness in carving the large images, the volume of the images is projected out of
the picture plane, the faces are round and smiling, heaviness in the sculptural volume is
reduced to relaxed flesh.

 The garments of the body are clearly visible and they cover the left shoulder.

 Images of the Buddha, Yakshas, Yakshinis, Shaivite and Vaishnav ite deities and portrait
statues are profusely sculpted.

 In the Second Century CE, images in Mathura get SENSUAL, ROTUNDITY INCREASES,
THEY BECOME FLESHIER.

 In the Third Century CE, treatment of sculptural volume changes by reducing the extreme
fleshiness, movement in the posture is shown by increasing distance between the two
legs as well as by using bents in the body posture.

 Softness in the surface continues to get refined.

 Transparent quality in the robes of the Buddha images is evident

 Halo around the head is profusely decorated

52. The Lingayat movement owes its origin to [1988]

A. Vidyaranya

B. Purandharadasa

C. Appar

D. Basava / Basavanna

Ans. D

Basavanna / Basaveshwara (1134-1168)

 Was an Indian philosopher, social reformer and statesman during the reign of the
Kalyani Chalukya/Kalachuri dynasty who attempted to create a casteless society and
fought against caste and religious discrimination.
 He was born in Bagevadi (of undivided Bijapur district in Karnataka).
 The tradition of Lingayatism is known to have been founded by him.
 Philosophy
o His spiritual discipline was based on the :
 Arivu (true knowledge)
 Achara (right conduct),
 Anubhava (divine experience)
https://t.me/prelimbits
62

o This path advocates a holistic approach of Lingangayoga (union with the divine).
This comprehensive discipline encompasses bhakti (devotion), jnana (knowledge),
and kriye (action) in a well balanced manner.
 He went to Kalyana (now called Basavakalyan) probably in the year A.D. 1154. His
achievements in the short span of twelve or thirteen years of his stay at Kalyana are striking.
 He established the Anubhava Mantapa, which was a common forum for all to discuss
the prevailing problems of socio, economic and political strata including religious and
spiritual principles along with personal problems.
o Thus, it was the first and foremost Parliament of India, where Sharanas sat
together and discussed the socialistic principles of a Democratic set up.
 He gave two more very important socio-economic principles.
o Kayaka (Divine work): According to this, every individual of the society should
take up the job of his choice and perform it with all sincerity.
o Dasoha (Equal distribution):
 There must be an equal income for equal work.
 The worker (Kayakajeevi) may lead his day-today life by his hard earned
income. But he should not preserve the money or property for tomorrow.
He must utilise the surplus money for the society and poors.
 Vachana Reformist Movement:
o The main aim of the Vachana (poetry) movement, led by Basaveshwara in 12th
century, was welfare of all.
o It attempted to address class, caste and to some extent gender issues in a given
societal milieu.
New Anubhava Mantapa [Karnataka]
 It will showcase the 12th Century Anubhava Mantapa (often referred to as the “first
Parliament of the world”) established by Basaveshwara in Basavakalyan where
philosophers and social reformers held debates.
 The building will adopt the Kalyani Chalukya style of architecture.
o Later Chalukyas, popularly known as the Chalukyas of Kalyan or Kalyani
Chalukyas form an integral part of ancient Karnataka history of early medieval period

53. The earliest evidence of a Jewish community settling in India is provided by a charter
of a

A. Chola king

B. Pandya king

C. Chera king

https://t.me/prelimbits
63

D. Viayanagara king

Ans. D *

Not able to find any source related to this Q. though many sites on internet is claiming D Ans.

54. Match the columns: [1988]

List-I List-I

A. Rig Veda 1. Musical hymns

B. Yajur Veda 2. Hymns and rituals

C. Sama Veda 3. Charms and spells

D. Atharva Veda 4. Hymns and prayers

Select the Correct answer from the codes given below :

A B C D

a) 3 4 1 2

b) 2 1 4 3

c) 4 2 1 3

d) 3 4 2 1

Ans. C

VEDIC LITERATURE

Rig Veda

 The Rig Veda is a collection of 1,028 hymns, divided into 10 Mandals (books).
 They are the earliest compositions and hence, they depict the life of the Early Vedic
people in India.
 Recently, the Rig Veda has been included by the UNESCO in the list of literature signifying
World Human Heritage.
o Earliest Mandals, i.e., II to VII are called family books as they are ascribed to
particular families of seers/rishis.
o Mandal VIII – Mostly relate to Kanva‟s family.
o Mandal IX – Compilation of Soma hymns.
o Mandal I and X – Are later additions and contains the Purusashukta which
explains the four varnas. They also deal with philosophical questions and also talk
about various virtues including a charity in the society
o The priests related to the Rig Veda are Kotri or Motri, and the Upveda of the Rig

https://t.me/prelimbits
64

Veda is the Ayurveda.


o Only surviving recension of the Rig Veda is the Shakala shakha.
 Rigvedic books 1 and 10 deal with philosophical questions and also talk about various
virtues including a charity in the society
 The meters used to form hymns are Gayatri, Anushtubh, Trishtubh and Jagati
(Trishtubh and Gayatri are most important)
 The Rigvedic hymns are dedicated to various deities, chief of whom are Indra, a heroic
god praised for having slain his enemy.
 Upaveds : Ayurveda (Medicine), associated with the Ṛig Veda
 Rigveda Brahmanas : Aitareya-brahmana and the Kaushitaki- (or Sankhayana-)
brahmana
 Aranyakas of the Rigveda:
o Aitareya Aranyaka
o Kaushitaki/ Shankhayana Aranyaka
 Upanishads of the Rigveda :
o Aitareya Upanishad,
o Kaushitaki Upanishad
Sama Veda

 The Sama Veda is the collection of verses mostly taken from the Rig Veda, but arranged
in a poetic form to facilitate singing
 Samaveda is categorised into two parts – Part-I includes melodies called Gana & Part-II
includes three verses book called Archika.
 It is a collection of 1,810 melodies, and also contains the famous Dhrupada raga, later
sung by Tansen.
 The Upveda of the Sama Veda is the Gandharva Veda
 There are two Upanishads embedded in Samaveda
o Chandogya Upanishad and
o Kena Upanishad
 Aranyakas of the Samaveda:
o Talavakara or Jaiminiya-Upanshad Aranyaka
o Chandogya- Aranyaka
 Brahmana
o Panchavimsha / Tandya Brahmana
o Samavidhana Brahmana
o Daivata Brahmana
o Samhitopanishad Brahmana
o Arsheya Brahmana
o Vamsha Brahmana
o Jaiminiya Brahmana

https://t.me/prelimbits
65

 Recensions (Shakhas) of the Sama Veda are


o Kauthuma,
o Jaiminiya (Talavakara), and
o Ranayaniya.
Yajur Veda

 The Yajur Veda deals with the procedure for the performance of sacrifices.
 The texts are further divided into:
o Shukla Yajur Veda/White Yajur Veda/Vajasaneya (contains only the Mantras). It
contains the Madhyandina and Kanva recensions. Vajasaneyi Samhita is the
Samhita in the Shukla Yajurveda
o Krishna Yajur Veda/Black Yajur Veda (contains both mantras and prose
explanations/ commentary). It contains the Kathaka, Maitrayani, Taittiriya, and
Kapishthala recensions.
 The Upveda of the Yajur Veda is the Dhanur Veda.
 Yajurveda consists of various Upanishads
o Brihadaranyaka Upanishad, the Isha Upanishad, the Taittiriya Upanishad, the
Katha Upanishad, the Shvetashvatara Upanishad, the Maitri Upanishad
 Aranyaka of Shukla Yajurveda: Brihadaranyaka
 Brahmana
o Shatapatha Brahmana
o Taittiriya Brahmana
Atharva Veda

 The Atharva Veda is a collection of magic spells and charms to ward off the evil spirits
and diseases.
 It is the last Veda and is considered a non-Aryan work divided into 20 kandas (books),
with 711 hymns. Unlike Samaveda where hymns are borrowed from Rigveda, hymns of
Atharvaveda are unique except a few
 Brahmana : Gopatha Brahmana.
 Recensions : Shaunaka and Paippalada
 Upveda : Shilpa Veda
 Upanishads of the Atharvaveda:
o Mundaka Upanishad,
o Mandukya Upanishad,
o Prashna Upanishad.
 No Aranyaka which belongs to the Atharvaveda.

55. Match the columns [1988]

List I List II
https://t.me/prelimbits
66

A. Amaravati 1. Hoysalas

B. Aihole 2. Cholas

C. Thanjavur / Tanjore 3. Satavahanas

D. Halebid 4. Chalukyas

5. Pandyas

Select the correct answer from the codes given below:

A B C D

a) 3 4 1 2

b) 2 1 4 3

c) 4 2 1 3

d) 3 4 2 1

Ans. D

A OPTION : SATAVAHANAS

Background : Maurya rule was succeeded by the Sungas and the Kanvas in the north. However, in
Deccan and in central India, the Satavahanas succeeded the Mauryas
 Satavahanas are considered identical with the Andhras of the Puranas (Matsya Purana).
 But both Purna & Satavahana inscriptions not mention anyting about them. (only refer
to Andhras).
 According to some Puranas, the Andhras ruled for 300 years with their capital at
Pratishthana (modern Paithan) on the Godavari in Aurangabad district.
o 2nd Capital Dhanyakataka (Amaravathi).
 This is also the period designated to the Satavahana Dynasty. The earliest Satavahana
inscriptions belong to the 1st century BCE, when they defeated the Kanvas and
established their rule over parts of central India.
 They ruled over parts of Andhra, Maharashtra, Karnataka and Madhya Pradesh.

 They used matronyms such as Gautamiputra and Vashishthiputra, although they were not
matrilineal or matriarchal in any sense.

 The Satavahanas assumed the title of Dakshinapathapati (Lord of the Dakshinapatha). The
Satavahanas are also known in history for starting the practice of giving royal grants of
land to Brahmanas and Buddhist monks, including those associated with tax exemptions.

 Land gifted to the Brahmanas was not to be entered or disturbed by the royal troops, was
not to be dug for salt, was free from the control of state officials, and was to enjoy all
https://t.me/prelimbits
67

sorts of pariharas (immunities). They also promoted Buddhism by granting land to the
monks.

B OPTION : CHALUKYAS OF BADAMI

 In northern Maharashtra and Vidarbha (Berar), the Satavahanas were succeeded by the
Vakatakas. The Vakatakas were followed by the Chalukyas of Badami
 The Chalukyas were a maritime power and they set up their kingdom in western Deccan
with capital at Vatapi (modern Badami in Karnataka).
 Along with the Chalukyas of Badami (who are primarily called the Western Chalukyas),
there were TWO OTHER BRANCHES :
o Eastern Chalukyas of Vengi and the
o Chalukyas of Lata.
 Founder Chalukyas of Badami king of this dynasty was Pulakeshin I, who established the
independent power of this dynasty.
Pulkeshin II (c. 610−642 CE)

 The most powerful and famous king of this dynasty, who achieved many military
successes.
 His eulogy was written by his court poet Ravikirti in the Aihole Inscription, which
Describes →
o His victories against the Kadambas of Banavasi, Alupas and Gangas of Mysore.
o The defeat of Harshavardhana by Pulakeshin II and the victory of the Chalukyas
over the Pallavas are mentioned
 Pulakeshin II defeated Harsha on the banks of Narmada in the winter of 618-
619 CE. Pulakeshin entered into a treaty with Harsha, with the Narmada
River designated as the border between the Chalukya Empire and that of
Harshavardhana.
C OPTION : CHOLAS (850 – 1279 AD)

https://t.me/prelimbits
68

 The Cholas, famous in Indian history for their local self-government


 Known as Imperial Cholas of Tanjore. [Medieval Cholas]
 The founder of Chola dynasty was Vijayalaya who was at first a feudatory of Pallavas.
 The greatest kings of Cholas were Rajaraja & his son Rajendra I.
Different Cholas
 Early Cholas of Sangam Age who were also called as the karikala Cholas. After them the
Chola dynasty became distinct.
 In 850 AD, Medieval Cholas rose under Vijayalaya reign during Pandya and pallava wars.
Thanjavur (Tanjore) was the capital of Cholas.
 Later Chola dynasty come under Kulothunga I ruler.

D OPTION : HOYSALA EMPIRE

 The Hoysala Empire ruled most of what is now Karnataka, India between the 10th and
the 14th centuries. The capital of the Hoysalas was initially located at Belur but was later
moved to Halebidu.
 The Hoysala rulers were originally from Malenadu, an elevated region
in the Western Ghats. In the 12th century, taking advantage of the
internecine warfare between the Western Chalukya Empire and
Kalachuris of Kalyani, they annexed areas of present-day Karnataka
and the fertile areas north of the Kaveri delta in present-day Tamil
Nadu. By the 13th century, they governed most of Karnataka, minor
parts of Tamil Nadu and parts of western Andhra Pradesh and
Telangana in the Deccan Plateau.
 The Hoysala era was an important period in the development of art,
architecture, and religion in South India. The empire is remembered
today primarily for HOYSALA ARCHITECTURE. Over a hundred
surviving temples are scattered across Karnataka.
 Well known temples "which exhibit an amazing display of sculptural
exuberance" include the Chennakeshava Temple (Vishnu), Belur, the Hoysaleswara
Temple (Shiva), Halebidu, and the Chennakesava Temple or Keshava Temple

https://t.me/prelimbits
69

(Krishna) , Somanathapura.
 The Hoysala rulers also patronised the fine arts, encouraging literature to flourish in
Kannada and Sanskrit.

56.

Assertion (A): In the ancient period of Indian history, urbanisation was followed by
pastoral life.

Reason (R): The Harappan Culture was an urban one.

A. Both A and R are true and R is the correct explanation of A

B. Both A and R are true but R is not a correct explanation of A

C. A is true but R is false

D. A is false but R is true

Ans. B

Assertion (A): Late Harappan [1900 – 1300 BCE] → Toward rural Life

 Around 1900 BCE signs of a gradual decline began to emerge, and by around 1700 BCE
most of the cities had been abandoned.

 Recent examination of human skeletons from the site of Harappa has demonstrated that
the end of the Indus civilisation saw an increase in inter-personal violence and in
infectious diseases like leprosy and tuberculosis.

 According to historian Upinder Singh, "the general picture presented by the late Harappan
phase is one of a breakdown of urban networks and an expansion of rural ones."

 During the period of approximately 1900 to 1700 BCE, multiple regional cultures
emerged within the area of the Indus civilisation.

o The Cemetery H culture was in Punjab, Haryana, and Western Uttar Pradesh, the

o Jhukar culture was in Sindh, and the

o Rangpur culture (characterised by Lustrous Red Ware pottery) was in Gujarat.

 Other sites associated with the Late phase of the Harappan culture are Pirak in
Balochistan, Pakistan, and Daimabad in Maharashtra, India.

 The largest Late Harappan sites are Kudwala in Cholistan, Bet Dwarka in Gujarat, and
Daimabad in Maharashtra, which can be considered as urban, but they are smaller and few
in number compared with the Mature Harappan cities.

https://t.me/prelimbits
70

 Bet Dwarka was fortified and continued to have contacts with the Persian Gulf region, but
there was a general decrease of long-distance trade.

 On the other hand, the period also saw a diversification of the agricultural base, with a
diversity of crops and the advent of Double-Cropping, as well as a shift of rural settlement
towards the east and the south. [According to Upinder singh]

 The pottery of the Late Harappan period is described as "showing some continuity with
mature Harappan pottery traditions," but also distinctive differences.

 Many sites continued to be occupied for some centuries, although their urban features
declined and disappeared.

 Formerly typical artifacts such as stone weights and female figurines became rare.

 There are some circular stamp seals with geometric designs, but lacking the Indus
script which characterised the mature phase of the civilisation. Script is rare and confined
to potsherd inscriptions

 There was also a decline in long-distance trade, although the local cultures show new
innovations in faience and glass making, and carving of stone beads

 Urban amenities such as drains and the public bath were no longer maintained, and
newer buildings were "poorly constructed".

https://t.me/prelimbits
71

 Stone sculptures were deliberately vandalised, valuables were sometimes concealed in


hoards, suggesting unrest, and the corpses of animals and even humans were left unburied
in the streets and in abandoned buildings.

 During the later half of the 2nd millennium BCE, most of the post-urban Late
Harappan settlements were abandoned altogether.

 Subsequent material culture was typically characterised by temporary occupation, "the


campsites of a population which was nomadic and mainly pastoralist" and which used
"crude handmade pottery."

 However, there is greater continuity and overlap between Late Harappan and subsequent
cultural phases at sites in Punjab, Haryana, and western Uttar Pradesh, primarily small rural
settlements.

Reason (R) : Mature Harappan Culture → Urban Life

 Due to good monsoon Flood-Supported Farming led to large agricultural surpluses,


which in turn supported the development of cities.

 The IVC residents did not develop irrigation capabilities, relying mainly on the seasonal
monsoons leading to summer floods

 According to J.G. Shaffer and D.A. Lichtenstein, the Mature Harappan civilisation was "a
fusion of the Bagor, Hakra, and Kot Diji traditions or 'ethnic groups' in the Ghaggar-
Hakra valley on the borders of India and Pakistan".

 By 2600 BCE, the Early Harappan communities turned into large urban centres. Such
urban centres include Harappa, Ganeriwala, Mohenjo-daro in modern-day Pakistan, and
Dholavira, Kalibangan, Rakhigarhi, Rupar, and Lothal in modern-day India.[

Cities

 A sophisticated and technologically advanced URBAN CULTURE is evident in the IVC,


making them the FIRST URBAN CENTRE in the region.

 The quality of municipal town planning suggests the knowledge of urban planning and
efficient municipal governments which placed a high priority on hygiene, or,
alternatively, accessibility to the means of religious ritual.

 As seen in Harappa, Mohenjo-daro and the Rakhigarhi, this urban plan included the world's
first known urban sanitation systems.

 Within the city, individual homes or groups of homes obtained water from wells. From a
room that appears to have been set aside for bathing, waste water was directed to
covered drains, which lined the major streets.

 Houses opened only to inner courtyards and smaller lanes.

 The ancient Indus systems of sewerage and drainage that were developed and used in

https://t.me/prelimbits
72

cities throughout the Indus region were far more advanced than any found in
contemporary urban sites in the Middle East.

 The advanced architecture of the Harappans is shown by their impressive dockyards,


granaries, warehouses, brick platforms, and protective walls.

 The massive walls of Indus cities most likely protected the Harappans from floods and
may have dissuaded military conflicts.

 The purpose of the CITADEL remains debated. In sharp contrast to this civilisation's
contemporaries, Mesopotamia and ancient Egypt, no large monumental structures
were built. There is NO conclusive evidence of palaces or temples.
 Some structures are thought to have been granaries. Found at one city is an enormous
well-built bath (the "Great Bath"), which may have been a public bath.

 Although the citadels were walled, it is far from clear that these structures were defensive.

 Most city dwellers appear to have been traders or artisans, who lived with others
pursuing the same occupation in well-defined neighbourhoods.

 Materials from distant regions were used in the cities for constructing seals, beads and
other objects.

 Among the artefacts discovered were beautiful glazed faïence beads. Steatite seals have
images of animals, people (perhaps gods), and other types of inscriptions, including the yet
un-deciphered writing system of the Indus Valley Civilisation. Some of the seals were
used to stamp clay on trade goods.
 Although some houses were larger than others, Indus civilisation cities were remarkable for
their apparent, if relative, egalitarianism. All the houses had access to water and
drainage facilities. This gives the impression of a society with relatively low wealth
concentration.

57. The original name of Mahabharata was [1988]

A. Brihit Katha

B. Sahastra Samhita

C. Jaya Samhita

D. Rajatarangini

Ans. A

 Jaya Samhita is the original name of Mahabharata. Mahabharata was written down by Lord
Ganesha. Mahabharata was narrated by Maharishi Ved Vyasa. [NO MORE PHD]

https://t.me/prelimbits
73

58. Jain is differed from Buddhism in which of the following aspects? [1989]

A. Faith in Karma

B. Doctrine of Ahimsa

C. Belief in fasting

D. Rejection of Yagna

Ans. C

FAITH IN KARMA

 Buddhist tradition : karma refers to action driven by intention (cetanā) which leads to
future consequences. Those intentions are considered to be the determining factor in
the kind of rebirth in samsara, the cycle of rebirth.

 Jains tradition : Jains believe that karma is a physical substance that is everywhere in
the universe. Karma particles are attracted to the soul by the actions of that soul. Karma
particles are attracted when we do, think, or say things, when we kill something, when we
lie, when we steal and so on.

o The theory of karma is one of the peculiar characteristics of Jainism, which is


elaborately discussed. It is nothing but the the law of moral causation. Nothing
happens without a cause. Each individual is responsible for his or her own actions.

o These actions are the cause that produces results. This is the eternal and universal
law of Karmas. [https://www.jaina.org/page/Theory_Of_Karma]

BELIEF IN FASTING : JAIN → UNIQUE CONCEPT OF NON-VIOLENCE & FASTING TILL DEATH

 The five vows are common to both the monks and lay followers. But The monks were to
observe the vows more rigorously than the lay followers.
 As Jainism placed great emphasis on non-violence, strict observers of the faith wear a
muslin cloth around their mouth and nose so that they would not inhale small insects
even by mistake.
 To avoid trampling on ants and other insects, Jain monks used feathers to sweep the
path before walking.
 Jains could not practice agriculture or other crafts that involve killing or injury to living
organisms. Hence they took to trading and money-lending and excelled in it. As a result,
they were closely associated with urbanisation.

Jainism is an egalitarian religion. It does not sanction any inequality based on birth. It is one‘s
deeds that determine one‘s status in society and not birth. Jainism believes that “by one‟s action
one becomes a Brahmin, a Kshatriya, or a Vaishya, or a Sudra.” Pride based on birth is
considered a sin. Women were admitted into the monastic order. However, as a woman one
cannot attain salvation. By accumulating merit by good deeds, a woman could be reborn

https://t.me/prelimbits
74

as a man and then strive to attain salvation

JAINISM : SALLEKHANA (SANTHARA) → FAST TILL DEATH

 It is the religious practice of voluntarily fasting to death by gradually reducing the intake
of food and liquids.
 It is viewed in Jainism as the thinning of human passions and the body, and another
means of destroying rebirth-influencing karma by withdrawing all physical and mental
activities. According to Jainism, Karma is the bane of the soul. Karma not only
encompasses the causality of transmigration but is also conceived as an extremely subtle
matter which infiltrates the soul, obscuring its natural, transparent, pure qualities. Karma is
thought of as a kind of pollution that taints the soul with various colors. Based on its
karma, a soul undergoes transmigration and reincarnation in various states of
existence like heavens or hells or as humans or animals.
 It is not considered as a suicide by Jain scholars because it is not an act of passion, nor
does it deploy poisons or weapons.
 Sallekhana is a vow available to both Jain ascetics and householders. (now uncommon
event)
Fasting in Buddhism is not obligatory for its adherents. However, if Buddhists wish and intend to
observe the uposatha (fasting) it is permissible.

59. Which of the following is not a feature of Gandhara School of Art? [1989]

A. It was patronised by Sakas and Kushanas

B. It has Indian and Greek influence

C. It resulted in the origination of Hinayanism

D. It portrayed human figures exactly

Ans. C

GANDHARA SCHOOL OF ART

 The Gandhara school of Art arose in modern-day Peshawar and Afghanistan on Punjab's
western boundaries.

 The Greek invaders brought the traditions of Greek and Roman sculptors with them,
which affected the region's native traditions ==> Greco-Indian School of Art

 Between 50 B.C. and 500 A.D., the Gandhara School flourished in two periods. While the
former school's sculptures were made of bluish-grey sandstone, the latter school's were
made of mud and plaster.

 The Buddha and Bodhisattvas iconography was based on the Greco-Roman pantheon
and resembled Apollo's.

 Major Centres :
https://t.me/prelimbits
75

o Jalalabad, Hadda, Bamaran, Begram & Taxila were the main centers where art pieces
of Gandhara School have been found.

 Major Features :

o The Gandhara sculptures have been discovered in the Taxila ruins as well as other
ancient sites in Afghanistan and Pakistan.

o They generally consist of Buddha images and relief sculptures depicting scenes
from Buddhist literature.

o Several Bodhisattva figures were cut out of the rock. The first preaching in the
deer park and the Buddha's death is depicted in a Gandhara figure.

o The predominant focus of this type of painting was Lord Buddha


and Bodhisattvas, as it was intimately tied with Mahayana Buddhism.

o The Bamiyan Buddha sculptures are an example of Gandhara style art.

o It thrived primarily in Afghanistan and present-day North-Western India.


Taxila, Peshawar, Begram, and Bamiyan were among the most prominent sites.

o From the first century BCE through the fourth century CE, the Gandhara School of
art flourished.

 Themes

o The Buddhist themes were largely represented in Gandhara art. The Buddha's
mother resembles an Anthenian matron.

o A Buddhist scene was created with an Apollo-like face. The image of Athena of
Rome in Lahore is perhaps one of the most beautiful Gandhara sculptures depicting
a western topic.

o It was created in order to express Buddhist ideas and practices.

o The specimens do not include any Greek art motif, with a few exceptions. The hand
of the Gandhara artist was Greek, but his heart was Indian.

 Patrons

o Gandhara School was fostered by the Shakas and the Kushanas. The first
sculptural portrayals of the Buddha in human form are attributed to
the Gandhara school. [The first image of the Buddha-was carved out during
the reign of famous Kushan king Kanishka I.]

 Features of sculpture

o The human body is sculpted in a realistic manner in these sculptures, with


considerable attention to realism and physical details, particularly in the depiction
of muscles, mustaches, and other facial features.

https://t.me/prelimbits
76

o The portrayal of the broad bold fold lines is a unique trait that sets it apart from what
has been discovered elsewhere in India.

 The Various Mudras of Buddha in Gandhara Art

o In all the Buddha depicted in the Gandhara Art is shown making four types of hand
gestures and this is a remarkable feature in this art.

 Greek influences on Gandhara School of art

o Greek god as protector: In many images of Buddha in Gandhara; he is seen under


the protection of Greek god Hercules.

o Vajrapani: Vajrapani found in the right hand of future Buddha is told as a


transformed symbol of Hercules who is seen as the protector of Buddha.

o Greek architectural influence: Some images of Buddha in Gandhara are presented


in the Greek architectural environment bearing the affinity of Corinthian.

o Artistic beauty: The Apollo-like face of Buddha; natural realism; wavy hair as
seen in images of Buddha in Gandhara resembles Hellenistic tradition.

o Intellectual affinity: The hello and bun of Gandhara Buddha signify intellectual
imbibitions of Buddha from Greek.

o Despite the fact that Gandharan sculpture's iconography was predominantly Indian, it
also included elements and methods from Classical Roman art. Vine scrolls,
cherubs with garlands, tritons, and centaurs are examples of Classical Roman art
found in Gandharan sculptures.

o Additionally, the Gandharan artists drew on the Roman religion's anthropomorphic


traditions.

o Gandharan art depictions of Buddha are evocative of statues of a youthful Apollo.

o The drapery on Buddha's garments was also similar to that on Roman imperial
monuments.

https://t.me/prelimbits
77

60. The Prakrit text "Gathasaptasati" is attributed to the Satavahana king [1989]

A. Vasishtiputra Pulumavi

B. Hala

C. Gautamiputra Satkarni

D. Amaru

Ans. B

Hala

 He was a Satavahana king [17th ruler] who ruled in present-day Deccan region.
 The Maharashtri Prakrit poem by Kouhala, Lilavai (c. 800 CE) describes his romance with
a princess Lilavai of Simhaladvipa (identified with present-day Sri Lanka).
 Hala is famous for compiling an anthology of Maharashtri Prakrit poems known as the
Gaha Sattasai [love poems]
 One of the more remarkable things about the Gathasaptashati is that it is written from the
perspective almost wholly of women. [Statement Source : The hindu]

61. “Mitakshara” is associated with [1989]

A. Yajnawalkya

B. Poetics

C. Drama

D. Grammar

Ans. A

Mitakshara

 The Mitakshara is a vivṛti (legal commentary) on the Yajnavalkya Smriti best known for
its theory of "inheritance by birth."

 It was written by Vijnanesvara, a scholar in the Western Chalukya [Kalyani] court in the
late eleventh and early twelfth century during the reign of Vikramaditya VI

 Along with the Dayabhaga, it was considered one of the main authorities on Hindu Law
from the time the British began administering laws in India.

 The entire Mitakshara, along with the text of the Yajnavalkya Smriti, is approximately 492
closely printed pages.

62. Mattavilasa is written by [1989]

A. Harsha

https://t.me/prelimbits
78

B. Rajashekhara

C. Somadeva

D. Mahendravarman I

Ans. D

 Mattavilasa Prahasana/ , (A Farce of Drunken Sport) is a short one-act


Sanskrit play.

 It is one of the two great one act plays written by Pallava King Mahendravarman I (571–
630CE) in the beginning of the seventh century in Tamil Nadu.

 Mattavilasa Prahasana is a satire that pokes fun at the peculiar aspects of the heretic
Kapalika and Pasupata Saivite sects, Buddhists and Jainism.

 The setting of the play is Kanchipuram, the capital city of the Pallava kingdom in the
seventh century.

 The play revolves around the drunken antics of a Kapalika mendicant, Satyasoma, his
woman, Devasoma, and the loss and recovery of their skull-bowl.

 The cast of characters consists of Kapali or Satysoma, an unorthodox Saivite mendicant,


Devasoma, Satysoma‘s female partner, a Buddhist Monk,
whose name is Nagasena, Pasupata, a member of another
unorthodox Saivite order and a Madman.

 The act describes a dispute between a drunken Kapali and


the Buddhist monk. The inebriated Kapali suspects the
Buddhist monk of stealing his begging bowl made from a
skull, but after a drawn-out argument it is found to have been
taken away by a dog.

Mahendravarman I (600–630 CE)

 Was a Pallava emperor who ruled the Southern portion of


present-day Andhra region and Northern regions of what
forms present-day Tamil Nadu in India in the early 7th
century.

 He was a scholar, painter, architect, musician. He was the


son of Simhavishnu, who defeated the Kalabhras and re-
established the Pallava kingdom.

 During his reign, the Chalukya king Pulakeshin II attacked


the Pallava kingdom. The Pallavas fought a series of wars in the northern Vengi region,
before Mahendravarma decimated his chief enemies at Pullalur (according to Pallava grants
at Kuram, Kasakudi and Tadantottam).

 Although Mahendravarma saved his capital, he lost the northern provinces to Pulakeshin.
https://t.me/prelimbits
79

 Tamil literature flourished under his rule, with the rise in popularity of Tevaram written
by Appar and Sambandhar.

 During his period "Bhagwatajjukam", another satire (prahasan), was written by Bodhayan.
King Mahendravarman mentioned this on a stone inscription in Mamandur along with
his own Mattavilas Prahasan.

 Mahendravarma was succeeded to the throne by his more famous son Narasimhavarman
I in 630 CE. who defeated Pulakeshin II of Chalukya dynasty and ransacked the Chalukyan
capital city of Vatapi (also known as Badami).

63. Match the following : [1989]

List I List II

A. Chaitya 1. Amaravati

B. Stupa 2. Besnagar

C. Pillar 3. Karle

D. Brick Temples 4. Bhitragaon

Select the correct answer from the codes given below:

A B C D

a) 3 4 1 2

b) 2 1 4 3

c) 4 2 1 3

d) 3 1 2 4

Ans. D

KARLA CAVES

 Ancient Buddhist Indian rock-cut caves at Karli near Lonavala, Maharashtra.


 Other caves in the area are Bhaja Caves, Patan Buddhist Cave, Bedse Caves and Nasik Caves.
 The shrines were developed over the period – from the 2nd century BCE to the 5th
century CE.
 The oldest of the cave shrines is believed to date back to 160 BCE, having arisen near a
major ancient trade route, running eastward from the Arabian Sea into the Deccan.

 The group at Karla is one of the older and smaller of the many rock-cut Buddhist sites in
Maharashtra, but is one of the best-known because of the famous "Grand Chaitya" (Cave
8), which is "the largest and most completely preserved" chaitya hall of the period, as well
https://t.me/prelimbits
80

as containing unusual quantities of fine sculpture, much of it on a large scale.

 Many traders and Satavahana rulers made grants for construction of these caves.

AMARAVATI STUPA

 The Amarāvati Stupa, is a ruined Buddhist stūpa at the village of Amaravathi, Palnadu
district, Andhra Pradesh, India, probably built in phases between the
third century BCE and about 250 CE.

 It was enlarged and new sculptures replaced the earlier ones, beginning
in about 50 CE.

 The site is under the protection of the Archaeological Survey of India,


and includes the stūpa itself and the Archaeological Museum.

 Art of Amaravati as one of the three major styles or schools of


ancient Indian art, the other two being the Mathura style, and the
Gandharan style.

 Largely because of the maritime trading links of the East Indian coast,
the Amaravati school or Andhran style of sculpture, seen in a number of sites in the region,
had great influence on art in South India, Sri Lanka and South -East Asia.

 The stupa, or mahāchetiya, was possibly founded in the third century BCE in the time of
Asoka but there is no decisive evidence for the date of foundation

BESNAGAR PILLAR INSCRIPTION

 The Heliodorus pillar is a stone column that was erected around 113 BCE in central India
in Besnagar (near Vidisha, Madhya Pradesh).
 The pillar was called the Garuda-standard by Heliodorus, referring to the deity Garuda.
 The pillar is commonly named after Heliodorus, who was an ambassador of the Indo-
Greek king Antialcidas from Taxila, and was sent to the Indian ruler Bhagabhadra.
 A dedication written in Brahmi script was inscribed on the pillar, venerating Vāsudeva, the
Deva deva the "God of Gods" and the Supreme Deity.

https://t.me/prelimbits
81

 The pillar also glorifies the Indian ruler as "Bhagabhadra the savior".
 The pillar is a stambha which symbolizes joining earth, space and heaven, and is thought
to connote the "cosmic axis" and express the cosmic totality of the Deity
BHITARGAON BRICK TAMPLE

 Bhitargaon is a town, near city of Kanpur in Kanpur district, Uttar Pradesh, India, known
for its ancient Hindu temple, the largest Indian brick temple to survive
from the time of the Gupta Empire.

 Despite being heavily restored, a number of original features remain. It is


dated to the late 5th century.

 it is the oldest remaining brick/terracotta Hindu shrine with a roof and a


high shikhara, though its upper chamber did sustain some damage in the
18th century.

 The walls are decorated with terracotta panels depicting aquatic monsters, Shiva and
Vishnu etc.

64. Match the following: [1989]

List I List II

A. Charak 1. Samkhya

B. Nagarjuna 2. Sanskrit author

C. Kapila 3. Ayurveda

D. Gorakhnath 4. Rasaratnakara

Select the correct answer from the codes given below:

A B C D

a) 3 4 1 2

b) 2 1 4 3

c) 4 2 1 3

d) 3 1 2 4

Ans. A

CHARAKA

 Charaka was one of the principal contributors to Ayurveda, a system of medicine and
lifestyle developed in Ancient India → FOUNDER AND FATHER OF AYURVEDA

https://t.me/prelimbits
82

 Historical records suggest that he lived in the period from 100 BC to not beyond 150- 200
BC. The exact date, however, remains to be unknown till date. Charaka has been identified
as a native of Kashmir.
 He is known as an editor of the medical treatise entitled Charaka Samhita
 Along with the Sushruta Samhita, it is one of the two foundational texts of this field
that have survived from ancient India. It is one of the three
works that constitute the Brhat Trayi (The Great Triad)
o Third in the triad is Ashtanga Hridayam Samhita by
Vagbhata
 It describes ancient theories on human body, etiology,
symptomology and therapeutics for a wide range of diseases
 The Charaka Samhita also includes sections on the importance
of diet, hygiene, prevention, medical education, and the
teamwork of a physician, nurse and patient necessary for recovery to health
 According to Charaka Samhita, anything and everything in the universe is composed of
Vata, pitta, and Kapha. They are considered the force of nature in Ayurveda that helps us
understand the universe better.
Nagarjuna (150-250 CE)

 Nagarjuna: He is often termed an Indian Einstein who proposed the Theory of Relativity in
his time in the form of a Sutra, the Prajna Parimata Sutra.
 He was also a great exponent of the Mahayana doctrine and propounded the
Madhyamaka school of Mahayana Buddhism (also known as Sunyavada School) which
focuses on sunyata or emptiness
 In some traditions he is known as an Indian metallurgist
 Nagarjun was a learned Buddhist of Nalanda University with profound knowledge in
chemistry. He wrote books like ‗Rasaratnakar‟ and ‗Arogyamanjari‟ on chemistry and
medicine respectively. Rasaratnakara describes methods for the extraction of metals like
gold, silver, tin and copper. https://www.myindiamyglory.com/2019/12/18/nagarjuna-
wizard-in-alchemy-metallurgy-introduced-gold-shine-mechanism/
Samkhya (Kapila)

 Samkhya is the oldest of the orthodox philosophical systems, and it postulates that
everything in reality stems from purusha (self, soul or mind) and prakriti (matter,
creative agency, energy).
 Purush cannot be modified or changed while prakriti brings change in all objects.
 Emphasizes the attainment of knowledge of self through meditation and concentration.
 School does not believe in the existence of God.
At the same point, it does not describe what happens after moksha and does not

https://t.me/prelimbits
83

mention anything about Ishwara or God, because after liberation there is no essential
distinction of individual and universal puruṣa.
o So what happens after Moksha is the irrelevant thing for this school.
o But at the same time, though godless, the Sankhya believes in the doctrine of
Karma and of transmigration of souls.
 This philosophy adversely affected the Tantra sadhana a lot.
 Sankhya accepts the theory of rebirth or transmigration of the soul
Gorakhnath

 Gorakhnath (also known as Goraksanath, c. early 11th century) was a Hindu yogi, saint who
was the influential founder of the Nath Hindu monastic movement in India

 He is considered one of the two notable disciples of Matsyendranath.

 His followers, found all over India, are called yogis, Gorakhnathi, Darshani or Kanphata.

65.

Assertion (A): The form of government in Rig vedic period was monarchy

Reason (R): Priest enjoyed both social and political status and influenced administration

A. Both A and R are true and R is the correct explanation of A

B. Both A and R are true but R is not the correct explanation of A

C. A is true but R is false

D. A is false but R is true

Ans. B

VEDIC PERIOD : LATER VEDIC

Political System

 Janas evolved to become Janapadas; Hastinapur and Indraprastha were capitals of


Kuru „janapada‟.
 Frequent battles among these ‗janapadas‘ were fought for territory.

 Authority of the „rajana‟ became more evident and a support staff called ‟ratnin‟; they
were 12 jewels of the king, worked for ‗rajana‘.
 Chiefship became hereditary.
 Still, there was no standing army.
 Rajana started various sacrifices like ‗Rajsuya‟ (coronation), ‗Ashwamedh‟ (to become
ruler of all directions i.e. ‗Chakravarti‘) and ‗Vajpeya‟ (Chariot race)
o Rajasuya Sacrifice: Supposed to confer supreme power on him. It is a king‗s

https://t.me/prelimbits
84

inaugural sacrifice. After conquering the kings of several other kingdoms and
collecting tribute from the conquered land,
o Vajapeya Sacrifice: Chariot race which a royal chariot was made to win the race
against his kinsmen.
o Ashvamedha Sacrifice: Unquestioned control over the area on which the royal
horse ran uninterrupted.
 Dependence on Sabha and Samiti reduced. Women were not allowed to attend these
assemblies. Sabha and samiti continued to hold their ground but their character changed.
They were now controlled by chiefs and rich nobles.
 Vidhata completely disappeared.
 Term Rashtra, indicating territory first appeared in this period.
 Rajana assumed titles like Samrat, Ekrat, Sarvbhumi, Virat.
Religious Aspect

 Varun and Indra, the most important Gods of Rig Vedic age, lost prominence in later
Vedic phase.
 Prajapati or Adipurush became the Supreme God in later Vedic phase.
 Some of the other minor gods became prominent, such as Vishnu (conceived as the
preserver and protector of people) and Rudra (the god of animals).
 Some of the social orders too adopted their own deities. For example, Pushan (who was
supposed to look after cattle) came to be regarded as the god of the Shudras.

 Rituals, sacrifices and requirement of supervising priest (Purohita) made religious life
complex.

 Few instances of idol worship are reported. Magic and Omen entered the socio-religious
life.

66. Who among the following was not a writer of legal texts? [1989]

A. Yajnavalkya

B. Manu

C. Narada

D. Bharavi

Ans. D

MANU

 Manusmriti, a book on Indian law is attributed to the legendary first man and lawgiver,
Manu. Manu-smriti, traditionally the most authoritative of the books of the Hindu code
(Dharmashastra) in India

https://t.me/prelimbits
85

 It was one of the first Sanskrit texts to be translated into English in 1776, by British
philologist Sir William Jones, and was used to construct the Hindu law code, for the East
India Company administered.

YAJNAVALKYA SMRITI

 The Yajnavalkya Smriti is one of the many Dharma-related texts of Hinduism composed
in Sanskrit.

 It is dated between the 3rd to 5th-century CE, and belongs to the Dharmashastra
tradition.

 The text was composed after the Manusmriti, but like it and Naradasmriti, the text was
composed in shloka (poetic meter) style.

 The legal theories within the Yajnavalkya Smriti are presented in three books, namely
achara-kanda (customs), vyavahara-kanda (judicial process) and prayascitta-kanda
(crime and punishment, penance).

 The text is named after the revered Vedic sage Yajnavalkya, who appears in many major
Upanishads of Hinduism, as well as other influential texts such as the Yoga Yajnavalkya.[

NARADASMṚTI

 is one of the latest books of the Dharmaśāstra tradition in Hinduism and also the only
one which does not deal directly with the means of knowing dharma, focusing instead on
the bhakti tradition

BHARAVI

 6th C Sanskrit poet


 Patron: Durvinita (Western Ganga Dynasty) || Simhavishnu (Pallava dynasty) ||
Yasodharman Vishnuvardhana (Malwa ruler)
 Contemporary to Kalidas.
 Work: Kiratarjuniya (about Kirat and Arjun). One of the six mahakavyas in Classical Sanskrit

67. The Harappans did not know the use of [1990]

A. Bronze

B. Gold

C. Iron

D. Silver

Ans: C

IVC : TECHNOLOGY AND CRAFT


 This is known as the first urbanization in India

https://t.me/prelimbits
86

 Along with stone, they were well acquainted with bronze (occasionally mixed arsenic with
copper instead of tin). As neither tin nor copper was easily available, bronze tools do not
abound in the region.

 Harappan culture is recognized for advanced metallurgical techniques. Archaeologists


have discovered usage of copper, bronze, tin, silver and gold. It was the first
time that silver was extracted and used in Indus valley
 Iron was not known to the people.
 Important crafts: spinning (Spindle whorls), bricklaying, boat-making, seal making,
terracotta manufacturing (potter‘s wheel), goldsmiths, bead making.
 They were aware of the use of the wheel.

68. The First Buddhist Council met at [1990]

A. Rajagriha

B. Pataliputra

C. Jalandhara

D. Kashmir

Ans: A

BUDDHIST COUNCILS

Number Venue, Patronage & Outcome

First Buddhist  At Satparni Rajagriha (483 BC)


Council  Under the patronage of King Ajatshatru with the monk Mhakasyapa
Upali presiding, at Rajgriha, in the Sattapani Caves
 The idea was to preserve Buddha‟s teachings (Sutta) and rules for
disciples (Vinaya). Ananda was one of the great disciples of Buddha
recited Suttas and Upali, another recited Vinaya.

Second Buddhist  At Vaishali (383 BC)


Council  Under the patronage of King Kalasoka and the presidency of
Sabakami. The ruler at time was Kalasoka (Shishunag Dynasty)
 This council settled down the disputes on the Vinaya Pitaka and code
of discipline.

Third Buddhist  At Pataliputra (250 BC)


Council  Under the patronage of King Asoka and the presidency of
Moggaliputta Tissa
 The teachings of Buddha which were under two baskets were now
classified in 3 baskets as Abhidhamma Pitaka was established in this

https://t.me/prelimbits
87

council and they were known as „Tripitaka‟.


 It also tried to settle all the disputes of Vinaya Pitaka. Establishment
of Sthaviravada School as an orthodox school.

Fourth Buddhist  At Kundalvana, Kashmir (72 AD)


Council  Under the patronage of Kushan king Kanishka and the president of this
councilwas Vasumitra, with Avaghosa /Ashvaghosha as his deputy.
 This council distinctly divided the Buddhism into 2 sects Mahayana and
Hinayana.

69. In which one of the following inscriptions is Ashoka mentioned by name? [1990]

A. Major Rock edict at Girnar

B. Minor Rock inscription at Maski

C. Bairat inscription

D. Lamghan inscription

Ans. B

MASKI INSCRIPTION

 Maski is a village and archaeological site in Karnataka's Raichur district. It is located on


the banks of the Maski River, a tributary of the Tungabhadra.
 A minor rock edict of Emperor Ashoka can be found at the site.

 It was Emperor Ashoka's first edict that included the name Ashoka rather than
'Devanampriya' or 'Piyadassi.'

 The inscription, which is still a dharma shasana, instructs people to follow the tenets of
Buddhism.
 Furthermore, the inscription suggests that Mauryan rule extended all the way to the
Krishna valley in north-eastern Karnataka.

70. The Stone Age people had the first domestic [1990]

A. Asses

B. Dogs

C. Horses

D. Sheep

Ans. B

 The first animal to be tamed was the wild ancestor of the dog.

 Later, people encouraged animals that were relatively gentle to

https://t.me/prelimbits
88

come near the camps where they lived.

 These animals such as sheep, goat, cattle and also the pig lived in herds, and most of them
ate grass.

 Often, people protected these animals from attacks by other wild animals. This is how they
became herders.

 It was started almost 2.5 million years ago and ended in 9600 BC. In the stone age, humans
were hunter food-gatherers.

 http://ncert.nic.in/ncerts/l/fess103.pdf

71. Which one of the following can be classified as a "Pratiloma Marriage"? [1990]

A. A Brahmin girl marrying a Sudra boy

B. A Sudra girl marrying a Brahmin boy

C. A Brahmin girl marrying a Brahmin boy

D. None of these

Ans. A

ANULOMA AND PRATILOMA MARRIAGE

 Anuloma and pratiloma marriage, as a rule of marriage is an inseparable part of the hindu
caste system. The rule of anuloma and pratiloma marriages has greater significance for the
caste structure of Hindu society than for the marriage pattern of the Hindus.

Anuloma Marriage (hypergamy form of marriage)

 During the Vedic Age inter-class marriages used to take place in the form of Anuloma
marriage. Anuloma marriage is a social practice according to which a boy from
upper varna / caste / class can marry a girl from lower varna / caste / class.

 Anuloma marriage was recommended by the ancient Hindu law writers for the first
three varnas or classes of the then society namely →

o the Brahmins;

o the Kshatriyas; and

o the Vaishyas.

 According to the Dharmashastrakaras, a girl should marry in her own varna, failing which
she may marry one in any of the higher varna.

 In the Rig Vedic period, the priests who performed Yajnas [sacrifices] arranged by the kings,
married Kshatriya girls offered to them as ―dakshina‖ or fees for their services.

Pratiloma Marriage (Hypogamy form of Marriage)

 Pratiloma is a type of marital practice in which a man of lower class / caste / varna
https://t.me/prelimbits
89

marries a girl of higher class / caste / varna. Such cases of Shudra-Aryan connections are
also recorded in the Vedic texts.

 Some Dharmashastrakaras had even permitted the practice of ―pratiloma‖ while many
condemned it. By the time of Dharmashastras, greater disapproval was shown
towards this practice.
 Even here, the marriage of Brahmin / Kshatriya / Vaishya girl with Shudra boy was
more despised with, than the marriage of a Brahmin girl with a Kshatriya / Vaishya boy.

 Pratiloma marriage was very much discouraged and even condemned. Marriage of a girl
of higher caste with a boy of lower caste faced more resistance in the society.

 Shastrakara Manu was of the opinion that the progeny of the most hated pratiloma would
become ―chandalas‖ or ―untouchables.‖

 In the Chandukya Upanishad and also in the ―Buddhist Jatakas‖ we find vast references
to Chandalas. In fact, the origin of the practice of untouchability is to be seen in the
practice of pratiloma marriage.

72. Consider the following statements regarding Kautilya's Arthshastra: [1990]

1. It places morality higher than political expediency.

2. It argues the case for a fully-centralised government for the empire

3. It deals with problems of administration as well as laws of property and crime

4. It includes section on government of the State and diplomatic relations with other States.

Of these statements

A. 1, 2 and 3 are correct

B. 2 and 3 are correct

C. 3 and 4 are correct

D. 1,2 and 4 are correct

Ans. * ans should be 2,3 & 4

Bcoz In the Arthashastra, Books 7, 11 and 12 have given a comprehensive analysis on all aspects
of the relations between states. In the first chapter of Book 6, the theoretical basis of foreign
policy are described. [Source : Kauṭalya (1992). The Arthashastra. L. N. Rangarajan. New Delhi:
Penguin Books India. pp. 506–515. ISBN 0-14-044603-6. OCLC 30678203]
Note : I will cover this book in More Detail in my Upcoming Ancient History Updated Notes.
Kautilya‟s Arthashastra

 It is considered one of the most sophisticated and detailed treatise to have ever been
produced in India.

https://t.me/prelimbits
90

 It consists of 15 books (Adhikaranas), out of which the first five deal with tantra (internal
administration), and next eight with avapa (interstate relations). The final two books deal
with other miscellaneous topics.
 It‘s a political thoughts of Chandragupta Maurya‟s chief minister Kautilya, also known as
Chanakya or Vishnugupta.
 This book was rediscovered in India in c.1905 CE by R. Shamashastry.
 Dated to approximately c.250 CE, many centuries after the time of Kautilya, but it is
generally accepted that the main ideas in this book are largely of Kautilya himself.
 The book contains detailed information about specific topics that are relevant for rulers
who wish to run an effective government. Diplomacy and war (including military tactics).
 The book also talks about recommendations on law, prisons, taxation, irrigation,
agriculture, mining, fortifications, coinage, manufacturing, trade, administration, and
spies.
The ideas expressed by Kautilya in the Arthashastra are completely practical and unsentimental.
Kautilya openly writes about controversial topics such as assassinations, when to kill family
members, how to manage secret agents, when it is useful to violate treaties, and when to spy on
ministers. However, he also writes about the moral duty of the king and stresses on paternal
despotism as he summarises the duty of a ruler, saying, “The happiness of the subjects is the
happiness of the king; their welfare is his. His own pleasure is not his good but the pleasure of his
subjects is his good”.

73. Match List I with List II and select the correct answer using the codes given below
them: [1990]

List I List II

A. Dayabhaga 1. Harsha

B. Mitakshara 2. Jimutavahana

C. Nagananda 3. Krittivasa

D. Ramayana 4. Vijnanesvara

Select the correct answer from the codes given below:

A B C D

a) 3 4 1 2

b) 2 4 1 3

c) 4 2 1 3

d) 3 1 2 4
https://t.me/prelimbits
91

Ans. B

Dayabhaga by Jimutavahana

 The Dayabhaga is a Hindu law treatise written by Jimutavahana which primarily focuses
on inheritance procedure.

 The Dayabhaga was the strongest authority in Modern British Indian courts in the
Bengal region of India, although this has changed due to the passage of the Hindu
Succession Act of 1956 and subsequent revisions to the act.

 Based on Jimutavahana criticisms of the Mitakṣara, it is thought that his work is


precluded by the Mitākṣarā. This has led many scholars to conclude that the Mitākṣarā
represents the orthodox doctrine of Hindu law, while the Dāyabhāga represents the
reformed version.

 The central difference between the texts is based upon when one becomes the owner of
property.

o The Dāyabhāga does not give the sons a right to their father's ancestral property
until after his death,

o unlike Mitākṣarā, which gives the sons the right to ancestral property upon their
birth.

Jimutavahana

 Jimutavahan (c. 12th century) was an Indian Sanskrit scholar and writer of legal and
religious treatises on Vaishnavism of early medieval period.

 He was the earliest writer on smriti (law) from Bengal whose texts are extant.

 Major works :

o Kalaviveka → exhaustive analysis of the auspicious kala (timings) for the


performance of religious rites and ceremonies. This text also contains discussions on
solar and lunar months.

o Vyvahāra-mātrikā or Nyayaratna-mātrikā or Nyayamātrikā → Dealt with


vyavahāra (judicial procedure). The text is divided into five sections,
Vyvaharamukha, Bhashapada, Uttarapada, Kriyapada and Nirnayapada.

o His magnum opus Dāyabhāga has dealt with the laws of inheritance based on
Manusmriti.

 In Bengal (and post-independence West Bengal and Tripura) and Assam,


Dāyabhāga was the principal guide for laws on inheritance till the enactment
of the Hindu Succession Act, 1956.

 This treatise differs in some aspects from Mitakshara, which was prevalent in
other parts of India based on Yajnavalka Smriti.

https://t.me/prelimbits
92

 The right of a widow without any male issue to inherit the properties of her
deceased husband is recognized in Dāyabhāga.

Mitakshara by Vijnanesvara

 The Mitakshara is a vivṛti (legal commentary) on the Yajnavalkya Smriti best known for
its theory of "inheritance by birth."
 It was written by Vijnanesvara, a scholar in the Western Chalukya [Kalyani] court in the
late eleventh and early twelfth century during the reign of Vikramaditya VI
 Along with the Dāyabhāga, it was considered one of the main authorities on Hindu Law
from the time the British began administering laws in India.
 The entire Mitakshara, along with the text of the Yajnavalkya Smriti, is approximately 492
closely printed pages.
King Harsha Vardhana

 Harsha was also very talented himself as it is believed that he wrote three dramas
o Priyadarshika
o Ratnavali (both romantic comedies), and
o Nagananda (based on Bodhisattva Jimutavahana).
 His court poet
o Banabhatta (covered ion Above Pages)
o Mayura (author of Mayurashataka)
o Bhartrihari Grammarian (author of Vakapadiya)
o Matanga Divakara
Krittivasi Ramayan

 Krittivasi Ramaya was composed by the fifteenth-century Bengali poet Krittibas Ojha,
from whom it takes its name, is a rendition of the Rāmāyaṇa into Bengali.

 Krittivasi Ramaya is not just a rewording of the original Indian epic, but also a vivid
reflection of the society and culture of Bengal across the period of its circulation,
from the Middle Ages into the modern period.

 It was characterised by Dinesh Chandra Sen in 1911 as 'by far the most popular book in
Bengal' and 'the Bible of the people of the Gangetic Valley'.

74. Which one of the following was the contemporary Bengal king during the time of
Harsha of Kannauj? [1990]

A. Bhaskaravarman

B. Divakaramitra

https://t.me/prelimbits
93

C. Devagupta

D. Sasanka

Ans. D

HARSHA VARDHANA (606−647 CE)

 He is known as the last great Hindu king of India, who was originally a Shaiva but also
supported Buddhism to a great extent and made generous endowments to Buddhists.
 His land grant inscriptions describe him as Parama-maheshvara (supreme devotee of
Shiva). His court poet Bana also describes him as a Shaivite.
o He later he became an ardent Hinayana Buddhist. Hiuen Tsang converted him to
Mahayana Buddhism. Harsha prohibited the use of animal food in his kingdom and
punished those who kill any living being.
 He is also described as the lord of the north (sakalauttarapathanatha).
 Rajya Vardhana had succeeded Prabhakar Vardhana initially. However, Harsha succeeded
his brother, when Rajya Vardhana left the reigns of governance in the hands of Harsha
Vardhana as he had to undertake a campaign against the →
o Ruler of Malwa, Devagupta and
o Shashanka, the ruler of Gauda, who had imprisoned their sister Rajyashri and
killed her husband Grahavarman.
 He defeated the Malwa army and killed Devagupta, but was unfortunately killed by
Shashanka, who also cut the Bodhi tree and occupied Kannauj.
 It was UNDER THESE CIRCUMSTANCES that HARSHA then ASCENDED the throne.
o He immediately marched towards Kannauj and rescued his sister Rajyashri.
Kannauj subsequently came under the sovereignty of Pushyabhutis.
o Later, he also defeated Shashanka and extended his control over parts of Kongoda
in Orissa.

 Harsha‘s reign was marked by many military victories and he brought most of north India
under his control as he brought the five Indies (regions) - Punjab, Kannauj, Bengal,
Orissa, Mithila and assumed the title of Siladitya (as mentioned in travel records of
Chinese traveler Xuanzang (Hsuan Tsang).
 He was victorious against the ruler of Sindh in the north west and also defeated the
https://t.me/prelimbits
94

Vallabhi king, Dhruvasena II, as mentioned in the Nausasi copper plate inscription.

 However, he (harsha) was defeated by Pulkesin II (western Chalukyan King of Badami)


on the banks of Narmada. Pulkesin II bestowed the title of Sakalauttarapathanatha
meaning ―Lord of the entire north‖ upon him.

 The capital of Harsha was originally Thaneswar, but he shifted it to Kannauj.

 The Chinese pilgrim Hsuan Tsang, who is also known as the Prince of Travelers and
author of Si-Yu-Ki, visited India during 629–644 CE.
o He earned Harsha‘s friendship and left a vivid account of the beauty, grandeur,
and prosperity of Kannauj.
o He gives a detailed account of a Grand Assembly held at Kannauj in 643 CE, which
was attended by the representatives of Hinduism, Jainism, and Buddhism.
 The Kannauj Assembly was followed by another spectacular assembly at
Prayaga in the same year.
 While the Kanauj Assembly was a religious assembly to highlight
Mahayanism, the Prayaga Assembly was an assembly of universal
character for offerings of royal charities to all classes of people.

75. It is said that of the five doctrines taught by Jainism four were taken over by Mahavira
from previous teachers and only one was added by him. Which one of the following
was the doctrine added by him?

A. Do not commit violence

B. Observe continence (Brahmacharya)

C. Do not acquire property

D. Do not speak a lie

Ans. B

Five principles of Jainism are :

 These are called anuvratas (small vows) for Jain laypersons, and mahavratas (great vows)
for Jain mendicants

o Non Violence (Ahimsa)

o No lies (Satya)

o No stealing (Asteya)

o No Property (Aparigraha)

o Observing Continence (Brahmacharya).

 The fifth principle (Brahmacharya) was added by Mahavira and the other four were the

https://t.me/prelimbits
95

teachings of his Predecessors. Mahavira was the 24th Tirthankara of Jain.

 Jainism prescribes seven supplementary vows, including three guņa vratas (merit vows)
and four siksa vratas.

 The Sallekhana (or Santhara) vow is a "religious death" ritual observed at the end of
life, historically by Jain monks and nuns, but rare in the modern age. In this vow, there is
voluntary and gradual reduction of food and liquid intake to end one's life by choice
and with dispassion. This is believed to reduce negative karma that affects a soul's future
rebirths

76. Indus valley civilisation is also known as the Harappan culture because [1991]

A. The site of Harappa is six times larger than Mohenjodaro site

B. The Indus valley civilisation is considered the elementary/initial stage of the Vedic culture
and Harappa is believed to be the same as Harappa mentioned in the Vedas

C. Harappa was the first site to be excavated in the Indus valley

D. The most important/significant evidence of the achievements of this civilisation have been
excavated from Harappa

Ans. C

HARAPPAN CIVILIZATION (INDUS VALLEY CIVILIZATION)

 IVC also known a Bronze Age civilisation in the northwestern regions of South Asia, lasting

https://t.me/prelimbits
96

from 3300 BCE to 1300 BCE, and in its mature form 2600 BCE to 1900 BCE

 Together with ancient Egypt and Mesopotamia, it was one of three early civilisations of
the Near East and South Asia, and of the three, the most widespread.

 Its sites spanned an area from northeast Afghanistan and much of


Pakistan to western and northwestern India.
 The civilisation flourished both in the alluvial plain of the Indus River, which flows
through the length of Pakistan, and along a system of perennial monsoon-fed rivers that
once coursed in the vicinity of the Ghaggar-Hakra, a seasonal river in northwest India and
eastern Pakistan.
 The cities of the ancient Indus were noted for their
o Urban planning,
o Baked brick houses,
o Elaborate drainage systems,
o Water supply systems,
o Clusters of large non-residential buildings, and
https://t.me/prelimbits
97

o Techniques of handicraft and metallurgy.


 Mohenjo-daro and Harappa very likely grew to contain between 30,000 and 60,000
individuals, and the civilisation may have contained between one and five million
individuals during its florescence.
 The Indus civilisation is also known as the Harappan civilisation, after Harappa was first
to be excavated early in the 20th century.
 The discovery of Harappa and soon afterwards Mohenjo-daro was the culmination of work
that had begun after the founding of the Archaeological Survey of India in the British Raj
in 1861.
 There were earlier and later cultures called Early Harappan and Late Harappan in the
same area.
o The early Harappan cultures were populated from Neolithic cultures, the earliest
and best-known of which is Mehrgarh, in Balochistan, Pakistan.
o Harappan civilisation is sometimes called MATURE HARAPPAN to distinguish it
from the earlier cultures.
 There are five major urban centres:
o Mohenjo-daro in the lower Indus Valley (declared a UNESCO World Heritage Site in
1980 as "Archaeological Ruins at Moenjodaro"),
o Harappa in the western Punjab region, Ganeriwala in the Cholistan Desert,
o Dholavira in western Gujarat (declared a UNESCO World Heritage Site in 2021 as
"Dholavira: A Harappan City"), and
o Rakhigarhi in Haryana.
 The Harappan language is not directly attested, and its affiliation uncertain as the Indus
script has remained undeciphered. A relationship with the Dravidian or Elamo-Dravidian
language family is favoured by a section of scholars
Harappan culture is divided into various phases

 Early Harappan 3000–2600 BC


 Mature Harappan 2600–1900 BC [Harappan civilisation is sometimes called Mature
Harappan to distinguish it from the earlier cultures.]
 Late Harappan 1900–1700 BC

https://t.me/prelimbits
98

77. Which one of the following does not indicate the difference between Mahayanism and
Hinayanism? [1991]

A. Belief in the Bodhisattvas

B. Worship of the image of Buddha

C. Use of the Sanskrit language

D. Admission of women into the monasteries

Ans. C

Both Buddhism and Jainism had some similarities in their teachings

 For instance, both rejected the authority of Vedas, emphasised on renunciation and
human effort as the means of obtaining salvation, and established a monastic order
for both men and women.

 Like Buddhism, Jainism is fundamentally atheistic; though it recognises the existence of


gods, yet it refuses to give them importance in the universal scheme of themes and
places the gods lower than Jina (the conqueror).

 Like Buddhist texts, Jaina texts criticise the Brahmanas, their practice of animal sacrifices,
and ritual authority giving preference to Brahmanas.

 Both Jainism and Buddhism focus on the superiority of the Kshatriya varna over all
other varnas including Brahmanas.

 They both attempted to give a new connotation to the meaning of Brahmana, shifting
the emphasis from an ascribed status to one achieved by good deeds.

 They use the word „Brahmana‟ in the sense of acknowledging a wise person who
possesses true knowledge and lives an exemplary life.

 Like Buddhism monastic order, people of all castes and social backgrounds were
welcomed in Jainism.

o There are frequent mentions of a learned Jaina monk named Harikeshiya who
belonged to a Chandala family.

o Brahmana varna was represented by Bhadrabahu, Divakara, Jinasena, and


Haribhadra.

o Similarly, Jainism opened its doors to women mendicants who were addressed as
aryika or sadhvi.

https://t.me/prelimbits
99

78. Which one of the following archaeologists initially discovered the Mohanjedaro site of
the Indus valley civilisation?

A. Sujohu Marshall

B. Daya Ram Sahni

C. Rakhal Das Banerji

D. Sir Mortimer Wheeler

Ans. C

Mohenjo-daro

 Farther south, along the main stem of the Indus Marshall deputed a succession of ASI
officers to survey the site. These included D. R. Bhandarkar (1911), R. D. Banerji (1919,
1922–1923), and M. S. Vats (1924).

 In 1923 Banerji wrote to Marshall about the site, postulating an origin in "remote
antiquity," and noting a congruence of some of its artifacts with those of Harappa

 Systematic excavations began in Mohenjo-daro in 1924–25 with that of K. N. Dikshit,


continuing with those of H. Hargreaves (1925–1926), and Ernest J. H. Mackay (1927–1931).
 By 1931, much of Mohenjo-daro had been excavated, but occasional excavations
continued, such as the one led by Mortimer Wheeler, a new director-general of the ASI
appointed in 1944, and including Ahmad Hasan Dani.
After the partition of India in 1947, when most excavated sites of the Indus Valley Civilisation lay
in territory awarded to Pakistan.

Important findings Discovered in Mohanjedaro

It Largest site of Indus civilization


 Post cremation burial, Great Granary, Great Bath , Pasupathi seal, Bronze dancing girl,
Unicorn Seals (Most numbers of it in here) & Steatite statue of Beard Man
 Great Granary: A granary has been found which is the largest building of the
MohenjoDaro. This granary is divided into 27 rooms of different size and shape. Used for
the store surplus food grains. (also found in Harappa)
 Mohenjo-Daro was the largest city of the Indus valley civilization. Both Harappa and
Mohenjo-Daro can be called the capital cities of the civilization.

79. The Upanishads are collectively known as [1991]

A. Vedangas

B. Vedanta

C. Sruti

https://t.me/prelimbits
100

D. Smriti

Ans. B

UPANISHADS

 The literal meaning of Upanishad is to sit near someone. There are 108 Upanishads, of
which 13 are the most prominent.

 The Upanishads dwell on the Atman and Brahman, and focus a lot on the philosophy
about life, universe, self, body, sacrifice, etc.

o Eg . What is this world? Who am I? What becomes of me after death?

 The Upanishads are often called ‗Vedanta‗. Literally, Vedanta means the end of Veda,

 They emphasise that the knowledge of the self or atman should be acquired and that
the relation of the atman with the Brahman should be properly understood.

Important Upanishads

 Aitareya Upanishad: Talks about the creation of Atman (Soul) and Consciousness.
 Chandogya Upanishad: Deals with rhythm and chanting of Mantras. It clearly refers to the
first three ashrams and discusses the types of marriage (mainly two).
o Anuloma marriage - The marriage of a man in his own varna or below his varna.
It is the most accepted and common form of marriage in the society.
o Pratiloma marriage – The marriage of a girl/woman in a varna lower than her
own, deemed rare and not sanctioned by the Vedas.
 Katha Upanishad: Tells the story of Nachiketa and Yama. Their conversation evolves into
discussion of Man, Atman (soul), Knowledge, and Moksha (liberation).
 Brihadaranyaka Upanishad: Talks about the transmigration of Atman; passages on
metaphysics and ethics. It contain Famous universal prayer for enlightenment
„Asato ma sad-gamaya;
Tamaso ma jyotir-gamaya;
Mrtyor-ma amrutam gamaya.‟
 Mundaka Upanishad: Contains the mantra ―Satyameva Jayate‖ (truth alone triumphs)
which is borrowed in the National Emblem of India.
Principal thirteen Upanishads, related to the Vedas are : (More imp for State Exams)

 Upanishads of the Rigveda

o Aitareya Upanishad,

o Kaushitaki Upanishad

 Upanishads of the Shukla-Yajurveda:

o Brihadaranyaka Upanishad,
https://t.me/prelimbits
101

o Isha Upanishad

 Upanishads of the Krishna-Yajurveda:

o Taittiriya Upanishad,

o Katha Upanishad,

o Shvetashvatara Upanishad,

o Maitrayaniya Upanishad

 Upanishads of the Samaveda:

o Chandogya Upanishad,

o Kena Upanishad

 Upanishads of the Atharvaveda:

o MundakaUpanishad,

o MandukyaUpanishad,

o Prashna Upanishad.

80. Match List I with List II and select the correct answer using the codes given below the
Lists: [1991]

List I List II

A. Chandragupta Maurya 1. The great council of Pryag

B. Samudragupta 2. Jainism

C. Harsha Vardhan 3. Gangaikonda Cholapuram

D. Rajendra Chola 4. Allahabad Pillar

Select the correct answer from the codes given below:

A B C D

a) 3 4 1 2

b) 2 4 1 3

c) 4 2 1 3

d) 3 1 2 4

Ans. b)

Chandragupta Maurya (CM) (324/321 - 297 BCE)

https://t.me/prelimbits
102

 Mauryan dynasty founded by Chandragupta Maurya with the help of his mentor
Chanakya/Kautilya.
 First established himself in Punjab and then moved eastwards until he gained control over
the Magadha region.
 Greek sources suggest a meeting between Chandragupta and Alexander (who invaded
northwest India during 327–326 BCE). They also account for the conflict between
Chandragupta Maurya and Seleucus Nikator (who was one of the generals of Alexander.
Chandragupta defeated him & gain additional territory west of the Indus River.
 Seleucus sent an ambassador to the Mauryan court, Megasthenes (who wrote the
Indica).
 He (CM) was the chief architect of the Mauryan empire and built a vast empire, which
included Bihar, Nepal, western and north- western India, and the Deccan.

 According to Jain texts and tradition, Chandragupta adopted Jainism and went to the hills
of Shravana Belagola (near Mysore) with Bhadrabahu and committed Sallekhana (death
by slow starvation).

Allahabad Pillar Inscription (Prayag Prasasti)

 "Prayag" refers to a place where something or someone can meet. Prayag is an ancient
name for Allahabad because it is the confluence of the rivers Ganga, Yamuna, and the
mythical Saraswati.
 Prashasti is a eulogy that means "in praise of someone."
 The Allahabad Prasasti was first inscribed on the Ashokan Pillar in Kausambhi, near
Allahabad.
 It was later relocated to the Allahabad fort.
 It is an Ashokan Stambh, but it has four different inscriptions,
1. The standard Ashokan inscriptions in Brahmi script are found on all pillars.
2. The Queen's edict lauding Ashoka's wife Kaurwaki's charitable deeds.

3. Inscriptions of Samudragupta (335–375 AD) written by Harisena in Sanskrit It


discusses Samudragupta's conquests and the boundaries of the Gupta Empire.
Inscriptions called him ―Dharma Prachar Bandhu”.

 Harisena, who was not only his court poet but was also a kumaramatya,
that is, a high ranking official who described himself by different titles such as
sandhivigrahika (minister for peace and war) and mahadandanayaka
(judicial/military officer).
4. Jahangir‟s Persian inscriptions.
Harsha Vardhan : The great council of Prayaga

 The Chinese pilgrim Hsuan Tsang, who is also known as the Prince of Travelers and

https://t.me/prelimbits
103

author of Si-Yu-Ki, visited India during 629–644 CE.


 He earned Harsha‘s friendship and left a vivid account of the beauty, grandeur, and
prosperity of Kannauj.
 He gives a detailed account of a Grand Assembly held at Kannauj in 643 CE, which was
attended by the representatives of Hinduism, Jainism, and Buddhism.

o The Kannauj Assembly was followed by another spectacular assembly at Prayaga


in the same year.

o While the Kanauj Assembly was a religious assembly to highlight Mahayanism,


the Prayaga Assembly was an assembly of universal character for offerings of
royal charities to all classes of people.
Rajendra I (1014- 1044 AD)
 Son of Rajaraja I, defeated & conquered whole Sri Lanka.

Rajendra I map
 Made Chola navy strongest in the area and ‗Bay of Bengal‘ was converted into a „Chola
Lake‟.

 In 1022 CE, he led an expedition to the north, crossed the Ganga following the same
route which was taken by the great conqueror Samudragupta and defeated the Pala
ruler, Mahipal I and the Western Chalukyas. To commemorate this occasion, he assumed
the title of Gangaikondachola (meaning ‗the Chola conqueror of Ganga‘) and established a
new capital named Gangaikondacholapuram (meaning ‗the city of the Chola conqueror of
the Ganga‘) near the mouth of the river Kaveri. He built a Shiva temple here and excavated a
tank called Chodagarg.

 He was great patron of learning and known as Pandita chola.


 Encouraged local self-government
 In 1025 CE, a successful naval expedition was dispatched against the revived Sri Vijaya

https://t.me/prelimbits
104

empire (which extended over the Malay peninsula, Sumatra, Java and the neighbouring
islands and controlled the overseas trade route to China). The expedition led to the
conquest of the kadaram or kedah and a number of other places in the Malay peninsula
and Sumatra. (asked in CSE pre 2003)
The later rulers such as Kulottunga II, Rajaraja II and Kulothunga III tried to maintain the Chola
empire but the Chola power gradually declined and the dynasty came to an end in the 13th
century. The place of the Cholas were taken by the Pandyas and the Hoysalas in the south, and
of the later Chalukyas by the Yadavas and the Kakatiyas.
Finally, in the beginning of 14th century, they were destroyed by the Sultans of Delhi.

81. Some Buddhist rock-cut caves are called chaityas while the others viharas. What is
the difference between the two? [1991]

A. Chaityas is a place of worship while vihara is the dwelling place of the monks
B. Chaityas is the stupa at the far end of the cave while vihara is the hall axial to it
C. There is no material difference between the two
D. Vihara is a place of worship while chaityas is the dwelling place of the monks
Ans. A
CHAITYA
 Chaitya were the places of worship and assembly of the monks and it consists of stupa
within the structure at one end
 The earliest surviving spaces comparable to the chaitya hall date to the 3rd century BCE.
These are the rock-cut Barabar Caves (Lomas Rishi Cave and Sudama Cave), excavated
during the reign of Ashoka by or for the Ajivikas, a non-Buddhist religious and
philosophical group of the period
 The chaitya at Bhaja Caves is perhaps the earliest surviving chaitya hall, constructed in
the second century BCE. It consists of an apsidal hall with a stupa.
 The most important of rock-cut complexes are the Karla Caves, Ajanta Caves, Ellora
Caves, Udayagiri and Khandagiri Caves, Aurangabad Caves and the Pandavleni Caves.
o KARLA CAVES near Lonavala, Maharashtra is best-known because of the famous
"Grand Chaitya" (Cave 8), which is "the largest and most completely preserved"
chaitya hall of the period, as well as containing unusual quantities of fine sculpture,
much of it on a large scale.

https://t.me/prelimbits
105

Drawing of the "Great Chaitya" at the Karla Caves,


Vihara
 The Vihara are the monastery where the monks used to live and it acts as their
accommodation during the rainy days. They are usually carved out of rocks as cave
structures.

82. Which one of the following does not constitute part of the Veda? [1991]

A. Nirukta

B. Upanishad

C. Brahmana

D. Samhita

Ans. A

VEDIC PERIOD

https://t.me/prelimbits
106

 The Vedic period, or the Vedic age (c. 1500 – c. 500 BCE), is the period in the late Bronze
Age and early Iron Age of the history of India when the Vedic literature, including the
Vedas (ca. 1300–900 BCE), was composed in the northern Indian subcontinent, between
the end of the urban Indus Valley Civilisation and a SECOND URBANISATION, which
began in the central Indo-Gangetic Plain c. 600 BCE.

 The Vedas are liturgical texts which formed the basis of modern day Hinduism, which
also developed in the Kuru Kingdom.

 The Vedas contain details of life during this period that have been interpreted to be
historical and constitute the primary sources for understanding the period. These
documents, alongside the corresponding archaeological record, allow for the evolution of
the Vedic culture to be traced and inferred.

 The Vedas were composed and orally transmitted in this period. The Vedic society was
patriarchal and patrilineal.

 Early Indo-Aryans were a Late Bronze Age society centred in the Punjab, organised into
tribes rather than kingdoms, and primarily sustained by a pastoral way of life.

 Around c. 1200–1000 BCE Vedic culture spread eastward to the fertile western Ganges
Plain.

 Iron tools were adopted, which allowed for the clearing of forests and the adoption of a
more settled, agricultural way of life.

 The second half of the Vedic period was characterised by the emergence of towns,
kingdoms, and a complex social differentiation distinctive to India, and the Kuru
Kingdom's codification of orthodox sacrificial ritual.

o During this time, the central Ganges Plain was dominated by a related but non-
Vedic culture, of GREATER MAGADHA.

 The end of the Vedic period witnessed the rise of true cities and large states (called
Mahajanapadas) as well as Sramaṇa Movements (including Jainism and Buddhism) which
challenged the Vedic orthodoxy.

 The Vedic period saw the emergence of a hierarchy of social classes that would remain
influential. Vedic religion developed into Brahmanical orthodoxy, and around the
beginning of the Common Era

 Archaeological cultures identified with phases of vedic culture include the Ochre
Coloured Pottery culture, the Gandhara grave culture, the black and red ware culture
and the Painted Grey Ware culture.

Origins

This is most Accepted view regarding the Vedic Culture evolution.

 The early Vedic age is historically dated to the second half of the second millennium

https://t.me/prelimbits
107

BCE.

 Historically, after the collapse of the Indus Valley civilisation, which occurred around
1900 BCE, groups of Indo-Aryan peoples migrated into north-western India and
started to inhabit the northern Indus Valley.

 The Indo-Aryans represented a sub-group that diverged from other Indo-Iranian tribes
at the Andronovo culture before the middle of the 2nd millennium BCE,

 The Indo-Aryans migrated through the adjacent Bactria–Margiana area (present-day


northern Afghanistan) to northwest India.

 Evidences of horses, spoked wheels, fire cults, and cremation (which formed important
parts of Aryan life in India) during the period between 1900 BCE and 1500 BCE, were found
in these regions.

 Apart from these, the artifacts and ceramics also suggest movement of people from
Central Asian region to South Asian region.

 The earliest Aryans lived in the land of the (Sapta Sindhu) (Sindhu is the river par
excellence of the Aryans) and probably because of their use of horse chariots and superior
military technology, they could establish their political dominance in the region.

 The region was drained by the seven rivers →

o Indus (Sindhu), and its five tributaries, namely →

 Jhelum (Vatista)

 Beas (Vipasa)

 Chenab (Askini)

 Ravi (Purushni)

 Sutlej (Sutudri), and the

o Saraswati (modern Ghaggar Hakra), which covered mostly the areas of eastern
Afghanistan, Punjab, and fringes of western U.P.

 The rare mention of the river Yamuna (twice) and Ganga (only once) is pointer to the fact
that Rig Vedic Aryans had not inhabited that region as yet.

Division of Vedic period

 The Vedic corpus is generally divided into Early Vedic and


Later Vedic texts.

o Early Vedic Literature/Rig Vedic Culture (c.


1500–1000 BCE): Includes the Rig Veda Samhita
and other texts of the family.

 They are called family books as their

https://t.me/prelimbits
108

composition is attributed to the families of certain seer poets, such as


Vishvamitra, Atri, Gritsamada, Vasishtha, and Bharadvaja.

o Later Vedic Literature/Later Vedic Culture (c. 1000–500 BCE): Includes


Books 1, 8, 9, and 10 of the Rig Veda Samhita, the Samhitas of the Sama Veda, the
Yajur and the Atharva Vedas and the Brahmanas, Aranyakas, and Upanishads
attached to all the four Vedas.

 The word Veda has been derived from the root vid, which means to know/knowledge.
They hold the status of Shruti (which has been heard) and have been transmitted
orally for many centuries.

 Later, they were written down, and the earliest surviving manuscript belongs to the 11th
century.

 There are four Vedas and every Veda generally has four parts: Samhita, Brahmana,
Aranyaka, and Upanishad.

Vedanga

 The word Vedanga means the limbs of the Vedas. They are the supplementary texts that
aid in the proper recitation and understanding of the Vedas

 However, these do not qualify as Shruti as they are considered of human origin (not
revealed by the gods) and are written in the form of Sutras.

 Sutras are very short condensed statements that are used to express varied ideas. There
are six Sutras.

 They are:

o Shiksha (Phonetics)

o Kalpa (Ritualistic science).

o Jyotisha (Astronomy)

o Vyakaran (Grammar)

o Nirukta (Etymology)

o Chhanda (Metrics)

 The Kalpasutra is further divided into:

o Shrautasutra: concerned with Vedic sacrifices that require three or more fires.

o Grihyasutra: concerned with comparatively simpler domestic sacrifices, which


require only one fire. It includes rituals pertaining to crucial life stages (Samskaras)
such as Upanayana (initiation), Vivaha (marriage), and Antyeshti (funerary
practices).

o Dharmasutra: concerned with the rituals‟ Dharma.

https://t.me/prelimbits
109

83. In ancient India the territorial name Tosali referred to [1992]

A. The region between the rivers Jhelum and Chenab

B. The basin of the river Son

C. The delta of the river Cauvery

D. The area of the river Mahanadi

Ans. D

Tosali or Toshali

 Tosali or Toshali was an ancient city in the present day Odisha

 It was the capital of the eastern province of the Kalinga Kingdom.

 Mauryan Empire was divided into 5 provinces → Uttarapatha (Taxila), Avantipatha


(Ujjain), Prachyapatha (Tosali), Dakshinpatha (Suvarnagiri) and Magadha.

2 Special Kalinga Edict (Odisha) [Asked In UPSC]

 1st at Dhauli (ancient name Toshali) and 2nd at Jaugada (earlier known as Samapa) in
Odisha, fell in ancient Kalinga, a kingdom attacked and plundered by Ashoka‘s soldiers.
These new Kalinga rock edicts were different from other edicts of Ashoka.
 Rock-cut Elephant at Dhauli is also important one was built during the reign of Ashoka in
250 BC.

84.Which one of the following was worshipped as a heavenly God by ancient Indians?

A. Surya
B. Angaraka (Mangala)
C. Sukra
D. Sani
Ans. A
 Surya is the Hindu god of the Sun. He is considered the creator of the universe and the
source of all life. He is the supreme soul who brings light and warmth to the world.
 The god's most famous temple is at Konarak in Orissa, north-east India but he
was worshipped across the Indian subcontinent.
Konark Sun Temple

 Konark Sun Temple, located in East Odisha near the sacred city of Puri.
 Built in the 13th century by King Narasimhadeva I (Eastern Ganga dynasty bout 1250 CE)
and is dedicated to the Sun God.
 It marks the highest point of achievement of Kalinga architecture.

https://t.me/prelimbits
110

 It was declared a UNESCO world heritage site in 1984.


 The temple is designed in the shape of a colossal chariot.
 There are two rows of 12 wheels on each side of the Konark sun temple.
 Some say the wheels represent the 24 hours in a day and others say the 12 months.
 The seven horses are said to symbolise the seven days of the week.
 Sailors once called this Sun Temple of Konark, the Black Pagoda because it was supposed
to draw ships into the shore and cause shipwrecks.
 Great poet Rabindranath Tagore wrote of Konark: ―Here the language of stone surpasses
the language of man.‖
 Chandrabhaga Mela around the month of February.
Martand Sun Temple

 Location- Jammu and Kashmir.


 The Martand Sun Temple, also known as Pandou Laidan is a Hindu temple dedicated to
Surya (the chief solar deity in Hinduism) and built during the 8th century CE.
 Timeline- built by the third ruler of the Karkota Dynasty, Lalitaditya Muktapida, in the
8th century CE. It is said to have been built during 725-756 CE.
 Deity- Sun God, Martand is another Sanskrit synonym for Surya.
 It is now in ruins, as it was destroyed by the orders of Muslim ruler Sikandar Shah Miri.

85. In which one of the following the Gandhara sculptures of the Buddha are typically
Indian and not Greek or Roman? [1992]

A. In the treatment of the robe


B. In the rendering of the physiognomy
C. In the iconography
D. In the style
Ans. C
GANDHARA SCHOOL OF ART

 The Gandhara school of Art arose in modern-day Peshawar and Afghanistan on Punjab's
western boundaries.

 The Greek invaders brought the traditions of Greek and Roman sculptors with them,
which affected the region's native traditions ==> Greco-Indian School of Art

 Between 50 B.C. and 500 A.D., the Gandhara School flourished in two periods. While the
former school's sculptures were made of bluish-grey sandstone, the latter school's were
made of mud and plaster.

https://t.me/prelimbits
111

 The Buddha and Bodhisattvas iconography was based on the Greco-Roman pantheon
and resembled Apollo's.

 Major Centres :

o Jalalabad, Hadda, Bamaran, Begram & Taxila were the main centers where art pieces
of Gandhara School have been found.

 Major Features :

o The Gandhara sculptures have been discovered in the Taxila ruins as well as other
ancient sites in Afghanistan and Pakistan.

o They generally consist of Buddha images and relief sculptures depicting scenes
from Buddhist literature.

o Several Bodhisattva figures were cut out of the rock. The first preaching in the
deer park and the Buddha's death is depicted in a Gandhara figure.

o The predominant focus of this type of painting was Lord Buddha


and Bodhisattvas, as it was intimately tied with Mahayana Buddhism.

o The Bamiyan Buddha sculptures are an example of Gandhara style art.

o It thrived primarily in Afghanistan and present-day North-Western India.


Taxila, Peshawar, Begram, and Bamiyan were among the most prominent sites.

o From the first century BCE through the fourth century CE, the Gandhara School of
art flourished.

 Themes

o The Buddhist themes were largely represented in Gandhara art. The Buddha's
mother resembles an Anthenian matron.

o A Buddhist scene was created with an Apollo-like face. The image of Athena of
Rome in Lahore is perhaps one of the most beautiful Gandhara sculptures depicting
a western topic.

o It was created in order to express Buddhist ideas and practices.

o The specimens do not include any Greek art motif, with a few exceptions. The hand
of the Gandhara artist was Greek, but his heart was Indian.

 Patrons

o Gandhara School was fostered by the Shakas and the Kushanas. The first
sculptural portrayals of the Buddha in human form are attributed to
the Gandhara school. [The first image of the Buddha-was carved out during
the reign of famous Kushan king Kanishka I.]

 Features of sculpture
https://t.me/prelimbits
112

o The human body is sculpted in a realistic manner in these sculptures, with


considerable attention to realism and physical details, particularly in the depiction
of muscles, mustaches, and other facial features.

o The portrayal of the broad bold fold lines is a unique trait that sets it apart from what
has been discovered elsewhere in India.

 The Various Mudras of Buddha in Gandhara Art

o In all the Buddha depicted in the Gandhara Art is shown making four types of hand
gestures and this is a remarkable feature in this art.

 Greek influences on Gandhara School of art

o Greek god as protector: In many images of Buddha in Gandhara; he is seen under


the protection of Greek god Hercules.

o Vajrapani: Vajrapani found in the right hand of future Buddha is told as a


transformed symbol of Hercules who is seen as the protector of Buddha.

o Greek architectural influence: Some images of Buddha in Gandhara are presented


in the Greek architectural environment bearing the affinity of Corinthian.

o Artistic beauty: The Apollo-like face of Buddha; natural realism; wavy hair as
seen in images of Buddha in Gandhara resembles Hellenistic tradition.

o Intellectual affinity: The hello and bun of Gandhara Buddha signify intellectual
imbibitions of Buddha from Greek.

o Despite the fact that Gandharan sculpture's iconography was predominantly Indian, it
also included elements and methods from Classical Roman art. Vine scrolls,
cherubs with garlands, tritons, and centaurs are examples of Classical Roman art
found in Gandharan sculptures.

o Additionally, the Gandharan artists drew on the Roman religion's anthropomorphic


traditions.

o Gandharan art depictions of Buddha are evocative of statues of a youthful Apollo.

o The drapery on Buddha's garments was also similar to that on Roman imperial

https://t.me/prelimbits
113

monuments.

86. Fresco paintings in the Ajanta Caves were done, while which of the following dynasties
was flourishing?

A. Guptas
B. Sungas
C. Kanvas
D. Mauryas
Ans. A
Gupta Period Lasted from 300 CE - 600 CE
Ajanta Caves (2nd C. – 5th C.)

 Approx. 30 rock-cut Caves → Almost exclusively Buddhist.


 Location : Aurangabad district of Maharashtra (UNESCO World Heritage Site)
 The walls of the caves have both Murals and Fresco Paintings (painted on wet plaster)
They use tempera style, i.e. use of pigments.
 The paintings portray human values and social fabric along with styles, costumes and
ornaments of that period. The emotions are expressed through hand gestures.
 The unique feature of the paintings is that each female figure has a unique hairstyle. Even
animals and birds are shown with emotions.
 The Common Themes of these paintings range from Jataka stories to life of Buddha to
elaborate decorative patterns of flora and fauna.
 The outline of the figures is in red ochre, with contours of brown, black or deep red.
Some important paintings at Ajanta are:
 Scenes from the Jataka stories → Buddha‟s former lives as a Bodhisattva, the life of
Gautam Buddha, etc.
 Paintings of various Bodhisattvas in tribhanga pose in Cave No.
o Vajrapani (protector and guide, a symbol of Buddha‘s power)
o Manjusri (manifestation of Buddha‘s wisdom) and
o Padmapani (Avalokitesvara) (symbol of Buddha‘s compassion).
 The Dying Princess in Cave No. 16.
 Scene of Shibi Jataka, where King Shibi offered his own flesh to save the pigeon.
 Painting of Maya Devi, the mother of the Buddha
 Cave No. XVII representing Buddha‟s visit to the door of Yashodhara‟s abode in the city
of Kapilavastu

https://t.me/prelimbits
114

History
 The Ajanta Caves are generally agreed to have been made in two distinct phases
o 1st Phase → Satavahana period
o 2nd Phase (5th century) → During the reign of Hindu Emperor Harishena of the
Vakataka dynasty. The second phase is attributed to the theistic Mahayana, or
Greater Vehicle tradition of Buddhist

87. Which one of the following is a Vedanga? [1992]

A. Sruti
B. Smriti
C. Nirukta
D. Samhita
Ans. C
See Q. 82
TYPES OF VEDIC LITERATURE

There are broadly two types of Vedic literature:

 Shruti Literature : The word ‗Shruti‘ from the term ‗Shruti Literature‘ means ‗to hear‘ and
describes the sacred texts which comprise of Vedas, Brahmanas, Aranyakas, &
Upanishads. Shruti Literature is canonical, consisting of revelation and unquestionable
truth, and is considered eternal.

 Smriti Literature : Whereas, the word ‗Smiriti‘ literally means to be remembered and
which is supplementary and may change over time. Smriti Literature is the entire body of
the post-Vedic Classical Sanskrit literature and consists of Vedanga, Shad darsana,
Puranas, Itihasa, Upveda, Tantras, Agamas, Upangas.

88. Burma was known to ancient Indians as

A. Suvarnabhumi
B. Suvarnadvipa
C. Yavadvipa
D. Malay amandalam
Ans. A

 The term Suvarnabhumi ('land of gold') is commonly thought to refer to the Southeast
Asian Peninsula, including lower Burma and the Malay Peninsula.

 However there is another gold-referring term Suvarnadvipa (the Golden Island or


Peninsula, where dvipa may refer to either a peninsula or an island), which may correspond

https://t.me/prelimbits
115

to the Indonesian Archipelago, especially Sumatra.


 Both terms might refer to a powerful coastal or island kingdom in present-day
Indonesia , possibly centered on Sumatra or Java. This corresponds to the gold
production areas traditionally known in Minangkabau Highlands in Barisan Mountains,
Sumatra, and interior Borneo.
Additional Information
 The first, a copper plate inscription unearthed at Nalanda, details an endowment by the
Shailendra King, Balaputradeva of Suvarnadvipa (Sumatra in modern-day Indonesia).

 This Srivijayan king, "attracted by the manifold excellences of Nalanda" had built a
monastery there and had requested Devapala to grant the revenue of five
villages for its upkeep, a request which was granted.
Though there is lots of debate regarding exact location of this Term

89. The last in the succession of Jaina Tirthankaras was [1992]

A. Parsvanatha
B. Rishabha
C. Mahavira
D. Manisubrata
Ans. C
Jainism

 Came to prominence in the 6th century B.C., when Lord Mahavira propagated the religion.
 Sect led by Vardhamana Mahavira (referred to as Nigantha Nataputta by Buddhist texts)
bloomed into a religion called Jainism. It was earlier known as Nirgranthas (free from
bonds).
 Mahavira was known as Jina (conqueror) of the soul and hence his sect came to be known
as Jainism.

 Mahavira was not the founder of Jainism, but the last of the 24 Tirthankaras.

 Risabha was the founder of the sect. He is considered the first Tirthankara.
 Yajur Veda mentions three of the Tirthankaras, viz., Risabha, Ajitanatha and Aristanemi.
Most Prominent Tirthankaras

Rishabhnath

 He is said to exist before Indus Valley Civilization


 It is mentioned that in Bhagavata Purana, he is referred to as Lord Vishnu.
 Vedas also mention the name of Rishabhnath.

https://t.me/prelimbits
116

 He had many sons including – Bharat and Bahubali (Note: The Gomateshwara Statue is
dedicated to Bahubali; and is the tallest statue of the world. It is located in
Shravanabelagola in Karnataka.)
 It is also believed that the name of the script ‗Brahmi‘ is inspired by his daughter‘s name.
Mallinath

 Malli was the 19th Tirthankara.


 It is often debated that Malli was a woman however, some Digambara sect Jains believe
that she was reborn as a man and then became a Tirthankara.
Neminatha

 He is the 22nd Tirthankara.


 He is stated to be the cousin of Lord Krishna (A Hindu God.)
 In paintings, he is depicted to adorn dark complexion.
Parsvanath

 Parsvanath was the 23rd tirthankar.


 It is believed that he existed two centuries before Vardhamana Mahavira.
 He was apparently born in Banaras (Uttar Pradesh) around 817 BCE.
 He is said to have propounded Jain religion which was later revived by Mahavira.
 He attained Kaivalya on Mount Sammeta (Parasnath) in Jharkhand.
 According to Svetambaras Sect (White-Clad Sect of Jainism); Parsvnath founded four-fold
restraints:
o Ahimsa
o Satya
o Asteya
o Aparigraha (The fifth one, ‗Brahmacharya‟ was added by Mahavira.)
 Navagraha Jain Temple in Karnataka houses the tallest statue of Parsvanath.
Mahavira

 He was the 24th Tirthankara of Jain religion.


 He was the son of Siddhartha (chief of the Jnatrika clan) and Trishla (Trishala was the sister
of Lichchhavi chief Chetak)
 He was born in Kundagrama, a suburb of Vaishali, Vajji [in present-day Bihar].
 He was a contemporary of Gautam Buddha.
 At the age of 30, he left his worldly possessions and sought ascetic life towards Kevala
Gnan.
 He attained Kaivalya under a Sal tree.
https://t.me/prelimbits
117

 He attained Nirvana in Pavapuri, Bihar.

90. The term nishka, which meant an ornament in the Vedic period, was used in later
times to denote a/an [1992]

A. Weapon
B. Agricultural implement
C. Script
D. Coin
Ans. D
Later Vedic Age Economy

 The Later Vedic culture is also called as PGW–Iron Phase culture, as a particular kind of
pottery (painted grey ware) was used during that phase.

 The main factor in the expansion of the Aryan culture during the Later Vedic period was
the beginning of the use of iron, which was introduced around 1000 BCE and is
mentioned as Krishna Ayas/Shyama Ayas

 The Rig Vedic people knew of a metal, called Ayas, which was either copper or bronze.

 The society was largely rural. However, towards the end of the period, there are traces of
the beginning of urbanism (as nagar in the sense of a town is mentioned in the Taittiriya
Aranyaka).

 Agriculture emerged as the chief means of livelihood of the Later Vedic people. The
forests were cleared by burning the trees, which is also mentioned in the Satapatha
Brahamana (ruler Videha Madhava burned the forests between river Saraswati to river
Sedanira or Gandak).

 Cultivation was done by the plough. Mixed farming (cultivation along with herding) was
one of the prime occupations of the Later Vedic people.

 The land was still communally owned over which the vish (clan) had many participatory
rights, but it was recognised as property and the head of the household who owned the
land was termed as Grahpati.

 Rice (Vrihi) and wheat (Godhuma) became the staple diet of the people and lentils were
also grown. Also, with the beginning of food production, agricultural produce began to
be offered in the rituals.

 The items of dana and dakshina included cooked rice. Tila, from which the first widely
used vegetable food-oil was derived increasingly, came to be used in rituals.

 Diverse arts and crafts were practiced in the Later Vedic period. Tin, lead, silver, Iron, gold,
bronze, and copper were known to Later Vedic people.

https://t.me/prelimbits
118

 There were probably good smiths and smelters as a lot of copper objects have been found
at PGW Sites.

 People had obtained knowledge of glass manufacturing too. Shresthins indicate the
guilds or organisation of merchants.

 Exchange was still via barter, but Niskha was used as a convenient unit of value
although not as a typical currency. [According to the Rigveda, 'Nishka' is a throat
ornament. Later, 'Nishka' was used to represent a gold coin.]

 Wagons drawn by oxen were probably the most used mode of transport.

 Possession of Horses remained an important priority of Vedic leaders and a remnant of


the nomadic lifestyle, resulting in trade routes beyond the Hindu Kush to maintain this
supply as horses needed for cavalry and sacrifice could not be bred in India.

 The Later Vedic people were acquainted with four types of pottery →

o Painted Grey Ware

o Black and Red Ware

o Black-slipped Ware

o Red Ware

 Unlike the Rig Vedic age where only voluntary offerings were taken and per se no revenue
collection was done, in the Later Vedic age, collection of taxes and tributes was made
Mandatory and were done by Sangrihitri. Vaishyas appear to be the only tribute payers
in Later Vedic times.

91. The Buddhist sect Mahayana formally came into existence during the reign of [93]

A. Ajatashatru
B. Ashoka
C. Dharmapala
D. Kanishka
Ans. D

Fourth Buddhist  At Kundalvana, Kashmir (72 AD)


Council  Under the patronage of Kushan king Kanishka and the president of this
councilwas Vasumitra, with Avaghosa /Ashvaghosha as his deputy.
 This council distinctly divided the Buddhism into 2 sects Mahayana and
Hinayana

SCHOOLS OF BUDDHISM

 Major Schools = Mahayana & Hinayan

https://t.me/prelimbits
119

 Other Schools = Theravada, Vajrayana & Zen.


Mahayana Buddhism
 The term Mahayana is a Sanskrit word which literally means "Great Vehicle".
 Sought salvation through grace & help of Buddha & Bodhisattvas.
 It originated in northern India and Kashmir and then spread east into Central Asia, East
Asia and some areas of Southeast Asia.
 Believed that Buddha will be born again.
 Worship Buddha in idol form.
 Language- Sanskrit.
 Sub-schools Chittmatra & Madhyamaka
 ZEN is a subschool of Mahayana which is prevalent in china, Korea and related to Taoism.
 Buddhist schools embedded in China, Korea, Tibet and Japan belong to the Mahayana
tradition.
Hinayana Buddhism
 Considered Lesser vehicle. Also known as Abandoned Vehicle or Defective vehicle. It
believes in the original teaching of Buddha or Doctrine of elders.
 Followed original teachings of Buddha.
 Sought individual salvation through self-discipline & meditation.
 Buddha will never be reborn.
 Did not believe in Idol worship.
 Language- Pali language.
 Buddha was an intellectual, not a god.
 Encompasses 18 subschools. Most important – Sarvastivada, Theravadin & Sautantrika.
Theravada
 Theravada Buddhism is older and the more conservative of the two main divisions of
Buddhism and is often referred to as the „traditions of the elder‟
 Theravada is a Hinayana sect.
 Theravada Buddhists strive to become Arhats and gain freedom from the cycle of
samsara.
 Theravada Buddhism developed in Sri Lanka and subsequently spread to the rest of
Southeast Asia. It is the dominant form of religion in Cambodia, Laos, Myanmar, Sri
Lanka, and Thailand
Vajrayana
 Vajrayana means “The Vehicle of the Thunderbolt”, also known as tantric Buddhism.

https://t.me/prelimbits
120

 This Buddhist school developed in India around 900 CE.


 Believed in acquiring magical power for liberation.
 Developed in Tibet and believes in the worship of female deities - Taras.
 It became popular in Eastern India, particularly Bengal and Bihar
Zen
 It is a school of Mahayana Buddhism that originated in China during the Tang dynasty as
the Chan school of Chinese Buddhism in and later developed into various schools.
 It spread to Japan in 7th century C.E.
 Meditation is the most distinctive feature of this Buddhist tradition

92. In Jainism 'perfect knowledge' is referred to as

A. Jina
B. Ratna
C. kaivalya
D. Nirvana
Ans. C
Jina
 Also called Jina (―Victor‖), in Jainism, a saviour who has succeeded in crossing over life's
stream of rebirths and has made a path for others to follow.
 A Jina is not a divine being or an avatar of a god, but an enlightened human being, who
has attained a state of transcendent bliss and omniscience through self discipline.
 They share the unchanging principles that underpin Jainism and teach others the path to
liberation.
 The term ―Jina‖ is sometimes used interchangeably with ―Tirthamkara,‖ which is Sanskrit for
'ford-maker'. It therefore means the person who builds a ford across the river of rebirth.
Ratna or Ratantraya of Jainism [Triratnas]

Kaivalya
 Kaivalya means knowing everything in Jainism. It is referred to as a person who has total
and ‗supreme knowledge‘.

https://t.me/prelimbits
121

 Kevala jnana has the concept which refers to the inert quality of all souls. This quality is
enveloped by ‗karmic particles‘ that surround the soul.
 According to their theory each soul has the „potential to gain‟ total knowledge by
wearing off the karmic particles.
 According to the Jain scriptures, there are ‗twelve stages‘ through which the ‗soul reaches‘
towards its goal of knowledge.
Nirvana Vs. Parinirvana

Nirvana (nirvana with remainder)

 In the Buddhist tradition, nirvana has commonly been interpreted as the extinction of the
"three fires",or "three poisons", greed (raga), aversion (dvesha) and ignorance (moha).
 When these fires are extinguished, release from the cycle of rebirth (saṃsāra) is
attained.
Parinirvana (nirvana without remainder)

 Parinirvana is commonly used to refer to nirvana-after-death, which occurs upon the


death of someone who has attained nirvana during his or her lifetime.
 It implies a release from the Saṃsāra, karma and rebirth as well as the dissolution of the
skandhas.
Distinguished btw Nirvana with remainder & Nirvana without remainder

Nirvana with Remainder


 A state achieved prior to death, where ―the remainder‖ refers to the mind and body of
this final existence
Nirvana without remainder
 Which is achieved at death when the causes of all future existence have been
extinguished and the chain of causation of both physical form and of consciousness have
been finally terminated

These states were available to all who followed the Buddhist path to its conclusion.
The Buddha himself is said to have realized nirvana when he achieved enlightenment at the age
of 35. Although he destroyed the cause of future rebirth, he continued to live for another 45
years. When he died, he entered nirvana, never to be born again. (this event is known as
Mahaparinirvana

The death of the Buddha, or Mahaparinirvana @ Ajanta Cave 26

https://t.me/prelimbits
122

Massive uddha sculpture. Here Buddha is lying on his right side. The sculpture has a length of 7
metres. The panel below the sculpture shows Buddha‟s disciples and celestial figures float
above. This is one of the grandest and most sensitive depiction in all of Buddhist art. The
Mahaparinirvana of the Buddha, when he achieved release from the mortal world, is a deeply
impressive sculpture.

93. Chanakya, was also known as [93]

A. Bhattasvamin
B. Rajasekhara
C. Vishnugupta
D. Visakhadatta
Ans. C
CHANAKYA (Aka. Kautilya or Vishnugupta)
 Chanakya (375–283 BCE) was an ancient Indian polymath who was active as a teacher,
author, strategist, philosopher, economist, jurist, and royal advisor.
 He is traditionally identified as Kauṭilya or Vishnugupta, who authored the ancient
Indian political treatise, the Arthashastra, a text dated to roughly between the fourth
century BCE and the third century CE.
 As such, he is considered the pioneer of the field of political science and economics in
India, and his work is thought of as an important precursor to classical economics.
 His works were lost near the end of the Gupta Empire in the sixth century CE and not
rediscovered until the early 20th century.
 Around 321 BCE, Chanakya assisted the first Mauryan emperor Chandragupta in his rise
to power and is widely credited for having played an important role in the
establishment of the Maurya Empire.
 Chanakya served as the CHIEF ADVISOR to both emperors Chandragupta and his son
Bindusara.
 Arthashastra →
o The book contains detailed information about specific topics that are relevant for
rulers who wish to run an effective government. Diplomacy and war (including
military tactics).
o The book also talks about recommendations on law, prisons, taxation, irrigation,
agriculture, mining, fortifications, coinage, manufacturing, trade,
administration, and spies.
o The ideas expressed by Kautilya in the Arthashastra are completely practical and
unsentimental. Kautilya openly writes about controversial topics such as
assassinations, when to kill family members, how to manage secret agents, when it is

https://t.me/prelimbits
123

useful to violate treaties, and when to spy on ministers. However, he also writes
about the moral duty of the king and stresses on paternal despotism as he
summarises the duty of a ruler, saying, “The happiness of the subjects is the happiness
of the king; their welfare is his. His own pleasure is not his good but the pleasure of his
subjects is his good”.
Rajashekhar

 10th C was an eminent Sanskrit poet, dramatist and critic.


 Patron : Mahindrapala (Gurjara-Pratihara)
 Work :
o Karpuramanjari: A famous play written in Sauraseni Prakrit to please his wife,
Avantisundari, a woman of taste and accomplishment. He is perhaps the only
ancient Indian poet to acknowledge a woman for her contributions to his literary
career.
o Kavya Mimansa (around c.880−920 CE): A practical guide for poets that explains
the elements and composition of a good poem.
o Vidhasalabhanjika
o Bhrinjika
o Balaramayana
o Prapanch Pandav
o Balabharata
Vishakhadatta

 Sanskrit poet and playwright during Gupta Era.


 Court poet of Chandragupta II [but according to RC majumdar he was not court poet of
Chandragupta II. So UPSC maybe will not ask again such Q which leads to controversy.]
 Work :
o Mudra Rakshas : Narrates the ascent of Chandragupta Maurya to throne.
o Devichandraguptam : The play, king Ramagupta decides to surrender his queen
Dhruvadevi ("Devi") to a Shaka enemy when besieged. Ramagupta's younger
brother Chandragupta enters the enemy camp disguised as the queen, and kills
the enemy ruler.

94. A lot of details, regarding the village administration under the Cholas is provided by
the Inscriptions at

A. Thanjavur
B. Uraiyur
C. Kanchipuram

https://t.me/prelimbits
124

D. Uttaramerur
Ans. D
Uttaramerur Inscription

 The Uttaramerur inscription, found in the village by this name in Chengalpattu (now
Kanchipuram) district south of Chennai.
 It is noted for its temple inscriptions that describe a self-governance system existing
around 7th to 9th century CE [under Chola administration]
 It mentions the qualifications, mode of election, disqualification criteria and
constitution of committee for local governance.

95. Asokan inscriptions were first deciphered by [93]

A. Buhler
B. Robert Sewell
C. James Prinsep
D. Codplngton
Ans. C
Edicts of Ashoka
 The Edicts of Ashoka are a collection of more than thirty inscriptions on the Pillars of
Ashoka, as well as boulders and cave walls, attributed to Emperor Ashoka of the Maurya
Empire who reigned from 268 BCE to 232 BCE.
 Ashoka used the expression Dhaṃma Lipi (Prakrit in the Brahmi script) to describe his
own Edicts.
 These inscriptions were dispersed throughout the areas of modern-day Bangladesh, India,
Nepal, Afghanistan and Pakistan, and provide the first tangible evidence of Buddhism.
 The edicts describe in detail Ashoka's view on dhamma, an earnest attempt to solve
some of the problems that a complex society faced.
 According to the edicts, the extent of Buddhist proselytism during this period reached as
far as the Mediterranean, and many Buddhist monuments were created.
 These inscriptions proclaim Ashoka's adherence to the Buddhist philosophy.
 The inscriptions show his efforts to develop the Buddhist dhamma throughout his
kingdom. Although Buddhism as well as Gautama Buddha are mentioned, the edicts
focus on social and moral precepts rather than specific religious practices or
the philosophical dimension of Buddhism. These were located in public places and
were meant for people to read.
 In these inscriptions, Ashoka refers to himself as "Beloved of the Gods" (Devanampiya).

 The identification of Devanampiya with Ashoka was confirmed by an inscription


https://t.me/prelimbits
125

discovered in 1915 by C. Beadon, a British gold-mining engineer, at Maski, a village in


Raichur district of Karnataka.
 Another minor rock edict, found at the village Gujarra in Datia district of Madhya
Pradesh, also used the name of Ashoka together with his titles: "Devanampiya
Piyadasi Asokaraja".
 The inscriptions found in the central and eastern part of India were written in Magadhi
Prakrit using the Brahmi script, while Prakrit using the Kharoshthi script, Greek and
Aramaic were used in the northwest.

 These edicts were deciphered by British archaeologist and historian James


Prinsep.
 The inscriptions revolve around a few recurring themes:
o Ashoka's conversion to Buddhism,
o the description of his efforts to spread Buddhism,
o his moral and religious precepts, and
o his social and animal welfare program.
 The edicts were based on Ashoka's ideas on administration and behaviour of people
towards one another and religion.

96. Who among the following is NOT associated with medicine in India?

A. Dhanvantri
B. Bhaskaracharya
C. Charaka
D. Susruta
Ans. B
Dhanvantari
 Dhanvantari is the physician of the devas in Hinduism.
 He is regarded to be an avatar of Vishnu.
 He is mentioned in the Puranas as the god of Ayurveda
Bhaskaracharya or Bhaskara II

 He Was an Indian mathematician and astronomer.

 His main work, Siddhanta Shiromani


 He was born in 1114 in Vijjadavida (Vijjalavida) and living in the Sahyadri mountain
ranges of Western Ghats, believed to be the town of Patan in Chalisgaon, located in
present-day Khandesh region of Maharashtra by scholars

https://t.me/prelimbits
126

 He is the only ancient mathematician who has been immortalized on a monument.


 Colebrooke who was the first European to translate (1817) Bhaskaracharya II's
mathematical classics refers to the family as Maharashtrian Brahmins residing on the
banks of the Godavari.
 Bhaskara II was the leader of a cosmic observatory at Ujjain, the main mathematical
centre of ancient India.
 Bhāskara and his works represent a significant contribution to mathematical and
astronomical knowledge in the 12th century.
 He has been called the greatest mathematician of medieval India.
 His main work Siddhanta-Siromaṇi, (Sanskrit for "Crown of Treatises") is divided into
four parts called
o Līlavati,
o Bijaganita,
o Grahagaṇita and
o Golādhyaya
 These four sections deal with ARITHMETIC, ALGEBRA, MATHEMATICS OF THE
PLANETS, and SPHERES respectively. He also wrote another treatise named Karaṇā
Kautūhala
Charaka

 Charaka was one of the principal contributors to Ayurveda, a system of medicine and
lifestyle developed in Ancient India → FOUNDER AND FATHER OF AYURVEDA
 Historical records suggest that he lived in the period from 100 BC to not beyond 150- 200
BC. The exact date, however, remains to be unknown till date. Charaka has been identified
as a native of Kashmir.
 He is known as an editor of the medical treatise entitled Charaka Samhita
 Along with the Sushruta Samhita, it is one of the two foundational texts of this field
that have survived from ancient India. It is one of the three
works that constitute the Brhat Trayi (The Great Triad)
o Third in the triad is Ashtanga Hridayam Samhita by
Vagbhata
 It describes ancient theories on human body, etiology,
symptomology and therapeutics for a wide range of diseases
 The Charaka Samhita also includes sections on the importance
of diet, hygiene, prevention, medical education, and the
teamwork of a physician, nurse and patient necessary for recovery to health
 According to Charaka Samhita, anything and everything in the universe is composed of

https://t.me/prelimbits
127

Vata, pitta, and Kapha. They are considered the force of nature in Ayurveda that helps us
understand the universe better.
Sushruta

 During the 6th century BCE, an Indian physician named Sushruta - widely regarded as the
'Father of Indian Medicine' and 'Father of Plastic Surgery' - wrote one of the world's
earliest works on medicine and surgery.
 Sushruta lived in the ancient city of Kashi, now known as Varanasi or Banaras in the
northern part of India.
Sushruta Samhita

 Sushruta is known for his pioneering operations and techniques and for his influential
treatise 'Sushruta Samhita' or Compendium of Sushruta, the main source of knowledge
about surgery in ancient India.
 Written in Sanskrit, the Sushruta Samhita dates back to the times before Christ and is
one of the earliest works in the field of medicine. It forms the foundations of the ancient
Hindu form of medicine known as Ayurveda and is highly regarded as one of the 'Great
Trilogy of Ayurvedic Medicine.'
o The Sushruta Samhita documented the etiology of more than 1,100 diseases, the use
of hundreds of medicinal plants, and instructions for performing scores of surgical
procedures - including three types of skin grafts and reconstruction of the nose.
 Skin grafts entail transplanting pieces of skin from one part of the body to another. Today,
surgeons use skin grafts to restore areas that have lost protective layers of tissue due to
trauma, infection, burns.
Origin of Rhinoplasty

 Rhinoplasty, colloquially known as the 'nose job,' is a surgery performed to achieve two
results:
o To improve the breathing function of the nose
o To improve the cosmetic look of the nose
 Sushruta's treatise provides the first written record of a forehead flap rhinoplasty, a
technique still used today to reconstruct a nose.

97. The earliest evidence of silver in India is found in the

A. Harappan culture
B. Chalcolithic cultures of Western India

https://t.me/prelimbits
128

C. Vedic texts
D. Silver punch-marked coins
Ans. A
Earliest Evidence Of Silver In India
 Metal technology attained great excellence with the mature Harappan civilization. The
earliest evidence of silver in India is found in the Harappan culture.
 One of the two silver seals that contain a unicorn motif has been found in Harappa.
 Evidence of a slanting flue has been discovered which suggests the presence of a furnace
which were suggested by vats Miller was used for metal crafting.
 Metals like copper, lead, tin, antimony were used for alloying.
 Harappan people wore exclusive jewellery made of silver and gold.
 Spacers made out of silver were discovered in the Kuntasi region. Bracelets, armlets and
necklaces were also made out of silver.

98. Which one of the following is the most fundamental difference between Mahayana
Buddhism and Hinayana Buddhism? [1994]

A. Emphasis on ahimsa
B. Castcless society
C. Worship of gods and goddesses
D. Worship of stupa
Ans. C
Refer Q. 91

99. Which one of the following usages was a Post Vedic development?

A. Dharma — Artha — Kama — Moksha


B. Brahmana — Kshatriya — Vaishya—Shudra
C. Brahmacharya—Grihastashrama— Vanaprastha — Sanyasa
D. Indra — Surya — Rudra — Marut
Ans. C
Asrama (Later Vedic Development)

 Asrama is a system of stages of life discussed in Hindu texts of the ancient and medieval
eras

 The four asramas are :


o Brahmacharya (student life – till 25)

https://t.me/prelimbits
129

o Gṛhastha (householder – 25-48)


o Vanaprastha (forest walker/forest dweller - 48–72)
o Sannyasa (renunciate - 72+ or anytime)
 The goal of each period was the fulfilment and development of the individual.
 The Asrama system is one facet of the Dharma concept in Hinduism.
 It is also a component of the ethical theories in Indian philosophy, where it is
combined with four proper goals of human life (Purushartha), for fulfilment,
happiness and spiritual liberation
o Purushartha → It is a key concept in Hinduism, and refers to the four proper goals or
aims of a human life. The four purusarthas are
 Dharma (righteousness, moral values),
 Artha (prosperity, economic values),
 Kama (pleasure, love, psychological values) and
 Moksha (liberation, spiritual values, self-actualization)
 Moreover, since the four asramas can be seen as the framework of an influential life-
span model, they are also part of an indigenous developmental psychology which from its
ancient beginnings until today has shaped the orientations and goals of many people,
especially in India.

100. In ancient Indian historical geography the name 'Ratnakara' denoted [1994]

A. the Arabian Sea


B. the Bay of Bengal
C. the Indian Ocean
D. the confluence of the Ganga, the Yamuna and the mythical Saraswati at Prayaga
Ans. C
 Since ancient times, the Sanskrit name for the Indian ocean has been 'Ratnakara,' - the
creator of gems - referring to the thousands of jewel-like volcanic islands and their mineral
rich soil which created a seemingly unending supply of precious gems.

101. In the Gandhara sculptures the preaching Mudra associated with the Buddha's First
Sermon at Samath is [94]

A. abhaya
B. dhyana
C. dharmachakra
D. bhumisparsa

https://t.me/prelimbits
130

Ans, C
Refer Q.12

102. The name of the poet Kalidasa is mentioned in the [94]

A. Allahabad pillar inscription


B. Aihole inscription
C. Alapadu grant
D. Hanumakonda inscription
Ans. B
AIHOLE INSCRIPTIONS [634 AD]

 Aihole in Karnataka was the Chalukyas' first capital.


 Many inscriptions were discovered at Aihole, but the inscription discovered at Meguti
Temple, also known as the Aihole inscription, witnessed many historical events during the
Chalukya period.
 The inscription is in Kannada script and written in Sanskrit.
 The defeat of Harshavardhana by Pulakeshin II and the victory of the Chalukyas over
the Pallavas are mentioned.
o Pulakeshin II defeated Harsha on the banks of Narmada in the winter of 618-619
CE. Pulakeshin entered into a treaty with Harsha, with the NARMADA RIVER
DESIGNATED AS THE BORDER BETWEEN THE CHALUKYA EMPIRE AND THAT OF
HARSHAVARDHANA.
 It also mentions the capital's relocation from Aihole to Badami.
 Bharavi, who is associated with the Pallavas of Kanchi along with Kalidasa is mentioned
in the Aihole Inscription.
 Ravikirti, the court poet of Pulakeshin II, who reigned from 610 to 642 CE, wrote them.
https://t.me/prelimbits
131

KAKATIYA DYNASTY (ANUMAKONDA INSCRIPTION)


 The Kakatiya dynasty ruled most of eastern Deccan region comprising present day
Telangana and Andhra Pradesh, and parts of eastern Karnataka and southern Odisha
between 12th and 14th centuries.

 Their capital was Orugallu, now known as Warangal.

 Early Kakatiya rulers served as feudatories to Rashtrakutas and Western Chalukyas for
more than two centuries.
 They assumed sovereignty under Prataparudra I in 1163 CE by suppressing other
Chalukya subordinates in the Telangana region.

 Ganapati Deva (r. 1199–1262) significantly expanded Kakatiya lands during the
1230s and brought under Kakatiya control the Telugu-speaking
lowland delta areas around the Godavari and Krishna rivers.
 Motupalli, now in Krishna district, was an important seaport in
Ganapati Deva reign.

 Ganapati Deva was succeeded by Rudrama Devi (r. 1262–1289)


who is one of the few queens in Indian history.

o Marco Polo, who visited India around 1289–1293, made


note of Rudrama Devi's rule and nature in flattering
terms. She successfully repelled the attacks of Yadavas
(Seuna) of Devagiri into the Kakatiyan territory.

 In 1303, Alauddin Khilji, the emperor of the Delhi Sultanate


invaded the Kakatiya territory which ended up as a disaster for
the Turks.
 But after the successful siege of Warangal in 1310, Prataparudra II was forced to pay
annual tribute to Delhi.
 Another attack by Ulugh Khan in 1323 saw stiff resistance by the Kakatiyan army, but they
were finally defeated.
 Kakatiya era the development of a distinct style of architecture which improved and
innovated upon the existing modes. Most notable examples are the Thousand Pillar
Temple in Hanamkonda, Ramappa Temple in Palampet, Warangal Fort, and Kota Gullu
in Ghanpur.
Prataparudra I
 The 1149 Sanigaram inscription of Prola II is the last known record of the Kakatiyas as
vassals.

 The 1163 Anumakonda inscription of Rudradeva alias Prataparudra I is the earliest


known record that describes the Kakatiyas as a sovereign power
ALLAHABAD PILLAR INSCRIPTION (PRAYAG PRASASTI)
https://t.me/prelimbits
132

 "Prayag" refers to a place where something or someone can meet. Prayag is an ancient
name for Allahabad because it is the confluence of the rivers Ganga, Yamuna, and the
mythical Saraswati.
 Prashasti is a eulogy that means "in praise of someone."
 The Allahabad Prasasti was first inscribed on the Ashokan Pillar in Kausambhi, near
Allahabad.
 It was later relocated to the Allahabad fort.
 It is an Ashokan Stambh, but it has four different inscriptions,
o The standard Ashokan inscriptions in Brahmi script are found on all pillars.
o The Queen's edict lauding Ashoka's wife Kaurwaki's charitable deeds.

o Inscriptions of Samudragupta (335–375 AD) written by Harisena in Sanskrit It


discusses Samudragupta's conquests and the boundaries of the Gupta Empire.
Inscriptions called him ―Dharma Prachar Bandhu”.

 Harisena, who was not only his court poet but was also a kumaramatya,
that is, a high ranking official who described himself by different titles such as
sandhivigrahika (minister for peace and war) and mahadandanayaka
(judicial/military officer).
o Jahangir‟s Persian inscriptions.

103. Which one of the following important trade centres of ancient India was on the
trade route connecting Kalyana with Vengi?

A. Tagara
B. Sripura
C. Tripuri
D. Tamralipti
Ans. A
TAGARA
 Ter, Maharashtra, a village in India, identified as the ancient Tagara trading centre
mentioned in the Periplus of the Erythraean Sea'.
 Tagara was the important trade centres of ancient India was on the trade route
connecting Kalyana with Vengi'
SIRPUR, MAHASAMUND
 Sirpur is a village in Mahasamund district in the state of Chhattisgarh 35 km from
Mahasamund city and 78 km away from Raipur on the banks of the river Mahanadi.
 Sirpur was capital of Panduvanshi Dynasty in ancient times.
 The village hosts the Sirpur Group of Monuments consisting of Buddhist, Hindu and Jain
https://t.me/prelimbits
133

temples and monasteries. These are dated from the 5th to 12th century.
 The site has been significant for its temple ruins of Rama and Lakshmana of the Ramayana
fame, as well as those related to Shaivism, Shaktism, Buddhism and Jainism.
 The location is mentioned in the memoirs of the Chinese traveler Xuanzang as a location
of monasteries and temples.
TAMRALIPTI
 Tamralipta or Tamralipti was a port city and capital of Suhma Kingdom in ancient Bengal,
located on the coast of the Bay of Bengal.
 The Tamluk town in present-day Purba Medinipur, West Bengal, is generally identified as
the site of Tamralipti.

 It was located near the Rupnarayan river.
 It gets its name from the Sanskrit term "Tāmra," or copper, which was mined nearby at
Ghatsila in the Singbhum region of the Chota Nagpur Plateau and traded through this
port.

 During the Gupta dynasty, Tamralipta was the main emporium, serving as a point of
departure for trade with Ceylon, Java, and China, as well as the west.
 It was linked by roads with the major towns of that time, i.e., Rajagriha, Shravasti,
Pataliputra, Varanasi, Champa, Kaushambi, and Taxila.

https://t.me/prelimbits
134

104. Toramana belonged to the ethnic horde of the

A. Scythians
B. Hunas
C. Yue-chis
D. Sakas
Ans. B
TORAMANA
 Toramana was a king of the Alchon Huns who ruled in northern India in the late 5th and
the early 6th century CE.
 Toramana consolidated the Hephthalite [white Hun] power in Punjab (present-day
Pakistan and northwestern India), and conquered northern and central India including
Eran in Madhya Pradesh. Toramana used the title "Great King of Kings"
 The Sanjeli inscription of Toramana speaks of his conquest and control over Malwa and
Gujarat. His territory also included Uttar Pradesh, Rajasthan and Kashmir

105. The word 'Hindu' as reference to the people of Hind (India) was first used by [1994]

A. the Greeks
B. the Romans
C. the Chinese
D. the Arabs
Ans. *
Well this kind of Q will never be asked again in UPSC level exam. Bcoz there are various theories
who used first ―this word‖
https://www.hinduscriptures.com/hindu-lifestyle/myths-about-india/origin-word-hindu/2213/
https://en.wikipedia.org/wiki/Hindus#Etymology
According to Gavin Flood [British scholar]
 The actual term 'hindu' first occurs, as "a Persian geographical term for the people who
lived beyond the river Indus (Sanskrit: Sindhu)", more specifically in the 6th-
century BCE inscription of Darius I.
 The Punjab region, called Sapta Sindhu in the Vedas, is called Hapta Hindu in Zend
Avesta.
 The 6th-century BCE inscription of Darius I [Persian ruler] mentions the province of
Hi[n]dush, referring to northwestern India
So if u wanna to attempt this Q then go with option D bcoz it is closer to geographical location

https://t.me/prelimbits
135

106. Who among the following was a Brahmavadini who composed some hymns of the
Vedas?

A. Lopamudra
B. Gargi
C. Leelavati
D. Savitri
Ans. *
Gargi was also Brahmavadini who composed hymns in Vedas.
Leelavati was also Brahmavadini
https://thesanatanchronicle.com/2020/08/30/brahmavadini-the-forgotten-tradition-of-women-
scholars-in-ancient-india/
https://en.wikipedia.org/wiki/G%C4%81rg%C4%AB_V%C4%81chaknav%C4%AB
https://dharmawiki.org/index.php/Brahmavadini_(%E0%A4%AC%E0%A5%8D%E0%A4%B0%E0%A4
%B9%E0%A5%8D%E0%A4%AE%E0%A4%B5%E0%A4%BE%E0%A4%A6%E0%A4%BF%E0%A4%A8%
E0%A5%80)#Rishikas
https://rashtram.org/bramahcharini/
BRAHMAVADINIS
 Brahmavadini was the title attributed to women scholars, who dedicated their lives to
the pursuit of knowledge and the study of the Vedas.
 Some were unmarried, living as ascetics and independent of their fathers, brothers or male
counterparts.
 They were paragons of intellectual proficiency, natural philosophy and spiritual
enlightenment.
 They were Rishikis–female sages–in their own right and were revered as teachers, doctors
and theorists.
 In the early Vedic period of Indian history, circa 1500-500 BC, women‟s education was
given equal priority, despite the dominant preference of begetting sons in every
household. A large number of young girls received both literary and cultural education.
 The Sacred Thread ceremony, called Upanayana, Janeau or Poita, was conferred on both
girls and boys of all castes at the age of seven.
 Post this ceremony, children were admitted to Vedic educational institutions also known
as Gurukuls. Many of these gurukuls practised co-education.
 However, with the advent of women teachers, called Upadhyayanis, there were also an
increasing number of educational institutions dedicated to girls pursuing higher education.
THE RIG VEDA CONTAINS HYMNS WRITTEN BY 27 WOMEN SCHOLARS. OF THESE, THE
PROMINENT BRAHMAVADINIS ARE LOPAMUDRA, GHOSHA, GARGI AND MAITREYI.
https://t.me/prelimbits
136

 Lopamudra was the wife of the sage Agastya. A hymn in the Rigveda is attributed to
her.

 Maitreyi, the wife of Yajnavalkya, is accredited with about ten hymns in Rig Veda.
 Gargi Vachaknavi
o Is honoured as a great natural philosopher in Vedic literature. She wrote many
hymns in the Rig Veda on the origin of existence. Due to the depth of her Vedic
expertise, she was frequently invited to debates with male philosophers of her time.
o On one such occasion, at the court of King Janak of Videha, she was the only
woman present in the assembly and one of the eight scholars who challenged the
great sage Yajnavalkya in a debate, as an equal participant.
o The quality and strength of her arguments almost overwhelmed Yajnavalkya–a feat
that no other scholar was able to achieve. Gargi was honoured as one of the
Navaratnas (nine gems) in the court of King Janak.
 Ghosha also called Mantradika, was a learned female philosopher and seer. She suffered
from leprosy and was confined to her father‘s home, where she attained most of her Vedic
knowledge.
o She wrote 2 hymns of 14 verses each, in the Rig Veda, praising the Ashwini Kumars
and expressing her desire for married life.
Other were : Vak Ambhrini, Sikta, Nivavari, Apala, Vishwawara, Sulabha , Urvashi , Saswati & so on
The Decline of the Brahmavadinis
 THE DECLINE OF THE BRAHMAVADINIS CAME WITH THE UNIVERSAL ACCEPTANCE
OF THE CODE OF MANU.
 In the later Vedic period, between 500BC to 200AD, Upanayana for girls was slowly
discontinued and Sadyovadhus [woman who marry at early age] were given more
importance.
 This turn of events resulted in the decline of the Brahmavadinis. Women lost their
independence. Their marriageable age was lowered from 16 to 9 years. They were
denied the right to Vedic education and confined to their home.
 In 200 AD, the code of Manu, called Manusmriti, was composed. It sealed the fate of
Indian women. According to Manu, a woman must always be under the protective
sphere of the men in her life – her father, husband and son. All the duties, that were listed
in the code, were against the independence and education of women.

107. In Sanskrit plays written during the Gupta Period women and sudras speak [1995]

A. Sanskrit
B. Prakrit
C. Pali
https://t.me/prelimbits
137

D. Sauraseni
Ans. B
GUPTA PERIOD : RELIGION

 The period 300–600 CE is often considered as the phase of consolidation of


Brahmanical ideology as temple-based sectarian cults became popular and
Sanskrit was firmly established as the language of royal inscriptions. [So Sanskrit was
official Language, Common people Such as woman & Sudra Speak Prakrit]

 Sectarian symbols such as the linga, trishula, bull, Gaja-lakshmi, chakra, and shankha (conch
shell) appeared on the royal prashastis, coins, and seals. Hinduism acquired its present
shape Gupta age only.
 The Gupta rulers gave patronage to Bhagvatism, which was centred around the worship
of Bhagvat or Vishnu and his incarnations.
o So Bhagvatism was also identified with Vaishnavism. It put emphasis on bhakti
(loving devotion) and ahimsa (non-killing of animals, which is why it stresses more
on vegetarianism) rather than Vedic rituals and sacrifices. This new religion→
o Was quite liberal, and assimilated the lower classes in its fold.
o By the end of the Gupta period, Mahayana Buddhism was overshadowed by
Bhagavatism and Vishnu became a member of the trinity of gods along with Shiva
and Brahma.
o Idol worship became a common feature and the idols of different incarnations of
Vishnu were housed in the temples constructed in the Gupta period.
 There was also an emergence of worship of composite deities such as Hari-hara, who is a
part Vishnu and part Shiva, and religious syncretism (for instance, the incorporation of
the Buddha as one of the ten Vishnu avatars).
 This phase also saw the competitive relationship between various cults, & myriad of
local cults, beliefs, and practices.
 The Gupta kings were devout Hindus, but they followed a policy of tolerance towards
other religious sects.
 The Chinese pilgrims Fa-Hien and Hiuen Tsang, who came to India during the reign of
Chandragupta II and Harsha respectively, clearly give the impression that Buddhism
was also flourishing. Though it is also true that Buddhism no longer received royal
patronage. However→
o Some stupas and viharas were constructed and Nalanda developed as a great
centre of education for Mahayana Buddhism during his time. Students from other
countries also came to study in this university. According to Hiuen Tsang, the
revenues of one hundred villages supported it.

Fahien / Fa-Hien account on Gupta

https://t.me/prelimbits
138

 Famous Chinese pilgrim, Fahien visited India during the reign of Chandragupta II.
 Out of his nine years stay in India, he spent six years in the Gupta empire.
 He came to India by the land route through Khotan, Kashgar, Gandhara and Punjab.
 He visited Peshawar, Mathura, Kanauj, Sravasti, Kapilavastu, Kusinagara, Pataliputra, Kasi and
Bodh Gaya among other places.
 He returned by the sea route, visiting on the way Ceylon and Java.
 The main purpose of his visit was to see the land of the Buddha and to collect Buddhist
manuscripts from India. He stayed in Pataliputra for three years studying Sanskrit and
copying Buddhist texts.
 Fahien provides valuable information on the religious, social and economic condition
of the Gupta empire.
 According to him, Buddhism was in a flourishing condition in the north-western India
but in the Gangetic valley it was in a state of neglect.
 He refers to the Gangetic valley as the „land of Brahmanism‟.
 Fahien mentions the unsatisfactory state of some of the Buddhist holy places like
Kapilavastu and Kusinagara. According to him the economic condition of the empire was
prosperous.
 He was not interested in political affairs. His interest was primarily religion.

108. The name by which Asoka is generally referred to in his inscriptions is

A. Chakravarti
B. Dharmadeva
C. Dharmakirti
D. Priyadarsi
Ans. D
 "Devanampriya" means "Beloved of the Gods". It is often used by Ashoka in conjunction
with the title Priyadasi, which means "He who regards others with kindness".
MASKI INSCRIPTION

 Maski is a village and archaeological site in Karnataka's Raichur district. It is located on


the banks of the Maski River, a tributary of the Tungabhadra.
 A minor rock edict of Emperor Ashoka can be found at the site.

 It was Emperor Ashoka's first edict that included the name Ashoka rather than
'Devanampriya' or 'Piyadassi.'

 The inscription, which is still a dharma shasana, instructs people to follow the tenets of
Buddhism.
 Furthermore, the inscription suggests that Mauryan rule extended all the way to the

https://t.me/prelimbits
139

Krishna valley in north-eastern Karnataka.

109. The term 'Yavanapriya' mentioned in ancient Sanskrit texts denoted: [1995]

A. A fine variety of Indian Muslim


B. Ivory
C. Damsels sent to the Greek court for a dance performance
D. Pepper
Ans. D
Sangam Age : Economy

 The region that Sangam Literature mentions was prosperous and agriculture, industry
and trade flourished there.
 People were pastoralists, hunter, fishermen although they also produced rice.
 Items Of Export: Maslin, glass beeds, pearls, sandalwood, perfumes, black pepper
(Yavanapriya), tortoise shell, medicines, animals and birds.

 Items Of Imports: Gold, Silver, Munga, wine, olive oil dry fruits, raw glass, ivory, copper,
tin, medicine and slaves.
o Gold and Silver in huge quantities were brought into India and made the trade
greatly in favour of India.
o Pliny, the Roman author and member of senate, in his Naturalis Historia (77CE)
regrets the drain out of huge amounts of gold and silver to India.
o Ptolemy in his Geographia (Geography) and Strabo in his Geographica
(Geography) also describe this trade imbalance of Roman empire with India.
 Discovery of Mansoon winds by Hippalus around 45-47 CE further promoted the
trade between India and the west

110. The concept of Anuvrata was advocated by

A. Mahayana Buddhism
B. Hinayana Buddhism
C. Jainism
D. the Lokayata school
Ans. C
FIVE PRINCIPLES OF JAINISM

 These are called Anuvratas (small vows) for Jain laypersons, and Mahavratas (great vows)
for Jain mendicants

o Non Violence (Ahimsa)

https://t.me/prelimbits
140

o No lies (Satya)

o No stealing (Asteya)

o No Property (Aparigraha)

o Observing Continence (Brahmacharya).

 The fifth principle (Brahmacharya) was added by Mahavira and the other four were the
teachings of his Predecessors. Mahavira was the 24th Tirthankara of Jain.

 Jainism prescribes seven supplementary vows, including three guņa vratas (merit vows)
and four siksa vratas.

The Sallekhana (or Santhara) vow is a "religious death" ritual observed at the end of life,
historically by Jain monks and nuns, but rare in the modern age. In this vow, there is voluntary
and gradual reduction of food and liquid intake to end one's life by choice and with
dispassion. This is believed to reduce negative karma that affects a soul's future rebirths

111. According to Mimamsa system of philosophy liberation is possible by means of [1995]

A. Jnana
B. Bhakti
C. Yoga
D. Karma
Ans. D
Purva Mimamsa / Mimamsa Sutra / Karma-Mimamsa (by Jaimini)
 It places emphasis on the power of yajnas and mantras in sustaining the activities of the
universe.
 This school of thought believes in complete authority of Vedas.
 He and later proponents of Karma-mimamsa philosophy teach that material existence is
endless, that there is NO LIBERATION.
 For Mimamsas, the cycle of karma is perpetual, and the best one can aim for is higher
birth among the Devas [god].

 Therefore, they hold that the whole purpose of the Vedas is to engage human
beings in rituals for Creating Good Karma, and consequently the mature soul's
prime responsibility is to ascertain the exact meaning of the Vedas' sacrificial
injunctions and to execute them.
Other Orthodox Schools of Indian Philosophy
 Samkhya (Kapila)
 Yoga (Pata njali)
 Nyaya (Gautama Muni)
https://t.me/prelimbits
141

 Vaisheshika (Kanada)
 Uttara Mimamsa (Vedanta)

112. The river most mentioned in early Vedic literature is

A. Sindhu
B. Sutudri
C. Sarasvati
D. Ganga
Ans. A
 The earliest Aryans lived in the land of the (Sapta Sindhu) (Sindhu is the river par
excellence of the Aryans) and probably because of their use of horse chariots and superior
military technology, they could establish their political dominance in the region.

 The region was drained by the seven rivers →

o Indus [most discussed] (Sindhu), and its five tributaries, namely →

 Jhelum (Vatista)

 Beas (Vipasa)

 Chenab (Askini)

 Ravi (Purushni)

 Sutlej (Sutudri), and the

o Saraswati (modern Ghaggar Hakra), which covered mostly the areas of eastern
Afghanistan, Punjab, and fringes of western U.P == > Most Prominent River

 The rare mention of the river Yamuna (twice) and Ganga (only once) is pointer to the fact
that Rig Vedic Aryans had not inhabited that region as yet.

113. Which one of the following is not a part of early Jaina literature? [96]

A. Therigatha
B. Acarangasutra
C. Sutrakritanga
D. Brihatkalpasutra
Ans. A
Therigatha (Buddhist Text)
 Often translated as Verses of the Elder Nuns.
 is a Buddhist text, a collection of short poems of early enlightened women who were
elder nuns (having experienced 10 Vassa or monsoon periods).
https://t.me/prelimbits
142

 In the Pali Canon, the Therigatha is classified as part of the Khuddaka Nikaya, the
collection of short books in the Sutta Pitaka. It consists of 73 poems organized into 16
chapters. It is the companion text to the Theragatha, verses attributed to senior monks.
Religion after Mahavira

 Mahavira had a huge following. In the early stages, his followers were drawn from
different sections of the society.
 However, in course of time, Jainism was confined to the trading and money-lending
community.
 Jainism‘s insistence on non-violence closed other occupations, including agriculture, as
it prescribed refraining from intended or unintended killing.
 About 500 years after Mahavira‟s death, in about 79 or 82 CE, schism occurred in
Jainism.
 Magadha was affected by severe famine and some of the Jaina monks under
Bhadrabahu went south to maintain their strict discipline. They remained without
garments and were known as Digambaras (space-clad or naked).
 Others stayed back under the leadership of Sthulabhadra and adopted a white garment
and were known as Svetambaras (white-clad).
 On the death of Bhadrabahu, Shulabhadra held a Great Council at Pataliputra, which
compiled the Jaina canon.
o It consisted of 12 angas (limbs).
 Another council was held in Valabhi, Gujarat, in the 5th century CE. It added 12 upangas
(minor sections).
 The Jaina monks not only wrote religious treatises but also promoted secular literature.
 Earliest Jaina texts →
o Acharanga Sutra,
 Is the first of the twelve Angas, part of the agamas (religious texts) which
were compiled based on the teachings of 24th Jina Mahavira.
 The existing text of the Acharanga Sutra which is used by the Svetambara
sect of Jainism was recompiled and edited by Kshama Shraman
Devardhigani, who headed the council held at Valabhi c. 454 CE.
o Sutrakritanga
 is the second agama of the 12 main aṅgās of the Jain Svetambara canon.
According to the Svetambara tradition it was written by Gandhara
Sudharmasvami in Ardhamagadhi Prakrit.
o Kalpasutra
 The Kalpa Sutra is a Jain text containing the biographies of the Jain
Tirthankaras, notably Parshvanatha and Mahavira.
https://t.me/prelimbits
143

 Traditionally ascribed to Bhadrabahu, which would place it in the 4th


century BCE
 It was probably put in writing 980 or 993 years after the Nirvana (Moksha) of
Mahavira.
 Most of the early Jaina texts were written in Ardha-Magadhi, the language of the common
people.

114. Which of the following were common to both Buddhism and Jainism? [96]

A. Avoidance of extremities of penance and enjoyment


B. Indifference to the authority of the Vedas
C. Denial of efficacy of rituals
D. Non-injury to animal life
Select the correct answer using the codes given below:
A. 1, 2, 3 and 4
B. 2, 3 and 4
C. 1,3 and 4
D. 1 and 2
Ans. B
Both Buddhism and Jainism had some similarities in their teachings

 For instance, both rejected the authority of Vedas, emphasised on renunciation and
human effort as the means of obtaining salvation, and established a monastic order
for both men and women.

 Like Buddhism, Jainism is fundamentally atheistic; though it recognises the existence of


gods, yet it refuses to give them importance in the universal scheme of themes and
places the gods lower than Jina (the conqueror).

 Like Buddhist texts, Jaina texts criticise the Brahmanas, their practice of animal
sacrifices (non-violence), and ritual authority giving preference to Brahmanas.

 Both Jainism and Buddhism focus on the superiority of the Kshatriya varna over all
other varnas including Brahmanas.

 They both attempted to give a new connotation to the meaning of Brahmana, shifting
the emphasis from an ascribed status to one achieved by good deeds.

o They use the word „Brahmana‟ in the sense of acknowledging a wise person who
possesses true knowledge and lives an exemplary life.

 Like Buddhism monastic order, people of all castes and social backgrounds were
welcomed in Jainism.

https://t.me/prelimbits
144

o There are frequent mentions of a learned Jaina monk named Harikeshiya who
belonged to a Chandala family.

o Brahmana varna was represented by Bhadrabahu, Divakara, Jinasena, and


Haribhadra.

o Similarly, Jainism opened its doors to women mendicants who were addressed as
aryika or sadhvi.

Unique concept of Non-Violence (Not Found in Buddhism)

 The five vows are common to both the monks and lay followers. But The monks were to
observe the vows more rigorously than the lay followers.
 As Jainism placed great emphasis on non-violence, strict observers of the faith wear a
muslin cloth around their mouth and nose so that they would not inhale small insects
even by mistake.
 To avoid trampling on ants and other insects, Jain monks used feathers to sweep the
path before walking.
 Jains could not practice agriculture or other crafts that involve killing or injury to living
organisms. Hence they took to trading and money-lending and excelled in it. As a result,
they were closely associated with urbanisation.
Jainism is an egalitarian religion. It does not sanction any inequality based on birth. It is one‘s
deeds that determine one‘s status in society and not birth. Jainism believes that “by one‟s action
one becomes a Brahmin, a Kshatriya, or a Vaishya, or a Sudra.” Pride based on birth is
considered a sin. Women were admitted into the monastic order. However, as a woman one
cannot attain salvation. By accumulating merit by good deeds, a woman could be reborn as a
man and then strive to attain salvation

115. Who among the following is known for his work on medicine during the Gupta
period? [96]

A. Saumilla
B. Sudraka
C. Shaunaka
D. Susrutha
Ans. D
SUSHRUTA

 During the 6th century BCE, an Indian physician named Sushruta - widely regarded as the
https://t.me/prelimbits
145

'Father of Indian Medicine' and 'Father of Plastic Surgery' - wrote one of the world's
earliest works on medicine and surgery.
 Sushruta lived in the ancient city of Kashi, now known as Varanasi or Banaras in the
northern part of India.
Sushruta Samhita

 Sushruta is known for his pioneering operations and techniques and for his influential
treatise 'Sushruta Samhita' or Compendium of Sushruta, the main source of knowledge
about surgery in ancient India.
 Written in Sanskrit, the Sushruta Samhita dates back to the times before Christ and is
one of the earliest works in the field of medicine. It forms the foundations of the ancient
Hindu form of medicine known as Ayurveda and is highly regarded as one of the 'Great
Trilogy of Ayurvedic Medicine.'
o The Sushruta Samhita documented the etiology of more than 1,100 diseases, the use
of hundreds of medicinal plants, and instructions for performing scores of surgical
procedures - including three types of skin grafts and reconstruction of the nose.
 Skin grafts entail transplanting pieces of skin from one part of the body to another. Today,
surgeons use skin grafts to restore areas that have lost protective layers of tissue due to
trauma, infection, burns.
Origin of Rhinoplasty

 Rhinoplasty, colloquially known as the 'nose job,' is a surgery performed to achieve two
results:
o To improve the breathing function of the nose
o To improve the cosmetic look of the nose
 Sushruta's treatise provides the first written record of a forehead flap rhinoplasty, a
technique still used today to reconstruct a nose.
SHAUNAKA
 is the name applied to teachers, and to a Shakha of the Atharvaveda.
 It is especially the name of a celebrated Sanskrit grammarian, author of the Ṛgveda-
Prātiśākhya, the Bṛhaddevatā, the Caraṇa-vyūha and six Anukramaṇīs (indices) to the
Rigveda.
 He is claimed as the teacher of Katyayana and especially
 He is said to have united the Bashkala and Shakala Shakhas of the Rigveda.
 In legend, he is sometimes identified with Gritsamada, a Vedic rishi.
SHUDRAKA

 Sanskrit Playwright during Gupta Period


 Work:

https://t.me/prelimbits
146

o Mrichchhkatikam (Little Clay Cart), which deals with the love affair of a poor
brahmana with the beautiful daughter of a courtesan. (asked in CSE)
o Vinavasavadatta
o Bhana (short one-act monologue)
o Padmaprabhritaka
SAUMILLA
Not able to find anything about him. He is though mentioned in kalidasa play Malavikagnimitram
 Kalidasa wrote: "Shall we neglect the works of such illustrious authors as Bhasa, Saumilla,
and Kaviputra? Can the audience feel any respect for the work of a modern poet, a
Kalidasa?‖

116. In the context of ancient Indian society, which one of the following terms does not
belong to the category of the other three?

A. Kula
B. Vamsa
C. Kosa
D. Gotra
Ans. C
Vaṃsa
 Vaṃsa is a Sanskrit word that signifies "family, lineage.
 This spotlights family histories.
 Two normal legendary genealogies are called Surya-Vamsi and Chandra-vamsa, sun-
powered (child based) and lunar (girl based) heredities of lords, families, and communities.
Additional two more up to date regal vamsha being, Agni-vamsa and Naga-vamsa.
 The Harivamsa is the incredible ancestry and story of the Hindu god Krishna.
Gotra
 In Hindu culture, the term gotra is viewed as comparable to ancestry.
 It comprehensively alludes to individuals who are relatives in a solid male line from a typical
male precursor or patrilineal. By and large, the gotra structures an exogamous unit, with the
marriage inside the equivalent gotra being restricted by custom, being viewed as
incest.
Kul
 Kul depicts the patrilocal nuclear family, regularly comprising three ages who live
respectively in a compound headed by the grandfather or his oldest child, into which
the ladies of the different ages are ingested. The family holds its property in like manner, as
a division of assets is customarily disliked.

https://t.me/prelimbits
147

Kosa
 Kosa was utilized for depository and the remaining of the three terms were identified with
family.

117. Match List I with List II and select the correct answer by using the codes given
below the lists

List I List II

A. Visakhadatta 1. Medicine

B. Varahamihira 2. Drama

C. Charaka 3. Astronomy

D. Brahmagupta 4. Mathematic

Codes

A B C D

a) 3 4 1 2

b) 2 4 1 3

c) 2 3 1 4

d) 3 1 2 4

Ans. C
VISHAKHADATTA

 Sanskrit poet and playwright


 Court poet of Chandragupta II
 Work :
o Mudra Rakshas : Narrates the ascent of Chandragupta Maurya to throne.
o Devichandraguptam : The play, king Ramagupta decides to surrender his queen
Dhruvadevi ("Devi") to a Shaka enemy when besieged. Ramagupta's younger
brother Chandragupta enters the enemy camp disguised as the queen, and kills the
enemy ruler.
CHARAKA

 Charaka was one of the principal contributors to Ayurveda, a system of medicine and
lifestyle developed in Ancient India → FOUNDER AND FATHER OF AYURVEDA
 Historical records suggest that he lived in the period from 100 BC to not beyond 150- 200
BC. The exact date, however, remains to be unknown till date. Charaka has been identified
https://t.me/prelimbits
148

as a native of Kashmir.
 He is known as an editor of the medical treatise entitled Charaka Samhita
 Along with the Sushruta Samhita, it is one of the two foundational texts of this field
that have survived from ancient India. It is one of the three
works that constitute the Brhat Trayi (The Great Triad)
o Third in the triad is Ashtanga Hridayam Samhita by
Vagbhata
 It describes ancient theories on human body, etiology,
symptomology and therapeutics for a wide range of diseases
 The Charaka Samhita also includes sections on the importance
of diet, hygiene, prevention, medical education, and the
teamwork of a physician, nurse and patient necessary for recovery to health
 According to Charaka Samhita, anything and everything in the universe is composed of
Vata, pitta, and Kapha. They are considered the force of nature in Ayurveda that helps us
understand the universe better.
VARAHAMIHIRA

 A 6th [or older] century astronomer, astrologer, mathematician, and scholar, who was one of
the navratnas of Chandragupta II and belonged to Avanti.
o Some believes him to be one of the "Nine Jewels" (Navaratnas) of the court of ruler
Yashodharman Vikramaditya (but not clear)
 His famous work:
o Panchasiddhantika summarized five astronomical schools.
o Brihatsamhita is an encyclopaedic work dealing with diverse topics.
o Brihat Jataka
BRAHMAGUPTA

 Late 6th/7th century astronomer and wellknown mathematician who made important
contributions to geometry.
 He was the first mathematician to discuss the method of obtaining a Cyclic Quadrilateral
 He also wrote:
o Brahmasputasiddhanta (628 CE), in which he hinted at the Law of Gravitation
o Khandakhadyaka (665 CE), covering such topics as the longitudes of the planets,
diurnal rotation, lunar and solar eclipses, rising and setting of the sun, the moon‘s
crescent, and conjunctions of the planets.

https://t.me/prelimbits
149

118. Which one of the following texts of ancient India allows divorce to a wife deserted
by her husband? [96]

A. Kamasutra
B. Manavadharmashastra
C. Sukra Nitisara
D. Arthashastra
Ans. D
ARTHASHASTRA ON GENDER ISSUES
https://sourcebooks.fordham.edu/india/kautilya2.asp
 Women, when twelve years old, attain their majority, and men when sixteen years old. If,
after attaining their majority, they prove disobedient to lawful authority, women shall be
fined fifteen panas, and men twice the amount.
 A woman who has a right to claim maintenance for an unlimited period of time shall be
given as much food and clothing as is necessary for her, or more than is necessary in
proportion to the income of her maintainer. . .
 Women of refractive natures shall not be taught manners by using such expressions
as "You, half-naked!; you, fully-naked; you, cripple; you, fatherless; you, motherless." Nor
shall she be given more than three beats, either with a bamboo bark or with a rope or with
the palm of the hand, on her hips.
 Violation of the above rules shall be liable to half the punishment levied for defamation and
criminal hurt. The same kind of punishments shall be meted out to a woman who, moved
with jealousy or hatred, shows cruelty to her husband. .
 A woman who hates her husband, who has passed the period of seven turns of her
menses, and who loves another, shall immediately return to her husband both the
endowment and jewelry she has received from him, and allow him to lie down with
another woman. A man, hating his wife, shall allow her to take shelter in the house of a
beggar woman, or of her lawful guardians or of her kinsmen. . .
 A woman, hating her husband, cannot divorce her husband against his will. Nor can a
man divorce his wife against her will. But from mutual enmity divorce may be obtained. . .
KAMA SUTRA

 is an ancient Indian Sanskrit text on sexuality, eroticism and emotional fulfillment in life.
Attributed to Vātsyāyana,

 Kama Sutra is neither exclusively nor predominantly a sex manual on sex positions, but
rather was written as a guide to the art of living well, the nature of love, finding a life
partner, maintaining one's love life, and other aspects pertaining to pleasure-oriented
faculties of human life.

Vātsyāyana
https://t.me/prelimbits
150

 Vātsyāyana was an ancient Indian philosopher, known for authoring the Kama Sutra.
o Hardly anything is known about Vātsyāyana from sources outside
the Kāmasūtra itself. Vātsyāyana's interest in refined human, including sexual,
behavior as a means of fulfilment, was recorded in his treatise Kama Sutra.
 He was a brahmin, and lived in India during the second or third century CE, probably in
Pataliputra (modern day Patna in Bihar).
 He is not to be confused with Pakṣilasvāmin Vātsyāyana, the author of Nyāya Sutra
Bhāshya, the first preserved commentary on Gotama's Nyāya Sutras.
 His Name is sometimes erroneously confused with Mallanaga, the seer of the Asuras, to
whom the origin of erotic science is attributed

119. Match List I with List II and select the correct answer by using the codes given
below the lists :

List I List II

A. Gupta 1. Badami

B. Chandella 2. Panamalai

C. Chalukya 3. Khajuraho

D. Pallava 4. Deogarh

Codes

A B C D

a) 3 4 1 2

b) 2 4 1 3

c) 2 3 1 4

d) 4 3 1 2

PANAMALAI
 Panamalai lies 23 kilometers from Gingee, Vilupuram in Tamil Nadu, India.
 The site is known as a location to various ancient structural temples built during the
Pallava dynasty. One of them is the Talagirisvara Temple.
 Narasimhavarman II, also known as Rajasimha or Rajamalla, is credited with constructing
structural temples of Pallava dynasty namely the Shore Temple at Mamallapuram,
Kailsanatha Temple and Talagirisvara temple at Panamalai.
CHALUKYA DYNASTY

https://t.me/prelimbits
151

 The Chalukya dynasty was a Classical Indian dynasty that ruled large parts of southern and
central India between the 6th and the 12th centuries.

 During this period, they ruled as three related yet individual dynasties.

 The earliest dynasty, known as the "Badami Chalukyas", ruled from Vatapi (modern
Badami) from the middle of the 6th century.

o The Badami Chalukyas began to assert their independence at the decline of the
Kadamba kingdom of Banavasi and rapidly rose to prominence during the reign of
Pulakeshin II.

 After the death of Pulakeshin II, the Eastern Chalukyas became an independent
kingdom in the eastern Deccan.

o They ruled from Vengi until about the 11th century.

 In the western Deccan, the rise of the Rashtrakutas in the middle of the 8th century
eclipsed the Chalukyas of Badami before being revived by their descendants, the
Western Chalukyas, in the late 10th century.
o These Western Chalukyas ruled from Kalyani (modern Basavakalyan) until the end
of the 12th century.

 The rule of the Chalukyas marks an important milestone in the history of South India and a
golden age in the history of Karnataka.
 The political atmosphere in South India shifted from smaller kingdoms to large empires
with the ascendancy of Badami Chalukyas.

 A Southern India-based kingdom took control and consolidated the entire region
between the Kaveri and the Narmada rivers.
 The rise of this empire saw the birth of efficient administration, overseas trade and
commerce and the development of new style of architecture called "CHALUKYAN
ARCHITECTURE".

 Kannada literature, which had enjoyed royal support in the 9th century Rashtrakuta
court found eager patronage from the Western Chalukyas in the Jain and Veerashaiva
traditions.

 The 11th century saw the patronage of Telugu literature under the Eastern Chalukyas.

KHAJURAHO TEMPLE → MADHYA PRADESH

 They are a group of temples built by the rulers of Chandela Dynasty (950-1050 AD).
 Hindu and Jain temples.
 Famous for nagara style architecture and known for extensive Erotic
Sculptures.

https://t.me/prelimbits
152

 UNESCO World Heritage Site.


 The first recorded mention of the Khajuraho temples is in the accounts of Abu Rihan al
Biruni in AD 1022 and the Arab 152traveller Ibn Battuta in AD 1335.
DASHAVATARA TEMPLE, DEOGARH
 The Dashavatara Temple is an early 6th century Vishnu Hindu temple located at Deogarh,
Uttar Pradesh which is 125 kilometers from Jhansi, in the Betwa River valley in northern-
central India.
 It has a simple, one cell square plan and is one of the earliest Hindu stone temples still
surviving today
 Built in the Gupta Period, the Dashavatara Temple at Deogarh shows the ornate Gupta
style architecture.

120. Which one of the following statements regarding Asokan stone pillars is incorrect?
[1997]

A. These are highly polished


B. These are monolithic
C. The shaft of pillars is tapering in shape
D. These are parts of architectural structures
Ans. D
PILLAR EDICTS

 Pillar edicts have been found at Lauriya-Araraj, Lauriya Nandangarh,


Rampurva, Nigali Sagar, Sarnath, Topra, and Meerut.
 Two pillars, one from Topra and the other from Meerut, were shifted to
Delhi by Feroz Shah Tughlaq.
 There are pillars without inscriptions such as the bull capital pillar at
Rampurva, the pillar with the lion capital at Vaishali, and the Kolam pillar,
which is also without a capital.
Features
 Mostly, the pillars are similar in form and dimensions, and are made of
sandstone quarried from Chunar.

[Two types of stones are used: Spotted White Sandstone (from Mathura) and Buff Coloured
Sandstone and quartzite (from Amaravati). Generally, they are made of sandstone quarried from
Chunar. ]

 Mostly monoliths (i.e., carved out of a single piece of stone) and have a lustrous, polished
surface

https://t.me/prelimbits
153

 Smooth cylindrical shaft tapers slightly upwards to a height of 12-14m. A cylindrical bolt
joins the top of the shaft to the capital, and have a bell capital (a stone carved in the shape
of an inverted lotus).
 On top of bell capital is a platform (abacus) which is
intended to support the crowned animal.
 Ashoka stone pillars were meant to spread his Dhamma
(not the part of architectural structure.)
Motifs in Pillers of Ashoka
 Most found motifs were of LOTUS (symbolised, purity
and fecundity [fertility] in Indian tradition),
 LION (is a solar symbol and represents the Buddha too,
as he is referred to as Sakyasimha, i.e., lion among the
Sakyas), bull (symbol of fertility),
 ELEPHANT (relevant in Buddhism as according to Buddhist tradition, Buddha entered his
mother‘s womb in the form of a white elephant),
 SPOKED WHEEL (represents dharmachakra, i.e., the wheel of dharma and also associated
with sovereignty),
 HORSE (symbolises departure of Siddhartha from his home).
Languages used

 In eastern part of the empire, Magadhi language in Brahmi script is used. (Magadhi is the
dialect of Prakrit found in Magadha).
 In western parts of the Mauryan Empire, Prakrit in kharoshti script is used.
 Major Rock Edict XIII contains an extract in Greek and Aramaic as well.

121. Which one of the following edicts mentions the personal name of Asoka? [97]

A. Kalsi
B. Rummindei
C. Special Kalinga Edict
D. Maski
Ans. D
MASKI INSCRIPTION

 Maski is a village and archaeological site in Karnataka's Raichur district. It is located on


the banks of the Maski River, a tributary of the Tungabhadra.
 A minor rock edict of Emperor Ashoka can be found at the
site.

https://t.me/prelimbits
154

 It was Emperor Ashoka's first edict that included the name Ashoka rather than
'Devanampriya' or 'Piyadassi.'

 The inscription, which is still a dharma shasana, instructs people to follow the tenets of
Buddhism.
 Furthermore, the inscription suggests that Mauryan rule extended all the way to the
Krishna valley in north-eastern Karnataka.
RUMMINDEI PILLAR EDICTS (LUMBINI)

 These inscriptions are classified as Minor Pillar Inscriptions. These have inscriptions that
record their dedication.
 The inscriptions mention Ashoka's visit to Lumbini (Rummindei), Rupandehi district,
Nepal, Lord Buddha's birthplace.
 Ashoka exempted Lumbini from paying taxes [bali] and set its grain contribution at one-
eighth.
 Brahmi script is used to write the inscriptions.
2 SPECIAL KALINGA EDICT (ODISHA)

 The Kalinga War was a watershed moment in Ashoka's career, as he not only abandoned
his ambition of becoming "Digvijaya," but also chose the path of non-violence and
Buddhism.
 1st at Dhauli (ancient name Toshali) and 2nd at Jaugada (earlier known as Samapa. These
new Kalinga rock edicts were different from other edicts of Ashoka.
o Mentions ‗All men are my children.‘
 The edicts are written in Prakrit with an early Brahmi script
 Sannati Inscription (Karnataka) : Site of all 14 major rock edicts as well as two separate
Kalinga edicts
KALSI INSCRIPTION [UK]

 Kalsi is well-known for its historical significance. It is a small town on


the banks of the Yamuna River located between Chakrata and
Dehradun.
 The site of Ashoka's inscriptions at Kalsi is unique in that it is the only
place in North India where the great Mauryan emperor inscribed the entire set
of fourteen rock edicts.
 The language and script of these edicts are Prakrit and Brahmi, respectively.
 The Rock edict, which is 8 feet wide and 10 feet long and made of quartz, was written
around 450 B.C.
 When Asoka converted to Buddhism, he used a human approach in his internal
administration, which is reflected in these rock edicts.

https://t.me/prelimbits
155

 It also reflects policies promoting nonviolence and the limitation of war.


 The inscription also describes his life before he chose the spiritual path.

122. In the Mahayana Buddhism, the Bodhisattva Avalokitesvara was also known as

A. Vajrapani

B. Manjusri

C. Padmapani

D. Maitreya

Ans. C

PROMINENT BODDHISATTVAS UNDER BUDDHISM

In Mahayana Buddhism, a Bodhisattva is a person who is able to reach nirvana but delays
doing so through compassion for suffering beings.
Avalokitesvara (Buddha‟s Compassion)

 One of Buddha's three protector deities, also known as Padmapani and is described as
carrying a Lotus Flower.
 His picture, which can be found in the Ajanta Caves, is the most well-known of all the
Bodhisattvas.
 The compassionate bodhisattva listens to the world's pleas and responds in skillful ways.
 Under the name Lokesvara, he appears informally in Cambodian Theravada Buddhism. He
is also shown as a woman, and His Holiness The Dalai Lama is thought to be his incarnation.

Vajrapani ( Buddha‟s Power)

 Vajrapani is one of three guardian deities ( others are Manjusri and Avlokiteshwara.
Vajrapani ) that surround Buddha and is represented in the Ajanta Caves.
 He is said to express all of Buddha's powers as well as the powers of the five tathagatas-
Vairocana, Akshobhya, Amitabha, Ratnasambhava, and Amoghasiddhi.

https://t.me/prelimbits
156

Manjusri (Buddha‟s Wisdom)

 Manjusri is one of three protecting deities that surround Buddha and is also represented in
the Ajanta Caves.
 He is a man Bodhisattva with a sword in his hand who is linked with Buddha's Wisdom.

Finally, Avalokitesvara embodies Buddha's compassion, Vajrapani embodies Buddha's might, and
Manjusri ponders Buddha's knowledge.

Samantabhadra (Buddha‟s Aspirations)

 The meaning of Samantabhadra is ‗Universal Worthy‘ and he is associated with practice


and meditation. He made ten vows to fulfil his journey of becoming a Buddha.
 In Buddhism, he is part of the Shakyamuni triad, which also includes the Buddha and
Manjusri.
Ksitigarbha (Buddha‟s Merit)

 He is represented as a Buddhist monk who vowed not to attain Buddhahood until hell
was totally empty.
Maitreya (Buddha‟s Activity)

 Aka. Ajita Boddhisattva


 A future Buddha who will arrive on Earth, attain perfect enlightenment, and teach the pure
dharma.
 Considered as the successor of Gautam Buddha
 Laughing Buddha is supposed to be an incarnation of Maitreya.
Akasagarbha (Buddha‟s Blessings)

 He is boundless as space. He was the twin brother of Ksitigarbha. He manifests as a


blessing.
Sarvanivarana-Vishkambhin (Buddha‟s Qualities):

 He is invoked to remove or eliminate all the obstacles to insure a successful meditation.


Tara

https://t.me/prelimbits
157

 Tara is a Vajrayana Buddhist deity who embodies the values of success in labor and
accomplishments.
Vasudhara

 He is one who represents riches, prosperity, and abundance. In Nepal, he is quite popular.
Skanda

 He is the protector of viharas and Buddhist teachings.


Sitatapatra

 She is adored in both Mahayana and Vajrayana traditions as protection against supernatural
peril.

Dipankara
 Dipankara was a Buddha from the past who had attained enlightenment before Gautam
Buddha.
 Dipankara was a past Buddha, Gautam was the most recent Buddha, and Maitreya is the
future Buddha, according to Buddhism.
 He is revered by Buddhists of all three sub-sects: Mahayana, Vajrayana, and even Theravada.

123. The silver coins issued by the Guptas were called

A. Rupaka

B. Karshapana

C. Dinara

D. Pana

Ans. A

CHANDRAGUPTA II (C. 376 – 413/15 CE)

 The peak of the territorial expansion of the Gupta empire was reached during
Chandragupta II‟s reign (also known as Chandragupta Vikramaditya).
o First Gupta ruler to assume the title of 'Param Bhagawata'
o He fought many wars and thus he was awarded the title of 'Prithivyah Pratham
Veer' which means the first powerful man of the world.
 Extended the limits of his empire by conquests & marital alliances with other royal
dynasties of the period.
 He married to the Naga princess, and had a daughter. His daughter was married to
Rudrasena II of the ruling Vakataka dynasty in Deccan.
 Chandragupta thus exercised indirect control over the Vakataka kingdom, which proved
very beneficial to him.

https://t.me/prelimbits
158

 He conquered western Malwa & Gujarat from the Shaka satraps. This conquest took the
expanse of the Gupta empire to the western sea coast, which was an important region for
trade and commerce. (aka. Saka Conqueror)
 This contributed to the prosperity of Malwa and its chief city Ujjain, which was also
Chandragupta II‘s second capital (1st Pataliputra).
 An iron pillar inscription at Mehrauli in Delhi indicates that his empire included even
north-western India and Bengal.

Mehrauli Iron Pillar


The famous Mehrauli iron pillar is a structure with 7.2 metres height and a diameter of 40.64
centimetres constructed by the famous king Chandragupta II. The pillar is famous for the rub-
resistant composition of the metals that have been used in the construction. It is believed that
the pillar was erected perhaps outside the Udayagiri caves and was later moved to its current
location in the Qutub complex at Mehrauli under the Delhi sultanate. The oldest inscriptions
on the pillar belonged to Chandragupta II.
Composed in the Sanskrit language. Chandragupta-II of the Gupta dynasty named this
pillar Vishnupada in honor of Lord Vishnu.

 He took the title of Vikramaditya, that is, the one who is as powerful as the sun, and
Simhavikrama.
 He was the first ruler to issue Silver Coins (called 16 Rupaka was equal to 1 gold coin). He
also issued copper coins and is referred to as Chandra on his coins. He also issued gold
coins called Dinara.
 The Udaigiri Cave Inscriptions refer to his digvijaya, that is, his conquest of the whole
world.
 Fa Hien/Faxian, the famous Chinese traveler, visited India during his reign.
COINAGE OF INDIA: COINS USED IN ANCIENT INDIA

Prehistoric and Bronze Age

 In the Indian context, no significant evidence is found to conclude that people in Stone
age used currency or they conducted exchange in barter.
 The Indus Valley Civilization seems to have conducted an extensive trade on a wide
network, but it was predominantly conducted around the Barter System.
o Through the excavations held at cites from the Indus Valley Civilization (Mohenjo-
Daro and Harappa), seals were found. However, there was no confirmation if these
seals were used as coins or not
Coins in the Vedic Period

 The Rigveda has references to nishka (gold) and nishka greeva (ornaments of gold) but it
is not sure if these could be understood as coins.

 However, the later Vedic texts had references to nishka, suvarna, shatamana, and pada.
https://t.me/prelimbits
159

But scholars are of the opinion that these terms do not indicate a wide circulation of
conventional coins.
Coinage during Janapadas

 The most definite literary and archaeological evidence that suggests the usage of coins in
the subcontinent, dates around 6th-5th Century BCE. It was in the context of the
emergence of states, urbanisation, and rising trade activities that the Buddhist texts and
Panini‟s Ashtadhyayi refer to terms like kahapana/ karshapana, nikkha/nishka,
shatamana, pada, vimshatika, trinshatika, and suvanna/suvarna.
 It is interesting to note the basic unit of the weight of the Indian coins was the red and
black seed of gunja berry (Abrus precatorius).
 In South India, the standard weight of the coins was ascertained by calculating the
relationship between the weight of two kinds of beans- the manjadi (Odenthera pavonia)
and Kalanju (Caesalpinia bonduc).
Punch-marked coins

 The most conventional system of coinage emerged with the circulation of punch-marked
coins, that were mostly made of silver, and sometimes even copper. They were sometimes
square, or round but mostly rectangular in shape.
 The symbols on these were hammered using punches and dies. That is why we know
them as punch-marked coins.
 A standard punch-marked coin often weighed around 32 rattis or about 52 grains (1 grain =
64.79 mg).
 Punch marked coins are found across the subcontinent and were popular till the early
centuries CE.
 The Punch marked coin series can be further divided into four categories: The Taxila-
Gandhara type, the Kosala type, the Avanti type, and the Magadhan type.
 With the expansion of the Magadhan Empire, the coins that belonged to the Magadhan
category replaced the other three types. It must be noted that these coins do not have
any legends inscribed on them which could convey the details of the kingdom.

Coins of Kuru Janapada

https://t.me/prelimbits
160

Coins of Magadha Janapada


Coinage and the Mauryas

 The Mauryan Empire used the punch-marked coins but with royal standard to make sure
of their authenticity.

 The liberty to use two different metals for coins was granted, and hence coins in silver and
copper were very popular.

Hoards mostly of Mauryan coins


Coinage by the Indo-Greeks

 The next prominent range of coins that were issued belongs to 2nd/1st century BCE by the
Indo-Greeks. The Indo-Greeks system of coinage becomes significant because the minting
was executed in a more refined manner.

 They issued First time Gold Coins in India but their number was so small. Their
coins were mostly made of silver, usually round, with exceptions to few rectangular or
circular) bore the name of the issuing ruler along with depicting legends.
 For example, the coins of Menander and Strabo I show them passing through different
stages of life, suggesting their long reigns.
 The languages on these coins was Prakrit, inscribed mostly in Kharoshthi script.

The early coins seem like they could have come directly from Greece itself, with their diadems
https://t.me/prelimbits
161

and helmets, Greek scripts and Greek goddesses, and Greek names.
Over time, these Indo-Greek kings became ever more Indian. They started worshipping the
Buddha and Hindu gods like Vishnu and Krishna.

Roman Empire Coins

 Roman Empire had long-lasting trade ties with ancient India. Roman gold and silver
poured into India, and many coins have been found all over India – ranging from the
time of the first Roman Emperor Augustus Caesar in the 30s BC, to Justinian I, 600 years
later!
 These Roman coins became a big hit in South India – not only as money, but as
fashionable gold-coin necklaces and earrings.

 As Roman trade with southern India gradually declined and Roman gold coins
slowed to a trickle (less flow), the fashion-conscious Indians began making crude
imitations of these coins, just to meet the demand for jewellery!
The Kushana type coins

 Roman denarii were the first gold coins to appear in India, which had only seen copper
and silver coins so far, though of course, gold jewellery had been around for ages. Soon the
Indian rulers of the day copied the idea and gold coins were minted in India for the first
time.
 The Kushanas (1st-4th Centuries CE) were the first dynasty in the subcontinent that issued
gold coins. The lower denomination was usually found in copper coins.
o Note: Though Indo Greek were first ruler to introduce Gold Coins in india but It was
kushana who issued Gold coins in such a large scale people considered them as
the first one to introduce gold coins.

o This Q if comes in CSE (which I don‟t think) & both the options are there than
my suggestion is go with Kushana. (in small exams this Q has been asked many
time but UPSC will not ask such kind of Q. if asked we will go with Kushana)

 The coins usually bore the figure, name, and title of the ruler on the obverse and the
deities on the reverse. The legends are either entirely in Greek, or in some cases in
Kharoshthi on the reverse.
o They were first to use the image of Goddess Lakshmi on their coins, along with
Ardochsho, the Iranic Goddess of wealth.
o The Kushans also depicted Oesho (Shiva), moon deity Miro and Buddha in their
coinage

https://t.me/prelimbits
162

Coin of Kanishka in Greek script, with an illustration of the Buddha on the reverse
Coinage by the Imperial Guptas

 The Imperial Guptas issued well-minted and well-executed gold coins, die struck with
various impressive legends in Sanskrit. These coins, also known as dinaras, were mostly
found in North India. Chandragupta II issued Silver coins known as Rupaka
 The obverse has the kings in various poses, mostly in martial mood, but sometimes even
in artistic calibre.
 Coins have also shown rulers like Samudragupta and Kumaragupta playing Vina.
 The Gupta coins were issued in gold in large numbers and have been credited to their
fine aesthetic appeal. However, the purity of gold saw a decline during the
Guptas & later Period. (due to collapse of Trade with Roman Empire)

Samudragupta Coin
Post Gupta Coinage

 The dynasties like the Gurjaras, Pratiharas, Chalukyas, Paramaras, and the Palas from circa
530 CE to 1202 CE can be put under a broad category of Indo-Sassanian styled coinage.
 The main features of these coins included the bust of the ruling sovereign in a simplified
geometric style on the obverse and a motif like a fire altar on the reverse.
Vijayanagara Coinage

 Vijayanagara kings used coinage with Hindu idols. Harihara –II (1377-1404) introduced
coins that had Brahma-Saraswati, Vishnu-Lakshmi and Shiva-Parvati.
 The Vijayanagara coins continued in circulation even after the kingdom was snuffed out in
1565 and commanded a premium when French traveller Tavernier visited the region.
British East India Co.

https://t.me/prelimbits
163

 Madras Presidency minted coins labelled as the Three Swamy Pagoda, which depicts Lord
Balaji flanked by Sridevi and Bhudevi on either side.
South Indian Kingdoms

South Indian coin issues were confined to dynastic crests:


 Boar – Chalukya Dynasty
 Bull – Pallavas
 Tiger – Cholas
 Fish – Pandya Dynasty
 Bow and Arrow – Chera Dynasty
 Lion – Hoysala

124. Match List I with List II and select the correct answer using the codes given below
the lists: [97]

List I List II

A. Varahamihira 1. Prabandha Chintamani

B. Visakhadatta 2. Mrchchhakatikam

C. Sudraka 3. Brhat-Samhita

D. Bilhana 4. Devi Chandraguptam

5. Vikramankadevacharita

Codes

A B C D

a) 3 4 2 5

b) 2 4 1 3

c) 2 3 1 4

d) 4 3 1 2

Ans. A
VARAHAMIHIRA

 A 6th [or older] century astronomer, astrologer, mathematician, and scholar, who was one of
the navratnas of Chandragupta II and belonged to Avanti.
o Some believes him to be one of the "Nine Jewels" (Navaratnas) of the court of ruler
Yashodharman Vikramaditya (but not clear)
https://t.me/prelimbits
164

 His famous work:


o Panchasiddhantika summarized five astronomical schools.
o Brihatsamhita is an encyclopaedic work dealing with diverse topics.
o Brihat Jataka
BILHANA

 11th century Kashmiri poet.


 Patron : Vikramaditya VI (Western Chalukya Empire) as Vidyapathi.
 Work :
o Vikramankadevacharita
o Caurapancasika (Love poem)
VISHAKHADATTA

 Sanskrit poet and playwright


 Court poet of Chandragupta II
 Work :
o Mudra Rakshas : Narrates the ascent of Chandragupta Maurya to throne.
o Devichandraguptam : The play, king Ramagupta decides to surrender his queen
Dhruvadevi ("Devi") to a Shaka enemy when besieged. Ramagupta's younger
brother Chandragupta enters the enemy camp disguised as the queen, and kills the
enemy ruler.
SHUDRAKA

 Sanskrit Playwright during Gupta Period


 Work :
o Mrichchhkatikam (Little Clay Cart), which deals with the love affair of a poor
brahmana with the beautiful daughter of a courtesan. (asked in CSE)
o Vinavasavadatta
o Bhana (short one-act monologue)
o Padmaprabhritaka

125. Which one of the following was a corporation of merchants in ancient India? [97]

A. Chaturvedimangalam
B. Parishad
C. Ashtadikgaja
D. Manigrama

https://t.me/prelimbits
165

Ans. D
A OPTION : CHATURVEDIMANGALAM
 Chaturvedimangalam was a village gifted to Brahmins in ancient India.
 Chaturvedi Mangalam, also known as agrahara, brahmadeya, brahmadesha,
brahmapura, or simply mangalam, means a village granted to, or owned by Brahmin‘s well
versed in the four Vedas.
 During the ancient and medieval periods, villages were granted to the Brahmins, usually
tax free, by the ruling kinds or chieftains as acts of piety, and also to patronize Vedic
learning and sacrifices.
 The Brahmins enjoyed considerable local autonomy in the management of their villages
through their own sabhas (village assemblies.)
 The detailed account of the constitution of such an assembly and its various sub-
committees functioning at a place called Uthiramerur (in Chengalpattu district) in the 10th
century CE. Qualifications of age, education and character were laid down for the
members of the assembly.
 From the inscriptions found in Karnataka we learn that the agraharas were considered so
important that queens and high ranking officers were put in charge to oversee and
ensure their welfare, e.g. Lakshmidevi, the chief queen of Vikramaditya VI, was in charge of
the administration of 18 agraharas in 1084 CE.
KRISHNA DEVA RAYA (1509–29 CE)

 The greatest of the Vijayanagar rulers .


o Aka. Abhinava Bhoja, Andhra Pitamah, Andhra Bhoj (as he was great patron of
literature and art).
 He fought a series of war with the independent kingdoms (Deccan Sultanates) that came
up on the ruins of the Bahmani kingdom. The Muslim armies were decisively defeated in the
battle of Diwani.
 Then he invaded the Raichur Doab and completely shattered the Adil Shahi forces of
Bijapur and set free the three Bahmani princes who were imprisoned there. He thus
restored the Bahmani Sultanate to Muhammad Shah.

o Krishna Deva himself took the title of Yavanarajya sthapanacharya.


 He maintained friendly relations with the Portuguese.
o Albuquerque sent his ambassadors to the court of Krishna Deva Raya. (asked in
lower exam)

o Portuguese travellers Domingo Paes and Barbosa came to India during his reign.

 Eight eminent scholars (Ashtadiggajas) of Telugu adorned his court :


1. Allasani Peddanna (the greatest, who was also called Andhra kavita Pitamaha)

https://t.me/prelimbits
166

 his important works include


 Manucharitam & Harikathasaram
2. Tenali Ramakrishna, author of Panduranga Mahamatyam
3. Pingali Suranna, author of:
 Garuda Puranam
 Prabhavatee
 Pradyumnam
 Raghava
 Pandaveeyam
 Kalapurnodayamu
4. Nandi Thimmana
5. Madayyagari Mallana
6. Dhurjati
7. Ayyalaraju Ramambhadrudu
8. Ramarajabhushanudu
 He himself authored
o Amukthamalyadha (Telugu)
o Jambavati Kalyanam and Ushaparinayam (Sanskrit)
 He built some fine stone temples such as Hazara Ramaswamy temples at Vijayanagar.
 The famous temple of Tirupati greatly developed during his period
 He also founded a suburban township near Vijayanagar called Nagalapuram after his
mother.
MANIGRAMAM
 Manigiramam, or manigramam, typically refers to a medieval merchant guild, organised
by itinerant ethnic Indian traders, primarily active in south India.

 Along with the Ainurruvar (the Ayyavole Five Hundred) and the Anjuvannam
(the anjuman), the manigiramam played a major role in the commercial activities of the
region.
 Unlike the anjuvannam which was confined to the port-towns of south India, the
Manigramam is found both in the port-towns and in the hinter-land trade centres.

 A body of merchants known as "the Vaniggrama" or "Vaniyagrama" attest its


presence in north India as early as first century BC

o They appear in a Karle inscription (first century BCE), a charter of king


Vishnusena from Kathiawad (6th century CE) and in a Sanjeli charter of king
https://t.me/prelimbits
167

Toramana (6th century CE).


 Records of activities of manigramam in South India are visible from the 5th century CE.
The first reference to manigiramam is found in two copper plate grants from south
Karnataka (from Melekote, Tumkur and from Hassan district), both datable to the 5th
century CE. The Melekote charter records a grant of land to a Buddhist shrine.
 The Quilon Syrian copper plates, of 9th century CE, also attests the presence of the
manigiramam representatives.
 It seems that by the 9th century the manigramam had become interested in ocean
trade too and collaborated with the anjuvannam (the anjuman) on the western coast.
 A Tamil inscription from Takua Pa (9th century CE) in Thailand refers to a manigiramam
guild.
 From the early 10th century the ainurruvar (the Ayyavole Five Hundred) expanded as the
superior guild of merchants of south India. The manigiramam and anjuvannam were
incorporated later into the ainurruvar.
 The Ainurruvar, in and after the 12th century CE, acted as an "umbrella organization" to
cover all other merchant guilds

126. [97]

Assertion (A): The Gandhara School of art bears the mark of Hellenistic influence

Reason (R): Hinayana form was influenced by that art.

A. Both A and R are true, and R is the correct explanation of A


B. Both A and R are true, but R is NOT a correct explanation of A
C. A is true, but R is false
D. A is false, but R is true
Ans. C
GANDHARA SCHOOL OF ART

 The Gandhara school of Art arose in modern-day Peshawar and Afghanistan on Punjab's
western boundaries.

 The Greek invaders brought the traditions of Greek and Roman sculptors with them,
which affected the region's native traditions ==> Greco-Indian School of Art

 Between 50 B.C. and 500 A.D., the Gandhara School flourished in two periods. While the
former school's sculptures were made of bluish-grey sandstone, the latter school's were
made of mud and plaster.

 The Buddha and Bodhisattvas iconography was based on the Greco-Roman pantheon
and resembled Apollo's.

https://t.me/prelimbits
168

 Major Centres :

o Jalalabad, Hadda, Bamaran, Begram & Taxila were the main centers where art pieces
of Gandhara School have been found.

 Major Features :

o The Gandhara sculptures have been discovered in the Taxila ruins as well as other
ancient sites in Afghanistan and Pakistan.

o They generally consist of Buddha images and relief sculptures depicting scenes
from Buddhist literature.

o Several Bodhisattva figures were cut out of the rock. The first preaching in the
deer park and the Buddha's death is depicted in a Gandhara figure.

o The predominant focus of this type of painting was Lord Buddha


and Bodhisattvas, as it was intimately tied with Mahayana Buddhism.

o The Bamiyan Buddha sculptures are an example of Gandhara style art.

o It thrived primarily in Afghanistan and present-day North-Western India.


Taxila, Peshawar, Begram, and Bamiyan were among the most prominent sites.

o From the first century BCE through the fourth century CE, the Gandhara School of
art flourished.

 Themes

o The Buddhist themes were largely represented in Gandhara art. The Buddha's
mother resembles an Anthenian matron.

o A Buddhist scene was created with an Apollo-like face. The image of Athena of
Rome in Lahore is perhaps one of the most beautiful Gandhara sculptures depicting
a western topic.

o It was created in order to express Buddhist ideas and practices.

o The specimens do not include any Greek art motif, with a few exceptions. The hand
of the Gandhara artist was Greek, but his heart was Indian.

 Patrons

o Gandhara School was fostered by the Shakas and the Kushanas. The first
sculptural portrayals of the Buddha in human form are attributed to
the Gandhara school. [The first image of the Buddha-was carved out during
the reign of famous Kushan king Kanishka I.]

 Features of sculpture

o The human body is sculpted in a realistic manner in these sculptures, with


considerable attention to realism and physical details, particularly in the depiction
https://t.me/prelimbits
169

of muscles, mustaches, and other facial features.

o The portrayal of the broad bold fold lines is a unique trait that sets it apart from what
has been discovered elsewhere in India.

 The Various Mudras of Buddha in Gandhara Art

o In all the Buddha depicted in the Gandhara Art is shown making four types of hand
gestures and this is a remarkable feature in this art.

 Greek influences [Hellenistic influence] on Gandhara School of art

o Greek god as protector: In many images of Buddha in Gandhara; he is seen under


the protection of Greek god Hercules.

o Vajrapani: Vajrapani found in the right hand of future Buddha is told as a


transformed symbol of Hercules who is seen as the protector of Buddha.

o Greek architectural influence: Some images of Buddha in Gandhara are presented


in the Greek architectural environment bearing the affinity of Corinthian.

o Artistic beauty: The Apollo-like face of Buddha; natural realism; wavy hair as
seen in images of Buddha in Gandhara resembles Hellenistic tradition.

o Intellectual affinity: The hello and bun of Gandhara Buddha signify intellectual
imbibitions of Buddha from Greek.

o Despite the fact that Gandharan sculpture's iconography was predominantly Indian, it
also included elements and methods from Classical Roman art. Vine scrolls,
cherubs with garlands, tritons, and centaurs are examples of Classical Roman art
found in Gandharan sculptures.

o Additionally, the Gandharan artists drew on the Roman religion's anthropomorphic


traditions.

o Gandharan art depictions of Buddha are evocative of statues of a youthful Apollo.

o The drapery on Buddha's garments was also similar to that on Roman imperial
monuments.

https://t.me/prelimbits
170

127. [98]

Assertion (A): According to Asoka's edicts social harmony among the people was more
important than religious devotion.

Reason (R): He spread ideas of equity instead of promotion of religion.

A. Both A and R are true, and R is the correct explanation of A


B. Both A and R are true, but R is NOT a correct explanation of A
C. A is true, but R is false
D. A is false, but R is true
Ans. A
ASHOKA'S POLICY OF DHAMMA
 The word dhamma is the Pali form of the Sanskrit word dharma
 There have been attempts to define and find equivalent English words for it, such as "piety",
"moral life" and "righteousness" or "duty" but scholars could not translate it into English
because it was coined and used in a specific context.
 The word Dharma has multiple meanings in the literature and thought of ancient India.
The best way to understand what Ashoka the great means by Dharma is to read his
edicts, which were written to explain the principles of Dharma to the people of that
time throughout the empire.
 Dharma was not a particular religious faith or practice, or an arbitrary formulated
royal policy. Dharma related to generalized norms of social behavior and activities;
 The principles of Dhamma were formulated to be acceptable to people belonging to
different communities and following any religion.

 Dhamma was not given any formal definition or structure. It emphasized tolerance of
people and the notion of showing consideration towards slaves and servants; there is stress
on obedience to elders; generosity towards the needy, Brahmans and Sarmanas.
 Ashoka pleaded for tolerance of different religious sects in an attempt to create a sense
of harmony
 The policy of Dhamma also laid stress on non-violence, which was to be practiced by
giving up war and conquests and also as a restraint on the killing of animals.
However, Ashoka realized that a certain display of his political might may be
necessary to keep the primitive forest-dwelling tribes in check.
 The policy of Dhamma also included other welfare measures, like the planting of trees and
digging of wells. Ashoka attacked ceremonies and sacrifices as meaningless.
 A group of officers known as Dhamma mahamattas were instituted to implement and
publicize the various aspects of Dhamma.
 Ashoka made them responsible for carrying his message to various sections of society,
https://t.me/prelimbits
171

However, they gradually developed into a type of priesthood of Dhamma with great powers
and soon began to interfere in politics
 Ashoka's Dhamma was not simply a collection of high-sounding phrases. He consciously
tried to adopt it as a matter of state policy; he declared that "all men and my children"
and "whatever exertion I make, I strive only to discharge debt that I owe to all living
creatures."
 It was a totally new and inspiring idea of kingship. In the Arthashastra, the king owed
nothing to anyone. His only job was to rule the state efficiently.
 Ashoka renounced war and conquest by violence and forbade the killing of many animals.
 Ashoka set an example of vegetarianism by almost stopping the consumption of meat
in the royal household.
 Since he wanted to conquer the world through love and faith, he sent many missions to
propagate Dhamma. Such missions were sent to far off places like Egypt, Greece and Sri
Lanka.

128. Which of the following pairs are correctly matched? [98]

1. Lothal Ancient dockyard

2. Sarnath First Sermon of Buddha

3. Rajgir Lion capital of Asoka

4. Nalanda Great seat of Buddhist learning

Options
A. 1, 2, 3 and 4
B. 3 and 4
C. 1, 2 and 4
D. 1 and 2
Ans. C
LOTHAL

 Lothal is located along the Bhogava River, a tributary of river Sabarmati, in the Gulf of
Khambat.

 It was one of the southernmost sites and only port-town of the Indus Valley civilization.

 It was a trade centre for beads, gems and ornaments in ancient times that traded with
West Asia and Africa.

 Lothal (Loth + thal) means ‗the mound of the dead‟ in Gujarati.

o Mohenjo-Daro, an Indus Valley Civilisation site, (now in Pakistan) also means ‗the

https://t.me/prelimbits
172

mound of the dead‘ in Sindhi.

 Lothal was a major trading centre of the Indus Valley Civilisation, and also a symbol of
India‟s maritime power and prosperity.

 According to the Archaeological Survey of India (ASI), Lothal had the world‟s earliest
known dock, connecting the city to an ancient course of the Sabarmati
River.
 The excavations shows Lothal had the Upper Town (citadel), the Lower Town, a bead-
making factory, a ware house and a tidal dockyard.

 Lothal is in the tentative list of UNESCO World Heritage Site.

National Maritime Heritage Complex

 National Maritime heritage complex (NMHC) is being developed to showcase India‟s rich
and diverse maritime heritage.

 The construction of the National Maritime Heritage Complex is being developed by the
Ministry of ports, shipping and waterways.

 The Heritage complex comprises of –

o Lothal mini recreation to recreate Harappan architecture and lifestyle,

o Memorial theme park,

o Maritime and Navy theme park,

o Climate theme park and

o Adventure and amusement theme park.

o World‘s tallest lighthouse museum,

o 14 galleries highlighting India‘s maritime heritage

SARNATH

 At the age of 35, Buddha ultimately attained Nirvana/Enlightenment (the cessation of


desire and hence the end of suffering)
at Gaya, Magadha (Bihar) under a Peepal tree at
Uruvela, on the banks of river Niranjana, and came to be known as the Buddha, the
Enlightened One.
 Peepal tree was known as the famous Bodhi tree. It is believed that tree was destroyed by
Asoka‘s queen Tissarakkha, Pushyamitra Shunga and by King Shashanka. [But again planted
by Buddhist]

 Delivered his First Sermon with his five former companions at deer park at Sarnath.
Event is known as DhammaChakka-Pavattana / Dhammacakkappavattana Sutta,
meaning „turning the wheel of dharma‟. It introduces fundamental concepts of Buddhist
thought, such as the Middle Way and the Four Noble Truths
https://t.me/prelimbits
173

 Lord Buddha met his 5 disciples first →


o Chaukhandi Stupa is the place where Lord Buddha met his 5 disciples first in Sarnath.
It is considered that he came to Sarnath after getting enlightenment at Bodh Gaya in
528 BCE to meet his disciples Mahanama, Koudanna, Bhaddiya, Vappa and
Asvajita in order to share his real knowledge got during enlightenment. It was built
of brick like octagonal tower during 4th to 6th century in the Gupta period
o These Five disciples became Arhats

 Arhats : one who has gained insight into the true nature of existence
and has achieved nirvana BUT but who may not have reached full
Buddhahood
Buddha initially intended to visit his former teachers, Āḷāra Kālāma and Uddaka Rāmaputta, to
teach them his insights, but they had already died
On his way, he encountered a spiritual seeker named Upaka. but Upaka was not convinced and
"took a different path"

RAJGIR
 Rajgir, meaning "The City of Kings," is a historic town in the district of Nalanda in Bihar
 As the ancient seat and capital of the Haryanka dynasty, the Pradyota dynasty, the
Brihadratha dynasty and the Mauryan Empire, as well as the dwelling ground of such
historical figures as The Buddha and The Mahavira
 Rajgir was the capital of Haryanka dynasty kings Bimbisara (558–491 BC) and
Ajatashatru (492–460 BC).
 Ajatashatru kept his father Bimbisara in captivity here. The sources do not agree on which of
the Buddha's royal contemporaries, Bimbisara and Ajatashatru, was responsible for its
construction.
 It was the ancient capital city of the Magadha kings until the 5th century BC when
Udayin (460–440 BC), son of Ajatashatru, moved the capital to Pataliputra (modern
Patna).
 Shishunaga (413-395 BC) founded Shishunaga dynasty in 413 BC with Rajgir as its initial
capital before it was moved to Pataliputra.
 It is associated with the founders of both the religions: Jainism and Buddhism, associated
with both the historical Mahavira and Buddha.
 Buddha
o It was here that Gautama Buddha spent several months meditating, and
preaching at Gridhra-kuta, ('Hill of the Vultures').
o He also delivered some of his famous sermons and initiated king Bimbisara of
Magadha and others to Buddhism.
o It was here that Budhha delivered his famous Atanatiya Sutra.

https://t.me/prelimbits
174

o On one of the hills is the Saptaparni Cave where the First Buddhist Council was held
under the leadership of Maha Kassapa
 Mahavira
o Mahavira, the 24th Tirthankara spent fourteen years of his life at Rajgir and
Nalanda, spending Chaturmas (i.e. 4 months of the rainy season) at a single place in
Rajgir (Rajgruhi) and the rest in the places in the vicinity.
o It was the capital of one of his Shravaks (follower) King Shrenik.
o The twentieth Jain Tirthankara, Munisuvrata is supposed to have been born here.
o It is also mentioned in Jain and Buddhist scriptures, which give a series of place-
names, but without geographical context.
NALANDA

 Nalanda was a renowned mahavihara (Buddhist monastic university) in ancient


Magadha (modern-day Bihar)

 Considered by historians to be the world's first residential university and among the
greatest centers of learning in the ancient world, it was located near the city of
Rajagriha (now Rajgir) and about 90 kilometres (56 mi) southeast of Pataliputra (now
Patna).

 Operating from 427 until 1197 CE, Nalanda played a vital role in promoting the
patronage of arts and academics during the 5th and 6th century CE, a period that has
since been described as the "Golden Age of India" by scholars.

 Nalanda was established during the Gupta Empire era, and was supported by numerous
Indian and Javanese patrons – both Buddhists and non-Buddhists

 Over some 750 years, its faculty included some of the most revered scholars of Mahayana
Buddhism.

 Nalanda mahavihara taught six major Buddhist schools and philosophies such as
Yogachara and Sarvastivada as well as subjects such as grammar, medicine, logic and
mathematics.

 The university was also a major source of the 657 Sanskrit texts carried by pilgrim
Xuanzang and the 400 Sanskrit texts carried by Yijing to China in the 7th century, which
influenced East Asian Buddhism.

 Many of the texts composed at Nalanda played an important role in the development of
Mahayana and Vajrayana Buddhism including the Mahavairocana Tantra and the
Bodhisattvacaryavatara of Shantideva.

 It was sacked and destroyed by the troops of Muhammad Bakhtiyar Khalji, partly
restored thereafter, and continued to exist till about 1400 CE.

 Today, it is a UNESCO World Heritage Site.


https://t.me/prelimbits
175

129. Which one of the following ancient Indian records is the earliest royal order to
preserve foodgrains to be utilised during the crises in the country? [98]

A. Sohagaura Copper-plate
B. Rummindei pillar-edict of Asoka
C. Prayaga-Prasasti
D. Mehrauli Pillar inscription of Chandra
Ans. A
SOHGAURA COPPER PLATE INSCRIPTION
 The Sohgaura copper plate inscription is an Indian copper plate inscription written in
Prakrit in the Brahmi script.
 It was discovered in Sohgaura, a village on the banks of the Rapti River, about 20 km
south-east of Gorakhpur, in the Gorakhpur District, Uttar Pradesh
 The inscription is sometimes presented as pre-Ashokan, even pre-Mauryan, but the writing
of the plate, especially the configuration of akshara would rather suggest a date after
Ashoka

 The text mentions the establishment of two grain depots (Kosthagara) to fight
against famine.
RUMMINDEI PILLAR EDICTS (LUMBINI)

 These inscriptions are classified as Minor Pillar Inscriptions. These have inscriptions that
record their dedication.
 The inscriptions mention Ashoka's visit to Lumbini (Rummindei), Rupandehi district,
Nepal, Lord Buddha's birthplace.
 Ashoka exempted Lumbini from paying taxes [bali] and set its grain contribution at one-
eighth.
 Brahmi script is used to write the inscriptions.
ALLAHABAD PILLAR INSCRIPTION (PRAYAG PRASASTI)

 "Prayag" refers to a place where something or someone can meet. Prayag is an ancient
name for Allahabad because it is the confluence of the rivers Ganga, Yamuna, and the
mythical Saraswati.
 Prashasti is a eulogy that means "in praise of someone."
 The Allahabad Prasasti was first inscribed on the Ashokan Pillar in Kausambhi, near
Allahabad.
 It was later relocated to the Allahabad fort.
 It is an Ashokan Stambh, but it has four different inscriptions,
5. The standard Ashokan inscriptions in Brahmi script are found on all pillars.

https://t.me/prelimbits
176

6. The Queen's edict lauding Ashoka's wife Kaurwaki's charitable deeds.


7. Inscriptions of Samudragupta (335–375 AD) written by Harisena in Sanskrit It
discusses Samudragupta's conquests and the boundaries of the Gupta Empire.
Inscriptions called him ―Dharma Prachar Bandhu”.
 Harisena, who was not only his court poet but was also a kumaramatya,
that is, a high ranking official who described himself by different titles such as
sandhivigrahika (minister for peace and war) and mahadandanayaka
(judicial/military officer).
8. Jahangir‟s Persian inscriptions.
MEHRAULI IRON PILLAR

 The famous Mehrauli iron pillar is a structure with 7.2 metres height and a diameter of 40.64
centimetres constructed by the famous king Chandragupta II.
 The pillar is famous for the rub-resistant composition of the metals that have been used
in the construction.
 It is believed that the pillar was erected perhaps outside the Udayagiri caves and was
later moved to its current location in the Qutub complex at Mehrauli under the Delhi
sultanate.
 The oldest inscriptions on the pillar belonged to Chandragupta II.
 Composed in the Sanskrit language. Chandragupta-II of the Gupta dynasty named this
pillar Vishnupada in honor of Lord Vishnu.

130. The concept of Eight-fold path forms the theme of [98]

A. Dipavamsa
B. Divyavadana
C. Mahaparinibban Sutta
D. Dharma Chakara Pravartana Sutta
Ans. D
DHARMA CHAKARA PRAVARTANA SUTTA

 At the age of 35, Buddha ultimately attained Nirvana/Enlightenment (the cessation of


desire and hence the end of suffering)
at Gaya, Magadha (Bihar) under a Peepal tree at
Uruvela, on the banks of river Niranjana, and came to be known as the Buddha, the
Enlightened One.
 Peepal tree was known as the famous Bodhi tree. It is believed that tree was destroyed by
Asoka‘s queen Tissarakkha, Pushyamitra Shunga and by King Shashanka. [But again planted
by Buddhist]

 Delivered his First Sermon with his five former companions at deer park at Sarnath.

https://t.me/prelimbits
177

Event is known as DhammaChakka-Pavattana / Dhammacakkappavattana Sutta,


meaning „turning the wheel of dharma‟. It introduces fundamental concepts of Buddhist
thought, such as the Middle Way and the Four Noble Truths
o In the Early Buddhist Texts, the term "Middle Path" was used in the
Dhammacakkappavattana Sutta, which the Buddhist tradition regards to be the first
teaching that the Buddha delivered after his awakening. In this sutta, the Buddha
describes the Noble Eightfold Path as the Middle Way which steers clear of the
extremes of sensual indulgence and self-mortification

MAHAPARINIBBANA SUTTA
 The Mahaparinibbaṇa Sutta is Sutta 16 in the Digha Nikaya, a scripture belonging to the
Sutta Pitaka of Theravada Buddhism.
 It concerns the end of Gautama Buddha's life - his Parinibbana - and is the longest
sutta of the Pali Canon.
 Because of its attention to detail, it has been resorted to as the principal source of reference
in most standard accounts of the Buddha's death
DIPAVAMSA
 Is the oldest historical record of Sri Lanka.
 The chronicle is believed to be compiled from Atthakatha and other sources around the
3rd to 4th century CE.
 Together with the Mahavamsa, it is the source of many accounts of the ancient history
of Sri Lanka and India.
 Its importance resides not only as a source of history and legend but also as an important
early work in Buddhist and Pali literature.
 The Dipavamsa refers to three visits to the Island by the Buddha,
DIVYAVADANA
 The Divyavadana or Divine narratives are a Sanskrit anthology of Buddhist avadana tales,
many originating in Malasarvastivadin vinaya texts.
 It may be dated to 2nd century CE.
 The stories themselves are therefore quite ancient and may be among the first Buddhist
texts ever committed to writing,
 Typically, the stories involve the Buddha explaining to a group of disciples how a
particular individual, through actions in a previous life, came to have a particular
karmic result in the present.
 A predominant theme is the vast merit (punya) accrued from making offerings to
enlightened beings or at stupas and other holy sites related to the Buddha

https://t.me/prelimbits
178

131. Many of the Greeks, Kushanas and Shakas embraced Buddhism rather than
Hinduism because [98]

A. Buddhism was in the ascendant at that time


B. they had renounced the policy of war and violence
C. caste-ridden Hinduism did not attract them
D. Buddhism provided easier access to Indian society
Ans. D
THE IMPACT AND CONTRIBUTION OF CONTACTS WITH CENTRAL ASIA : INDIAN SOCIETY

 Central Asians conquerors not only assimilate with the India society, but they also added
new ingredients to Indian culture and enriched it immensely.
 They completely identified themselves with Indian culture. Since they did not have their
own script, written language or any organised religion,
 No other period of ancient history were foreigners assimilated in Indian society on such
a large scale as they were in the post-Maurya period.
 The rulers not only adopted the Indian societal norms but also adopted the popular
religions (Buddhism) of that time.

 Greek ambassador Heliodorus converted to Vaishnavism & set up a pillar in honour


of Vasudeva.

 Greek ruler Menander who became a Buddhist. Similarly, the Kushana rulers
worshipped both Shiva and the Buddha and the images of these two.
o Greek, Kushanas and Shakas who entered India through North Western frontier
and established their rule or kingdom. They needed legitimacy for their rule for
people to accept them as their king.

o Accepting religion of local area was the best way to achieve


legitimacy.
o Buddhism provided easiest entry in comparison to Hinduism which
had strict birth related caste structure. So these rulers embraced Buddhism
in order to strengthen their dominance over the local people.

132. The Asokan major rock edicts which tell us about the Sangam Kingdom include
rock edicts [98]

A. 1 and X
B. I and XI
C. II and XIII
D. 11 and XIV
https://t.me/prelimbits
179

Ans. C
SANGAM AGE: THE HISTORY OF SOUTH INDIA

 Sangam literature is main source of History of ancient South India i.e., Tamilkam.
 It was compiled during 3rd century BC to 3rd century CE & was composed in poetic
format around theme of love and war. (praise of various heroes and heroines)

o Thus they do not constitute religious literature and they are secular in nature.
o They also speak of the Yavanas (foreigners) coming in their own vessels purchasing
pepper with gold and supplying wine and women slaves to the natives.
 Sangam was a college or assemblies of Tamil poets held under royal patronage. It is
believed that 3 sangams lasted for 9990 years and were attended by 8598 poets and had
197 Pandya kings as patrons.
o 1st Sangam- Old Madurai (Lemurai) No literary work of this Sangam is available.
o 2nd Sangam- Kapatpuram (Alovai) only Tolkappiyam survives
o 3rd Sangam- New Madurai
 Sangam Literature is broadly divided into 2 groups – Narrative & Didactic.
 Narrative texts

o Are called Melkannaku/Eighteen major works consisting of Ettuthogai-


Collection of 8 long poems & Pattupattu- Collection of 10 small poems.

https://t.me/prelimbits
180

o Heroic Poetry in which heroes and wars are glorified. They also give idea of state
formation in South India.

Poem within Sangam literature are composed on are composed on two broder themes :
Akam (Puram) & Puram (mostly based on war)

 Didactic texts

o Are called Kilkannaku/Eighteen minor works consisting of Tirukural and


Naladiyar.
o Prescribe a code of conduct for kings & society. They also Mentions about social
groups, occupations.

 2nd and 13th rock edicts of Ashoka name 4 neighborly kingdoms of South India;
these were Cholas, Cheras (Keralputras of Malabar) Pandyas and Satiyaputras.

 Hathigumpha inscription of Kharavela of Kalinga mentions Tamil kingdoms.


o The Hathigumpha Inscription is a seventeen line inscription in Prakrit language
incised in Brahmi script in a cavern called Hathigumpha in Udayagiri hills, near
Bhubaneswar in Odisha, India. Dated between 2nd-century BCE and 1st-century
CE, it was inscribed by the Jain king Kharavela of Kalinga kingdom.
 Other Sangam Literature →
o Silappadikaram – written by Ilango Adigal. It is about love affair of Kovalan,
Kanaggi and Madhavi. Later, a Kannagi Cult developed in South India.
o Manimekhalai – Written by Sittalai Sattanar continues the story of Silappadikaram
in next generation in which Manimekhalai is the daughter of Madhavi and Kovalan.
o Tolkappiyam – written by Tolkappiyar was product of 2nd Sangam and it is
basically a work on Tamil grammar & poetics.

o Tirrukural – Deals with philosophy and wise maxims & was written by Tiruvallur
o Tirukkural is comprised of 133 sections of 10 couplets each is divided into three
books :
 Aram (virtue),
 Porul (government and society), and
 Kamam (love)

The Sangam literature discusses about the 3 main Kingdoms- CHOLA, PANDYA & CHERA and
about their rivalry. [Muvendar]
3 MAIN KINGDOMS

Cholas

 Capitals at Kaverippattanam (Puhar) and Uraiyur (famous for cotton trade).


 Emblem - Tiger
https://t.me/prelimbits
181

 Kaverippattanam [port], Uraiyur and Arikamedu (Puducherry) became famous centers


of trade and industry under Cholas.
o Arikamedu
 It was excavated in the 1940s .
 It is famous for manufacturing the construction of amazing bead
ornaments and the glass beads site.
 It was considered as the "mother of all bead centers in
the world".
 Potteries were found from the Mediterranean region such
as Amphorae and stamped red glaze pottery known
as Arretine Ware.
 Elara was the earliest known king. He conquered Sri Lanka & ruled
over it for 50 yrs.
 Karikala was the greatest king. He founded Puhar &
constructed dam [Kallanai Dam] across Cauvery River.
Pattinappalai (by Kadiyalur) gives a vivid account of his
reign.

o Pattinappalai is a Tamil poem in the ancient


Sangam literature
o It contains 301 lines, of which 296 lines are about the
port city of Kaverippattanam, the early Chola
kingdom and the Chola king Karikalan.
o The remaining 5 lines are on the proposed separation by a man who wants to move
there and the separation pain of his wife who would miss her husband's love
 Poruṇaraṟṟuppaṭai and a number of individual poems in the Akanaṉūṟu and
Purananuru also are the main source for the information that is attributed to Karikala.
 Many Sangam poems mention the Battle of Venni where he defeated the confederacy of
Cheras, Pandyas and eleven minor chieftains. Vahaipparandalai was another important
battle fought by Karikala.
Pandyas

 Capital at Madurai (center of trade and industry). Korkai & Alagankulam was their main
port
o Korkai and Alagankulam are believed to have been the exchange centres of the
Pandyas. Korkai, a port at the mouth of the river Tambraparni, was linked to the
famous pearl fisheries and Alagankulam was also developed as a port
o The socio-economic condition of the seaport of Korkai was mentioned in
Maduraikkanji which was written by Mangudi Maruthanar.

https://t.me/prelimbits
182

 They invaded Southern Kerala and controlled the port of Nelkynda, near Kottayam
 Emblem - Carp (fish)
 Pandyas had trade relations with Romans. They were first mentioned by Megasthanese.
They also find mention in the Ramayana & Mahabharata
o Trade was prosperous and their pearls were famous.
 Nedunjelian, known for his kingdom‘s wealth and prosperity, was most noteworthy
Pandya ruler
Cheras

 Capital at Vanji (Malabar).


 Emblem - Bow and Arrow.
 Senguttuvan / Chenguttuvan (Red Chera) was most important ruler.
He established the Kannagi or Pattini Cult; Kannagi became object of
worship.
 He was the first King from South India to send an ambassador to
China.
 He enjoyed the reputation of being highly ethical or virtuous.
 Gajabahu was his cotemporary Sri Lankan King.
 Karrur and Musiris & Tyndis (Tondi) were important centers of
international trade.
o Romans settled at Musiris. A temple of Roman emperor Augustus was constructed
here.

133. What is the correct chronological order in which the following appeared in India?
[98]

1. Gold coins
2. Punch marked silver coins
3. Iron plough
4. Urban culture
Select the correct answer using the codes given below:
A. 3,4,1, 2
B. 3,4, 2,1
C. 4, 3,1, 2
D. 4, 3, 2,1
Ans. D
 IVC : 1st Urban culture [2600 BCE to 1900 BCE]

https://t.me/prelimbits
183

 Later Vedic Age [1000 – c. 600 BCE] : Use of Iron [So they must have used iron to plough
their field]
 Coinage during Janapadas [6th-5th Century BCE] : Punch-marked coins
 Indo–Greeks [ after 190 BCE] : first to issue gold coins in India (which increased in number
under the Kushanas)
All these options have been extensively covered in previous Qs.

134. The term 'Aryan' denotes [98]

A. an ethnic group
B. a nomadic people
C. a speech group
D. a superior race
Ans. C
ARYAN or ARYA
 Aryan or Arya is a term originally used as an ethnocultural self-designation by Indo-
Iranians in ancient times, in contrast to the nearby outsiders known as 'non-Aryan' (*an-
arya).

 In Ancient India, the term Arya was used by the Indo-Aryan speakers of the Vedic
period as an endonym (self-designation) and in reference to the geographic region known
as Aryavarta ('abode of the Aryas'), where the Indo-Aryan culture emerged.

 The Sanskrit word arya was originally an Ethnocultural term designating those who
spoke Vedic Sanskrit and adhered to Vedic cultural norms (including religious rituals
and poetry), in contrast to an outsider, or an-Arya ('non-Arya').
https://en.wikipedia.org/wiki/Aryan#Etymology

135. Which one of the following ports handled the north Indian trade during the Gupta
period? [99]

A. Tamralipti
B. Broach
C. Kalyan
D. Cambay
Ans. A
TAMRALIPTI PORT
 Tamralipta or Tamralipti was a port city and capital of Suhma Kingdom in ancient Bengal,
located on the coast of the Bay of Bengal.

https://t.me/prelimbits
184

 The Tamluk town in present-day Purba Medinipur, West Bengal, is generally identified as
the site of Tamralipti.
 It was located near the Rupnarayan river.
 It gets its name from the Sanskrit term "Tāmra," or copper, which was mined nearby at
Ghatsila in the Singbhum region of the Chota Nagpur Plateau and traded through this
port.

 During the Gupta dynasty, Tamralipta was the main emporium, serving as a point of
departure for trade with Ceylon, Java, and China, as well as the west.
 It was linked by roads with the major towns of that time, i.e., Rajagriha, Shravasti,
Pataliputra, Varanasi, Champa, Kaushambi, and Taxila.

BHARUCH (Barygaza)
 is a city at the mouth of the river Narmada in Gujarat. Bharuch is the administrative
headquarters of Bharuch District.

 The city of Bharuch and surroundings have been settled since times of antiquity. It was a
ship building centre and sea port in the pre-compass coastal trading routes to points
West, perhaps as far back as the days of the pharaohs. The route made use of the regular
and predictable monsoon winds or galleys.
 Many goods from the Far East (the famed Spice and Silk trade) were shipped there
during the annual monsoon winds, making it a terminus for several key land-sea trade
routes.
 Bharuch was known to the Greeks, the various Persian Empires, in the Roman Republic
and Empire, and in other Western centres of civilisation through the end of the European
Middle Ages
 During the 8th century, the town of Bharuch was ruled by King Mayur giving rise to the
Chaudhary Dynasty. The king ruled the city for 50 years and was popularly known as the
'Ace of Bharuch'.
 Arab traders entered Gujarat via Bharuch to do business. The British and the Dutch
(Valandas) later noted Bharuch's importance and established their business centres here.
 At the end of the 17th century, it was plundered twice, but recovered quickly.
KALYAN PORT (Maharashtra)
 Kalyan was once famous as a port in ancient times. Records of its existence as a premier
port in the region have been found in ancient Greek manuscripts.

https://t.me/prelimbits
185

KHAMBHAT or CAMBAY PORT


 Khambhat or Cambay, also known as Stambhatirtha, Stambhapura, Mahinagara,
Tarakpura, or Karnavati, is situated north of the River Dhadar and at the head of the Gulf
of Cambay, where the River Mahi discharges into the sea.
 It is mentioned by the Arab geographer Khardadha in his work The Book of Roads and
States, written in 865 CE.
 The Arab traveller Al-Masudi visited Khambhat in 913-914 CE and records that ―Khambhat
enjoyed great fame for its semi-precious stones, which were very popular in the markets of
Aden and Mecca and in Baghdad.‖
 Al-Idrisi (1100 CE) mentions it as a pretty and well-known naval station and second among
the towns of Gujarat.
 According to Marco Polo (1254-1324 CE), an Italian explorer, ―when merchants come to
Cambay with their wares loaded on many ships, they bring, above all, gold, silver, and
copper.
 Khambhat reached its zenith during the period of Sultan Muhammed Begda (1459-1511
CE) and had trade with Persia, Arabia and Africa in the west and as far as China in the
east.

136. From the third century AD when the Hun invasion ended the Roman Empire, the
Indian merchants relied more and more on the [99]

A. African trade
B. West-European trade
https://t.me/prelimbits
186

C. South-East Asian trade


D. Middle-Eastern trade
Ans. C
GUPA PERIOD : ECONOMY

 The Gupta period witnessed an increase in land taxes but a decrease in trade and
commerce taxes (such as shulka or tolls).
 The period from 4th century to 8th century was a period of agricultural expansion. Due
to the practice of granting lands to brahmanas (brahmadeyas, agraharas or shasanas) and
to some other officers, vast areas of virgin land were brought under cultivation and
major improvements were made in the existing methods of production.
 The king collected taxes varying from one-fourth to one-sixth of the produce.
 Two new agricultural taxes that appear in Gupta inscriptions are uparikara (probably a
tax imposed on temporary tenants) and udranga (its exact nature is not clear, but might be
water tax or a sort of police tax).
 There is also mention of vata-bhuta → cesses for the maintenance of rites and
halirakara, probably plough tax.
 In addition to these taxes, peasants were also subjected to vishti (forced labour) for
serving the royal army and officials. It was considered a source of income for the State, a
sort of tax paid by the people. It was more prevalent in Madhya Pradesh and Kathiyawar.
 The Gupta and post-Gupta period witnessed a Comparative Decline in the country‘s
Trade & Commerce.
o Till 550 CE, India continued to have some trade with the Eastern Roman empire,
to which it exported silk, and spices. Around the sixth century, the Romans learnt
the art of rearing silk worms, which adversely affected India‟s export foreign
trade.

o The Mandasor inscription is testimony to the fact that a guild of silk weavers left
their original home at Lata in western Gujarat and migrated to Mandasor, where
they gave up their original occupation and took to other professions.
o The disruption of the north-western trade route by the Huns was another factor
for this decline.
o India tried to make up for the loss by carrying on trade with South-East Asian
countries, but it did not help revive the economy substantially.
o The loss in trade lessened the inflow of gold and silver into the country. It is
confirmed by a general scarcity of gold coins after the Guptas.
o In ancient India, the Guptas issued a large number of gold coins called dinaras
though the gold content is not as pure as Kushan ones.

o The gold coins of each successive Gupta ruler, after Chandragupta II, contain less

https://t.me/prelimbits
187

gold and more alloy. After the conquest of the western satraps of Gujarat, they
issued a large number of silver coins (rupyakas) mainly for local exchange and very
few copper coins.
Shrenis or Trade Guilds
 The Ramayana and many plays from the Gupta period and Tamil Sangam literature write in
detail about the trade guilds or shrenis.
 These were professional bodies of jewelers, weavers, ivory carvers, or even salt-makers who
came together to control quality production, create sound business ethics, maintain fair
wages and prices, sometimes operated as a cooperative, and controlled the entry of
newcomers by laying down high standards of craftsmanship and enforcing rules regarding
apprenticeship.
 Each guild had its own chief, assisted by others.
o These functionaries were selected with great care.
o Guild members were even entitled to impeach and punish a chief found guilty of
misconduct.
 The shrenis were not necessarily restricted to a locality and were known to move from one
town to another, over a period of time.
 Occasionally, shrenis (of merchants and artisans) came together in a joint organization,
called the nigama, or the equivalent of a chamber of commerce and industry.
o Some nigamas also included a class of exporters, who transported the specialties of a
town over long distances and sold them at higher margins of profit than those they
could obtain locally.
 By all accounts, the shrenis were very sound and stable institutions and enjoyed
considerable moral and social prestige not only among their own members but in society at
large.

MANIGRAMAM
 Manigiramam, or manigramam, typically refers to a medieval merchant guild, organised
by itinerant ethnic Indian traders, primarily active in south India.
 Along with the Ainurruvar (the Ayyavole Five Hundred) and the Anjuvannam (the
anjuman), the manigiramam played a major role in the commercial activities of the
region.
 Unlike the anjuvannam which was confined to the port-towns of south India, the
Manigramam is found both in the port-towns and in the hinter-land trade centres.
 A body of merchants known as "the Vaniggrama" or "Vaniyagrama" attest its presence in
north India as early as first century BC
o They appear in a Karle inscription (first century BCE), a charter of king Vishnusena
from Kathiawad (6th century CE) and in a Sanjeli charter of king Toramana (6th
century CE).

https://t.me/prelimbits
188

 Records of activities of manigramam in South India are visible from the 5th century CE.
The first reference to manigiramam is found in two copper plate grants from south
Karnataka (from Melekote, Tumkur and from Hassan district), both datable to the 5th
century CE. The Melekote charter records a grant of land to a Buddhist shrine.
 The Quilon Syrian copper plates, of 9th century CE, also attests the presence of the
manigiramam representatives.
 It seems that by the 9th century the manigramam had become interested in ocean
trade too and collaborated with the anjuvannam (the anjuman) on the western coast.
 A Tamil inscription from Takua Pa (9th century CE) in Thailand refers to a manigiramam
guild.
 From the early 10th century the ainurruvar (the Ayyavole Five Hundred) expanded as the
superior guild of merchants of south India. The manigiramam and anjuvannam were
incorporated later into the ainurruvar.
 The Ainurruvar, in and after the 12th century CE, acted as an "umbrella organization" to
cover all other merchant guilds

Land measurement during the Gupta period


 There were various texts and inscriptions used for land measurement during the Gupta
period.
 The terms for short distances were-
o The angula (approx ¾ inch).
o The hasta (cubit) was the standardized distance between the tip of the elbow and the
middle finger (18 inches).
 The terms for large distances were-
o Adhavapa (3/8–1/2 acre)
o Dronavapa (1½–2 acres)
o Kulyavapa (12–16 acres)
 The 'Kulyavapa ' was the area required to sow one kulya of grain and the 'Dronavapa' was
the area required to sow one Drona of grain.
 Such a large number of land measure terms indicates that there was no single standard set
of measurement and that different measures were current in different regions.

137. The following persons came to India at one time or another:

1. Fa-Hien
2. Tsing
3. Megasthenes
4. Hieun-Tsang
The correct chronological sequence of their visits is:
https://t.me/prelimbits
189

A. 3,1, 2, 4
B. 3,1,4, 2
C. 1,3, 2,4
D. 1, 3, 4, 2
Ans. B
 Megasthenes Visit : Around 300 BCE
 Faxian (Fa Hien) visit : 399 –412 CE
 Xuanzang's (Hiuen Tsang ) visit : 630–643 CE
 Yijing's ( I-tsing) visit : 673–700 CE
MEGASTHENES (350 BCE– c. 290 BCE)
 Was an ancient Greek historian, diplomat and Indian ethnographer and explorer in the
Hellenistic period.
 He described India in his book Indica, which is now lost, but has been partially
reconstructed from literary fragments found in later authors that quoted his work.
 Megasthenes was the first person from the Western world to leave a written
description of India
 Megasthenes was a Greek ambassador of Seleucus I Nicator in the court of
Chandragupta Maurya.
 Arrian explains that Megasthenes lived in Arachosia, with the satrap Sibyrtius, from where
he visited India

 Megasthenes visited the Mauryan capital Pataliputra, but it is not certain which other
parts of India he visited. He appears to have passed through the Punjab region in north-
western India, as he provides a detailed account of the rivers in this area. He must have
then traveled to Pataliputra along the Yamuna and the Ganga rivers.
 He then compiled information about India in the form of Indica, a document which is now a
lost work. It partially survives in form of quotations by later writers.
 Other Greek envoys to the Indian court are known after Megasthenes →

https://t.me/prelimbits
190

o Deimachus as ambassador to Bindusara, and


o Dionysius, as ambassador to Ashoka.
FAXIAN (FA HIEN VISIT) (399–412 CE)

 Also referred to as Fa-Hien, Fa-hsien and Sehi, was a Chinese Buddhist monk and
translator who traveled by foot from China to India to acquire Buddhist texts.

 Starting his arduous journey about age 60, he visited sacred Buddhist sites in Central, South
and Southeast Asia between 399 and 412 CE, of which 10 years were spent in India.

 He visited India in the early fifth century. He entered India from the northwest
and reached Pataliputra. He took back with him a large number of Sanskrit
Buddhist texts and images sacred to Buddhism.

 Upon his return to China, he is also credited with translating these Sanskrit texts into
Chinese.

 Faxian's visit to India occurred during the reign of Chandragupta II.

 His memoir describe his 10 year stay in India. He visited the major sites associated with
the Buddha, as well the renowned centers of education and Buddhist monasteries
(including the Nalanda area). He visited Kapilvastu (Lumbini), Bodh Gaya, Benares
(Varanasi), Shravasti, and Kushinagar, all linked to events in Buddha's life.

 Faxian learned Sanskrit, and collected Indian literature from Pataliputra (Patna),
Oddiyana, and Taxila in Gandhara.

 His memoir mentions the Hinayana (Theravada) and emerging Mahayana traditions, as
well as the splintering and dissenting Theravada sub-traditions in 5th-century Indian
Buddhism.

 On Faxian's way back to China, after a two-year stay in Sri Lanka

His visit to Nalanda

 When Faxian, a Chinese Buddhist pilgrim monk, visited the city of Nalanda, there probably
was no university yet.

 However, he makes no mention of any monastery or university at Nalanda even though


he was looking for Sanskrit texts and took a large number of them from other parts of India
back to China. This silence in Faxian's memoir suggests that Nalanda monastery-
university did not exist around 400 CE.

XUANZANG'S (HIUEN T-SANG ) VISIT (630–643 CE)

 Hiuen Tsang, was a 7th-century Chinese Buddhist monk, scholar, traveler, and translator.
He is known for the epoch-making contributions to Chinese Buddhism

 Xuanzang travelled around India between 630 and 643 CE, during the reign of King
Harsha Vardhan.
https://t.me/prelimbits
191

 He visiting Nalanda in 637 and 642, spending a total of around two years at the
monastery

 He was warmly welcomed in Nalanda where he received the Indian name of Mokshadeva
and studied under the guidance of Shilabhadra, the venerable head of the institution at
the time. He learnt from him Yogachara, a school of thought that had then only partially
been transmitted to China.

 Besides Buddhist studies, the monk also attended courses in grammar, logic, and Sanskrit,
and later also lectured at the Mahavihara.

 Xuanzang returned to China with 657 Sanskrit texts and 150 relics carried by 20 horses in
520 cases. He translated 74 of the texts himself

YIJING'S ( I-TSING) VISIT (673–700 CE)

 Unlike Faxian and Xuanzang, Yijing followed the sea route around Southeast Asia and Sri
Lanka.

 He arrived in 673 CE, and stayed in India for fourteen years, ten of which he spent at
the Nalanda Mahavihara.

 When he returned to China in 695, he had with him 400 Sanskrit texts and 300 grains of
Buddha relics which were subsequently translated in China

 Unlike Xuanzang, who also described the geography and culture of seventh-century India,
Yijing's account primarily concentrates on the practice of Buddhism in India and detailed
descriptions of the customs, rules, and regulations of the monks at the monastery. In his
chronicle, Yijing notes that revenues from 200 villages (as opposed to 100 in Xuanzang's
time) had been assigned toward the maintenance of Nalanda. He described there being
eight vihara with as many as 300 cells. According to him, Nalanda monastery has numerous
daily Nikaya procedures and rules for the monks.

138. Which one of the following was initially the most powerful city state of India in the
6th century B.C.? [99]

A. Gandhar
B. Kamboj
C. Kashi
D. Magadh
Ans. D
MAHAJANAPADAS [6th – 3rd BC]

 6th century BC - Second Urbanisation [1st IVC]


 The centre of economic and political activity shifted from the North-West, Punjab,
Haryana, and Western UP, to Eastern UP and Bihar (Ganga Basin). Bcuz of:
https://t.me/prelimbits
192

o fertile land
o better rainfall
o river systems
o closer to the iron production centres
 New iron agricultural tools and implements (such as the ploughshare) enabled the
people to clear thick forest and cultivate the hard soil of this area
 Iron weapons made the warrior class more important
 People gained a strong allegiance to the territory they belonged to rather than the Jana
or tribe
 Led to the foundation of the Janapadas or territorial
states
 Some territorial states to become very large =
Mahajanapadas
 According to Buddhist texts, Anguttara Nikaya (the
land between Himalayas and Narmada) was divided
into 16 independent states (Mahajanapadas).
o Anga, Magadha, Kosala, Kasi, Vajji, Malla,
Chedi, Vatsa, Kuru, Panchala, Matsya, Avanti,
Surasena, Asmaka, Gandhara and Kambhoja.
 Most powerful states in the 6th century were Magadha, Kosala, Vatsa, and Avanti and
two of the Mahajanapads, the Vajji and Malla were Gana-Sanghas.
 The Gana-Sanghas (literally meaning an assembly of Gana, those who claim to be of
equal status) were an alternative polity to the kingdoms.
 Unlike kingdoms where they had a single hereditary monarch, the Gana-Sanghas had a
government by assembly and within this assembly too, they had an oligarchy. Some of
the Ganas were of one clan, e.g., the Shakyas and the Mallas; while the others were
confederation of several clans, e.g., Vajjis.

Gana-Sanghas (Chiefdoms and Oligarchies) Kingdoms

 The chief office was not hereditary and  Kingdoms registered a centralised
was known as Ganapati or Ganaraja. government with king as Sovereign
and all the power was vested in him
 The power was vested in an aristocratic
and ruling family. The ruling family
council comprising heads of leading
became a dynasty with hereditary
Kshatriya families and the authority
succession and mostly following
structure of ganas had greater elements
primogeniture.
of tribal organisation.

 Most of the ganas were located in or  Generally occupied the fertile alluvial
near the Himalayan foothills in eastern tracts of the Ganga valley
India

https://t.me/prelimbits
193

 Based in smaller geographical areas  Political power was concentrated in the


and had more representative king who was advised and assisted by
government. ministers, advisory councils such as
Parishad and Sabha.
 However, with the emergence of the
concept of ‗divinity of king‘ and more
emphasis on priestly rituals, the
centrality of the popular assemblies was
reduced.

 Gana-Sanghas were more ready to  The Brahmanical political, social and


tolerate unorthodox views and more religious theory was more deeply
open to individualistic or independent entrenched in kingdoms.
opinion than the kingdoms.
 i.e., Mahavira (Jainism, belonged to Vajji
confederacy) and Buddha (Buddhism,
belonged to Shakya clan) were able to
propagate their philosophy in a more
unrestricted way in Gana-Sanghas as
compared to kingdoms.

Magadha Empire → Haryanka Dynasty

Bimbisara (founder of the empire)


 Bimbisara powerful king [Haryanka dynasty]
 Contemporary of both Buddha and Mahavira, and paid equal respect to them
 He pursued a three-pronged policy, namely, matrimonial alliances, friendship with
strong rulers, and conquest of weak neighbours to expand the empire.
 Capital at Rajgriha
 Earlier, had rivalry with Avanti‘s king Pradyota, but later became friends and Bimbisara even
sent his royal physician Jivaka to Ujjain (along Dakshinapatha to Avanti) when Pradyota had
jaundice.
 Strengthened his position by three matrimonial alliances
o His first wife was from Kosala named Mahakoshala (sister of Prasenjit), who brought
in dowry the territory of Kashi, which yielded a revenue of 1,00,000 coins.
o Lichchhavi Princess Chellana from Vaishali, who gave birth to Ajatashatru.
o Daughter of the chief of the Madra clan (Punjab).
Ajatashatru
 Powerful and aggressive ruler
 Followed his father‘s expansionist policy through military conquest
 Annexed Kosala (ruled by Prasenjit)also added Vaishali to Magadha

https://t.me/prelimbits
194

 Used two innovative military weapons:


o War engine
o Chariot
 Buddha died during his reign. He is said to have met Gautama Buddha. This scene is also
depicted in the sculptures of Barhut. According to the Mahavamsa, he constructed several
chaityas and viharas.
 He arranged 1st Buddhist council
Udayabhadra (Udayin)
 New capital at Pataliputra
Magadha Empire → Shishunaga Dynasty

According to Sri Lankan chronicles, the people of Magadha revolted during the reign of
Nagadasaka (last ruler of Haryanka Dynasty ) and placed an amatya (minister) named Sisunaga
as the king. Sisunaga dynasty lasted from 413 BCE to 345 BCE
Shishunaga
 Temporarily shifted capital to Vaishali
 Defeat Avanti (Pradyota Dynasty) and make it a part of Magadha. This brought an end to
100-year-old rivalry between Magadha and Avanti
Kalasoka [Shishunga son]
 2nd Buddhist Council held at Vaishali
Magadha Empire → Nanda Dynasty (First non-Kshatriya dynasty)

Mahapadma Nanda
 Aka. Ugrasena in Pali texts, because of his large army
 Brahmanical texts - low caste or non-Kshatriya caste
 Puranas – Son of a king of the Shishunaga dynasty by a Shudra woman [considered
adharmika]
 Buddhist texts - annatakula (of unknown lineage).
 Added Kalinga to Magadha and brought an image of Jina as a victory trophy.
 Also acquired Kosala
 Methodical collection of taxes by regularly appointing officials. He also built canals and
did a lot of irrigation work
Dhanananda
 Last important king of the Nanda dynasty
 Believed that Alexander [Greek Invader] attacked Punjab during his reign [ 326 BC ] but
his huge army checked them from advancing towards Magadha.
 Invention of Nandopakramani (a particular measuring standard).
 Chandragupta Maurya under the able guidance of Kautilya, took the advantage of this

https://t.me/prelimbits
195

public resentment and uprooted the Nanda rule and set up the Maurya empire
Causes of Magadha‟s Success
 Advantageous geographical position. Both of its capitals, Rajgriha and Patliputra, were
situated at strategic points.
 centre of the middle Gangetic plain, which had fertile alluvium and received heavy rainfall
 Able and ambitious rulers such as Bimbisara, Ajatashatru, and Mahapadma Nanda.
 Rise of trade, towns, and metal money, the tolls levied too added to the treasury of
Magadhan kings, which again helped them to maintain the huge army.
 unorthodox character of Magadhan society infused in it more enthusiasm for expansion
than any other kingdoms.

139. The Indo-Greek kingdom set up in north Afghanistan in the beginning of the second
century BC was

A. Bactria

B. Scythia

C. Zedrasia

D. Aria

Ans. A

INDO–GREEKS/ BACTRIAN GREEKS

 After the death of Alexander in 323 BCE, many Greeks came to settle on the north-
western borders of India with Bactria (presently northern part of Afghanistan, the area
lying to the south of the Oxus river and to the north-west of the Hindu Kush mountains).
 They were the first to issue gold coins
Demetrius (King of Bactria)
 Invaded India around 190 BCE and probably also came into conflict with Pushyamitra
Shunga
Menander /Milinda/Minedra (165 BCE – 145 BCE)
 The most celebrated Indo-Greek ruler
 He was converted to Buddhism by Nagasena ( Sarvastivadan Buddhist) has been
identified with the King Milinda mentioned in the famous Buddhist text Milindapanho
(Question of Milinda), which contains philosophical questions that Milinda asked
Nagasena (the Buddhist author of the text). The text claims that impressed by the answers,
the king accepted Buddhism as his religion.
Impact of Indo-Greek Rule

 First rulers in India to issue coins (gold, silver, copper, and nickel) which can be definitely
attributed to any dynasty and were also the first to issue gold coins in India (which
increased in number under the Kushanas)
https://t.me/prelimbits
196

 They introduced new features of Hellenistic art and literature such as Gandhara art in
the north-west frontier of India, which was not purely Greek, but rather it was the outcome
of the intermingling and influence of both Indian and central Asian contacts.
 The Hellenistic Greeks are also known for their monumental buildings and their small,
finely-crafted objects
 Also introduced Practice of Military Governorship (the governors were called strategos).
SAKAS (100 BC – 150 CE

 Sakas who were also known as Scythians, were from Western China.
 The first Shaka king was Maues or Moga (approx 80 BC) who is known from inscriptions
and a series of coins.
 Mathura, Ujjain and Girnar were centres of Saka rulers in north India.
 They ruled in capacity of ‗Satrapas’ i.e., governors and Mahasatrapas.
 With control over western Ganga valley, parts of central India and Gujrat, Sakas were almost
always at war with Satvahanas and put pressure over Deccan region.
 Rudradaman (130 CE - 150 CE) of Ujjain centre of Sakas is of significance as he finds
mention in Junagarh inscription. This inscription is in Sanskrit unlike previous Prakrit
inscriptions. According to this inscription, he had even defeated the Satavahanas in
battle.
o Rudradaman got the Lake Sudarshan (at Kathiawar) repaired for better irrigation
(constructed during the time of Chandragupta Maurya).
 It was again repaired under the Skandgupta's reign by his governor
Parramatta during 415 AD- 455AD.
 Huge number and great variety of silver coins are reported from western India that are
attributed to the Sakas.
 Patronising Indian art and culture many of the Saka rulers got themselves Indianized.
 Important centre of development of art were Sanchi, Mathura and Gandhar.
 The King of Ujjain defeated Sakas and assumed the title of 'Vikramaditya' and est. the
Vikram Samvat or era in 57 BCE

140. [1999]

Assertion (A): The Aham and Puram poems of the Padinen Kilukanakku group formed a
continuation of the Sangam composition.

Reason (R): They were included under the Post Sangam works as against the Sangam
works proper.

A. Both A and R are true, and R is the correct explanation of A

B. Both A and R are true, but R is not a correct explanation of A

C. A is true, but R is false


https://t.me/prelimbits
197

D. A is false, but R is true

Ans. A

AHAM & PURAM

 Early classical Tamil literature is known as Sangam literature meaning fraternity,


indicating mainly 2 schools of poets,

o aham (subjective love poems), and

o Puram (objective, public poetry and heroic).

 Aham deals purely with the subjective emotions of the lover.

 Puram deals with all kinds of emotions, mainly the valour and glory of kings, and about
good and evil.

 The Aham and Puram poems belong to post Sangam period (100 to 500 A.D).

o The didactic works of Sangam period are called Kilukanakku (18 minor groups)
consisting of Tirukurral and Naladiyar.

o The Aham and Puram poems of the Kilukanakku group were composed in post-
Sangam period. That‘s why R explains A.

141. [2000]

Assertion (A): The emphasis of Jainism on nonviolence (ahimsa) prevented


agriculturalists from embracing Jainism.

Reason (R): Cultivation involved killing of insects and pests.

A. Both A and R are true, and R is the correct explanation of A

B. Both A and R are true, but R is not a correct explanation of A

C. A is true, but R is false

D. A is false, but R is true

Ans. A

JAINISM : UNIQUE CONCEPT OF NON-VIOLENCE

 The five vows are common to both the monks and lay followers. But the monks were to
observe the vows more rigorously than the lay followers.
 As Jainism placed great emphasis on non-violence, strict observers of the faith wear a
muslin cloth around their mouth and nose so that they would not inhale small insects
even by mistake.
 To avoid trampling on ants and other insects, Jain monks used feathers to sweep the
path before walking.

https://t.me/prelimbits
198

 Jains could not practice agriculture or other crafts that involve killing or injury to living
organisms.
 Hence they took to trading and money-lending and excelled in it. As a result, they were
closely associated with urbanisation.

142.

Assertion (A): The origin of feudal system in ancient India can be traced to military
campaigns.

Reason (R): There was considerable expansion of the feudal system during the Gupta
period.

A. Both A and R are true, and R is the correct explanation of A

B. Both A and R are true, but R is not a correct explanation of A

C. A is true, but R is false

D. A is false, but R is true

Ans. B

FEUDALISM IN INDIA

 Use of the term feudalism to describe India applies a concept of medieval European origin,
according to which the landed nobility held lands from the Crown in exchange for
military service, and vassals were in turn tenants of the nobles, while the peasants
(villeins or serfs) were obliged to live on their lord's land and give him homage, labor,
and a share of the produce, notionally in exchange for military protection.

 Feudalism is most likely introduced to India when the Kushan Dynasty from
Central Asia invaded India and introduced new policies of their own.
 The term Indian feudalism is used to describe taluqdar, zamindar, jagirdar, ghatwals,
mulraiyats, sardar, mankari, deshmukh, chaudhary and samanta. Most of these systems
were abolished after the independence of India and the rest of the subcontinent.

 From the post-Maurya penod, and especially from Gupta times, certain political and
administrative developments tended to feudalise the state apparatus

 The most striking development was the practice of making land grants to
the brahmanas.

 The early Pali texts of the pre-Maurya period refer to the villages granted to the
brahmanas by the rulers of Kosala and Magadha, but they do not mention the
abandonment of any administrative rights by the donors

 The same is the case with the earliest epagraphic record of a land grant, a

https://t.me/prelimbits
199

Satavahana inscription of the first century BC, which refers to the grant of a village as
a gift in the asvamedha sacrifice

 Surprisingly enough, admimstrative rights were perhaps given up for the first
time in the grants made to Buddhist monks by the Satavahana ruler Gautamiputra
Satakarni m the second century AD

 The land granted to them could not be entered by royal troops, disturbed by
government officials, or interfered with by the district police.

 Significant features of such grants, which became more frequent from the fifth
century AD , were the transfer of all sources of revenue, and the surrender of police
and administrative functions

 The grants of the second century AD mention the transfer of the king‘s control only
over Salt, which implies that King retained certain other sources of revenue

 But in later grants, from the time of Pravarasena II Vakataka onwards (fifth century A D),
the ruler gave up his control over almost all sources of revenue, including pasturage,
hides and charcoal, mines for the production of salt, forced labour, and all hidden
treasures and deposits

 According to some grants of the fourth and fifth centuries AD the brahmanas were
granted the right of enjoying the hidden treasures and deposits of the villages, this
meant the transfer of royal owner shift over mines, which was an Important sign of the
king‘s sovereignty

 Equally important is the fact that the donor not only abandonded his revenues but
also the right to govern the mhabitants of the villages that were granted
 The Gupta period furnishes at least half dozen instances of grants of apparently
settled villages made to the brahmanas, in which the residents, including the
cultivators and artisans, were expressly asked by their respective rulers not
only to pay the customary taxes to the donees, but also to obey their
commands.
 In two other land grants of post-Gupta times royal command were issued to government
officials that they should not cause any disturbance to the brahmanas. All this provide
clear evidence of the surrender of the adrnimstrative power of the state.

 The inscriptions of the fifth century AD show that the ruler generally retained the right
to punish thieves, which was one of the main bases of the state power. The process of
delegation reached its logical end when in later times the king made over to the
brahmanas not only this right, but also bis right to punish all offences against family,
property, person, etc

 Of the seven organs of the state power mentioned in literary and epagraphic sources,
https://t.me/prelimbits
200

taxation system and coercive power based on the army are rightly regarded as two vital
elements. If they are abandoned, the state power disintegrates. But this is the position
created by the grants made to the brahmanas. The fiefs (granted land) are usually
granted for as long as the existence of the sun and the moon, which implies the
permanent break-up of the integrity of the state

 The grants to priests can be traced back to pre-Maurya and Maurya times Kautilya
recommends grants of land in new settlements according to the brahmadeya tenure,
which cames freedom from taxes and punishments. But the position changes in the Gupta
period

 Commenting on the term brahmadeyya in the early PaIi texts, Buddhaghosa, who
flourished during the fifth century A D, states that the brahmadeyya grant carries with
it judicial administrative rights, which is corroborated by contemporary epigraphic
evidence. This interpretation of the term brahmadeyya does not reflect the position in the
pre-Maurya period but in the time of the commentator.

 Thus the widespread practice of making land grants in the Gupta period paved
the way for the rise of brahmana feudatories, who performed administrative
functions not under the authority of the royal officers but almost independently

Source: RS Shrama (Indian Feudalism)

143.

Assertion (A): Ashoka annexed Kalinga to the Mauryan Empire.

Reason (R): Kalinga controlled the land and sea routes to South India.

A. Both A and R are true, and R is the correct explanation of A

B. Both A and R arc true, but R is not a correct explanation of A

C. A is true, but R is false

D. A is false, but R is true

Ans. A

ASHOKA (268 - 232 BCE)

 Eliminating other claimants, Ashoka was coronated to the throne in 269 BC. Radhagupta,
helped Ashoka in usurping the throne.
 Regarded as one of the greatest kings of all times, and probably was the first ruler to
maintain direct contact with his people through his inscriptions. The various names of
the emperor include :
o Buddhashakya and Ashoka (in the Maski Edict),
 First edict of Emperor Ashoka that contained the name Ashoka

https://t.me/prelimbits
201

 Archaeological site in the Raichur district of the state of Karnataka, India. It


lies on the bank of the Mask iriver which is a tributary of the Tungabhadra
o Dharmasoka (Sarnath inscription),
o Devanampiya (meaning beloved of the gods), and Piyadassi (meaning of pleasing
appearance)
 8 years after coronation, Ashoka fought the horrible Kalinga War in 261 BC.
o Kalinga was once under the rule of the Nanda Empire until it fell in 321 BC.
Chandragupta Maurya had tried to conquer the state, but failed to do so.
o Kalinga was an important kingdom because it controlled the trade routes with
South India & Southeast Asia. The important ports on the Kalinga coastline
were Tamralipta, Khalkatapatna, Manikapatna (Chelitalo), Palur (Dantapura),
Gopalpur (Mansurkota), Dosarene, Sonapur, Baruva (Barua), Kalingapatnam,
Pithunda
o A powerful state on the fringes of the Mauryan Empire would be problematic as it
could disrupt the communication lines between the Mauryan capital of Pataliputra
and its holdings in the central Indian peninsula.
o So Ashoka wanted to capture it and lead his huge army to Kalinga. Raja Ananta
Padmanabha was the king of Kalinga during Kalinga war.
 Ashoka was moved by the untold miseries caused by the war, renounced conquest by
warfare, in favour of cultural conquest. In other words, Bherighosha was replaced with
Dhammaghosha.
 His empire covered the whole territory from Hindukush to Bengal, and extended over
Afghanistan, Baluchistan, and the whole of India including Kashmir and the valleys of Nepal
(the first empire to do so), with the exception of a small area in the far south (which,
according to rock edict 13, were inhabited by the Cholas and Pandyas, and according to
rock edict 2, by the Keralaputras and Satiyaputras).
 Ashoka exchanged missions, both diplomatic and other, with his western
contemporaries
 He was a great proponent of Buddhism. According to tradition, and as mentioned in the
Mahavamsa and Dipavamsa, he was converted to Buddhism by his nephew Nigrodha
According to the Divyavadana, Samudra (a merchant - turned-monk) converted him.
 Buddhism for the first time went outside India during his reign. He sent his son
Mahendra and daughter Sanghamitra for propagation of Buddhism to Sri Lanka.
 Appointed Dharma Mahamattas to propagate dharma among various social groups
including women (in the 14th year of his reign).
 Organised the third Buddhist council at Pataliputra
 He also visited Lumbini (the birth place of Buddha).
 Banned animal sacrifice, regulated the slaughter of animals for food, and established
https://t.me/prelimbits
202

dharmashalas, hospitals, and sarais throughout his kingdom.

144. Which one of the following dynasties was ruling over North India at the time of
Alexander's invasion? [2000]

A. Nanda

B. Maurya

C. Sunga

D. Kanva

Ans. A

NANDA EMPIRE

 The Nanda dynasty ruled in the northern part of the Indian subcontinent during the
fourth century BCE, and possibly during the fifth century BCE.

 The Nandas overthrew the Shaishunaga dynasty in the Magadha region of eastern India,
and expanded their empire to include a larger part of northern India.

 Ancient sources differ considerably regarding the names of the Nanda kings and the
duration of their rule, but based on the Buddhist tradition recorded in the Mahavamsa,
they appear to have ruled during 345–322 BCE, although some theories date the start of
their rule to fifth century BCE.

 The Nandas built on the successes of their Haryanka and Shaishunaga predecessors,
and instituted a more centralised administration.

 Ancient sources credit them with amassing great wealth, which was probably a result of
introduction of new currency and taxation system.

 Ancient texts also suggest that the Nandas were unpopular among their subjects
because of their low status birth, excessive taxation, and their general misconduct.

 The last Nanda king was overthrown by Chandragupta Maurya, the founder of the
Maurya empire, and the latter's mentor Chanakya.

 Modern historians generally identify the ruler of the Gangaridai and the Prasii
mentioned in ancient Greco-Roman accounts as a Nanda king.

 While describing Alexander the Great's invasion of north-western India (327–325 BCE),
the Greco-Roman writers depict this kingdom as a great military power. The prospect of a
war against this kingdom, coupled with the exhaustion resulting from almost a decade of
campaigning, led to a mutiny among Alexander's homesick soldiers, putting an end to
his Indian campaign.

Origins

 Both Indian and Greco-Roman traditions characterize the dynasty's founder as of low

https://t.me/prelimbits
203

birth. According to Greek historian Diodorus (1st century BCE), Porus told Alexander
that the contemporary Nanda king was thought to be the son of a barber.

 The Jain tradition, as recorded in the Avashyaka Sutra and Parishishta-parvan,


corroborates the Greco-Roman accounts, stating that the first Nanda king was the son of
a barber. According to the 12th century text Parishishta-parvan, the mother of the first
Nanda king was a courtesan. However, the text also states that the daughter of the last
Nanda king married Chandragupta, because it was customary for Kshatriya girls to
choose their husbands; thus, it implies that the Nanda king claimed to be a Kshatriya, that
is, a member of the warrior class.

Rulers

According to the Sri Lankan Buddhist text Mahavamsa, written in Pali language, there were 9
Nanda kings – they were brothers who ruled in succession, for a total of 22 years.[7] These nine
kings were

 Mahapadma or Ugra-sena (Uggasena in Pali)

 Panduka

 Pandugati

 Bhuta-pala

 Rashtra-pala

 Govishanaka

 Dasha-siddhaka

 Kaivarta

 Dhana

Imperial extent

 The Nanda capital was located at Pataliputra (near present-day Patna) in the Magadha
region of eastern India. This is confirmed by the Buddhist and Jain traditions, as well as
the Sanskrit play Mudrarakshasa.

 The Nanda empire appears to have stretched from present-day Punjab in the west to
Odisha in the east.

 An analysis of various historical sources – including the ancient Greek accounts, the Puranas,
and the Hathigumpha inscription – suggests that the Nandas controlled eastern India,
the Ganges valley, and at least a part of Kalinga.

 It is also highly probable that they controlled the Avanti region in Central India, which made
it possible for their successor Chandragupta Maurya to conquer present-day Gujarat
western India.

 According to the Jain tradition, the Nanda minister subjugated the entire country up to
https://t.me/prelimbits
204

the coastal areas.

 The Puranas state that the Nanda king Mahapadma destroyed the Kshatriyas, and
attained undisputed sovereignty. The Kshatriyas said to have been exterminated by him
include Maithalas, Kasheyas, Ikshvakus, Panchalas, Shurasenas, Kurus, Haihayas,
Vitihotras, Kalingas, and Ashmakas.

Military strength

 Alexander the Great invaded north-western India at the time of Dhana Nanda
(Agrammes or Xandrames) whom modern historians generally identify as the last Nanda
king

 In the summer of 326 BCE, Alexander's army reached the Beas River (Greek: Hyphasis),
beyond which the Nanda territory was located.

 The Nanda army did not have the opportunity to face Alexander, whose soldiers
mutinied at the Beas River, refusing to go any further in the east. Alexander's soldiers had
first started to agitate to return to their homeland at Hecatompylos in 330 BCE, and the stiff
resistance that they had met in north-western India in the subsequent years had
demoralised them. They mutinied, when faced with the prospect of facing the
Nanda army, forcing Alexander to withdraw from India.
Administration

 The Puranas describe the Nanda king as ekarat ("single ruler"), which suggests that the
Nanda empire was an integrated monarchy rather than a group of virtually independent
feudal states.

 However, the Greek accounts suggest the presence of a more federated system of
governance.

https://t.me/prelimbits
205

 Historian H. C. Raychaudhuri theorises that the Nandas held centralised control over
their core territories in present-day Bihar and Uttar Pradesh, but allowed considerable
autonomy in the frontier parts of their empire. This is suggested by Buddhist legends,
which state Chandragupta was unable to defeat the Nandas when he attacked their
capital but was successful against them when he gradually conquered the frontier
regions of their empire.

 Historians have put forward various theories to explain the political success of these
dynasties of Magadha. Pataliputra, the capital of Magadha, was naturally protected
because of its location at the junction of the Ganges and the Son rivers.

 The Ganges and its tributaries connected the kingdom with important trade routes. It
had fertile soil and access to lumber (wood) and elephants of the adjacent areas.

 D. D. Kosambi theorised that Magadha's monopoly over iron ore mines played a major
role in its imperial expansion, but historian Upinder Singh has disputed this theory,
pointing out that Magadha did not have a monopoly over these mines, and the iron
mining in the historical Magadha region started much later. However, he notes that the
adjoining Chota Nagpur Plateau was rich in many minerals and other raw
materials, and access to these would have been an asset for Magadha
 Several historical sources refer to the great wealth of the Nandas. According to the
Mahavamsa, the last Nanda king was a treasure-hoarder, and amassed wealth worth 80
kotis (800 million). He buried these treasures in the bed of the Ganges river.

 The Kashika, a commentary on Panini's grammar, mentions Nandopakramani manani –


a measuring standard introduced by the Nandas. This may be a reference to their
introduction of a new currency system and punch-marked coins, which may have been
responsible for much of their wealth. A hoard of coins found at the site of ancient
Pataliputra probably belongs to the Nanda period.

Ministers and scholars

 According to the Jain tradition, Kalpaka was the minister of the first Nanda king.

 He became a minister reluctantly, but after assuming the office, he encouraged the king to
adopt an aggressive expansionist policy. The Jain texts suggest that the ministerial
offices of the Nanda Empire were hereditary.

 It is probable that some of the grammarians who preceded Patanjali lived during the
Nanda period.

Religion

 The Nanda Empire's population included adherents of Hinduism, Buddhism and Jainism.
The Nandas and the Mauryas appear to have patronised the religions originating in the
Greater Magadha region, namely Jainism, Ajivikism, and Buddhism.

 However, the rulers of the empire never engaged in conversion of their subjects to
https://t.me/prelimbits
206

other religions and there is no evidence that these rulers discriminated against any
contemporary religion.

 In the pre-Nanda period, the Vedic Brahmanism was supported by several smaller
kings, who patronised the Brahmin priests. The declining power of these kings under the
more centralised Nanda and Maurya rule appears to have deprived the Brahmins of their
patrons, resulting in the gradual decline of the traditional Vedic society.

 The Jain tradition suggests that several Nanda ministers were inclined towards Jainism.
When Shakatala, a minister of the last Nanda king, died, his son Sthulabhadra refused to
inherit his father's office, and instead became a Jain monk. Sthulabhadra's brother Shriyaka
accepted the post

Unpopularity and Overthrow

 All historical accounts agree that the last Nanda king was unpopular among his subjects.
According to Diodorus, Porus told Alexander that the contemporary Nanda king was a man
of "worthless character", and was not respected by his subjects as he was thought to be of
low origin.

 The Sri Lankan Buddhist tradition blames the Nandas for being greedy and for
imposing oppressive taxation. The Puranas of India label the Nandas as adharmika,
indicating that they did not follow the norms of dharma or righteous conduct.

 The Nanda dynasty was overthrown by Chandragupta Maurya, who was


supported by his mentor (and later minister) Chanakya.

 The Buddhist text Milinda Panha mentions a war between the Nanda general
Bhaddasala (Sanskrit: Bhadrashala) and Chandragupta. According to the text, this war led
to the slaughter of 10,000 elephants; 100,000 horses; 5,000 charioteers; and a billion foot
soldiers. While this is obviously an exaggeration, it suggests that the overthrow of the
Nanda dynasty was a violent affair.

145. The practice of military governorship was first introduced in India by the

A. Greeks

B. Shakas

C. Parthians

D. Mughals

Ans. A

IMPACT OF INDO-GREEK RULE

 First rulers in India to issue coins (gold, silver, copper, and nickel) which can be definitely
attributed to any dynasty and were also the first to issue gold coins in India (which
increased in number under the Kushanas)
https://t.me/prelimbits
207

 They introduced new features of Hellenistic art and literature such as Gandhara art
in the north-west frontier of India, which was not purely Greek, but rather it was the
outcome of the intermingling and influence of both Indian and central Asian contacts.

 The Hellenistic Greeks are also known for their monumental buildings and their
small, finely-crafted objects
 Also introduced Practice of Military Governorship (the governors were called
strategos).

 The Kushanas introduced the „Satrap‟ system of government, under which the empire
was divided into numerous satrapis and each satrapi was placed under the rule of a
satrap. These systems led to the development of a feudatory organisation

146. [2000]

Assertion (A): Harshavardhana convened the Prayag Assembly.

Reason (R): He wanted to popularise only the Mahayana form of Buddhism.

A. Both A and R are individually true, and R is the correct explanation of A

B. Both A and R are individually true, but R is NOT a correct explanation of A

C. A is true, but R is false

D. A is false, but R is true

Ans. C

 The Chinese pilgrim Hsuan Tsang, who is also known as the Prince of Travelers and
author of Si-Yu-Ki, visited India during 629–644 CE.
o He earned Harsha‘s friendship and left a vivid account of the beauty, grandeur,
and prosperity of Kannauj.
o He gives a detailed account of a Grand Assembly held at Kannauj in 643 CE, which
was attended by the representatives of Hinduism, Jainism, and Buddhism.
 The Kannauj Assembly was followed by another spectacular assembly at
Prayaga in the same year.

 While the Kanauj Assembly was a religious assembly to highlight


Mahayanism, the Prayaga Assembly was an assembly of universal
character for offerings of royal charities to all classes of people.

147. Which one of the following animals was NOT represented on the seals and
terracotta art of the Harappan culture?

A. Cow

https://t.me/prelimbits
208

B. Elephant

C. Rhinoceros

D. Tiger

Ans. A

IVC : SEALS
 Most of the seals are square plaque (2x2 square inches) made mostly from Steatite.
 Seals had an animal (no Cow) or human figure on one side and an inscription on the
opposite side or inscriptions on both the sides. [Cow, camel, horse, and lion were not
depicted on seals.]

o Unicorn (bull) was the animal most commonly represented on the seals.

 Seals were primarily used for Commercial Purposes, as Amulets, as form of


Identification, for Educational Purposes as well.
 Seals with symbol similar to „Swastika‟ design have also been found.
 Types – Square or Rectangular.

 The most famous seal is the Pashupati Seal [Proto-Siva]of


Harappan civilization from Mohenjo Daro. It is a seal with a figure
seated cross-legged in the centre with animals around; an
elephant and a tiger to the right of the figure and rhino and a buffalo to its left.

148. Match List I (Ancient site) with List II (Archaeological finding) and select the correct
answer using the codes given below the lists [2001]

List I List II

A. Lothal 1. Ploughed field

B. Kalibangan 2. Dockyard

C. Dholavira 3. Terracotta replica of a plough

D. Banawali 4. An inscription comprising ten large sized signs of the Harappan script

Codes

a) A - 1, B - 2, C - 3, D - 4

b) A - 2, B - 1, C - 4, D - 3

c) A - 1, B - 2, C - 4, D – 3

d) A - 2, B - 1, C - 3, D – 4

Ans. B

MAJOR SITES OF INDUS VALLEY CIVILIZATION

https://t.me/prelimbits
209

SITE (River) FINDINGS

Harappa  Granaries, Red sandstone Male torso, Stone symbols of Lingam and Yoni,
(Ravi) Painted pottery, Mother Goddess, Dice, Copper Bullock cart, Coffin
burials (Only founded in Harrapa)

Mohenjodaro Discovered by R D Banerjee in 1922. Largest site of Indus civilization


 Post cremation burial, Great Granary, Great Bath , Pasupathi seal, Bronze
dancing girl, Unicorn Seals (Most numbers of it in here) & Steatite statue
of Beard Man
 Great Granary: A granary has been found which is the largest building of
the MohenjoDaro. This granary is divided into 27 rooms of different size
and shape. Used for the store surplus food grains. (also found in
Harappa)
 Mohenjo-Daro was the largest city of the Indus valley civilization. Both
Harappa and Mohenjo-Daro can be called the capital cities of the
civilization.

Chanhudaro Discovered by N G Mazumdar in 1931.


(Indus)  Inkpot, Lipstick, Metal workers, Shell-ornament makers and bead makers
shop, dog‟s paw imprint on brick, Terracotta model of bullock cart,
Bronze toy cart.
 Note: It is the only city without a citadel

Lothal Discovered by S Rao in 1953.


(Bhogava)  Important naval trade site, Cremation site, Dockyard, Granaries, Rice
husk, Double burial (male female together)
Guj

Dholavira  Dholavira is a Harappan City in the Rann of Kutch in the state of Gujarat
(Luni) in western India.

guj  Dholavira‘s location is on the Tropic of Cancer. It is one of the five largest
Harappan sites and most prominent archaeological sites in India

 It is located on Khadir Bet Island in the Kachchh Desert Wildlife Sanctuary


in the Great Rann of Kachchh.

 Dholavira is the first site of the ancient Indus Valley Civilisation (IVC) in
India to get the UNESCO World Heritage tag.

 It was discovered in 1968 by archaeologist Jagat Pati Joshi.

 It was a commercial and manufacturing hub for about 1,500 years before
its decline and eventual ruin in 1500 BC.

 Distinct features:

https://t.me/prelimbits
210

o Cascading series of water reservoirs.

o Fortified citadel, a middle town and a lower


town

o Outer fortification.

o Two multi-purpose grounds, one of which


was used for festivities and other as a marketplace.

o Nine gates with unique designs.

o Funerary architecture featuring tumulus — hemispherical


structures like the Buddhist Stupas.

o Multi-layered defensive mechanisms, extensive use of stone in


construction and special burial structures.

o Longest inscription comprising ten large-sized signs of the


Harappan script. It was embedded on a three-meter-long board

Surkotada  Only site with HORSE REMAINS, Oval grave, Pot burials, Soldiers sign on
potsherd
(Gujrat)
 Surkotada also has the citadel and the lower town. Both of them are
joined (and not separated as in Harappa, Mohenjodaro and Kalibangan).

Kalibangan  Bangle factory, Ploughed field surface, Camel bones, Fire altars.
(Ghaggar)
Raj

Banawali  Oval shaped settlement, Lack of systematic drainage system, Barley grains,
(Ghaggar Lapis Lazuli, Fire altars, Only city with Radial streets. The terracotta of
Haryana ) 'Plough' was also found in Banawali.

149. In ancient Indian Buddhist monasteries, a ceremony called Pavarana used to be


held. It was the [2002]

A. Occasion to elect the Sanghaparinayaka and two speakers one on Dhamma and the other
on Vinaya

B. Confession by monks of their offences committed during their stay in the monasteries
during the rainy season

C. Ceremony of initiation of new person into the Buddhist Sangha in which the head is shaved
an when yellow robes are offered

D. Gathering of Buddhist monks on the next day to the full moon day of Ashadha when they
take up affixed abode for the next four months or rainy season.

https://t.me/prelimbits
211

Ans. B

BUDDHISM : KEY TERM

Key Term / Concept Meaning / Purpose

Chaityas Prayer halls of monks

Viharas Monasteries

Pravrajya Ceremony to mark a person‟s going forth from home into


homelessness.

Upasampada Ordination ceremony when the novice [new] becomes a full-fledged


member of the monastic community.

Uposatha Ceremony held on full moon and new moon.

Parajika Literally meaning defeat, it included four most serious offences


involving expulsion from the Sangha (sexual intercourse, taking
what is not given, killing someone, and making false claim
of spiritual attainment).

Pavarana Pavarana is a Buddhist holy day celebrated on Aashvin full moon of


the lunar month. It marks the end of the rainy season, sometimes
called ‗BUDDHIST LENT‘. On this day, each monk must come before
the community of monks (Sangha) and atone for an offense he
may have committed during the Vassa (rainy season).

Upasakas & Male followers & Female followers who have taken refuge in the
Upasikas Buddha, Dhamma and Sangha, but who have not taken monastic vows.

Boddhisattva Enlightened beings who have renounced their own salvation and put
off entering paradise in order to help others attain enlightenment.
They devote all their power and energy to save suffering beings in this
world. As deities of compassion, Bodhisattvas are typically represented
with precious jewellery, elegant garments, and graceful postures.
Maitreya
 Maitreya will be successor of Gautama Buddha. He is also
known as Ajita Boddhisattva.
 He holds a ―water phial‖ in his left hand.
 Earliest mention of Metteyya is in the Digha Nikaya 26 of the Pali
Canon.
 It is said that he will arrive when oceans will decrease in size

https://t.me/prelimbits
212

(that is why keeps a Kumbha or philial in his hand) and will rule
the Ketumati Pure Land (Varanasi).

Bhikkhu Sangha & Sangha of monks & Nuns


Bhikkhuni Sangha

Theragatha & Verses of Elder Monks & Nuns


Therigatha
Therigatha
 Often translated as Verses of the Elder Nuns.
 Is a Buddhist text, a collection of short poems of early
enlightened women who were elder nuns (having experienced
10 Vassa or monsoon periods).
 In the Pali Canon, the Therigatha is classified as part of the
Khuddaka Nikaya, the collection of short books in the Sutta
Pitaka. It consists of 73 poems organized into 16 chapters. It is
the companion text to the Theragatha, verses attributed to
senior monks.

Paribbajaka/Parivr Wanderer
ajaka

Shakra God Indra

Arhats Liberated beings

Sramaṇa One who labors, toils, or exerts themselves (for some higher or
religious purpose) or "seeker, one who performs acts of austerity,
ascetic

150. The ancient Indian play Mudrarakshasa of Visakhadutta has its subject on [2002]

A. a conflict between Gods and Demons of ancient Hindu lore

B. a romantic story of an Aryan prince and a tribal woman

C. the story of the power struggle between two Aryan tribes

D. the court intrigues at the time of Chandragupta Maurya

Ans. D

VISHAKHADATTA

 Sanskrit poet and playwright


 Court poet of Chandragupta II

https://t.me/prelimbits
213

 Work :
o Mudra Rakshas : Narrates the ascent of Chandragupta Maurya to throne.
o Devichandraguptam : The play, king Ramagupta decides to surrender his queen
Dhruvadevi ("Devi") to a Shaka enemy when besieged. Ramagupta's younger
brother Chandragupta enters the enemy camp disguised as the queen, and kills
the enemy ruler.

151. Consider the following statements : [2002]

1. Vardhamana Mahavira's mother was the daughter of Lichchavi chief Chetaka.

2. Gautama Buddha's mother was a princess from the Koshalan dynasty.

3. Parshvanatha, the twenty-third Tirthankara, belonged to Banaras.

Which of these statements is/are correct?

A. Only 1

B. Only 2

C. 2 and 3

D. 1, 2 and 3

Ans. C

TRISHALA (MOTHER OF THE MAHAVIRA)

 Trishala, also known as Videhadatta, Priyakarini, or Trishala Mata (Mother Trishala), was
the mother of Mahavira, the 24th Tirthankara of Jainism, and wife of the Jain monarch,
Siddhartha of Kundagrama, of present-day Bihar. She finds mention in
the Jain texts

 As per Indologist Hermann Jacobi, Vardhaman Mahavira's mother


Trishala was sister of King Chetaka king of the Licchavi tribe. His
third wife, Kshema, was a daughter of the chief of the Madra clan of
Punjab.

Auspicious dreams

 Auspicious dreams are often described in texts of Jainism which forecast


the virtue of children.

 Their number varies according to different traditions and they described


frequently as fourteen or sixteen dreams. According to the Digambara
sect, the number of dreams is 16. While the Svetambara sect believe them to be only
fourteen.

 They are seen by mothers of the prominent figures in Jainism including Tirthankaras,
on the conception of their soul in the womb. This is celebrated as Garbha Kalyanaka
https://t.me/prelimbits
214

 They are interpreted as describing virtues and kingship of a future child.

 They are described in detail as dreams of queen Trishala, mother of Mahavira, in some
Jain texts

MAYA (MOTHER OF THE BUDDHA)

 Queen Maya of Shakya was the birth mother of Gautama Buddha.

 She was sister of Mahapajapati Gotami, the first Buddhist nun ordained by the Buddha

 In Buddhist tradition, Maya died soon after the birth of Buddha, generally said to be
seven days afterwards. Thus Maya did not raise her son who was instead raised by his
maternal aunt Mahapajapati Gotami.

 Maya married King Suddhodana, the ruler of the Sakya clan of Kapilvastu. She was the
daughter of King Suddhodhana's uncle and therefore his cousin; her father was king of
Devadaha.

 She was a princess of the Kollia / Koliya republic. She belonged to the ruling family of
Deodah state, which is now in the Deoria district of Uttar Pradesh.

 Deoria was part of Koshal Rajya in ancient history. The entire area of the Kosala Kingdome
was an ancient center of Aryan civilization.

 Buddha was born as Siddhartha at Lumbini (near Kapilavastu, Nepal)

 The Dream of Queen Maya (the Buddha's Conception) : Queen Maya saw Six-
tusked elephant that descends from heaven to enter her womb. This miraculous
conception marks the Buddha Shakyamuni's final rebirth and physical entrance into this
world.

Buddha's Conception

MOST PROMINENT TIRTHANKARAS


https://t.me/prelimbits
215

Rishabhnath

 He is said to exist before Indus Valley Civilization


 It is mentioned that in Bhagavata Purana, he is referred to as Lord Vishnu.
 Vedas also mention the name of Rishabhnath.
 He had many sons including – Bharat and Bahubali (Note: The Gomateshwara Statue is
dedicated to Bahubali; and is the tallest statue of the world. It is located in
Shravanabelagola in Karnataka.)
 It is also believed that the name of the script ‗Brahmi‘ is inspired by his daughter‘s name.
Mallinath

 Malli was the 19th Tirthankara.


 It is often debated that Malli was a woman however, some Digambara sect Jains believe
that she was reborn as a man and then became a Tirthankara.
Neminatha

 He is the 22nd Tirthankara.


 He is stated to be the cousin of Lord Krishna (A Hindu God.)
 In paintings, he is depicted to adorn dark complexion.
Parsvanath

 Parsvanath was the 23rd tirthankar.


 It is believed that he existed two centuries before Vardhamana Mahavira.
 He was apparently born in Banaras (Uttar Pradesh) around 817 BCE.
 He is said to have propounded Jain religion which was later revived by Mahavira.
 He attained Kaivalya on Mount Sammeta (Parasnath) in Jharkhand.
 According to Svetambaras Sect (White-Clad Sect of Jainism); Parsvnath founded four-fold
restraints:
o Ahimsa
o Satya
o Asteya
o Aparigraha (The fifth one, ‗Brahmacharya‟ was added by Mahavira.)
 Navagraha Jain Temple in Karnataka houses the tallest statue of Parsvanath.
Mahavira

 He was the 24th Tirthankara of Jain religion.


 He was the son of Siddhartha (chief of the Jnatrika clan) and Trishla (Trishala was the sister
of Lichchhavi chief Chetak)

https://t.me/prelimbits
216

 He was born in Kundagrama, a suburb of Vaishali, Vajji [in present-day Bihar].


 He was a contemporary of Gautam Buddha.
 At the age of 30, he left his worldly possessions and sought ascetic life towards Kevala
Gnan.
 He attained Kaivalya under a Sal tree.
 He attained Nirvana in Pavapuri, Bihar.

152. Consider the following statements: [2003]

1. The last Mauryan ruler Brihadratha was assassinated by his commander-in-chief,


Pushyamitra Sunga.

2. The last Sunga king, Devabhuti was assassinated by his Brahmana minister Vasudeva Kanva
who usurped the throne

3. The last ruler of the Kanva dynasty was deposed by the Andhras.

Which of these statements is/are correct?

A. 1 and 2

B. Only 2

C. Only 3

D. 1, 2 and 3

Ans. D

BRIHADRATHA (Last Mauryan King)

 The Maurya Empire declined rapidly after Ashoka and the


later kings had very short reigns.

 The empire became weak and fragmented, and the


Mauryan dynasty came to an end when the last king,
Brihadratha, was killed by his military commander,
Pushyamitra Sunga, in 187 BCE who went on to
establish the Shunga Empire.
SUNGA DYNASTY (185 BCE−73 BCE)
 Sungas are considered Brahmanas and they are referred
to in both Buddhist and Brahminical texts such as the
Bāṇa‘s Harshacharita, the Brihadaranyaka Upanishad,
Panini‟s Ashtadhyayi, Kalidasa‟s Malavikaagnimitram,
Divyavadana and Tarantha‟s (a Buddhist scholar)
account.
Pushyamitra Sunga

https://t.me/prelimbits
217

 Capital at Patliputra and a second capital at Vidisha (under son Agnimitra‘s viceroy).
 Performed two Ashvamedha sacrifices, as mentioned in the Ayodhya stone inscription of
King Dhana
 Drama Malavikaagnimitram mentions the conflict between Pushyamitra and Yajnasena,
the king of Vidarbha (eastern Maharashtra), and the victory of the Sungas over them.
 Defeated the Bactrian Greek king, Demetrius. Mentioned in Patanjali‟s works he called
them yavanas) as well as in Kaildasa‟s Malavikaagnimitram
o Yavanas were Indo-Greeks. They liked pepper, that is why pepper was known as
Yavanapriya.
 Patanjali, the notable grammarian who wrote the Mahabhasya was a contemporary of
Pushyamitra Sunga.

 The Divyavadana (Buddhist Text) gives an account of Pushyamitra‟s cruelty towards


Buddhists and his hatred towards Buddhism.
o But there is enough evidence to show that Pushyamitra patronised Buddhist
art. During his reign the Buddhist monuments at Bharhut and Sanchi were
renovated and further improved.

DIVYAVADANA
 The Divyavadana or Divine narratives are a Sanskrit anthology of Buddhist avadana tales,
many originating in Malasarvastivadin vinaya texts.
 It may be dated to 2nd century CE.
 The stories themselves are therefore quite ancient and may be among the first Buddhist
texts ever committed to writing,
 Typically, the stories involve the Buddha explaining to a group of disciples how a
particular individual, through actions in a previous life, came to have a particular
karmic result in the present.
 A predominant theme is the vast merit (punya) accrued from making offerings to
enlightened beings or at stupas and other holy sites related to the Buddha

BHARHUT STUPA
 Around 100 B.C., a great stupa was made at Bharhut, in the eastern part of present-day
Madhya Pradesh (in Satna District). Bharhut stupa was initially built by Asoka and was
later improvised by the Sungas.
 The railings of the stupa and its one surviving gate are at the Indian Museum in Kolkata.
This is the earliest stupa railing to have survived.
The architectural significance
 Unlike the imperial art of the Mauryas, the inscriptions on railings of Bharhut stupa show
that the reliefs and figures were donated by lay people, monks and nuns. Thus, it is one of
the earliest examples of Maurya popular art.

https://t.me/prelimbits
218

 The railings contain numerous birth stories of the Buddha‟s previous lives, or Jataka
tales.
 The Bharhut stupa represents the aniconic phase of Buddhist
art. Buddha has been represented in the form of symbols.
 The style is generally flat, with low bass relief, and all characters
are depicted wearing the Indian dhoti, except for one
foreigner, thought to be an Indo-Greek soldier, with Buddhist
symbolism.
 The Bharhut stupa railings have numerous images of the
yakshas and yakshis, which have been part of India‘s society
from time immemorial.
 At Bharhut, we find the earliest images of the Yakshas and Yakshis which later become
the part of later art. These embody the spirit of nature and serve to remind us of the divinity
that underlies all that is around us. The Yakshas and Yakshis represent the protection of
nature and its great fertility, which ensures the continuance of life.
 The north gateway of the Vedika at Bharhut shows Kubera whom the Yaksha and Yakshis
attend. We find the images of Yakhsi Chandra and Vrikshika, the one who is shown
intertwined with a tree. One more Yakshi called AsokaDohada, the one who hold an Asoka
Tree leave in her hand and also a child in her womb (two hearts) and intertwines with the
tree like a creeper, representing the fertility.
 One of the sculptures is of Laksmi on the railing of the Bharhut, which is
earliest image of this diety.
 A pillar of the vedika has a depiction of a Greek warrior. He wears boots
and a tunic and has short hair and a headband.
 On another railing, there is a Nagaraja, the serpent king, who is in human
form but has a serpent hood.

 Queen Maya‟s dream, preceding the birth of the Buddha, is also a


major theme on the railing of the Bharhut ―stupa‖. In the early art of Buddhism, the figure of
the Buddha was never represented. Instead, there were symbols of him, such as a seat,
footprints, the Bodhi tree, the wheel and the ―stupa‖.

Agnimitra

 The hero of Kalidasa‟s Malavikagnimitram, described in this drama as the viceroy of


Vidisha.
o It tell the love stary of Agnimitra & maid servant Malvika
The last ruler of the Shunga dynasty was Devabhuti, he was killed by Vasudeva who established
Kanva Dynasty which ruled in eastern and central India.

Contribution
 Shungas issued punch-marked copper coins & silver coins and they inherited and
continued with Mauryan structure of administration

https://t.me/prelimbits
219

 Art and literature received due patronage under Shungas. Mathura school of art achieved
new heights, especially in realistic human depiction. Accessories like gateways and
stone railings were added to a number of stupas.
 Manu‟s ‗Manavdharmashastra’ or ‗Manusmriti‟ and ‗Kalidasa‟s Malvikagnimitram’ are
said to have been composed during this period.
 Yavanarajya inscription, Dhanadeva-Ayodhaya inscription mentions about Shungas.

KANVA DYNASTY [73 BCE to 28 BCE]

 The Kanva dynasty or Kanvayana that overthrew the Shunga dynasty in parts of eastern
and central India, and ruled from 73 BCE to 28 BCE.
 Although the Puranic literature indicates that the Kanva Dynasty ruled from the former
capital of the Shunga Empire in Pataliputra, Magadha in Eastern India, their coins are
primarily found in and around the region of Vidisha in Central India, which had also been
the capital of later Shunga rulers.
 The Kanva dynasty was established by Vasudeva Kanva in 73 BCE

 The Kanva ruler allowed the kings of the Shunga dynasty to continue to rule in obscurity in
a corner of their former dominions.
 There were four Kanva rulers. According to the Puranas, their dynasty was brought to an
end by the Satavahanas in 28 BCE

List of Kanava dynasty rulers

Emperor Reign (BCE) Period

Vasudeva Kanva 73–64 BCE 9

Bhumimitra 64–50 BCE 14

Narayana 50–38 BCE 12

https://t.me/prelimbits
220

Susarman 38–28 BCE 10

 According to the Puranas, the last king of the Kanva dynasty was killed by Balipuccha,
who founded the Andhra dynasty (Satavahanas ).
THE SATAVAHANAS

Maurya rule was succeeded by the Sungas and the Kanvas in the north. However, in Deccan and in
central India, the Satavahanas succeeded the Mauryas
 Satavahanas are considered identical with the Andhras of the Puranas (Matsya Purana).
 But both Purna & Satavahana inscriptions not mention anyting about them. (only refer
to Andhras).
 According to some Puranas, the Andhras ruled for 300 years with their capital at
Pratishthana (modern Paithan) on the Godavari in Aurangabad district.
o 2nd Capital Dhanyakataka (Amaravathi)
 This is also the period designated to the Satavahana Dynasty. The earliest Satavahana
inscriptions belong to the 1st century BCE, when they defeated the Kanvas and
established their rule over parts of central India.
 They ruled over parts of Andhra, Maharashtra, Karnataka and Madhya Pradesh.
 They used matronyms such as Gautamiputra and
Vashishthiputra, although they were not matrilineal or
matriarchal in any sense.
 The Satavahanas assumed the title of
Dakshinapathapati (Lord of the Dakshinapatha). The
Satavahanas are also known in history for starting
the practice of giving royal grants of land to
Brahmanas and Buddhist monks, including those
associated with tax exemptions.
 Land gifted to the Brahmanas was not to be entered or disturbed by the royal troops, was
not to be dug for salt, was free from the control of state officials, and was to enjoy all
sorts of pariharas (immunities). They also promoted Buddhism by granting land to the
monks.
 Founder of empire was Simuka & Greatest one was Gautamiputra Satakarni

153. Emperor Harsha's southward march was stopped on the Narmada river by: [2003]

A. Pulakesin-1
B. Pulakesin-II
C. Vikramaditya-I
D. Vikramaditya-II

https://t.me/prelimbits
221

Ans. B
CHALUKYAS OF BADAMI (Important Kings)
Pulakeshin I

 The Chalukya dynasty was established by Pulakeshin I in 543.

 Pulakeshin I took Vatapi (modern Badami in Bagalkot district, Karnataka) under his
control and made it his capital.

o Aihole was the first capital of Chalukyas where they built numerous temples dating
back to the 6th century CE.

o Later the capital was changed to Badami by Pulkeshin 1 in 543.

 Pulakeshin I and his descendants are referred to as "Chalukyas of Badami".

 They ruled over an empire that comprised the entire state of Karnataka and most of
Andhra Pradesh in the Deccan.

Pulkeshin II (c. 610−642 CE)

 The most powerful and famous king of this dynasty, who


achieved many military successes.

 His eulogy was written by his court poet Ravikirti in the Aihole
Inscription, which Describes →
o His victories against the Kadambas of Banavasi, Alupas
and Gangas of Mysore.
o The defeat of Harshavardhana by Pulakeshin II and
the victory of the Chalukyas over the Pallavas are mentioned
 Pulakeshin II defeated Harsha on the banks of Narmada in the winter of
618-619 CE. Pulakeshin entered into a treaty with Harsha, with the Narmada
River designated as the border between the Chalukya Empire and that of
Harshavardhana.
 Sent an embassy to Persian king Khusro II (the reception of this embassy is depicted in
the Ajanta Caves).
 Hsuan Tsang visited his kingdom and he describes that even though Pulkeshin II was
Hindu, he was tolerant towards Buddhism and Jainism.
 He had conflicts with the Pallavas of Kanchi and annexed the region between Krishna
and Godavari (popularly known as Vengi) from the Pallavas and placed it under his
brother Vishnuvardhana (Chalukyas of Vengi/Eastern Chalukyas).
 Thus the first expedition against the Pallavas, who were at the time ruled by
Mahendravarman I, was a COMPLETE SUCCESS.
 However, the second expedition against the Pallavas PROVED TO BE A COMPLETE

https://t.me/prelimbits
222

FAILURE as King Narsimhavarman of the Pallava dynasty killed Pulkeshin II and


adopted the epithet of Vatapikonda. (conqueror of Vatapi → capital of Chalukya )

The Badami Chalukya dynasty went into a brief decline following the death of Pulakeshin II
due to internal feuds. Badami was occupied by the Pallavas for a period of 13 years.

Vikramaditya I (c. 655−680 CE)

 Succeeded in pushing the Pallavas out of Badami and reestablished authority over whole
kingdom. He not only defeated successive Pallava rulers but also captured their capital
Kanchi.
 Vikramaditya I took the title “Rajamalla” (meaning the Sovereign of the Mallas or
Pallavas).
 Also defeated the Cheras, Cholas and Pandyas.
Vikramaditya II (c. 733−743 CE)

 Known for his repeated invasions of the territory of Tondaimandalam and his subsequent
victories over Pallava King Nandivarman II.
 Engraved a Kannada inscription on the victory pillar of the Kailasanath Temple.
o Lokesvara Temple is the largest temple in the Pattadakal complex. It was built
by Queen Lokamahadevi in the 8th century to commemorate the victory of her
husband Vikramaditya II over the Pallavas.
 During his reign, Arab intruders of the Umayyad Caliphate invaded southern Gujarat,
which was under Chalukya rule, but the Arabs were defeated and driven out by Pulakesi, a
Chalukya governor of Navsari.
 Vikramaditya II later overran the other traditional kingdoms of Tamil country, the
Pandyas, the Cholas and the Cheras in addition to subduing a Kalabhra ruler.

Kirtivarman II (c.743−757 CE)

 Last ruler of this dynasty who was defeated by the Rashtrakuta king Dantidurga. Thus,
Chalukyan rule came to an end in about 757 CE
 At their peak, the Chalukyas ruled a vast empire stretching from the Kaveri in the south to
the Narmada in the north.

154. Consider the following statements: [2003]

1. The Chinese pilgrim Fa-Hien attended the fourth Great Buddhist Council held by Kanishka.
2. The Chinese pilgrim Hiuen-Tsang met Harsha and found him to be antagonistic to
Buddhism.
Which of the statements given above is/are correct?
A. 1 only

https://t.me/prelimbits
223

B. 2 only
C. Both 1 and 2
D. Neither 1 nor
Ans. D
Faxian (Fa Hien) visit : 399 – 412 CE & 4th Buddhist council : Kundalvana, Kashmir : 72 AD
Harsha was not antagonistic (hostile).

FAXIAN (FA HIEN VISIT) (399–412 CE)

 Also referred to as Fa-Hien, Fa-hsien and Sehi, was a Chinese Buddhist monk and
translator who traveled by foot from China to India to acquire Buddhist texts.

 Starting his arduous journey about age 60, he visited sacred Buddhist sites in Central, South
and Southeast Asia between 399 and 412 CE, of which 10 years were spent in India.

 He visited India in the early fifth century. He entered India from the northwest
and reached Pataliputra. He took back with him a large number of Sanskrit
Buddhist texts and images sacred to Buddhism.

 Upon his return to China, he is also credited with translating these Sanskrit texts into
Chinese.

 Faxian's visit to India occurred during the reign of Chandragupta II.

 His memoir describe his 10 year stay in India. He visited the major sites associated with
the Buddha, as well the renowned centers of education and Buddhist monasteries
(including the Nalanda area). He visited Kapilvastu (Lumbini), Bodh Gaya, Benares
(Varanasi), Shravasti, and Kushinagar, all linked to events in Buddha's life.

 Faxian learned Sanskrit, and collected Indian literature from Pataliputra (Patna),
Oddiyana, and Taxila in Gandhara.

 His memoir mentions the Hinayana (Theravada) and emerging Mahayana traditions, as
well as the splintering and dissenting Theravada sub-traditions in 5th-century Indian
Buddhism.

 On Faxian's way back to China, after a two-year stay in Sri Lanka

His visit to Nalanda

 When Faxian, a Chinese Buddhist pilgrim monk, visited the city of Nalanda, there probably
was no university yet.

 However, he makes no mention of any monastery or university at Nalanda even though


he was looking for Sanskrit texts and took a large number of them from other parts of India
back to China. This silence in Faxian's memoir suggests that Nalanda monastery-
university did not exist around 400 CE.

https://t.me/prelimbits
224

FOURTH BUDDHIST COUNCIL

 At Kundalvana, Kashmir (72 AD)

 Under the patronage of Kushan king Kanishka and the president of this
councilwas Vasumitra, with Avaghosa /Ashvaghosha as his deputy.

 This council distinctly divided the Buddhism into 2 sects Mahayana and Hinayana

 The fourth Buddhist council consists of two separate Buddhist councils.


 The first one was held at the Anuradhapura Maha viharaya in Sri Lanka and is
traditionally ascribed to the first century BCE and the next one ( fourth Buddhist
council) was held at kundalavana in 72 AD.
 The purpose of the fourth council was to deal with the critical dispute between the
Gandhara and the sarvastivada teachers of Kashmir.
 During the council, the sarvastivadin beliefs were ordered into a Mahavibhasa including
three large commentaries on the pitakas.

155. With reference to ancient Jainism, which one of the following statements is
correct? [2004]

A. Jainism was spread in South India under the leadership of Sthalabahu


B. The Jainas who remained under the leadership of Bhadrabahu were called Shvetambaras
after the Council held at Pataliputra
C. Jainism enjoyed the patronage of the Kalinga king Kharavela in the first century BC
D. In the initial stage of Jainism, the Jainas worshipped images unlike Buddhists
Ans. C
Initially, like the Buddhists, the Jainas were not image worshipers. Later they began to worship
Mahavira and also the twenty-three tirthankaras. Beautiful and sometimes massive images in stone
were sculpted for this purpose, especially, in Karnataka, Gujarat, Rajasthan, and Madhya Pradesh.
DIFFERENT SCHOOLS OF JAINISM

Digambara Shvetambara

Literally means ‗sky clad‟. Digambaras stress Literally means ‗white clad‟. Shvetambaras
the practice of nudity as an absolute pre- assert that the practice of complete nudity is
requisite to the mendicant‘s path and to the not essential to attain liberation.
attainment of salvation.

Magadha Famine (200 Yrs af. death of Some stay back at Magadha af. the famine
Mahavira), many followers went south under under the leadership of Sthalabahu.
the leadership of Bhadrabahu lived in the old
traditional way.

https://t.me/prelimbits
225

[Mahavira asked to DISCARD CLOTH


completely]

Women lacks the body and rigid will Women are capable in the present lifetime of
necessary to attain moksha she has to be the same spiritual accomplishments
reborn as a man before such an attainment Eg. 19 th Tirthankara is a female named Malli,
is possible who is the only female Tirthankara.

On attaining enlightenment, an omniscient is Even the omniscient requires food.


freed from the needs of ordinary existence
such as, hunger, thirst, sleep, disease, or fear.

Mahavira never married and renounced the Married Princess Yashoda at a fairly young age
world while his parents were alive. and had a daughter from her named
Priyadarshana. It was only after his parents‘
death that he became an ascetic.

Idols of Tirthankaras as nude, unadorned and Wearing a loin-cloth, bedecked with jewels and
with downcast eyes in the contemplative with glass eyes inserted in the marble.
mood.

Canonical Literature = original and genuine Believe in the validity and sacredness of
texts were lost long ago. canonical literature, that is, the 12 angas and
They also refuse to accept the achievements sutras, as they exist now.
of the first council which met under the Tattvartha Sutra is the probably the most
leadership of Acharya Sthulabhadra and authoritative text in Jainism, accepted by
consequently the recasting of the angas. both Svetambara and Digambara.

They eat food only once in a day. eat more than once in a day.

Must Renounce All Possessions, including allowed to have fourteen possessions including
clothes and is allowed only three possessions: White clothes, Rajoharana – broom made up of
Picchi (a small peacock-feather broom for soft white wool to remove small insects from

https://t.me/prelimbits
226

brushing insects away) and a Kamandalu (a their path.


wooden water-pot for toilet hygiene) and
Shastra – scriptures.

Rajoharaṇa
Picchi and Kamandalu

Tirthankara Mallinatha: Digambara Jains Svetambara Jains believe that Mallinatha was a
believe that Mallinatha was a male. female by the name Malli.

Birth and Death of Mahavira Birth of Mahavira - 599 BC


Birth of Mahavira - 582 BC Death of Mahavira - 527 BC
Death of Mahavira - 510 BC

Major Sub-Sects: Mula Sangh, Bisapantha, Major Sub-Sects: Murtipujaka, Sthanakvasi,


Terapantha, Taranpantha or Samaiyapantha. Terapanthi
Minor Sub-Sets: Gumanapantha, Totapantha.

HATHIGUMPHA INSCRIPTION

 Hathigumpha inscription of Kharavela of Kalinga mentions Tamil kingdoms.


 The Hathigumpha Inscription is a seventeen line inscription in Prakrit language incised in
Brahmi script in a cavern called Hathigumpha in Udayagiri hills, near Bhubaneswar in
Odisha, India. Dated between 2nd-century BCE and 1st-century CE, it was inscribed by
the Jain king Kharavela of Kalinga kingdom.

156. Which one of the following four Vedas contains an account of magical charms and
spells? [2004]

A. Rig-veda

B. Yajur-veda

C. Atharva-vcda

D. Sama-veda

Ans. C

ATHARVA VEDA

 The Atharva Veda is a collection of magic spells and charms to ward off the evil spirits
and diseases.

 It is the last Veda and is considered a non-Aryan work divided into 20 kandas (books),
with 711 hymns. Unlike Samaveda where hymns are borrowed from Rigveda, hymns of
https://t.me/prelimbits
227

Atharvaveda are unique except a few

 Brahmana : Gopatha Brahmana.


 Recensions : Shaunaka and Paippalada
 Upveda : Shilpa Veda
 Upanishads of the Atharvaveda:
o Mundaka Upanishad,

o Mandukya Upanishad,

o Prashna Upanishad.

 No Aranyaka which belongs to the Atharvaveda.

OTHER VEDAS

Rig Veda

 The Rig Veda is a collection of 1,028 hymns, divided into 10 Mandals (books).

 They are the earliest compositions and hence, they depict the life of the Early Vedic
people in India.

 Recently, the Rig Veda has been included by the UNESCO in the list of literature signifying
World Human Heritage.

o Earliest Mandals, i.e., II to VII are called family books as they are ascribed to
particular families of seers/rishis.

o Mandal VIII – Mostly relate to Kanva‟s family.

o Mandal IX – Compilation of Soma hymns.

o Mandal I and X – Are later additions and contains the Purusashukta which
explains the four varnas. They also deal with philosophical questions and also talk
about various virtues including a charity in the society

o The priests related to the Rig Veda are Kotri or Motri, and the Upveda of the Rig
Veda is the Ayurveda.

o Only surviving recension of the Rig Veda is the Shakala shakha.

 Rigvedic books 1 and 10 deal with philosophical questions and also talk about various
virtues including a charity in the society

 The meters used to form hymns are Gayatri, Anushtubh, Trishtubh and Jagati
(Trishtubh and Gayatri are most important)

 The Rigvedic hymns are dedicated to various deities, chief of whom are Indra, a heroic
god praised for having slain his enemy.

https://t.me/prelimbits
228

 Upaveds : Ayurveda (Medicine), associated with the Ṛig Veda


 Rigveda Brahmanas : Aitareya-brahmana and the Kaushitaki- (or Sankhayana-)
brahmana

 Aranyakas of the Rigveda:


o Aitareya Aranyaka

o Kaushitaki/ Shankhayana Aranyaka

 Upanishads of the Rigveda :


o Aitareya Upanishad,

o Kaushitaki Upanishad

Sama Veda

 The Sama Veda is the collection of verses mostly taken from the Rig Veda, but arranged
in a poetic form to facilitate singing

 Samaveda is categorised into two parts – Part-I includes melodies called Gana & Part-II
includes three verses book called Archika.

 It is a collection of 1,810 melodies, and also contains the famous Dhrupada raga, later
sung by Tansen.

 The Upveda of the Sama Veda is the Gandharva Veda

 There are two Upanishads embedded in Samaveda

o Chandogya Upanishad and

o Kena Upanishad

 Aranyakas of the Samaveda:


o Talavakara or Jaiminiya-Upanshad Aranyaka

o Chandogya- Aranyaka

 Brahmana
o Panchavimsha / Tandya Brahmana

o Samavidhana Brahmana

o Daivata Brahmana

o Samhitopanishad Brahmana

o Arsheya Brahmana

o Vamsha Brahmana

o Jaiminiya Brahmana
https://t.me/prelimbits
229

 Recensions (Shakhas) of the Sama Veda are

o Kauthuma,

o Jaiminiya (Talavakara), and

o Ranayaniya.

Yajur Veda

 The Yajur Veda deals with the procedure for the performance of sacrifices.

 The texts are further divided into:

o Shukla Yajur Veda/White Yajur Veda/Vajasaneya (contains only the Mantras). It


contains the Madhyandina and Kanva recensions. Vajasaneyi Samhita is the
Samhita in the Shukla Yajurveda

o Krishna Yajur Veda/Black Yajur Veda (contains both mantras and prose
explanations/ commentary). It contains the Kathaka, Maitrayani, Taittiriya, and
Kapishthala recensions.

 The Upveda of the Yajur Veda is the Dhanur Veda.

 Yajurveda consists of various Upanishads

o Brihadaranyaka Upanishad, the Isha Upanishad, the Taittiriya Upanishad, the


Katha Upanishad, the Shvetashvatara Upanishad, the Maitri Upanishad

 Aranyaka of Shukla Yajurveda: Brihadaranyaka


 Brahmana
o Shatapatha Brahmana

o Taittiriya Brahmana

157. Who among the following was not a contemporary of the other three? [2004]

A. Bimbsara

B. Gautama Buddha

C. Milinda

D. Prasenjit

Ans. C

Though we don‟t know the exact dates of following. So we follow Approx. or their Nearby date

 Bimbisara : born c. 543—died 491 BCE

 Buddha : Born 563 BCE or 480 BCE & Died c. 483 BCE or 400 BCE (aged 80)

https://t.me/prelimbits
230

 Prasenajit / Pasenadi : 600 BC – 501

 Menander /Milinda/Minedra (165 BCE – 145 BCE) [so here is big difference & we got our
Ans.]

MENANDER /MILINDA/MINEDRA (165 BCE – 145 BCE)

 The most celebrated Indo-Greek ruler

 He was converted to Buddhism by Nagasena ( Sarvastivadan Buddhist) has been


identified with the King Milinda mentioned in the famous Buddhist text Milindapanho
(Question of Milinda), which contains philosophical questions that Milinda asked
Nagasena (the Buddhist author of the text). The text claims that impressed by the answers,
the king accepted Buddhism as his religion.
PASENADI

 Was an Aiksvaka ruler of Kosala. Savatthi was his capital.


 He succeeded after Sanjaya Mahakosala. He was a prominent Upasaka (lay follower) of
Gautama Buddha, and built many Buddhist monasteries for the Buddha.
BIMBISARA (Founder of the Haryanka Dynasty )
 Bimbisara powerful king
 Contemporary of both Buddha and Mahavira, and paid equal respect to them
 He pursued a three-pronged policy, namely, matrimonial alliances, friendship with
strong rulers, and conquest of weak neighbours to expand the empire.
 Capital at Rajgriha
 Earlier, had rivalry with Avanti‘s king Pradyota, but later became friends and Bimbisara even
sent his royal physician Jivaka to Ujjain (along Dakshinapatha to Avanti) when Pradyota had
jaundice.
 Strengthened his position by three matrimonial alliances
o His first wife was from Kosala named Mahakoshala (sister of Prasenjit), who brought
in dowry the territory of Kashi, which yielded a revenue of 1,00,000 coins.
o Lichchhavi Princess Chellana from Vaishali, who gave birth to Ajatashatru.
o Daughter of the chief of the Madra clan (Punjab).

158. With reference to the invaders in ancient India, which one of the following is the
correct chronological order? [2005]

A. Greeks- Sakas- Kushans


B. Greeks -Kushans -Sakas
C. Sakas -Greeks -Kushans
D. Sakas - Kushans -Greeks
Ans. A

https://t.me/prelimbits
231

 Indo–Greeks/ Bactrian Greeks : Invaded India around 200 or 190 BCE


 SAKAS Ruled : 100 BC – 150 CE
 Kushanas ruled : AD 30 to circa 375
KUSHAN EMPIRE

 The Kushan Empire was a syncretic empire, formed by the Yuezhi (tribe), in the Bactrian
territories in the early 1st century.

 It spread to encompass much of modern-day territory of, Afghanistan, Pakistan,


Uzbekistan, and northern India, at least as far as Saketa and Sarnath near Varanasi
(Benares), where inscriptions have been found dating to the era of the Kushan Emperor
Kanishka the Great

 The Kushans were most probably one of five branches of the Yuezhi confederation

 The founder of the dynasty, Kujula Kadphises , followed Greek religious ideas and
iconography after the Greco-Bactrian tradition, and being a follower of Shaivism. The
Kushans in general were also great patrons of Buddhism , and, starting with
Emperor Kanishka, they also employed elements of Zoroastrianism in their pantheon. They
played an important role in the spread of Buddhism to Central Asia and China.

King Kanishka

 The best known of the Kushana kings was Kanishka, who is thought to have ruled from
127 CE. latest research by Henry Falk indicated that Kanishka ascended the throne in
127 CE
 Kanishka was an ardent follower of Buddhism and hosted the fourth Buddhist
mahasangha or council (the third council had been held in Pataliputra during Ashoka‘s
reign). By now Mahayana Buddhism had become the dominant sect, and Kanishka
supported the missions sent to China to preach Buddhism

o Kushan rulers worshipped both Shiva and the Buddha, and the images of these
two gods appeared on the Kushan coins.

https://t.me/prelimbits
232

o Seveal Kushan rulers were worshippers of Vishnu, as was certainly the case with the
Kushan ruler Vasudeva, whose very name is a synonym for Krishna, an incarnation of
Vishnu.

 Kushana coins were of the highest quality [Number of Gold coins Increased during
Kushana period)

 Kanishka also patronised the Gandhara and Mathura schools of sculpture. At Mathura, a
headless statue of Kanishka depicting him as a warrior been found.
 He controlled the famous Silk Route in Central Asia.

 Another striking aspect of Kanishka‘s personality was his attitude of


religious tolerance as the king‟s coinage depicts motifs drawn from a
great variety of religious traditions: Indian (the motifs of the Shiva and the
Buddha), Greek & West Asian.
SAKAS (100 BC – 150 CE

 Sakas who were also known as Scythians, were from Western China.
 The first Shaka king was Maues or Moga (approx 80 BC) who is known from inscriptions
and a series of coins.
 Mathura, Ujjain and Girnar were centres of Saka rulers in north India.
 They ruled in capacity of ‗Satrapas’ i.e., governors and Mahasatrapas.
 With control over western Ganga valley, parts of central India and Gujrat, Sakas were almost
always at war with Satvahanas and put pressure over Deccan region.

 Rudradaman (130 CE - 150 CE) of Ujjain centre of Sakas is of significance as he finds


mention in Junagarh inscription. This inscription is in Sanskrit unlike previous Prakrit
inscriptions. According to this inscription, he had even defeated the Satavahanas in
battle.
o Rudradaman got the Lake Sudarshan (at Kathiawar) repaired for better irrigation
(constructed during the time of Chandragupta Maurya).
 It was again repaired under the Skandgupta's reign by his governor
Parramatta during 415 AD- 455AD.
 Huge number and great variety of silver coins are reported from western India that are
attributed to the Sakas.
 Patronising Indian art and culture many of the Saka rulers got themselves Indianized.
 Important centre of development of art were Sanchi, Mathura and Gandhar.
 The King of Ujjain defeated Sakas and assumed the title of 'Vikramaditya' and estd. the
Vikram Samvat or era in 57 BCE .
INDO-GREEK KINGDOM

 The Indo-Greek Kingdom, or Graeco-Indian Kingdom, also known historically as the


Yavana Kingdom (Yavanarajya) was a Hellenistic-era Greek kingdom covering various

https://t.me/prelimbits
233

parts of Afghanistan and the northwestern regions of the Indian subcontinent (parts of
modern-day Pakistan and northwestern India).

 This kingdom was in existence from ca. 200 BC to ca. 1 BC.

 During its existence the kingdom was ruled over by 30 successive kings. Menander I,
being the most well known amongst the Indo-Greek kings, is often referred to simply as
―Menander,‖ despite the fact that there was indeed another Indo-Greek King known as
Menander II. Menander I's capital was at Sagala in the Punjab (present-day Sialkot).

 The expression "Indo-Greek Kingdom" loosely describes a number of various dynastic


polities, traditionally associated with a number of regional capitals like Taxila, (modern
Punjab (Pakistan)), Pushkalavati and Sagala.

 The kingdom was founded when the Graeco-Bactrian king Demetrius (and later
Eucratides) invaded India from Bactria in 200 BC.

 The Greeks in the Indian Subcontinent were eventually divided from the Graeco-
Bactrians centered on Bactria (now the border between Afghanistan and Uzbekistan), and
the Indo-Greeks in the present-day northern Indian Subcontinent.

Impact of Indo-Greek Rule

 First rulers in India to issue coins (gold, silver, copper, and nickel) which can be definitely
attributed to any dynasty and were also the first to issue gold coins in India (which
increased in number under the Kushanas)

 They introduced new features of Hellenistic art and literature such as Gandhara art
in the north-west frontier of India, which was not purely Greek, but rather it was the
outcome of the intermingling and influence of both Indian and central Asian contacts.

 The Hellenistic Greeks are also known for their monumental buildings and their
small, finely-crafted objects
 Also introduced Practice of Military Governorship (the governors were called
https://t.me/prelimbits
234

strategos).

159. Consider the following statements:

1. The Ikshvaku rulers of southern India were antagonistic towards Buddhism.

2. The Pala rulers of Eastern India were patrons of Buddhism.

Which of the statements given above is/are correct?

A. 1 only

B. 2 only

C. Both 1 and 2

D. Neither 1 nor 2

Ans. B

ANDHRA IKSHVAKU

 The Ikshvaku dynasty ruled in the eastern Krishna River valley of India, from their capital
at Vijayapuri (modern Nagarjunakonda in Andhra Pradesh) during approximately 3rd and
4th centuries CE.

 The Ikshvakus are also known as the Andhra Ikshvakus or Ikshvakus of Vijayapuri to
distinguish them from their legendary namesakes.

 The Ikshvaku kings were Shaivites and performed Vedic rites, but Buddhism also
flourished during their reign. Several Ikshvaku queens and princes contributed to
the construction of the Buddhist monuments at present-day Nagarjunakonda

Chamtamula (Founder)
 The dynasty's founder Vasishthiputra Chamtamula rose to power after the decline of
the Satavahana power.

https://t.me/prelimbits
235

 He is attested by the Rentala and Kesanapalli inscriptions. The Rentala inscription, dated
to his 5th regnal year, calls him "Siri Caṃtamula". The 4-line Kesanapalli inscription, dated
to his 13th regnal year, and inscribed on the pillar of a Buddhist stupa, names him as the
founder of the Ikshvaku dynasty
Ehuvala Chamtamula
 Vasishthi-putra Ehuvala Chamtamula ruled for at least 24 years
 The Ikshvaku kingdom reached its zenith during his reign. Several Hindu and Buddhist
shrines were constructed during his reign.

 His Patagandigudem inscription is the oldest known copper-plate charter from the
Indian subcontinent.
THE PALAS OF BENGAL : 750-1150

 In 750 AD, Gopala founded Pala dynasty.


 Capital: Muddagiri/ Munger (Bihar)
 Pala kingdom included Bengal and Bihar, which included major cities of Pataliputra,
Vikrampura, Monghyr, Tamralipti.

 Palas were followers of Mahayana Buddhism and Tantric schools of Buddhism.

 Pala had close cultural relations with Tibet. The noted Buddhist scholars, Santarakshita
and Dipankara were invited to Tibet. They introduced a new form of religion there.
 They had close trade contacts and cultural links with South-East Asia.
 Sailendra dynasty (Buddhist) which ruled over Malaya, Java, Sumatra sent many
embassies to the Pala court.
 Pala period is considered as golden period in history of Bengal.
 Pala armies were famous for its vast war elephant cavalry.
 Arab merchant Sulaiman had visited Pala kingdom.
 The Palas power were destroyed by the Sena dynasty under Vijyasena.
Important Rulers

Gopala (around 750 CE)


 As per Khalimpur copper inscription of Dharampala, he founded Pala dynasty, replacing
later Guptas of Magadha and Khadga dynasty.
 Built famous Odantpuri monastery at Bihar, 2nd largest after Nalanda university
Dharampala (770-810)
 Defeated by Rashtrakuta king Dhruva and Pratihara king Nahabhatta II.
 Founded the Vikramshila University near Bhagalpur, Bihar and built Somapuri monastery
(paharpur, Bihar).
 Buddhist scholar, Santarakshita (aka. abbot of Nalanda) belonged to his reign.
https://t.me/prelimbits
236

 Revived Nalanda University (credited 200 village)


Devapala (810-850)
 Expanded empire in the east included Assam.
 His Inscriptions proclaims his victory over Hunas, Gurjaras and Dravidas.
 His Court poet was Buddhist scholar Vajradatta, author of Lokesvarasataka
 Sailendra dynasty king, Balaputradeva requested him to grant five villages to the
monastery at Nalanda.
 Power decline @ late 9th C as they struggle to control over Kannauj & subordinate rulers of
Assam and Orissa assumed Independence.
Mahipala I (c. 977−1027 CE)
 Fortunes of the Palas were restored under Mahipala I in the late 10th century.
 He defended Pala supporter in Bengal and Bihar against Chola invasions, but was defeated
by Rajendra Chola.
Ramapala (c. 1072−1126 CE )
 The last strong Pala ruler, who gained control of Kamarupa and Kalinga and again
revived Pala fortunes in the 11th century.

160. Who among the following laid the foundation of Rashtrakuta Empire?

A. Amoghavarsha I
B. Dantidurga
C. Dhruva
D. Krishna I
Ans. B
THE RASHTRAKUTAS (DECCAN) (752-973)

 Rashtrakutas meaning ‗the chief of a rashtra‟, they considered as the feudatory of the
Chalukyas.
 Capital: Manyakheta or Malkhed near Solapur.
 In the Tripartite Struggle they mostly defeated the Palas and the Pratiharas.
 They also fought constantly against the eastern Chalukyas of Vengi and Pallavas of Kanchi
and Pandya of Madurai.
 Krishna III was the last great ruler of Rashtrakutas. In 972 CE, Capital city was burnt to
ashes by the united opponents. This marks the end of Rashtrakutas.
 They were tolerant in their religious views and patronised Shaivism, Vaishnavism and
Jainism.
o There was an active commerce between the Deccan and the Arabs. Arab traders
https://t.me/prelimbits
237

were also allowed to build mosques and follow their religion without any hindrance.
Important Rulers

Dantidurga (753-756)

 Feudatory of the Chalukyan king, Kirtivarman II


 Founded Rashtrakutas kingdom in 753 CE.
 Performed the Hiranyagarbha ritual at Ujjayini

Hiranyagarbha literally means golden womb. A person coming out of this golden womb, after
elaborate rituals performed by priests, is declared as reborn possessing a celestial body. The
Satavahana king Gautamiputra Sathakarni performed this ritual to claim kshatriya status

Krishna I (756- 774)

 Extended the empire, included present day Karnataka and Konkan under his control.
 Gave final blow to Pallavas.
 The rock-cut Kailasanath (Shiva) temple at Ellora caves was built during his reign. [+
Vishnu images]

Kailasanatha temple (Cave 16)


 Cave 16 of Ellora
 Pallava and Chalukya styles
 Monolith Rock cut temple dedicated to Shiva.
 The sculptured panels of Dasavatara Bhirava, Ravana shaking the Mount Kailasa,
dancing Shiva and Vishnu, and Lakshmi listening to music are the best specimens of
sculpture inside the temple.

Dhruva (780-793)

 Led successful expeditions to Kannauj, defeated the Nagabhatta II (Pratiharas) and


Dharmapala (Pala).
 Brought the Eastern Chalukyas and Gangas of Talakad under his control. The
Rashtrakutas became a pan- Indian power during his rule
Govinda III (793 - 814)

 He emerged victorious at the tripartite struggle.


 He made incursions to north India and defeated Pala king Dharmapala and also wrestled
Malwa from Pratihara king Nagabhatta II.
Amoghavarsha I (814-878)

 Called as ―Ashoka of the South” and also compared to Gupta king Vikramaditya.
 Built new capital city at Manyakheta (Modern Malkhed).
 Broach became the best port of the kingdom during his reign
https://t.me/prelimbits
238

 Defeated Eastern Chalukyas, maintained friendly relations with neighbours.


 He wrote Kavirajamarga (Kannada) and the Prashnottara Ratnamalika (Sanskrit).
 He was follower of Jainism [coverted by Jinasena]

Important Ancient Ports


 Ports of Kalyan, Chaul, Broach, and Cambay in Western India.
 Tamralipti was an important port in Bengal. It carried on trade with China, Lanka, Java and
Sumatra.
 In the Andhra region, the ports were Kadura and Ghantasala, Kaveripattanam (Puhar)
and Tondail were the ports of the Pandya region.
 The ports of Kottayam and Muziris were on the Malabar coast.

161. Anekantavada is a core theory and philosophy of which one of the following? [2006]

A. Buddhism
B. Jainism
C. Sikhism
D. Vaishnavism
Ans. B
JAINISM : IMPORTANT DOCTRINE

Doctrine of Anekantavada

 Opposed to the philosophy of ekanta (one-sidedness or solitary attribute)

 This doctrine encourages acceptance of Relativism and Pluralism.

 Truth and reality are perceived differently from different points of view, and no
single point of view is the complete truth.
 Only the Kevalins can understand the whole truth while all others are capable of
only partial knowledge.
 The origins of Anekantavada can be traced back to the teachings of Mahavira (599–527
BCE).
 The dialectical concepts of syadvada "conditioned viewpoints" and nayavada "partial
viewpoints" arose from Anekantavada in the medieval era, providing Jainism with more
detailed logical structure and expression. The details of the doctrine emerged in Jainism in
the 1st millennium CE, from debates between scholars of Jain, Buddhist and vedic schools
of philosophies

 The Jain doctrine of Anekantavada states that "no philosophic or metaphysical


proposition can be true if it is asserted without any condition or limitation".

https://t.me/prelimbits
239

For a metaphysical proposition to be true, according to Jainism, it must include one or


more conditions (syadvada) or limitations (nayavada, standpoints)
Doctrine of Syadvada

 All judgments are conditional, holding good only in certain conditions, circumstances,
or senses, expressed by the word syat (―may be‖).

 Syadavada literally means the ‗method of examining different probabilities‟.


Nayavada
 Nayavadais the theory of standpoints or viewpoints
 According to Jainism, there are seven nayas or viewpoints through which one can make
complete judgments about absolute reality using syadvada [no need to know these 7
viewpoints]
 The naya theory emerged after about the 5th century CE, and underwent extensive
development in Jainism.
 A particular viewpoint is called a naya or a partial viewpoint. According to Vijay Jain,
Nayavada does not deny the attributes, qualities, modes and other aspects; but
qualifies them to be from a particular perspective.
 A naya reveals only a part of the totality, and should not be mistaken for the
whole. A synthesis of different viewpoints is said to be achieved by the doctrine of
conditional predications (syadvada)

162. The Allahabad Pillar inscription is associated with which one of the following?
[2006]

A. Mahapadma Nanda
B. Chandragupta Maurya
C. Ashoka
D. Samudragupta
Ans : C& D
ALLAHABAD PILLAR INSCRIPTION (PRAYAG PRASASTI)

 "Prayag" refers to a place where something or someone can meet. Prayag is an ancient
name for Allahabad because it is the confluence of the rivers Ganga, Yamuna, and the
mythical Saraswati.
 Prashasti is a eulogy that means "in praise of someone."
 The Allahabad Prasasti was first inscribed on the Ashokan Pillar in Kausambhi, near
Allahabad.
 It was later relocated to the Allahabad fort.

https://t.me/prelimbits
240

 It is an Ashokan Stambh, but it has four different inscriptions,


o The standard Ashokan inscriptions in Brahmi script are found on all pillars.
o The Queen's edict lauding Ashoka's wife Kaurwaki's charitable deeds.

o Inscriptions of Samudragupta (335–375 AD) written by Harisena in Sanskrit It


discusses Samudragupta's conquests and the boundaries of the Gupta Empire.
Inscriptions called him ―Dharma Prachar Bandhu”.

 Harisena, who was not only his court poet but was also a kumaramatya, that
is, a high ranking official who described himself by different titles such as
sandhivigrahika (minister for peace and war) and mahadandanayaka
(judicial/military officer).
o Jahangir‟s Persian inscriptions.

163. Between which of the following was the ancient town of Takshasila (Taxila) located?
[2006]

A. Indus and Jhelum


B. Jhelum and Chenab
C. Chenab and Ravi
D. Ravi and Beas
Ans. A

https://t.me/prelimbits
241

 Taxila or Takshashila is a city in Punjab, Pakistan. In 326 BCE, Alexander the Great
gained control of the city without a battle, as it was immediately surrendered to him by
Omphis.
 Old Taxila was an important city of ancient India, situated on the eastern shore of the
Indus River the pivotal junction of the Indian subcontinent and Central Asia; it was
founded around 1000 BCE.
 Some ruins at Taxila date to the time of the Achaemenid Persian Empire, followed
successively by the Maurya Empire, the Indo-Greek Kingdom, the Indo-Scythians, and
the Kushan Empire.
 Owing to its strategic location, Taxila has changed hands many times over the centuries,
with many polities vying for its control.
 When the great ancient trade routes connecting these regions ceased to be important,
the city sank into insignificance and was finally destroyed in the 5th century by the
invading Hunas.
 In mid-19th century British India, ancient Taxila's ruins were rediscovered by British
archaeologist Alexander Cunningham. In 1980, UNESCO designated Taxila as a World
Heritage Site.

164. What does Baudhayan theorem (Baudhayan Sulva Sutras) relate to? [2008]

A. Lengths of sides of a right-angled triangle


B. Calculation of the value of pi
C. Logarithmic calculations
D. Normal distribution curve
Ans. A
BAUDHAYANA SUTRAS
 The Baudhayana sutras are a group of Vedic Sanskrit texts which cover dharma, daily
ritual, mathematics and is one of the oldest Dharma-related texts of Hinduism that have
survived into the modern age from the 1st-millennium BCE.
 They belong to the Taittiriya branch of the Krishna Yajurveda school and are among the
earliest texts of the genre.
 The Baudhayana sutras consist of six texts:
o the Srautasutra, probably in 19 Prasnas (questions),
o the Karmantasutra in 20 Adhyayas (chapters),
o the Dwaidhasutra in 4 Prasnas,
o the Grihyasutra in 4 Prasnas,
o the Dharmasutra in 4 Prasnas and

https://t.me/prelimbits
242

o the Sulbasutra in 3 Adhyayas.

 The Baudhayana Sulbasutra is noted for containing several early mathematical


results, including an approximation of the square root of 2 and the statement of the
Pythagorean theorem.
Other Suthras just in case
 Baudhayana Shrautasutra → related to performing Vedic sacrifices
 Baudhayana Dharmasutra → The structure of this Dharmasūtra is not very clear because it
came down in an incomplete manner.

165. Mahamastakabhisheka, a great religious event, is associated with and done for
whom of the following? [2009]

A. Bahubali
B. Buddha
C. Mahavira
D. Nataraja
Ans. A
MAHAMASTAKABHISHEKA
 The Mahamastakabhisheka ("Grand Consecration", "The Great Indian
Festival") refers to the abhiseka (anointment) of the Jain images when
held on a large scale.

 The most famous of such consecrations is the anointment of the


Bahubali Gommateshwara statue
Gomateshwara Temple [Karnataka]

 It is also known as Bahubali temple.


 The tallest monolithic statue (Of Gomateshwara son of lord
Rishabhanatha) in the world carved out of a single block of granite.
 It is a Jain temple that is 57 feet high.
 Bahubali statue is one of the largest free-standing statues in the world
depicting the prolonged meditation of Bahubali.
 The construction of the statue was commissioned by the Ganga
dynasty minister and commander,
 The Mahamastakabhisheka Mahotsava is an anointing [Abhisheka] ceremony
of the statue of Lord Bahubali. Only once in 12 years

https://t.me/prelimbits
243

166. There are only two known examples of cave paintings of the Gupta period in ancient
India. One of these is paintings of Ajanta caves. Where is the other surviving example
of Gupta paintings? [2010]

A. Bagh caves
B. Ellora caves
C. Lomas Rishi cave
D. Nasik caves
Ans. A
 Gupta Period [4th century CE to late 6th century CE]
 Ajanta Caves [2nd C. – 5th C.]
 Bagh Caves [about the 6th century CE]
 Ellora Cave [5th – 11th AD]
 Nashik / Nasik Caves [2nd century BCE and the 2nd century CE]
BAGH CAVES
 The Bagh Caves are a group of nine rock-cut monuments, situated among the southern
slopes of the Vindhyas in Bagh town of Dhar district in Madhya Pradesh
 These are renowned for MURAL PAINTINGS by master painters of ancient India.

 The Bagh caves, like those at Ajanta, were excavated by master craftsmen on
perpendicular sandstone rock face of a hill on the far bank of a seasonal stream, the
Baghani.
 Buddhist in inspiration, of the nine caves, only five have survived. All of them are
'viharas' or resting places of monks monasteries having quadrangular plan. A small
chamber, usually at the back, forms the 'chaitya', the prayer hall.

 Most significant of these five extant caves is the Cave 4, commonly known as the
Rang Mahal (Palace of Colors).
 The Bagh Caves were quarried in the 5th-6th century AD, in the very late stages of
Buddhism in India, and long after most of the Indian Buddhist Caves had been built,
many of them since the 2nd or 1st centuries BCE. Caves were established by Buddhist
monk Dataka

 Cave 2 is the best preserved cave, also known as "Pandava Cave"

 These paintings are materialistic RATHER THAN spiritualistic.


 Characteristics of paintings are like those of Ajanta Caves.

https://t.me/prelimbits
244

AJANTA CAVES (2ND C. – 5TH C.)

 Approx. 30 rock-cut Caves → Almost exclusively Buddhist.


 Location : Aurangabad district of Maharashtra (UNESCO World Heritage Site)
 The walls of the caves have both Murals and Fresco Paintings(painted on wet plaster) They
use tempera style, i.e. use of pigments.
 The paintings portray human values and social fabric along with styles, costumes and
ornaments of that period. The emotions are expressed through hand gestures.
 The unique feature of the paintings is that each female figure has a unique hairstyle. Even
animals and birds are shown with emotions.
 The Common Themes of these paintings range from Jataka stories to life of Buddha to
elaborate decorative patterns of flora and fauna.
 The outline of the figures is in red ochre, with contours of brown, black or deep red.
Some important paintings at Ajanta are:
 Scenes from the Jataka stories → Buddha‟s former lives as a Bodhisattva, the life of
Gautam Buddha, etc.
 Paintings of various Bodhisattvas in tribhanga pose in Cave No.
o Vajrapani (protector and guide, a symbol of Buddha‘s power)
o Manjusri (manifestation of Buddha‘s wisdom) and
o Padmapani (Avalokitesvara) (symbol of Buddha‘s compassion).
 The Dying Princess in Cave No. 16.
 Scene of Shibi Jataka, where King Shibi offered his own flesh to save the pigeon.
 Painting of Maya Devi, the mother of the Buddha
 Cave No. XVII representing Buddha‟s visit to the door of Yashodhara‟s abode in the city
of Kapilavastu
History
 The Ajanta Caves are generally agreed to have been made in two distinct phases
o 1st Phase → Satavahana period
https://t.me/prelimbits
245

o 2nd Phase (5th century) → During the reign of Hindu Emperor Harishena of the
Vākāṭaka dynasty. The second phase is attributed to the theistic Mahāyāna, or
Greater Vehicle tradition of Buddhist
BARABAR CAVES (322–185 BCE)

 The Barabar Hill Caves are the oldest surviving rock-cut caves in India, dating from the
Maurya Empire (322–185 BCE), some with Ashokan inscriptions, located in the
Makhdumpur region of Jehanabad district, Bihar, 24 km (15 mi) north of Gaya.
 These caves are SITUATED IN THE TWIN HILLS of →
o Barabar (Four caves - Karan Chaupar, Lomas Rishi, Sudama and Visvakarma)
o Nagarjuni (Three Caves- Gopika, Vadithi-ka-Kubha cave & Vapiya-ka-Kubha
cave)
 These rock-cut chambers bear dedicatory inscriptions in the name of →
o "King Piyadasi" for the Barabar group, and
o "Devanampiya Dasaratha" for the Nagarjuni group
 Thought to date back to the 3rd century BCE during the Maurya period, and to
correspond respectively to Ashoka (reigned 273–232 BCE) and
his grandson, Dasharatha Maurya.
 The sculptured surround to the entrance to the LOMAS
RISHI CAVE is the earliest survival of the ogee shaped
"chaitya arch" or chandrashala that was to be an important
feature of Indian rock-cut architecture and sculptural
decoration for centuries.
 The caves were used by ascetics from the Ajivika sect,
founded by Makkhali Gosala, a contemporary of Gautama
Buddha, the founder of Buddhism, and of Mahavira, the last
and 24th Tirthankara of Jainism.
 The Ajivikas had many similarities with Buddhism as well as
Jainism.
 Most caves at Barabar consist of two chambers, carved entirely out of granite, with a
highly polished internal surface, the "Mauryan polish" also found on sculptures,
and exciting echo effects.
NASHIK / NASIK CAVES

 Located near Nashik in Maharashtra


 It is called as Pandavleni Caves/ Trirashmi Buddhist Caves
 It is a cluster of 24 caves carved between the 2nd century BCE and the 2nd century CE
 It is representing the Hinayana Buddhist caves

https://t.me/prelimbits
246

 Patronage → Can be associated with Satavahana and Kshatarapa rulers


 The inscription of Gautamiputra Satakarni mother Gautami Balashri at Nasik [written in
Prakrit] mentions him as the conqueror of the Shakas, Pahlavas, and Yavanas.

167. India maintained its early cultural contacts and trade links with Southeast Asia
across the Bay of Bengal. For this pre-eminence of early maritime history of Bay of
Bengal, which of the following could be the most convincing explanation/explanations?
[2011]

A. As compared to other countries, India had a better ship building technology in ancient and
mediaeval times
B. The rulers of southern India always patronised traders, Brahmin priests and Buddhist monks
in this context
C. Monsoon winds across the Bay of Bengal facilitated sea voyages
D. Both (a) and (b) are convincing explanations in this context
Ans. D
 The maritime history of Indian shipbuilding begins right from the time of the Harappan
Civilization.
 As compared to other countries, India had better ship-building technology in ancient and
medieval times.
 It was during the Medieval period that a number of Indian Vessels were constructed purely
for waging war at sea.
 During the days of the composition of Yuktikalpataru (around ninth century CE), it appears
that ship-building was highly advanced.
o Yuktikalpataru written by King Bhoja.
 The rules of southern India always patronized traders, brahmin priests and Buddhist
monks in this context.
o Kanishka patronized Buddhist monks, Kushans patronized traders, Cholas
patronized brahmin priests.
 Indians had Knowledge of Monsoon winds since the Gupta period. Monsoon winds
across the Bay of Bengal facilitated sea voyages, but this was not the main reason that
India maintained its early cultural contacts and trade links with Southeast Asia across
the Bay of Bengal.

168. The Jain philosophy holds that the world is created and maintained by ? [2011]

A. Universal law
B. Universal truth
C. Universal faith
https://t.me/prelimbits
247

D. Universal soul
Ans. A
JAINISM AND NON-CREATIONISM

 According to Jain doctrine, the universe and its constituents—soul, matter, space,
time, and principles of motion have always existed.
 Jainism does not support belief in a creator deity. All the constituents and actions
are governed by UNIVERSAL NATURAL LAWS.
 It is not possible to create matter out of nothing and hence the sum total of matter in
the universe remains the same (similar to law of conservation of mass).
 Jain texts claim that the universe consists of jiva (life force or souls) and ajiva (lifeless
objects). The soul of each living being is unique and uncreated and has existed during
beginningless time.
 The Jain theory of causation holds that a cause and its effect are always identical in
nature and HENCE a conscious and immaterial entity like God cannot create a material
entity like the universe.

 Furthermore, according to the Jain concept of divinity, any soul who destroys its
karmas and desires achieves liberation (nirvana). A soul who destroys all its
passions and desires has no desire to interfere in the working of the universe.

 Moral rewards and sufferings are not the work of a divine being , but a result of
an innate moral order in the cosmos: a self-regulating mechanism whereby the
individual reaps the fruits of his own actions through the workings of the
karmas.
 Through the ages, Jain philosophers have rejected and opposed the concept
of any omnipotent creator god, and this has resulted in Jainism being labeled as
nastika darsana, or an atheist philosophy by the rival religious philosophies.

169. Regarding the Indus Valley Civilization, consider the following statements: [2011 ]

1. It was predominantly a secular civilization and the religious element, though present, did
not dominate the scene
2. During this period, cotton was used for manufacturing textiles in India
Which of the statements given above is/are correct?
A. 1 only
B. 2 only
C. Both 1 and 2
D. Neither 1 nor 2

https://t.me/prelimbits
248

Ans. C
RELIGION OF THE INDUS VALLEY CIVILIZATION
They worshiped lots of things hence we can say that Indus Valley Civilizationwas predominantly a
secular also Not a single structure qualifying as a temple has been found.

 Seal-Male deity Pashupati Mahadeva (proto-siva)-three-horned heads, and is


represented in the sitting posture of a yogi, with one leg placed above the other
surrounded by an elephant, a tiger, a rhinoceros, and below his throne there is a
buffalo, and at his feet two deer.
 Prevalence of the Phallus (lingam) and Yoni worship. The Rig Veda speaks of non- Aryan
people who were phallus worshippers.
 Chief female deity was Mother Goddess. They also worshiped fire.
 The people of the Indus region also worshipped trees (ex: pipal) and animals
(unicorn, humped bull etc).
 Harappans believed in ghosts and evil forces and, therefore, they used amulets against
them.
 In contrast to contemporary Egyptian and Mesopotamian civilizations, Indus Valley
lacks any monumental palaces, even though excavated cities indicate that the society
possessed the requisite engineering knowledge.
 This suggests that religious ceremonies if any may have been largely confined to
individual homes, small temples, or the open air.
 At present only the Great Bath at Mohenjo-Daro is widely thought to have been so used,
as a place for ritual purification.
 The funerary practices of the Harappan civilization are marked by fractional burial (in
which the body is reduced to skeletal remains by exposure to the elements before final
interment), and even cremation.
VIC PEOPLES WERE FIRST TO SPIN AND WEAVE COTTON
 In 1929 archaeologists recovered fragments of cotton textiles at Mohenjo-Daro, in what
is now Pakistan, dating to between 3250 and 2750 BCE.
 Cottonseeds founds at nearby Mehrgarh have been dated to 5000 BCE.
o Mehrgarh is one of the EARLIEST SITES WITH EVIDENCE OF FARMING and
herding in South Asia.

170. The “Dharma” and “Rita” depict a central idea of the ancient Vedic civilization of
India. In this context, consider the following statements : [2011]

1. Dharma was a conception of obligations and of the discharge of one‘s duties to oneself and
to others.
2. Rita was the fundamental moral law governing the functioning of the universe and all it
contained.
https://t.me/prelimbits
249

Which of the statements given above is/are correct?


A. 1 only
B. 2 only
C. Both 1 and 2
D. Neither 1 nor 2
Ans. C
RITA
 In the Vedic religion, Rita is the principle of natural order which regulates and
coordinates the operation of the universe and everything within it.

 In the hymns of the Vedas, Rita is described as that which is ultimately responsible
for the proper functioning of the natural, moral and sacrificial orders.
 Conceptually, it is closely allied to the injunctions and ordinances thought to uphold it,
collectively referred to as Dharma, and the action of the individual in relation to those
ordinances, referred to as Karma – two terms which eventually eclipsed Rita in importance
as signifying natural, religious and moral order in later Hinduism.
DHARMA
 In Hinduism, dharma is one of the four components of the Purusartha, the aims of life,
and signifies behaviours that are considered to be in accord with Rita, the
order that makes life and universe possible.
 It includes duties, rights, laws, conduct, virtues and "right way of living". It had a
transtemporal validity.

171. With reference to the scientific progress of ancient India, which of the statements
given below are correct? [2012]

1. Different kinds of specialized surgical instruments were in common use by the 1st century
AD.
2. Transplant of internal organs in the human body had begun by the beginning of the 3rd
century AD.
3. The concept of sine of an angle was known in the 5th century AD.
4. The concept of cyclic quadrilaterals was known in the 7th century AD.
Select the correct answer using the codes given below:
A. 1 and 2 only
B. 3 and 4 only
C. 1, 3 and 4 only
D. 1, 2, 3 and 4

https://t.me/prelimbits
250

Ans. C
DIFFERENT KINDS OF SPECIALIZED SURGICAL INSTRUMENTS WERE IN COMMON USE BY
THE 1ST CENTURY AD
 We have been told that in ancient India, it was Sushrut who is known to be the first
surgeon.
 He not only used the S-type and U-type surgical instruments but also has documented
plastic surgery.
 However, historians have not been able to assign a definite period to Sushruta.
 The era of Sushrut is most probably BC and definitely, it was prior to the 1st century
AD.
TRANSPLANT OF INTERNAL ORGANS
 Transplant of internal organs in the human body had begun by the beginning of the
6th century BC.
 Sushruta mentioned plastic surgery in which the flaps of nose skins were used to
replace other parts.
THE CONCEPT OF THE SINE

 The concept of the sine of an angle was known in 5th century AD hints towards Surya
Siddhanta was written by Latadeva, a student of Aryabhatta I [As described by al-Biruni]
Surya Siddhanta
 It was cited and extensively quoted in a 6th-century CE text by the legendary Indian
astronomer & mathematician Varahamihira, & was perhaps revised for several centuries
under the same title.
 The ‗Surya Siddhanta‗ describes rules to calculate the motions of various planets and the
moon, relative to various constellations. It shows how to calculate the diameters of
various planets, and also the orbits of various astronomical bodies.
 The text asserts, that the earth is spherical in shape. It treats earth as a stationary globe
around which Sun orbits.
 It calculates the earth‘s diameter to be 8,000 miles (modern: 7,928 miles), diameter of moon
as 2,400 miles (actual ~2,160) and the distance between moon and earth to be 258,000
miles (actual ~238,000).
 The Surya Siddhanta provides methods of calculating the sine values in chapter 2.
CYCLIC QUADRILATERALS
 The concept of cyclic quadrilaterals was known in 6th or 7th century AD hints towards
Brahamgupta, who gave the formula for the area of the cyclic quadrilateral.
 He also wrote:
o Brahmasputasiddhanta (628 CE), in which he hinted at the Law of Gravitation
https://t.me/prelimbits
251

o Khandakhadyaka (665 CE), covering such topics as the longitudes of the planets,
diurnal rotation, lunar and solar eclipses, rising and setting of the sun, the moon‘s
crescent, and conjunctions of the planets.

172. With reference to the guilds (Shreni) of ancient India that played a very important
role in the country's economy, which of the following statements is/are correct? [2012]

1. Every guild was registered with the central authority of the State and the king was the chief
administrative authority on them.
2. The wages, rules of work, standards, and prices were fixed by the guild.
3. The guild had judicial powers over its own members.
Select the correct answer using the codes given below:
A. 1 and 2 only
B. 3 only
C. 2 and 3 only
D. 1, 2 and 3
Ans. C
SHRENIS OR TRADE GUILDS
 The Ramayana and many plays from the Gupta period and Tamil Sangam literature write in
detail about the trade guilds or shrenis.
 These were professional bodies of jewelers, weavers, ivory carvers, or even salt-makers who
came together to control quality production, create sound business ethics, maintain fair
wages and prices, sometimes operated as a cooperative, and controlled the entry of
newcomers by laying down high standards of craftsmanship and enforcing rules regarding
apprenticeship.
 Each guild had its own chief, assisted by others.
o These functionaries were selected with great care.
o Guild members were even entitled to impeach and punish a chief found guilty of
misconduct.
 The shrenis were not necessarily restricted to a locality and were known to move from one
town to another, over a period of time.
 Occasionally, shrenis (of merchants and artisans) came together in a joint organization,
called the nigama, or the equivalent of a chamber of commerce and industry.
o Some nigamas also included a class of exporters, who transported the specialties of a
town over long distances and sold them at higher margins of profit than those they
could obtain locally.
 By all accounts, the shrenis were very sound and stable institutions and enjoyed

https://t.me/prelimbits
252

considerable moral and social prestige not only among their own members but in society at
large.

173. The religion of early Vedic Aryans was primarily of [2012]

A. Bhakti
B. image worship and Yajnas
C. worship of nature and Yajnas
D. worship of nature and Bhakti
Ans. C
Early Vedic Period [1500 BCE – c. 1000 BCE] : Religious Aspect

 Rig Vedic hymns („sukti‟) are praises for Gods and Goddesses and the deity is
anthropomorphic i.e. in human form. Yet, idol worship was not practiced.
 Simple, short and less ritualistic worship and sacrifices were practiced
 Households performed the rituals themselves and any expert priest was not required.
 Chanting of mantras was an important part of the ritual.
 Magic and Omen were not prevalent.
 The gods were categorised into three spheres namely, terrestrial, atmospheric or mid-
sphere and cosmic or celestial sphere.
 A peculiar case of Henotheism or Kathenotheism found in Rig Vedic religion ==>
Worship of a Single God (Supreme God)
 Important gods of Rigvedic time were Indra, Varuna, Agni, Yama and Soma.
 Vishnu was minor god during Rigvedic period.
 Maharshi Vasistha & Vishwamitra were important priests
 Demi-gods:

o Gandharvas (Divine musicians)

o Apasaras (Mistress of Gods)

o Vishwadevas (Intermediate deities)

o Aryaman (Guardian of compacts and marriages)

174. Lord Buddha‟s image is sometimes shown with the hand gesture called
„Bhumisparsha Mudra‟. It symbolizes [2012 ]

A. Buddha‘s calling of the Earth to watch over Mara and to prevent Mara from disturbing his
meditation
B. Buddha‘s calling of the Earth to witness his purity and chastity despite the temptations of
Mara

https://t.me/prelimbits
253

C. Buddha‘s reminder to his followers that they all arise from the Earth and finally dissolve into
the Earth, and thus this life is transitory
D. Both the statements (a) and (b) are correct in this context
Ans. B
BHUMISPARSA MUDRA
 The Bhumisparsa or "earth witness" mudra of Gautama Buddha is one of the most
common iconic images of Buddhism. Other names include "Buddha
calling the earth to witness", and "earth-touching".
 It depicts the story from Buddhist legend of the moment when Lord
Buddha attained complete enlightenment, with Buddha sitting in
meditation with his left hand, palm upright, in his lap, and his right
hand touching the earth.
 In the legend, Lord Buddha was challenged by the evil one, Mara, who argue for a
witness to attest his right to achieve it. In response to Mara, Lord Buddha touched the
ground, and Vasundhara or Dharani (Phra Mae Thorani), the earth goddess, appeared
to be the witnesses for Lord Buddha's enlightenment.

175. The Nagara, the Dravida, and the Vesara are the [2012]

A. three main racial groups of the Indian subcontinent


B. three main linguistic divisions into which the languages of India can be classified
C. three main styles of Indian temple architecture
D. three main musical Gharanas prevalent in India
Ans. C
NAGARA AND DRAVIDIAN STYLES

 Both Nagara and Dravidian styles of architecture emerged from the earlier style known as
the Panchayatana style of architecture.
 Basically, their development can be divided into three stages
o First, there were simple pillars and structures.
o In the second stage, more carvings were added and the top part became heavy, so
the pillars were also made more ornate.
o In the third stage, more decorations were added to the Shikhar (top part) and also
to the pillars. The trajectory of growth was very complex in art.
The major differences between Nagara vs Dravidian Style Temple Architecture

Nagara Style of Temple Architecture Dravidian Style of Temple Architecture

Temples located in Northern India are Temples located in Southern India are classified

https://t.me/prelimbits
254

classified as Nagara Style. as Dravidian Style

Nagara Style has multiple Shikharas The Dravidian style has 1 single Shikhara.

In Nagara Style, there are multiple towers In Dravidian Style, it is always a single tower.

In Nagara Style, Central Tower is Curvilinear In Dravidian Style, Central Tower is shaped like
in shape a Pyramid

In Nagara style, the most prominent In Dravidian Style, the most prominent element
element is the Shikhara is the Gopuram.

In Nagara Style, at the entrance of Sanctum In Dravidian Style, the entrance has
Sanctorum; Ganga and Yamuna rivers are Dwarapalas.
depicted in personified form

Not much importance given to the temple In this style, temple boundaries are given high
boundaries importance

In Nagara style, pedestals are higher than Pedestals are at ground level in the Dravidian
the ground. Style.

In Nagara Style, deities are inside In Dravidian Style, deities are outside.

VESARA STYLE
 Vesara is a hybrid form of Indian temple architecture, with South Indian plan and a shape
that features North Indian details.
 This fusion style likely originated in the historic architecture schools of the Dharwad
region. It is common in the surviving temples of later Chalukyas and Hoysalas in the
Deccan region, particularly Karnataka.
 According to Indian texts, Vesara was popular in central parts of India such as between
the Vindhyas and the river Krishna.

176. With reference to the history of ancient India, which of the following was/were
common to both Buddhism and Jainism? [2012]

1. Avoidance of extremities of penance and enjoyment


2. Indifference to the authority of the Vedas
3. Denial of the efficacy of rituals
Select the correct answer using the codes given below:
A. 1 only
B. 2 and 3 only
C. 1 and 3 only
https://t.me/prelimbits
255

D. 1, 2 and 3
Ans. B
Both Buddhism and Jainism had some similarities in their teachings

 For instance, both rejected the authority of Vedas, emphasised on renunciation and
human effort as the means of obtaining salvation, and established a monastic order
for both men and women.

 Like Buddhism, Jainism is fundamentally atheistic; though it recognises the existence of


gods, yet it refuses to give them importance in the universal scheme of themes and
places the gods lower than Jina (the conqueror).

 Like Buddhist texts, Jaina texts criticise the Brahmanas, their practice of animal
sacrifices (non-violence), and ritual authority giving preference to Brahmanas.

 Both Jainism and Buddhism focus on the superiority of the Kshatriya varna over all
other varnas including Brahmanas.

 They both attempted to give a new connotation to the meaning of Brahmana, shifting
the emphasis from an ascribed status to one achieved by good deeds.

o They use the word „Brahmana‟ in the sense of acknowledging a wise person who
possesses true knowledge and lives an exemplary life.

 Like Buddhism monastic order, people of all castes and social backgrounds were
welcomed in Jainism.

o There are frequent mentions of a learned Jaina monk named Harikeshiya who
belonged to a Chandala family.

o Brahmana varna was represented by Bhadrabahu, Divakara, Jinasena, and


Haribhadra.

o Similarly, Jainism opened its doors to women mendicants who were addressed as
aryika or sadhvi.

Unique concept of Non-Violence (Not Found in Buddhism)

 The five vows are common to both the monks and lay followers. But The monks were to
observe the vows more rigorously than the lay followers.
 As Jainism placed great emphasis on non-violence, strict observers of the faith wear a
muslin cloth around their mouth and nose so that they would not inhale small insects
even by mistake.
 To avoid trampling on ants and other insects, Jain monks used feathers to sweep the
path before walking.
 Jains could not practice agriculture or other crafts that involve killing or injury to living
organisms. Hence they took to trading and money-lending and excelled in it. As a result,
they were closely associated with urbanisation.
https://t.me/prelimbits
256

Jainism is an egalitarian religion. It does not sanction any inequality based on birth. It is one‘s
deeds that determine one‘s status in society and not birth. Jainism believes that “by one‟s action
one becomes a Brahmin, a Kshatriya, or a Vaishya, or a Sudra.” Pride based on birth is
considered a sin. Women were admitted into the monastic order. However, as a woman one
cannot attain salvation. By accumulating merit by good deeds, a woman could be reborn as a
man and then strive to attain salvation

Sallekhana/Santhara [In Jainism & Not Found in Buddhism]

 It is the religious practice of voluntarily fasting to death by gradually reducing the intake
of food and liquids.
 It is viewed in Jainism as the thinning of human passions and the body, and another
means of destroying rebirth-influencing karma by withdrawing all physical and mental
activities. According to Jainism, Karma is the bane of the soul. Karma not only
encompasses the causality of transmigration but is also conceived as an extremely subtle
matter which infiltrates the soul, obscuring its natural, transparent, pure qualities. Karma is
thought of as a kind of pollution that taints the soul with various colors. Based on its
karma, a soul undergoes transmigration and reincarnation in various states of
existence like heavens or hells or as humans or animals.
 It is not considered as a suicide by Jain scholars because it is not an act of passion, nor
does it deploy poisons or weapons.
 Mahavira also fasted unto death (Sallekhana) according to Jaina ideals. His death or
final liberation was a joyous event for the Jains.
 Sallekhana is available to both Jain ascetics and householders.(now uncommon event)

177. Consider the following historical places: [2013]

1. Ajanta Caves
2. Lepakshi Temple
3. Sanchi Stupa
Which of the above places is/are also known for mural paintings?
A. 1 only
B. 1 and 2 only
C. 1, 2 and 3
D. None
Ans. B
Ajanta Caves have mural paintings in caves 1, 2, 16, and 17. Some of the paintings were
commissioned by Harisena of the Vakataka dynasty. The theme of the paintings was Jataka tales.
Lepakshi Temple is renowned for being one of the best repositories of mural paintings of the
Vijaynagar kings. Sanchi Stupa has many beautiful sculptures but not mural paintings.
https://t.me/prelimbits
257

MURALS PAINTINGS

 A mural is a large picture painted or affixed directly on a wall or ceiling. The existence
of mural paintings in India dates back to 2nd century BC to 8-10th century AD.

 Some of the places where this painting is found include- Ajanta, Bagh, Sittanavasal,
Armamalai cave, Ravan Chhaya rock-shelter and Kailashnath temple in Ellora caves.

 Majority of the themes in these paintings relates to religion- Buddhism, Jainism and
Hinduism.

Ajanta Caves (2nd C. – 5th C.)

 Approx. 30 rock-cut Caves → Almost exclusively Buddhist.


 Location : Aurangabad district of Maharashtra (UNESCO World Heritage Site)
 The walls of the caves have both Murals and Fresco Paintings(painted on wet plaster) They
use tempera style, i.e. use of pigments.
 The paintings portray human values and social fabric along with styles, costumes and
ornaments of that period. The emotions are expressed through hand gestures.
 The unique feature of the paintings is that each female figure has a unique hairstyle. Even
animals and birds are shown with emotions.
 The Common Themes of these paintings range from Jataka stories to life of Buddha to
elaborate decorative patterns of flora and fauna.
 The outline of the figures is in red ochre, with contours of brown, black or deep red.
Some important paintings at Ajanta are:
 Scenes from the Jataka stories → Buddha‟s former lives as a Bodhisattva, the life of
Gautam Buddha, etc.
 Paintings of various Bodhisattvas in tribhanga pose in Cave No.
o Vajrapani (protector and guide, a symbol of Buddha‘s power)
o Manjusri (manifestation of Buddha‘s wisdom) and
o Padmapani (Avalokitesvara) (symbol of Buddha‘s compassion).
 The Dying Princess in Cave No. 16.
 Scene of Shibi Jataka, where King Shibi offered his own flesh to save the pigeon.
 Painting of Maya Devi, the mother of the Buddha
 Cave No. XVII representing Buddha‟s visit to the door of Yashodhara‟s abode in the city
of Kapilavastu
History
 The Ajanta Caves are generally agreed to have been made in two distinct phases
o 1st Phase → Satavahana period

https://t.me/prelimbits
258

o 2nd Phase (5th century) → During the reign of Hindu Emperor Harishena of the
Vākāṭaka dynasty. The second phase is attributed to the theistic Mahāyāna, or
Greater Vehicle tradition of Buddhist
VEERABHADRA TEMPLE, LEPAKSHI
 Veerabhadra temple is a Hindu temple located in the Lepakshi, in the state of Andhra
Pradesh
 The temple is dedicated to the Virabhadra, a fierce incarnation of Lord Shiva.
 Built in the 16th century, the architectural features of the temple are in the Vijayanagara
style with profusion of carvings and paintings at almost every
exposed surface of the temple.
 It is one of the centrally protected monuments of national
importance and is considered one of the most spectacular
Vijayanagara temples.

 The fresco paintings are particularly detailed in very bright


dresses and colours with scenes of Rama and Krishna from the
epic stories of the Ramayana, the Mahabharata and the Puranas
and they are well preserved.
 There is a very large Nandi (bull), mount of Shiva, about 200 metres
(660 ft) away from the temple which is carved from a single block of
stone, which is said to be one of the largest of its type in the world.
 The temple is home to many Kannada inscriptions as its located close to Karnataka border.
 History →
o The temple was built in 1530 AD (1540 AD is also mentioned) by Virupanna
Nayaka and Viranna , both brothers who were Governors under the Vijayanagar
Empire during the reign of King Achutaraya, at Penukonda who were native to
Karnataka.

SANCHI STUPA
 Sanchi is a Buddhist complex, famous for its Great Stupa, on a hilltop at Sanchi Town in

https://t.me/prelimbits
259

Raisen District of the State of Madhya Pradesh


 The Great Stupa at Sanchi is one of the oldest stone structures in India, and an important
monument of Indian Architecture.
 It was originally commissioned by the Mauryan emperor Ashoka the Great in the 3rd
century BCE.

 Its nucleus was a simple hemispherical brick structure built over the relics of the
Buddha. It was crowned by the 'chhatra', a parasol-like structure symbolising high rank,
which was intended to honour and shelter the relics.
 The original construction work of this stupa was overseen by Ashoka, whose wife Devi
was the daughter of a merchant of nearby Vidisha. Sanchi was also her birthplace as
well as the venue of her and Ashoka's wedding.

 In the 1st century BCE, four elaborately carved Toranas (ornamental gateways) and a
balustrade encircling the entire structure were added.
 The Sanchi Stupa built during Mauryan period was made of
bricks. The composite flourished until the 11th century.
 Sanchi is the center of a region with a number of stupas, all
within a few miles of Sanchi, including →

o Satdhara (9 km to the W of Sanchi, 40 stupas, the Relics of


Sariputra and Mahamoggallana, now enshrined in the new
Vihara, were unearthed there),

o Bhojpur (also called Morel Khurd, a fortified hilltop with 60


stupas)

o Andher (respectively 11 km and 17 km SE of Sanchi), as well as

o Sonari (10 km SW of Sanchi).

o Further south, about 100 km away, is Saru Maru.

o Bharhut is 300 km to the northeast.


 The monuments at Sanchi today comprise a series of Buddhist monuments starting from
the Mauryan Empire period (3rd century BCE), continuing with the Gupta Empire period
(5th century CE), and ending around the 12th century CE.
 During the following centuries, especially under the Shungas and the Satavahanas, the
Great Stupa was enlarged and decorated with gates and railings, and smaller
stupas were also built in the vicinity, especially Stupa No.2, and Stupa No.3.
 Simultaneously, various temple structures were also built, down to the Gupta Empire
period and later. Altogether, Sanchi encompasses most of the evolutions of ancient
Indian architecture and ancient Buddhist architecture in India, from the early stages of
Buddhism and its first artistic expression, to the decline of the religion in the subcontinent

https://t.me/prelimbits
260

 Sanchi Stupa is depicted on the reverse side of the Indian currency note of ₹200 to
signify its importance to Indian cultural heritage.

178. In the context of the cultural history of India, a pose in dance and dramatics called
„Tribhanga‟ has been a favorite of Indian artists from ancient times till today. Which one
of the following statements best describes this pose? [2013]

A. One leg is bent and the body is slightly but oppositely curved at the waist and neck
B. Facial expressions, hand gestures, and make-up are combined to symbolize certain epic or
historic characters
C. Movements of body, face, and hands are used to express oneself or to tell a story
D. A little smile, a slightly curved waist, and certain hand gestures are emphasized to express
the feelings of love or eroticism.
Ans. A
 The techniques of movement are built around the two basic postures of the Chowk
and the Tribhanga.

 The chowk is a position imitating a square – a very masculine stance with the
weight of the body equally balanced.

 The tribhanga is a very feminine stance where the body is deflected at the neck,
torso and knees.
o Tribhanga consists of three bends in the body; at the neck, waist and knee,
hence the body is oppositely curved at waist and neck which gives it a
gentle "S" shape and is considered the most graceful and sensual of the
Odissi positions

179. The Chinese traveler Yuan Chwang (Hiuen Tsang) who visited India recorded the
general conditions and culture of India at that time. In this context, which of the
following statements is/are correct? [2013]

1. The roads and river routes were completely immune from robbery.
2. As regards punishment for offenses, ordeals by fire, water, and poison were the instruments
for determining the innocence or guilt of a person.
3. The tradesmen had to pay duties at ferries and barrier stations.
Select the correct answer using the codes given below.
A. 1 only
B. 2 and 3 only
C. 1 and 3 only
D. 1, 2 and 3

https://t.me/prelimbits
261

Ans. B
XUANZANG'S (HIUEN T-SANG ) VISIT (630–643 CE)

 Hiuen Tsang, was a 7th-century Chinese Buddhist monk, scholar, traveler, and translator.
He is known for the epoch-making contributions to Chinese Buddhism

 Xuanzang travelled around India between 630 and 643 CE, during the reign of King
Harsha Vardhan.

 He visiting Nalanda in 637 and 642, spending a total of around two years at the
monastery

 His prime objective to visit India was to secure genuine Buddhist scriptures and visit
places of Buddhist interest as he was interested in Mahayana Buddhism.

 He was warmly welcomed in Nalanda where he received the Indian name of Mokshadeva
and studied under the guidance of Shilabhadra, the venerable head of the institution at
the time. He learnt from him Yogachara, a school of thought that had then only partially
been transmitted to China.

 Besides Buddhist studies, the monk also attended courses in grammar, logic, and Sanskrit,
and later also lectured at the Mahavihara.

 Xuanzang returned to China with 657 Sanskrit texts and 150 relics carried by 20 horses in
520 cases. He translated 74 of the texts himself

Yuan Chwang (Hiuen Tsang) made the following observations of India:


 There were different types of houses constructed of wood, bricks, and dung. The streets
of the city were circular and dirty.
 New cities grew up in place of the old ones which got ruined.
 Prayag and Kannauj (capital) were important cities where he witnessed the assemblies.
 Nalanda and Valabhi were the important centres of Buddhist learning.
 Indians wore garments made of cotton, silk, and wool and are lovers of education,
literature, and fine arts.

 He also praised the emperor Harsha and his administration. He wrote that the criminal
law was severe and the offenders were given imprisonment, also trial by
ordeals was in vogue, but still the roads were not safe and he himself got robbed
more than once. Also, the emperor maintained a large, powerful, and well-equipped
army.
 According to him, the government was liberal in the case of tax and revenue
collection. Tradesmen had to pay light duties at ferries and barrier stations.

 The people did not include meat in their diet, the society was divided
based on caste, and women were respected in general.

https://t.me/prelimbits
262

 Brahmanical system of religion was prominent and Sanskrit was the language of the
cultured classes.

180. With reference to the history of Indian rock-cut architecture, consider the following
statements: [2013]

1. The caves at Badami are the oldest surviving rock-cut caves in India.
2. The Barabar rock-cut caves were originally made for Ajivikas by Emperor Chandragupta
Maurya.
3. At Ellora, caves were made for different faiths.
Which of the statements given above is/are correct?
A. 1 only
B. 2 and 3 only
C. 3 only
D. 1, 2 and 3
Ans. C
BARABAR CAVES (322–185 BCE)

 The Barabar Hill Caves are the oldest surviving rock-cut caves in India, dating from the
Maurya Empire (322–185 BCE), some with Ashokan inscriptions, located in the
Makhdumpur region of Jehanabad district, Bihar, 24 km (15 mi) north of Gaya.
 These caves are SITUATED IN THE TWIN HILLS of →
o Barabar (Four caves - Karan Chaupar, Lomas Rishi, Sudama and Visvakarma)
o Nagarjuni (Three Caves- Gopika, Vadithi-ka-Kubha cave & Vapiya-ka-Kubha
cave)
 These rock-cut chambers bear dedicatory inscriptions in the name of →
o "King Piyadasi" for the Barabar group, and
o "Devanampiya Dasaratha" for the Nagarjuni group
 Thought to date back to the 3rd century BCE during the Maurya period, and to
correspond respectively to Ashoka (reigned 273–232 BCE)
and his grandson, Dasharatha Maurya.
 The sculptured surround to the entrance to the LOMAS
RISHI CAVE is the earliest survival of the ogee shaped
"chaitya arch" or chandrashala that was to be an important
feature of Indian rock-cut architecture and sculptural
decoration for centuries.
 The caves were used by ascetics from the Ajivika sect,
founded by Makkhali Gosala, a contemporary of Gautama
https://t.me/prelimbits
263

Buddha, the founder of Buddhism, and of Mahavira, the last and 24th Tirthankara of
Jainism.
 The Ajivikas had many similarities with Buddhism as well as Jainism.
 Most caves at Barabar consist of two chambers, carved entirely out of granite, with a
highly polished internal surface, the "Mauryan polish" also found on sculptures,
and exciting echo effects.
ELLORA CAVE [5TH – 11TH AD]

 100 Kms away from Ajanta caves [in Maharashtra's Sahyadri hills]

 34 caves → 17 Brahmanical, 12 Buddhist and 5 Jain [Digambara sect]


o Caves 1–12: Buddhist
o Caves 13–29: Hindu
o Caves 30–34: Jain (Digambara sect)
 Hindu and Buddhist Caves by Rashtrakuta dynasty and Jain Caves by Yadava dynasty.
 Prominent Caves
o Cave No. 10 [Chaitya] also known as Visvakarma Cave or Carpenter‘s Cave. Buddha
is seated in Dharmachakra Mudra here with the Bodhi
tree carved at his back.
o Cave No. 14 is themed ―Raavan ki Khai‖.
o Cave No. 15 is Dashavatar Temple [mainly depicts Lord
Shiva & Lord Vishnu in various forms]
o Cave No. 16 is Kailasha Temple
 Lord Shiva
 By Rashtrakuta king Krishna I
 Monolith sculpture
o Dhumar Lena Cave No. 29
o Rameshwar Lena Cave No. 21
o Jain caves
 Indra Sabha (Cave No. 32)
 Jagannath Sabha (Cave No. 33).

181. Which of the following statements is/are applicable to Jain doctrine? [2013]

1. The surest way of annihilating Karma is to practice penance.


2. Every object, even the smallest particle has a soul.
3. Karma is the bane of the soul and must be ended.
https://t.me/prelimbits
264

Select the correct answer using the codes given below.


A. 1 only
B. 2 and 3 only
C. 1 and 3 only
D. 1, 2 and 3
Ans. D
KARMA IN JAINISM
 Karma is the basic principle within an overarching psycho-cosmology in Jainism. Human
moral actions form the basis of the transmigration of the soul (jiva).
 The Soul is constrained to a cycle of rebirth, trapped within the temporal world
(saṃsāra), until it finally achieves Liberation (moksa). Liberation is achieved by following
a path of purification.
 Jains believe that karma is a physical substance that is everywhere in the universe. Karma
particles are attracted to the soul by the Actions of that soul.
 Karma particles are attracted when we do, think, or say things, when we kill something,
when we lie, when we steal and so on.
 Karma not only encompasses the causality of transmigration, but is also conceived of as
an extremely subtle matter, which infiltrates the soul—obscuring its natural,
transparent and pure qualities.

 Karma is thought of as a kind of pollution, that taints the soul with various colours.
 Based on its karma, a soul undergoes transmigration and reincarnates in various
states of existence—like heavens or hells, or as humans or animals.
 Jains cite inequalities, sufferings, and pain as evidence for the existence of karma.
 The Jain doctrine also holds that it is possible for us to both modify our karma, and to
obtain release from it, through the austerities and purity of conduct.
SOUL OR JIVA (IN JAINISM)

 Jiva or Atman is a philosophical term used within Jainism to identify the soul. As per Jain
cosmology, jiva or soul is the principle of sentience and is one of the tattvas or one of the
fundamental substances forming part of the universe.

 The Jain metaphysics, divides the universe into two independent, everlasting, co-
existing and uncreated categories called the jiva (soul) and the ajiva (non-soul).

 The jiva, according to Jainism, is an essential part of how the process of karma, rebirth
and the process of liberation from rebirth works.

Soul substance

 Jains regard soul as one of the Six Fundamental and Eternal Substances (dravyas) which

https://t.me/prelimbits
265

forms the universe.

 The two states of soul substance are mentioned in the Jain texts. These are — Svabhva
(pure or natural) and Vibhava (impure or unnatural state).

 Souls in transmigration are in impure state and liberated ones are said to be in natural
or pure state.

 The five vows of Jain practice are believed in Jainism to aid in freeing the jiva from
karmic matter, reduce negative karmic effects and accrue positive karmic
benefits.
Souls and rebirth

 According to Jain philosophy, rebirth occurs through soul. Depending on the karmic
particles attached to a soul, Jain theology states a being is reborn in one of four gatis
(states of existence), namely, heavenly being (deva), human (manushya), hell being
(naraki) and animals and plants (triyancha).

 Besides this there also exist a sub-microscopic life form, Nigoda, possessing only one
sense, i.e., of touch.

 In Jain beliefs, souls begin their journey in a primordial state, and exist in a state of
consciousness continuum that is constantly evolving through Sansara. Some evolve to a
higher state. WHILE → Some regress → a movement that is driven by the karma.

 Further, Jaina traditions believe that there exist Abhavya (incapable), or a class of souls
that can never attain moksha (liberation). The Abhavya state of soul is entered after an
intentional and shockingly evil act.

 The Jaina theosophy, like ancient Ajivika, but unlike Hindu and Buddhist theosophies,
asserts that each soul passes through 8,400,000 birth-situations, as they circle through
Saṃsāra.

 As the soul cycles, Jainism traditions believe that it goes through five types of bodies:
earth bodies, water bodies, fire bodies, air bodies and vegetable lives.

 With all human and non-human activities, such as rainfall, agriculture, eating
and even breathing, minuscule living beings are taking birth or dying, their
souls are believed to be constantly changing bodies.
 Perturbing, harming or killing any life form, including any human being, is considered a
sin in Jainism, with negative karmic effects.

 A liberated soul in Jainism is one who has gone beyond Saṃsāra , is at the apex,
is omniscient, remains there eternally, and is known as a Siddha.

 A male human being is considered closest to the apex with the potential to achieve
liberation, particularly through asceticism.

https://t.me/prelimbits
266

 Women must gain karmic merit, to be reborn as man, and ONLY then can they achieve
spiritual liberation in Jainism, particularly in the Digambara sect of Jainism

o However, this view has been historically debated within Jainism and different Jaina
sects have expressed different views, particularly the Shvetambara sect that
believes that women too can achieve liberation from Saṃsāra.

 In contrast to Buddhist texts which do not expressly or unambiguously condemn


injuring or killing plants and minor life forms, Jaina texts do. Jainism considers it a
bad karma to injure plants and minor life forms with negative impact on
a soul's Sansara.
 However, some texts in Buddhism and Hinduism do caution a person from injuring all life
forms, including plants and seeds.

182. Which of the following characterizes/characterize the people of Indus Civilization?


[2013]

1. They possessed great palaces and temples.


2. They worshipped both male and female deities.
3. They employed horse-drawn chariots in warfare.
Select the correct statement/statements using the codes given below.
A. 1 and 2 only
B. 2 only
C. 1, 2 and 3
D. None of the statements given above is correct
Ans. B
RELIGION OF THE INDUS VALLEY CIVILIZATION
They worshiped lots of things hence we can say that Indus Valley Civilizationwas predominantly a
secular also Not a single structure qualifying as a temple has been found.

 Seal-Male deity Pashupati Mahadeva (proto-siva)-three-horned heads, and is


represented in the sitting posture of a yogi, with one leg placed above the other
surrounded by an elephant, a tiger, a rhinoceros, and below his throne there is a
buffalo, and at his feet two deer.
 Prevalence of the Phallus (lingam) and Yoni worship. The Rig Veda speaks of non- Aryan
people who were phallus worshippers.
 Chief female deity was Mother Goddess. They also worshiped fire.
 The people of the Indus region also worshipped trees (ex: pipal) and animals
(unicorn, humped bull etc).

https://t.me/prelimbits
267

 Harappans believed in ghosts and evil forces and, therefore, they used amulets against
them.
 In contrast to contemporary Egyptian and Mesopotamian civilizations, Indus Valley
lacks any monumental palaces, even though excavated cities indicate that the society
possessed the requisite engineering knowledge.
 This suggests that religious ceremonies if any may have been largely confined to
individual homes, small temples, or the open air.
 At present only the Great Bath at Mohenjo-Daro is widely thought to have been so used,
as a place for ritual purification.
 The funerary practices of the Harappan civilization are marked by fractional burial (in
which the body is reduced to skeletal remains by exposure to the elements before final
interment), and even cremation.
IVC : DOMESTICATION OF ANIMALS

 Oxen, buffaloes, goats, sheep, and pigs, dogs, cats, asses and camels were domesticated.
Humped Bulls were favoured by the Harappans.
 Not horse centred but evidence in Surkotada, Mohenjodaro and Lothal. Lion was not
known. Elephants and Rhinoceros (Amari) & deer were well known
 The evidence of the horse comes from a superficial level of Mohenjo-Daro and from a
doubtful terracotta figurine from Lothal. The remains of the horse are reported from
Surkotada, situated in western Gujarat and belong to around 2000 B.C. But the identity is
doubtful. In any case, the Harappan culture was not horse-centered.

183. Which one of the following describes best the concept of Nirvana in Buddhism?
[2013]

A. The extinction of the flame of desire


B. The complete annihilation of self
C. A state of bliss and rest
D. A mental stage beyond all comprehension.
Ans. A
NIRVANA VS. PARINIRVANA

Nirvana (nirvana with remainder)

 In the Buddhist tradition, nirvana has commonly been interpreted as the extinction of the
"three fires",or "three poisons", greed (raga), aversion (dvesha) and ignorance (moha).
 When these fires are extinguished, release from the cycle of rebirth (saṃsāra) is
attained.
Parinirvana (nirvana without remainder)

 Parinirvana is commonly used to refer to nirvana-after-death, which occurs upon the


https://t.me/prelimbits
268

death of someone who has attained nirvana during his or her lifetime.
 It implies a release from the Saṃsāra, karma and rebirth as well as the dissolution of the
skandhas.
Distinguished btw Nirvana with remainder & Nirvana without remainder

 Nirvana with Remainder


o A state achieved prior to death, where ―the remainder‖ refers to the mind and
body of this final existence
 Nirvana without remainder
o Which is achieved at death when the causes of all future existence have been
extinguished and the chain of causation of both physical form and
of consciousness have been finally terminated

These states were available to all who followed the Buddhist path to its conclusion.
The Buddha himself is said to have realized nirvana when he achieved enlightenment at the age
of 35. Although he destroyed the cause of future rebirth, he continued to live for another 45
years. When he died, he entered nirvana, never to be born again. (this event is known as
Mahaparinirvana

The death of the Buddha, or Mahaparinirvana @ Ajanta Cave 26


Massive uddha sculpture. Here Buddha is lying on his right side. The sculpture has a length of 7
metres. The panel below the sculpture shows Buddha‟s disciples and celestial figures float
above. This is one of the grandest and most sensitive depiction in all of Buddhist art. The
Mahaparinirvana of the Buddha, when he achieved release from the mortal world, is a deeply
impressive sculpture.
MAHAPARINIBBAṆA SUTTA
 The Mahaparinibbaṇa Sutta is Sutta 16 in the Digha Nikaya, a scripture belonging to the
Sutta Pitaka of Theravada Buddhism.
 It concerns the end of Gautama Buddha's life - his Parinibbana - and is the longest
sutta of the Pali Canon.
https://t.me/prelimbits
269

 Because of its attention to detail, it has been resorted to as the principal source of reference
in most standard accounts of the Buddha's death

184. With reference to the history of philosophical thought in India, consider the
following statements regarding Sankhya school: [2013]

1. Sankhya does not accept the theory of rebirth or transmigration of the soul.
2. Sankhya holds that it is self-knowledge that leads to liberation and not any exterior
influence or agent.
Which of the statements given above is/are correct?
A. 1 only
B. 2 only
C. Both 1 and 2
D. Neither 1 nor 2
Ans. B
ORTHODOX SCHOOLS OF INDIAN PHILOSOPHY : SANKHYA (Kapila)

 Samkhya is the oldest of the orthodox philosophical systems, and it postulates that
everything in reality stems from purusha (self, soul or mind) and prakriti (matter,
creative agency, energy).
 Purush cannot be modified or changed while prakriti brings change in all objects.
 Emphasizes the attainment of knowledge of self through meditation and concentration.
 School does not believe in the existence of God.
At the same point, it does not describe what happens after moksha and does not
mention anything about Ishwara or God, because after liberation there is no essential
distinction of individual and universal puruṣa.
o So what happens after Moksha is the irrelevant thing for this school.
o But at the same time, though godless, the Sankhya believes in the doctrine of
Karma and of transmigration of souls.
 This philosophy adversely affected the Tantra sadhana a lot.
 Sankhya accepts the theory of rebirth or transmigration of the soul

185. Some Buddhist rock-cut caves are called Chaityas, while the others are called
Viharas. What is the difference between the two? [2013 ]

A. Vihara is a place of worship, while Chaitya is the dwelling place of the monks
B. Chaitya is a place of worship, while Vihara is the dwelling place of the monks
C. Chaitya is the stupa at the far end of the cave, while Vihara is the hall axial to it

https://t.me/prelimbits
270

D. There is no material difference between the two


Ans. B
CHAITYA
 Chaitya were the places of worship and assembly of the monks and it consists of stupa
within the structure at one end
 The earliest surviving spaces comparable to the chaitya hall date to the 3rd century BCE.
These are the rock-cut Barabar Caves (Lomas Rishi Cave and Sudama Cave), excavated
during the reign of Ashoka by or for the Ajivikas, a non-Buddhist religious and
philosophical group of the period
 The chaitya at Bhaja Caves is perhaps the earliest surviving chaitya hall, constructed in
the second century BCE. It consists of an apsidal hall with a stupa.
 The most important of rock-cut complexes are the Karla Caves, Ajanta Caves, Ellora
Caves, Udayagiri and Khandagiri Caves, Aurangabad Caves and the Pandavleni Caves.
o KARLA CAVES near Lonavala, Maharashtra is best-known because of the famous
"Grand Chaitya" (Cave 8), which is "the largest and most completely preserved"
chaitya hall of the period, as well as containing unusual quantities of fine sculpture,
much of it on a large scale.

Drawing of the "Great Chaitya" at the Karla Caves,


VIHARA
 The Vihara are the monastery where the monks used to live and it acts as their
accommodation during the rainy days.
 They are usually carved out of rocks as cave structures.

186. The national motto of India, „Satyameva Jayate‟ inscribed below the Emblem of India
is taken from [2014]

A. Katha Upanishad
B. Chandogya Upanishad

https://t.me/prelimbits
271

C. Aitareya Upanishad
D. Mundaka Upanishad
Ans. D
UPANISHADS

 The literal meaning of Upanishad is to sit near someone. There are 108 Upanishads, of
which 13 are the most prominent.

 The Upanishads dwell on the Atman and Brahman, and focus a lot on the philosophy
about life, universe, self, body, sacrifice, etc.

o Eg . What is this world? Who am I? What becomes of me after death?

 The Upanishads are often called ‗Vedanta‗. Literally, Vedanta means the end of Veda,

 They emphasise that the knowledge of the self or atman should be acquired and that
the relation of the atman with the Brahman should be properly understood.

Important Upanishads

 Aitareya Upanishad: Talks about the creation of Atman (Soul) and Consciousness.
 Chandogya Upanishad: Deals with rhythm and chanting of Mantras. It clearly refers to the
first three ashrams and discusses the types of marriage (mainly two).
o Anuloma marriage - The marriage of a man in his own varna or below his varna.
It is the most accepted and common form of marriage in the society.
o Pratiloma marriage – The marriage of a girl/woman in a varna lower than her
own, deemed rare and not sanctioned by the Vedas.
 Katha Upanishad: Tells the story of Nachiketa and Yama. Their conversation evolves into
discussion of Man, Atman (soul), Knowledge, and Moksha (liberation).
 Brihadaranyaka Upanishad: Talks about the transmigration of Atman; passages on
metaphysics and ethics. It contain Famous universal prayer for enlightenment
„Asato ma sad-gamaya;
Tamaso ma jyotir-gamaya;
Mrtyor-ma amrutam gamaya.‟
 Mundaka Upanishad: Contains the mantra ―Satyameva Jayate‖ (truth alone triumphs)
which is borrowed in the National Emblem of India.
Principal thirteen Upanishads, related to the Vedas are : (More imp for State Exams)

 Upanishads of the Rigveda

o Aitareya Upanishad,

o Kaushitaki Upanishad

 Upanishads of the Shukla-Yajurveda:


https://t.me/prelimbits
272

o Brihadaranyaka Upanishad,

o Isha Upanishad

 Upanishads of the Krishna-Yajurveda:

o Taittiriya Upanishad,

o Katha Upanishad,

o Shvetashvatara Upanishad,

o Maitrayaniya Upanishad

 Upanishads of the Samaveda:

o Chandogya Upanishad,

o Kena Upanishad

 Upanishads of the Atharvaveda:

o MundakaUpanishad,

o MandukyaUpanishad,

o Prashna Upanishad.

187. Which one of the following pairs does not form part of the six systems of Indian
Philosophy? [2014 ]

A. Mimamsa and Vedanta


B. Nyaya and Vaisheshika
C. Lokayata and Kapalika
D. Sankhya and Yoga
Ans. C
ORTHODOX SCHOOLS OF INDIAN PHILOSOPHY

 Orthodox (astika) schools, originally called sanatana dharma, are collectively referred to as
Hinduism in modern times. The ancient Vedas are their source and scriptural authority.
Hinduism consists of six systems of philosophy & theology.
Samkhya (Kapila)

 Samkhya is the oldest of the orthodox philosophical systems, and it postulates that
everything in reality stems from purusha (self, soul or mind) and prakriti (matter,
creative agency, energy).
 Purush cannot be modified or changed while prakriti brings change in all objects.
 Emphasizes the attainment of knowledge of self through meditation and concentration.
 School does not believe in the existence of God.

https://t.me/prelimbits
273

At the same point, it does not describe what happens after moksha and does not
mention anything about Ishwara or God, because after liberation there is no essential
distinction of individual and universal puruṣa.
o So what happens after Moksha is the irrelevant thing for this school.
o But at the same time, though godless, the Sankhya believes in the doctrine of
Karma and of transmigration of souls.
 This philosophy adversely affected the Tantra sadhana a lot.
 Sankhya accepts the theory of rebirth or transmigration of the soul
Yoga (Pata njali)

 Yoga school introduces the methods of the discipline of body and mind. Sage Patanjali is
the founder of Yoga.
 Emancipation of Purusha from Prakriti by self-awareness through the discipline of
body and mind is conceptualized by Yoga.
 It is believed that practising Ashtanga Yoga is the way to relieve oneself from past sins in
order to make way for liberation.
Nyaya (Gautama Muni)

 The term ‗Nyaya‘ means “justice”, “rules”, “method” or “judgment”


 It approaches philosophical questions in a scientific and rational approach.
 According to Nyaya, there were four valid sources of knowledge — perception, inference,
comparison, and testimony
 Four sources of invalid knowledge: memory, doubt, error (false certainty),
and hypothetical argument (―If there was no fire, there wouldn‘t be smoke, but there is
smoke, so there must be fire‖).
 This school believes attaining knowledge through the five senses is the sole way of
attaining liberation from the cycle of birth and death.
Vaisheshika (Kanada)

 Vaisheshika school of Hinduism, like Buddhism, accepted only two reliable means to
knowledge: perception and inference
 Vaisheshika School is known for its insights in naturalism. It is a form of atomism in
natural philosophy.
 It postulated that all objects in the physical universe are reducible to paramāṇu(atoms),
and one‘s experiences are derived from the interplay of substance, quality, activity,
commonness, particularity and inherence
Purva Mimamsa (Jaimini)

 It places emphasis on the power of yajnas and mantras in sustaining the activities of the
universe.

https://t.me/prelimbits
274

 This school of thought believes in complete authority of Vedas.


 He and later proponents of Karma-mimamsa philosophy teach that material existence is
endless, that there is NO LIBERATION.
 For Mimamsas, the cycle of karma is perpetual, and the best one can aim for is higher
birth among the Devas [god].

 Therefore, they hold that the whole purpose of the Vedas is to engage human
beings in rituals for Creating Good Karma, and consequently the mature soul's
prime responsibility is to ascertain the exact meaning of the Vedas' sacrificial
injunctions and to execute them.
 Two of their most ardent supporters, Sabar Swami and Kumarila Bhatta, made further
inroads into the doctrine
Uttara Mimamsa (Vedanta)

 Vedanta school is a monistic school of philosophy that believes that the world is unreal
and the only reality is Brahman.
 The school separated into six sub-schools,
o Advaita (Adi Shankara): It states that both the individual self (Atman) and Brahman
are the same, and knowing this difference causes liberation.
o Visishtadvaita (Ramanuja): It believes that all diversity is subsumed to a unified
whole.
o Dvaita (Madhvacharya): It considers Brahman and Atman as two different entities,
and Bhakti as the route to eternal salvation.
o Dvaitadvaita (Nimbarka): It states that the Brahman is the highest reality, the
controller of all.
o Shuddhadvaita (Vallabhacharya): It states that both God and the individual self are
the same, and not different.
o Achintya Bheda Abheda (Chaitanya Mahaprabhu): It emphasizes that the
individual self (Jīvatman) is both different and not different from Brahman.

188. With reference to Buddhist history, tradition, and culture in India, consider the
following pairs: [2014 ]

Which of the pairs given above is/are correctly matched?


A. 1 only
B. 2 and 3 only

https://t.me/prelimbits
275

C. 1 and 3 only
D. 1, 2 and 3
Ans. C
IMPORTANT BUDDHIST MONASTERIES IN INDIA

Buddhist Location Significance


Temples/
Monasteries

Mahabodhi Bihar  Bodh Gaya is a Buddhist pilgrimage centre in Bihar's


Temple Gaya District.
 Gautama Buddha attained enlightenment here under
the Bodhi tree, which is where the Mahabodhi Temple
is located.
 It has since been designated as a UNESCO World
Heritage Site.

Hemis Monastery Ladakh  It is a Drukpa Lineage Tibetan Buddhist monastery


(gompa) in Hemis, Ladakh, India.
 It is located 45 kilometres south of Leh, in the state of
Jammu & Kashmir, on the west bank of the Indus
River.
 The annual Guru Padmasambhava festival, which
takes place in June-July, is well-known at this
monastery.

Tabo Monastery Himachal  It's in the Tabo village of the Spiti Valley in Himachal
Pradesh Pradesh, India.
 It was created in 996 CE in the Tibetan year of the Fire
Ape by Tibetan Buddhist lotsawa (translator)
Rinchen Zangpo (Mahauru Ramabhadra).

Tsuglagkhang Himachal  It is one of the most well-known Buddhist


Monastery Pradesh monasteries.
(Dharamshala)  His Holiness the Dalai Lama's residence is in the
Mcleodganj area of Dharamshala district in Himachal
Pradesh.
 The Dalai Lama's temple is another name for it.

Thiksey Leh in Ladakh  It is a Tibetan Buddhist gompa (monastery) linked


Monastery with the Gelug sect.
 It is located on top of a hill in Thiksey hamlet, Ladakh,
India, about 19 kilometres east of Leh

https://t.me/prelimbits
276

 It is notable for its resemblance to Tibet's Potala


Palace.
 It is a twelve-story complex that houses stupas,
statues, thangkas, wall murals, and swords, among
other Buddhist artefacts.

Tawang Arunachal  It is the largest monastery in India and the second


Monastery Pradesh largest in the world after the Potala Palace in
Lhasa, Tibet.
 It is located in Tawang city of Tawang district in the
Indian state of Arunachal Pradesh.

Bylakuppe Karnataka  It is the world's largest teaching centre for Tibetan


Monastery Buddhism's Nyingma lineage.
(Namdroling)  It is located in Bylakuppe, Karnataka, in the district of
Mysore.
 It is the main teaching centre of the Nyingma lineage
of Tibetan Buddhism.

Shashur Himachal  In Lahaul and Spiti, Himachal Pradesh, northern India,


Monastery Pradesh it is a Buddhist monastery of the Drugpa sect.
 It was constructed in the 17th century by Lama Deva
Gyatsho of Zanskar, a missionary of Bhutan's King
Nawang Namgyal.

Ghoom West Bengal  It's in the West Bengal town of Ghum.


Monastery  Lama Sherab Gyatso founded this monastery in 1875
AD.
 It is part of the Gelukpa or Yellow Hat sect and is
noted for its 15-foot (4.6-meter) high Maitreya
Buddha statue.

Kye Gompa Himachal  It is a Tibetan Buddhist monastery in the Spiti Valley


Monastery Pradesh of Himachal Pradesh's Lahaul and Spiti region, India,
built on top of a hill at an elevation of 4,166 metres
above sea level, close to the Spiti River.
 It is Spiti Valley's largest monastery and a religious
training centre for Llamas.

Lingdum Sikkim  Near Ranka, about an hour's drive from Gangtok.


Monastery  It is based on the Zurmang Kagyu lineage.

Alchi Gompa Leh  It is located in Alchi village, Leh District


Monastery  The Likir Monastery is in charge of it.

https://t.me/prelimbits
277

 According to historians, Guru Rinchen Zangpo built it


between 958 and 1055 AD.

Shankar Leh
Monastery

Matho Monastery Leh

Nako Monastery Himachal  It is located in Himachal Pradesh's Kinnor district.


Pradesh  It was founded in 996 AD

Rumtek Sikkim  The Dharmachakra Centre is another name for it.


Monastery  It's a Buddhist temple known as a gompa (Buddhist
ecclesiastical fortifications of learning, lineage and
sadhana).

189. With reference to the Indian history of art and culture, consider the following pairs:
[2014]

Which of the pairs given above is/are correctly matched?


A. 1 and 2 only
B. 3 only
C. 1 and 3 only
D. 1, 2 and 3
Ans. C
GROUP OF MONUMENTS AT MAHABALIPURAM

 Collection of 7th and 8th century CE religious monuments in Tamil Nadu.


 The monuments were built during the Pallava dynasty.
 The group contains several categories of monuments:
 Ratha temples
o They are with monolithic temples carved in the shape of chariots.

https://t.me/prelimbits
278

o The best-known are the five monolithic structures known as the Five Rathas or the
Pandava Rathas: Dharmaraja Ratha, Bhima Ratha, Arjuna Ratha, Nakula-Sahadeva
Ratha and Draupadi Rathas.
o Other ratha monuments at Mahabalipuram include the late-7th-century Ganesha
Ratha. It is three storeyed and of better workmanship, it resembles the Bhima-ratha
in roof form.
 Structural temples
o The structural (free-standing) temples at Mamallapuram have been built with cut
stones as building blocks.
o Some of the major temples are:
 Shore Temple: It consists of a large temple, two smaller temples and many
minor shrines, open halls, gateways, and other elements, much of which is
buried by sand.
 Olakkanesvara temple: It was built in the early 8th century from grey granite
by King Rajasimha. The walls of the temple depict the Ravananugraha legend
from the Ramayana and a relief of Dakshinamurti (Shiva as a yoga teacher)
 Mukundanayanar temple: It has ratha-like architecture.
 Rock reliefs
o The best-known rock relief is the Descent of the Ganges also known as Arjuna's
Penance or Bhagiratha's Penance. (carved on two monolithic rock boulders)
o It has two primary interpretations: the effort needed to bring the Ganges from
the heavens to earth, and the Kirātārjunīya legend and the chapter from the
Mahabharata about Arjuna's efforts to gain the weapon he needed to help good
triumph over evil.

 Cave temples
o Varaha cave
o Kotikal cave
o Dharmaraja cave
o Ramanuja Cave
Lad Khan temple (or Chalukya Shiva Temple ), dedicated to Shiva, is one of the oldest Hindu

https://t.me/prelimbits
279

temples and is located in the group of monuments at Aihole in the state of Karnataka.
NIRVANA VS. PARINIRVANA

Nirvana (nirvana with remainder)

 In the Buddhist tradition, nirvana has commonly been interpreted as the extinction of the
"three fires",or "three poisons", greed (raga), aversion (dvesha) and ignorance (moha).
 When these fires are extinguished, release from the cycle of rebirth (saṃsāra) is
attained.
Parinirvana (nirvana without remainder)

 Parinirvana is commonly used to refer to nirvana-after-death, which occurs upon the


death of someone who has attained nirvana during his or her lifetime.
 It implies a release from the Saṃsāra, karma and rebirth as well as the dissolution of the
skandhas.
Distinguished btw Nirvana with remainder & Nirvana without remainder

 Nirvana with Remainder


o A state achieved prior to death, where ―the remainder‖ refers to the mind and
body of this final existence
 Nirvana without remainder
o Which is achieved at death when the causes of all future existence have been
extinguished and the chain of causation of both physical form and
of consciousness have been finally terminated

These states were available to all who followed the Buddhist path to its conclusion.
The Buddha himself is said to have realized nirvana when he achieved enlightenment at the age
of 35. Although he destroyed the cause of future rebirth, he continued to live for another 45
years. When he died, he entered nirvana, never to be born again. (this event is known as
Mahaparinirvana

The death of the Buddha, or Mahaparinirvana @ Ajanta Cave 26


https://t.me/prelimbits
280

Massive uddha sculpture. Here Buddha is lying on his right side. The sculpture has a length of 7
metres. The panel below the sculpture shows Buddha‟s disciples and celestial figures float
above. This is one of the grandest and most sensitive depiction in all of Buddhist art. The
Mahaparinirvana of the Buddha, when he achieved release from the mortal world, is a deeply
impressive sculpture.
BADAMI CAVE (TEMPLES) 6TH CENTURY

 Complex of Hindu and Jain cave temples located in Badami, a town in the Bagalkot
district in northern part of Karnataka
 Caves are important examples of Indian rock-cut architecture, especially Badami Chalukya
architecture, and the earliest date from the 6th century.
 Badami, also referred to as Vatapi, Vatapipura, Vatapinagari and Agastya Tirtha in
historical texts, the capital of Chalukya dynasty in the 6th century

 There are 4 Cave in which 3 belongs to Hindu & 1 to Jain

 Exact dating is known only for Cave 3, which is a temple dedicated to Vishnu. An
inscription found here records the dedication of the shrine by Mangalesha [King] in Saka
500 (solar calendar, 578/579 CE)
 The Badami Caves complex is part of a UNESCO-designated World Heritage Site
candidate under the title "Evolution of Temple Architecture – Aihole-Badami-
Pattadakal" in the Malaprabha river valley, considered a cradle of temple architecture
that formed the model for later Hindu temples in the region.
 The artwork in Caves 1 and 2 →
o exhibit the northern Deccan style of the 6th and 7th centuries,
 While those in Cave 3 →
o simultaneously represent two ancient Indian artistic traditions; the northern Nagara
and the southern Dravida styles
o Cave 3 also shows icons and reliefs in the so-called Vesara style, a fusion of ideas
from the two styles, as well as some of the earliest surviving historical examples in
Karnataka of yantra-chakra motifs (geometric symbolism) and colored fresco
paintings.
 The first three caves feature sculptures of Hindu icons and legends focusing on Shiva
and Vishnu.
 While Cave 4 features Jain icons and themes
Temple caves

 Temples are carved out of soft Badami sandstone on a hill cliff.


Cave 1

 Nataraja Carving

https://t.me/prelimbits
281

o The cave portrays the Tandava-dancing Shiva as Nataraja


o Shiva has his son Ganesha and the bull Nandi by his side.
o Adjoining the Nataraja, the wall depicts the goddess Durga of Shaktism tradition
slaying the buffalo- demon Mahishasura.
o Shiva's dance is Ugra tandava natya (angry mode) dancing on a Rakshasa
(daemon), but here the dance is particularly called Lalitha Natya Chatur tandava
(which is a happy mode)
 Mahisasura Mardhini Carving
o On the right side of the Nataraja carving is an amazing carving of Mahishaasura
mardini with Chaturbhuja (Chatur means 4, Bhuja means hands).
o The sculpture depicts the killing of the Assura (daemon) Mahishaasura by
Goddess Durga.
o Mahishasura was the son of Mahisi and great grandson of Bhahmarishi Kashyapa.
 Harihara Carving
o Sculpture of Harihara, a 7.75-foot (2.36 m) high sculpture of a fused image that is
half-Shiva and half-Vishnu.
 Ardhanareshwara Carving
o A fused image of Shiva and his consort Parvati.
o Next to the Ardhanarishvara half that represents Shiva is Nandi the bull, and
skeletal Bhringi, a devotee of Shiva. The Bhringi sadhu carvings shows the
advance knowledge of anatomy for the whole body which includes the Skulls, Ribs,
bones on hands and legs.

o The carvings of Ardhanareshwara and Harihara were carefully built with a vision
by the Chalukya kings delivering message to the Chalukya citizens about the
equal rights for men and women in the society and need to mutually respect each
other. There is NO GREAT MALE AND NO GREAT FEMALE.

Cave 2

 The largest relief in Cave 2 depicts the legend of Vishnu in his Trivikrama form
 Another major relief shows the legend of Vishnu in his Varaha (a boar)

https://t.me/prelimbits
282

Cave 3

 Cave 3 is earliest dated Hindu temple in the Deccan region.


 It is dedicated to Vishnu; it is the largest cave in the complex.
 It has intricately carved friezes and giant figures of Trivikrama, Anantasayana, Vasudeva,
Varaha, Harihara and Narasimha.
 Cave 3's primary theme is Vaishnavite, though it also shows Harihara on its southern wall
half Vishnu and half Shiva shown fused as one, making the cave important to Shaivism
studies.
 Cave 3 also shows fresco paintings on the ceiling, some of which are faded and broken.
These are among the earliest known surviving evidence of fresco painting in Indian art. The
Hindu god Brahma is seen on Hamsa vahana in one of the murals. The wedding of Shiva
and Parvati, attended by various Hindu deities is the theme of another
Cave 4

 It is dedicated to Tirthankaras, the revered figures of Jainism. It was constructed after the
first three, sponsored by Hindu kings in later part of the 7th-century.

https://t.me/prelimbits
283

190. Which of the following Kingdoms were associated with the life of the Buddha? [2015]

1. Avanti
2. Gandhara
3. Kosala
4. Magadha
Select the correct answer using the code given below.
A. 1, 2 and 3
B. 2 and 4.
C. 3 and 4 only
D. 1, 3 and 4
Ans. C
 Gandhara is not directly associated with the life of Buddha. During the rule of Asoka
Gandhara was influenced by Buddhist culture.
 Avanti was not related to the life of Buddha
PLACES ASSOCIATED WITH THE LIFE OF BUDDHA

Astamahasthanas are eight great holy places associated with the life of Buddha. These
include four pilgrimage sites related to the life of Gautama Buddha viz. Lumbini, Bodhgaya,
Sarnath and Kushinagara and four other sites viz. Sravasti, Sankasya, Rajgir and Vaishali.

1. Lumbini : Lumbini is currently located in Kapilavastu district of Nepal. It is birth place of


Buddha. At the time of birth of Buddha, Lumbini was a part of Shakya Clan, which was a
republic.
2. Bodhgaya : It is located in Bihar on the bank of river Neranjana {this river was known as
Uruwela at that time}. It is known for place of enlightment of Buddha.
3. Sarnath : Sarnath {also known as Mrigadava, Migadaya, Rishipattana, Isipatana} is the deer
park where Gautama Buddha delivered his first sermon or Dhammachakraparivartan
Sutra. At the time of Buddha, it was a part of Kashi Janapada.
https://t.me/prelimbits
284

4. Kushinagar : Kushinara or Kushinagara is located in Kushinagar district of Uttar Pradesh. It


is the site of Buddha‟s death and mahaparinirvana. At the time of Buddha‘s death, it was
a capital of Malla janapada.
5. Sravasti : Sravasti was located in Uttar Pradesh around area of Balrampur in modern Uttar
Pradesh. It is closely asscoiated with the life of Buddha because Buddha had spent 24
Chaturmasas {implies 24 years because one year as only one Chaturmasa between Ashadha
to Kartika}. Thus, we can say that most of monastic life of Buddha was spent in Shravasti. In
Buddha‘s times, Shravasti was capital of Kosala Kingdom. Shravasti is also birthplace of
Jaina Tirthankar Sambhavanath, and thus is important for Jains also.

6. Sankasya : It‘s current location is Farrukhabad district of Uttar Pradesh. It has some faiths of
Buddhism that Buddha after is death descended from heaven here.
7. Rajgir : After the great departure (Mahabhinishkramana), Buddha had first gone to Rajgir.
o Rajgir, meaning "The City of Kings," is a historic town in the district of Nalanda in
Bihar
o As the ancient seat and capital of the Haryanka dynasty, the Pradyota dynasty,
the Brihadratha dynasty and the Mauryan Empire, as well as the dwelling ground
of such historical figures as The Buddha and The Mahavira

https://t.me/prelimbits
285

o Rajgir was the capital of Haryanka dynasty kings Bimbisara (558–491 BC) and
Ajatashatru (492–460 BC).
o Ajatashatru kept his father Bimbisara in captivity here. The sources do not agree on
which of the Buddha's royal contemporaries, Bimbisara and Ajatashatru, was
responsible for its construction.
o It was the ancient capital city of the Magadha kings until the 5th century BC when
Udayin (460–440 BC), son of Ajatashatru, moved the capital to Pataliputra
(modern Patna).
o Shishunaga (413-395 BC) founded Shishunaga dynasty in 413 BC with Rajgir as its
initial capital before it was moved to Pataliputra.
o It is associated with the founders of both the religions: Jainism and Buddhism,
associated with both the historical Mahavira and Buddha.
o Buddha
 It was here that Gautama Buddha spent several months meditating, and
preaching at Gridhra-kuta, ('Hill of the Vultures').
 He also delivered some of his famous sermons and initiated king
Bimbisara of Magadha and others to Buddhism.
 It was here that Budhha delivered his famous Atanatiya Sutra.
 On one of the hills is the Saptaparni Cave where the First Buddhist Council
was held under the leadership of Maha Kassapa
o Mahavira
 Mahavira, the 24th Tirthankara spent fourteen years of his life at Rajgir and
Nalanda, spending Chaturmas (i.e. 4 months of the rainy season) at a single
place in Rajgir (Rajgruhi) and the rest in the places in the vicinity.
 It was the capital of one of his Shravaks (follower) King Shrenik.
 The twentieth Jain Tirthankara, Munisuvrata is supposed to have been
born here.
 It is also mentioned in Jain and Buddhist scriptures, which give a series of
place-names, but without geographical context.
8. Vaishali : At the time of Buddha, Vaishali was in Vajji Janapada. It is said that the Buddha
visited this place thrice and spent quite a long time here. The Buddha also delivered his
last sermon at Vaishali and announced his Nirvana here.
Other places

Kosambi, Uttar Pradesh


 By the time of Gautama Buddha, Kosambi was a thriving city with a high number of affluent
merchants.
 It was a vital crossing point for commodities and persons from the north and south. It is

https://t.me/prelimbits
286

mentioned frequently in the stories of Buddha's life.


 Udayana was a Buddhist upasaka, and Gautama Buddha visited Kaushambi multiple
times throughout his reign in order to promote the dharma, the Noble Eightfold Path, and
the Four Noble Truths.
 The first image of the Buddha (carved from sandalwood) was produced at Udayana's
request, according to the Chinese translation of the Buddhist canonical scripture Ekottara
Agama.

191. With reference to the art and archaeological history of India, which one among the
following was made earliest? [2015]

A. Lingaraja Temple at Bhubneshwar


B. Rock-cut Elephant at Dhauli
C. Rock-cut Monuments at Mahabalipuram
D. Varaha Image at Udayagiri
Ans. B
 Rock-cut Elephant at Dhauli = 250 BC
 Udayagiri Caves associated with a Gupta period = 300 CE – 600 CE
 Group of Monuments at Mahabalipuram = 7th and 8th century CE [see Q no 189]
 Lingaraj Temple = 7th C or 11th century AD.
DHAULI
 Dhauli hills are located on the banks of the river Daya, 8 km south of Bhubaneswar in
Odisha
 It is a hill with vast open space adjoining it, and has major Edicts of Ashoka engraved on a
mass of rock
 Dhauli hill is presumed to be the area where the Kalinga War was fought.
 The Rock Edicts found here include Nos. I-X, XIV and two separate Kalinga Edicts.
 In Kalinga Edict VI, he expresses his concern for the "welfare of the whole world". The
Rock-Cut Elephant above the Edicts is the earliest Buddhist sculpture of Odisha.
 The stone elephant shows the animal's foreparts only, though it has a fine sense of form
and movement. It is considered as one of the few remains of Mauryan art.
 Ashoka had a special weakness for Dhauli, where the battle was fought. The Daya river is
said to have turned red with the blood of the many deceased after the battle, and
enabled Ashoka to realize the magnitude of horror associated with war.
 He saw to it that Dhauli became an important centre of Buddhist activities. He built several
Chaityas, Stupas and Pillars there.
 He got abodes excavated for the recluse, instructions inscribed for officials, expounded the

https://t.me/prelimbits
287

main principles of dandaniti (strict enforcement of laws) for the public, provided special
status to his new kingdom including the stupas at Dhauli.

The front is shaped as an elephant. Dhauli, Puri District


UDAYAGIRI CAVES

 The Udayagiri Caves are twenty rock-cut caves near Vidisha, Madhya Pradesh from the
early years of the 5th century CE.

 They contain some of the oldest surviving Hindu and Jain temples and iconography in
India.

 They are the only site that can be verifiably associated with a Gupta period monarch from
its inscriptions.

 Udayagiri caves contain iconography of Jainism. They are notable for the ancient
monumental relief sculpture of Parshvanatha in his incarnation.

 The site has important inscriptions of the Gupta dynasty belonging to the reigns of
Chandragupta II (c. 375-415) and Kumaragupta I (c. 415-55).

 Udayagiri caves also has a series of rock-shelters and petroglyphs, ruined buildings,
inscriptions, water systems, fortifications and habitation mounds, all of which remain a
subject of continuing archaeological studies.

 Udayagiri caves contain iconography of Vaishnavism (Vishnu), Shaktism (Durga and


Matrikas) and Shaivism (Shiva)

 There are in total 20 caves out of which two are related to Jainism and the remaining
eighteen to Hinduism directly. [https://imvoyager.com/udayagiri-caves-madhya-pradesh/]

 Caves are notable for the ancient monumental relief sculpture of Vishnu in his
incarnation as the MAN-BOAR VARAHA, rescuing the earth symbolically represented
by Bhudevi clinging to the boar‘s tusk as described in Hindu mythology

https://t.me/prelimbits
288

Man-Boar Varaha (Cave 5)

There are a number of places in India with the same name, the most notable being the mountain
called Udayagiri at Rajgir in Bihar and the Udayagiri and Khandagiri Caves in Odisha.

LINGARAJA TEMPLE

https://www.holidify.com/places/bhubaneswar/lingaraj-temple-sightseeing-1385.html

 Lingaraja Temple is a temple DEDICATED TO SHIVA.


 Built in 7th C. by King Jajati Keshari of Soma Vansh.
 It is built in RED STONE and is a classic example of KALINGA STYLE of architecture.
 It signifies the syncretisation of Shaivism and Vaishnavism sects in
Odisha
 Bhubaneswar is called the Ekamra Kshetra as the deity of
Lingaraja was originally under a mango tree (Ekamra) as noted in
Ekamra Purana, a 13th-century Sanskrit treatise.
 Bindu Sagar Lake or Bindu Sarovar is a sacred lake located to the north of the Lingaraja
Temple and to the east of the Ananta Vasudeva Temple

192. In the context of the history of India, consider the following pairs: [2016]

Which of the pairs given above is/are correctly matched?

https://t.me/prelimbits
289

A. 1 and 2
B. 3 only
C. 2 and 3
D. 1 and 3
Ans. D
GHATIKA

 "Ghatika", a famous education & learning center during the Pallava period, was situated
at Kanchipuram.

 Education and Learning during Pallava Rule:

o The Ghatika at Kanchipuram was a famous center of Education & Learning. Mostly
attached to temples

o Various noted pupils of Ghatika were enlighted by every ruler of Pallava.

 The following tables show the list of famous writers & poets with respective
descriptions.

TANIYURS

 Was a large village and were distinct revenue units under Cholas.
BRAHMADEYA
 Villages donated to a single Brahmin or a group of Brahmins are known as Brahmadeya.
ERIPATTI

 Was Tank lands i.e land donated by individuals, the revenue from which was set apart
for the maintenance of‟ the village tank.

193. Which one of the following books of ancient India has the love story of the son of
the founder of the Sunga dynasty? [2016]

A. Swapnavasavadatta
B. Malavikagnirnitra
C. Meghadoota
D. Ratnavali
Ans. B
 Malavikagnimitra is a Sanskrit play by Kalidas
KALIDASA

Kalidasa was one of the navratnas of Chandragupta II. His works are very famous and have been
translated in many European languages. Some of his works are:
Kalidasa wrote three plays:
https://t.me/prelimbits
290

 Malavikagnimitram (Pertaining to Malavika and Agnimitra) tells the story of King


Agnimitra, who falls in love with the picture of an exiled servant girl named Malavika.
o In the introduction to his first play Malavikagnimitram, Kālidāsa wrote: "Shall we
neglect the works of such illustrious authors as Bhāsa, Saumilla, and Kaviputra?
Can the audience feel any respect for the work of a modern poet, a Kālidāsa?
 Abhijnanasakuntalam (Of the recollection of Shakuntala) tells the story of King
Dushyanta who, while on a hunting trip, meets Shakuntala, the adopted daughter of a
sage, and marries her. It was among the first Sanskrit works to be translated into English.
 Vikramorvasiyam (Urvashi Won by Valour) tells the story of mortal King Pururavas
and celestial diety Urvashi who falls in love.
Kalidasa is the author of two epic poems:
 Raghuvamsa is an epic poem about the kings of Raghu dynasty.
 Kumarasambhava describes the birth and adolescence of the goddess Parvati, and her
marriage with Lord Shiva.
Kalidasa also wrote two khandakavyas (minor poems):
 Ritusamhara describes the six seasons by narrating the experiences of two lovers in each of
the seasons.
 Meghaduta (The Cloud Messenger) describes the story of a Yaksha trying to send a
message to his lover through a cloud. This poem is elegiac in nature through which
Kalidasa created his own genre of poetry. Kalidasa set this poem to the mandakranta
meter, which is known for its lyrical sweetness.

194. Which one of the following was a very important seaport in the Kakatiya kingdom?
[2017]

A. Kakinada
B. Motupalli
C. Machilipatnam (Masulipatnam)
D. Nelluru
Ans. B
KAKATIYA DYNASTY
 The Kakatiya dynasty ruled most of eastern Deccan region comprising present day
Telangana and Andhra Pradesh, and parts of eastern Karnataka and southern Odisha
between 12th and 14th centuries.

 Their capital was Orugallu, now known as Warangal.

 Early Kakatiya rulers served as feudatories to Rashtrakutas and Western Chalukyas for
more than two centuries.

https://t.me/prelimbits
291

 They assumed sovereignty under Prataparudra I in 1163 CE by suppressing other


Chalukya subordinates in the Telangana region.

 Ganapati Deva (r. 1199–1262) significantly expanded Kakatiya lands during the
1230s and brought under Kakatiya control the Telugu-speaking lowland delta areas around
the Godavari and Krishna rivers.

 Motupalli, now in Krishna district, was an important seaport in Ganapati Deva reign.

 Ganapati Deva was succeeded by Rudrama Devi (r. 1262–1289) who is


one of the few queens in Indian history.

o Marco Polo, who visited India around 1289–1293, made note


of Rudrama Devi's rule and nature in flattering terms. She
successfully repelled the attacks of Yadavas (Seuna) of Devagiri into
the Kakatiyan territory.

 In 1303, Alauddin Khilji, the emperor of the Delhi Sultanate invaded


the Kakatiya territory which ended up as a disaster for the Turks.
 But after the successful siege of Warangal in 1310, Prataparudra II was
forced to pay annual tribute to Delhi.
 Another attack by Ulugh Khan in 1323 saw stiff resistance by the Kakatiyan army, but they
were finally defeated.
 Kakatiya era the development of a distinct style of architecture which improved and
innovated upon the existing modes. Most notable examples are the Thousand Pillar
Temple in Hanamkonda, Ramappa Temple in Palampet, Warangal Fort, and Kota Gullu
in Ghanpur.
Prataparudra I
 The 1149 Sanigaram inscription of Prola II is the last known record of the Kakatiyas as
vassals.

 The 1163 Anumakonda inscription of Rudradeva alias Prataparudra I is the earliest


known record that describes the Kakatiyas as a sovereign power

195. With reference to the difference between the culture of Rigvedic Aryans and Indus
Valley people, which of the following statements is/are correct? [2017]

1. Rigvedic Aryans used the coat of mail and helmet in warfare whereas the people of Indus
Valley Civilization did not leave any evidence of using them.
2. Rigvedic Aryans knew gold, silver, and copper whereas Indus Valley people knew only
copper and iron.
3. Rigvedic Aryans had domesticated the horse whereas there is no evidence of Indus Valley
people having been aware of this animal.
Select the correct answer using the code given below:
https://t.me/prelimbits
292

A. 1 only
B. 2 and 3 only
C. 1 and 3 only
D. 1, 2 and 3
Ans. C
VIC : NOT HORSE CENTRED
 Oxen, buffaloes, goats, sheep, and pigs, dogs, cats, asses and camels were domesticated.
Humped Bulls were favoured by the Harappans.
 Not horse centred but evidence in Surkotada, Mohenjodaro and Lothal. Lion was not
known. Elephants and Rhinoceros (Amari) & deer were well known

Horse
The evidence of the horse comes from a superficial level of Mohenjo-Daro and from a doubtful
terracotta figurine from Lothal. The remains of the horse are reported from Surkotada,
situated in western Gujarat and belong to around 2000 B.C. But the identity is doubtful. In any
case, the Harappan culture was not horse-centered.

ARYAN KNEW ABOUT HORSES & HAVE SUPERIOR MILITARY TECHNOLOGY


 The Indo-Aryans migrated through the adjacent Bactria–Margiana area (present-day
northern Afghanistan) to northwest India.

 Evidences of horses, spoked wheels, fire cults, and cremation (which formed important
parts of Aryan life in India) during the period between 1900 BCE and 1500 BCE, were found
in these regions.

 Apart from these, the artifacts and ceramics also suggest movement of people from
Central Asian region to South Asian region.

 The earliest Aryans lived in the land of the (Sapta Sindhu) (Sindhu is the river par
excellence of the Aryans) and probably because of their use of horse chariots and superior
military technology, [e.g.used the coat of mail (armour) and helmet in warfare. ] they could
establish their political dominance in the region.

 The region was drained by the seven rivers →

o Indus [most discussed] (Sindhu), and its five tributaries, namely →

 Jhelum (Vatista)

 Beas (Vipasa)

 Chenab (Askini)

 Ravi (Purushni)

 Sutlej (Sutudri), and the

https://t.me/prelimbits
293

o Saraswati (modern Ghaggar Hakra), which covered mostly the areas of eastern
Afghanistan, Punjab, and fringes of western U.P. == > Most Prominent River

 The rare mention of the river Yamuna (twice) and Ganga (only once) is pointer to the fact
that Rig Vedic Aryans had not inhabited that region as yet.

HARAPPAN PEOPLES KNEW GOLD, SILVER, AND COPPER


 Harappan culture is recognized for advanced metallurgical techniques. Archaeologists
have discovered usage of copper, bronze, tin, silver and gold. It was the first time that
silver was extracted and used in Indus valley. Iron was not known to the people.
 Important crafts: spinning (Spindle whorls), bricklaying, boat-making, seal making,
terracotta manufacturing (potter‟s wheel), goldsmiths, bead making.
 Though written considerably later, the arts treatise Natya Shastra (c. 200 BCE – 200 CE)
classifies musical instruments into four groups based on their means of acoustical
production - strings, membranes, solid materials and air

 It is probable that such instruments had existed since the IVC. Archeological evidence
indicates the use of simple rattles and vessel flutes, while iconographical evidence
suggests early harps and drums were also used.

VEDIC PERIOD : LATER VEDIC AGE ECONOMY

 The Later Vedic culture is also called as PGW–Iron Phase culture, as a particular kind of
pottery (painted grey ware) was used during that phase.

 The main factor in the expansion of the Aryan culture during the Later Vedic period was
the beginning of the use of iron, which was introduced around 1000 BCE and is
mentioned as Krishna Ayas/Shyama Ayas

 The Rig Vedic people knew of a metal, called Ayas, which was either copper or bronze.

 The society was largely rural. However, towards the end of the period, there are traces of
the beginning of urbanism (as nagar in the sense of a town is mentioned in the Taittiriya
Aranyaka).
https://t.me/prelimbits
294

 Agriculture emerged as the chief means of livelihood of the Later Vedic people. The
forests were cleared by burning the trees, which is also mentioned in the Satapatha
Brahamana (ruler Videha Madhava burned the forests between river Saraswati to river
Sedanira or Gandak).

 Cultivation was done by the plough. Mixed farming (cultivation along with herding) was
one of the prime occupations of the Later Vedic people.

 The land was still communally owned over which the vish (clan) had many participatory
rights, but it was recognised as property and the head of the household who owned the
land was termed as Grahpati.

 Rice (Vrihi) and wheat (Godhuma) became the staple diet of the people and lentils were
also grown. Also, with the beginning of food production, agricultural produce began to
be offered in the rituals.

 The items of dana and dakshina included cooked rice. Tila, from which the first widely
used vegetable food-oil was derived increasingly, came to be used in rituals.

 Diverse arts and crafts were practiced in the Later Vedic period. Tin, lead, silver, Iron, gold,
bronze, and copper were known to Later Vedic people.

 There were probably good smiths and smelters as a lot of copper objects have been found
at PGW Sites.

 People had obtained knowledge of glass manufacturing too. Shresthins indicate the
guilds or organisation of merchants.

 Exchange was still via barter, but Niskha was used as a convenient unit of value
although not as a typical currency. [According to the Rigveda, 'Nishka' is a throat
ornament. Later, 'Nishka' was used to represent a gold coin.]

 Wagons drawn by oxen were probably the most used mode of transport.

 Possession of Horses remained an important priority of Vedic leaders and a remnant of


the nomadic lifestyle, resulting in trade routes beyond the Hindu Kush to maintain this
supply as horses needed for cavalry and sacrifice could not be bred in India.

 The Later Vedic people were acquainted with four types of pottery →

o Painted Grey Ware

o Black and Red Ware

o Black-slipped Ware

o Red Ware

 Unlike the Rig Vedic age where only voluntary offerings were taken and per se no revenue
collection was done, in the Later Vedic age, collection of taxes and tributes was made
Mandatory and were done by Sangrihitri. Vaishyas appear to be the only tribute payers

https://t.me/prelimbits
295

in Later Vedic times.

INDUS VALLEY CIVILIZATION APPEARS TO HAVE BEEN A PEACEFUL


 Indus Valley civilization appears to have been a peaceful one. Very few weapons have been
found and no evidence of an army has been discovered.
 Excavated human bones reveal no signs of violence, and building remains show no
indication of battle. All evidence points to a preference for peace and success in achieving
it.

196. Which of the following is/are famous for Sun temples? [2017]

1. Arasavalli
2. Amarkantak
3. Omkareshwar
Select the correct answer using the code given below:
A. 1 only
B. 2 and 3 only
C. 1 and 3 only
D. 1, 2 and 3
Ans. A
 Arasavalli in Srikakulam district (Andra P.) is a sun temple.
 Sri Omkareshwar temple in Mysore is for Lord Shiva
 Amarkantak in Madhya Pradesh has temples, but not any Sun temple
FAMOUS SUN TEMPLES IN INDIA
 Konark (Odisha)
 Dakshinaarka (Gaya)
 Modhera Sun Temple (Pushpavati river in Mehsana district of Gujarat)
 Surya Pahar Temple (Assam)
 Suryanar Temple, Kumbakonam (Tamil Nadu)
 Surya Narayana Temple , Arasavalli (Andhra Pradesh)
 The Bhramanya Dev Temple (Unao in Madhya Pradesh)
 Martand Sun temple (Jammu)
 Temple at Gwalior (similar to that of Konark Sun temple)
 Katarmal Sun Temple (Uttarakhand)
 Domlur Sun Temple (Banglore)

https://t.me/prelimbits
296

Konark Sun Temple

 Konark Sun Temple, located in East Odisha near the sacred city of Puri.
 Built in the 13th century by King Narasimhadeva I (Eastern Ganga dynasty bout 1250 CE)
and is dedicated to the Sun God.
 It marks the highest point of achievement of Kalinga architecture.
 It was declared a UNESCO world heritage site in 1984.
 The temple is designed in the shape of a colossal chariot.
 There are two rows of 12 wheels on each side of the Konark sun temple.
 Some say the wheels represent the 24 hours in a day and others say the 12 months.
 The seven horses are said to symbolise the seven days of the week.
 Sailors once called this Sun Temple of Konark, the Black Pagoda because it was supposed
to draw ships into the shore and cause shipwrecks.
 Great poet Rabindranath Tagore wrote of Konark: ―Here the language of stone surpasses
the language of man.‖
 Chandrabhaga Mela around the month of February.
Martand Sun Temple

 Location- Jammu and Kashmir.


 The Martand Sun Temple, also known as Pandou Laidan is a Hindu temple dedicated to
Surya (the chief solar deity in Hinduism) and built during the 8th century CE.
 Timeline- built by the third ruler of the Karkota Dynasty, Lalitaditya Muktapida, in the
8th century CE. It is said to have been built during 725-756 CE.
 Deity- Sun God, Martand is another Sanskrit synonym for Surya.
 It is now in ruins, as it was destroyed by the orders of Muslim ruler Sikandar Shah Miri.
Modhera Sun Temple (Gujarat)

 The Sun temple in Modhera is situated on the bank of the Pushpavati River.

 The temple was built after 1026-27 CE during the reign of Bhima I of the Solanki or
Chalukya dynasty.

 The temple was attacked and invaded twice, first by Mehmood Ghazni during his raid of
Gujarat and then by Allauddin Khilji.

 The magnificent temple is built in the Solanki type Maru-Gurjara Style of architecture.

o It is a style of north Indian temple architecture that originated in Gujarat and


Rajasthan from the 11th to 13th centuries, under the Chaulukya dynasty.

 The temple was completely built of sandstone and it consists of three main components –

o The main temple consists of a Garbagriha and a mandapa called as Gudamandapa.


https://t.me/prelimbits
297

o A Sabha Mandapa.

o A kunda or a stepped water tank called Surya kund, which encompasses a number of
miniature shrines on its smaller steps.

The famous Modheshwari Mata Temple is also located in Modhera near the Sun Temple.

India‟s First Solar-Powered Village – Modhera

https://www.newindianexpress.com/nation/2022/oct/10/gujarats-modhera-is-indias-first-fully-
solar-village-2506501.html

Recently, Prime Minister declared Modhera, in Gujarat as India‟s first round-the-clock solar-
powered village involving a ground-mounted solar power plant and more than 1,300 rooftop
solar systems.

Solarisation of Modhera

 Modhera village is in Mehsana district in Gujarat.

 The solarisation of Modhera happened through partnership between the Central and state
governments.

 The day time power comes from the rooftop solar panels.

 The night time power comes from a Battery Energy Storage System (BESS) in Mehsana.

 Besides this, the village has a solar based modern electric vehicle charging station.

 Solar power is major component of India‘s target of 500 GW of installed renewable


energy by 2030 and net zero by 2070.

197. The painting of Bodhisattva Padmapani is one of the most famous and oft-
illustrated paintings at [2017]

A. Ajanta
B. Badami

https://t.me/prelimbits
298

C. Bagh
D. Ellora
Ans. A
 BAGH CAVES (Q no. 166)
 BADAMI CAVE (Q no. 189)
 ELLORA CAVE (Q. no. 180)

AJANTA CAVES (2ND C. – 5TH C.)

 Approx. 30 rock-cut Caves → Almost exclusively Buddhist.


 Location : Aurangabad district of Maharashtra (UNESCO World Heritage Site)
 The walls of the caves have both Murals and Fresco Paintings(painted on wet plaster) They
use tempera style, i.e. use of pigments.
 The paintings portray human values and social fabric along with styles, costumes and
ornaments of that period. The emotions are expressed through hand gestures.
 The unique feature of the paintings is that each female figure has a unique hairstyle. Even
animals and birds are shown with emotions.
 The Common Themes of these paintings range from Jataka stories to life of Buddha to
elaborate decorative patterns of flora and fauna.
 The outline of the figures is in red ochre, with contours of brown, black or deep red.
Some important paintings at Ajanta are:
 Scenes from the Jataka stories → Buddha‟s former lives as a Bodhisattva, the life of
Gautam Buddha, etc.
 Paintings of various Bodhisattvas in tribhanga pose in Cave No.
o Vajrapani (protector and guide, a symbol of Buddha‘s power)
o Manjusri (manifestation of Buddha‘s wisdom) and

o Padmapani (Avalokitesvara) (symbol of Buddha‘s compassion).

 The Dying Princess in Cave No. 16.


 Scene of Shibi Jataka, where King Shibi offered his own flesh to save the pigeon.
 Painting of Maya Devi, the mother of the Buddha
 Cave No. XVII representing Buddha‟s visit to the door of Yashodhara‟s abode in the city
of Kapilavastu
History
 The Ajanta Caves are generally agreed to have been made in two distinct phases
o 1st Phase → Satavahana period

https://t.me/prelimbits
299

o 2nd Phase (5th century) → During the reign of Hindu Emperor Harishena of the
Vākāṭaka dynasty. The second phase is attributed to the theistic Mahāyāna, or
Greater Vehicle tradition of Buddhist

198. With reference to the religious history of India, consider the following statements:
[2017]

1. Sautrantika and Sammitiya were the sects of Jainism.


2. Sarvastivadin held that the constituents of phenomena were not wholly momentary but
existed forever in a latent form.
Which of the statements given above is/are correct?
A. 1 only
B. 2 only
C. Both 1 and 2
D. Neither 1 nor 2
Ans. B
EARLY BUDDHIST SCHOOLS

 The early Buddhist schools are those into which the Buddhist monastic sangha split early in
Buddhism's history. The divisions were caused initially by differences in Vinaya, but later also
by doctrinal differences and geographical separation of monastic groups.
 A significant number of years after Gautama Buddha's death, the original sangha split into
the first early schools (generally believed to be the Sthavira Nikaya and the Mahsghika)
between 383 and 250 BC. These early schools were later subdivided into schools such as
the Sarvstivdins, Dharmaguptakas, and Vibhajyavda.
Sthavira Nikaya

 One of the first Buddhist schools was the Sthavira Nikaya.


 They broke away from the majority Mahasamghikas during the Second Buddhist
Council, resulting in the Sangha's first schism.
o During the Second Buddhist Council, a small number of reformist elders proposed a
new arrangement of the Vinaya Pitaka, which contained additional rules - something
the Buddha forbade – and when they were unsuccessful, they split from the majority
Mahasamghikas to found the Sthavira Nikaya.
 Sthavira Nikaya school used paishachi.
o Paishachi is a largely unattested literary language of the middle kingdoms of India
mentioned in Prakrit and Sanskrit grammars of antiquity.

 The major sub-sects included Mahisasaka, Sarvastivada, Sankrantika, Sautrantika,

https://t.me/prelimbits
300

Dharmaguptaka, Vatsiputriya, Dharmottariya, Bhadrayaniya, Sannagarika,


Sammitiya , Vibhajyavadins & Vatsiputriyas
o Sarvastivadin sect of Buddhism believes that all things exist, and exist
continuously, in the past and the future as well as in the present.
Mahasanghika

 Many academics attribute Mahayana Buddhism's early growth to the Mahasamghika


branch.
 The Mahasamghika sect's initial headquarters were in Magadha, but they also had
prominent locations in Mathura and Karli.
 Its sub-sects included Gokulika, Prajnaptivada, Bahusrutiya, Ekavyaharikas, Caitika.

Other sub-sects not identified under the above two schools were Hemavatika, Rajagiriya,
Siddhatthaka, Pubbaseliya, Aparaseliya, and Apararajagirika.
Later Buddhist schools included Theravada, Mahayana, Vajrayana, etc.

199. Consider the following pairs of Tradition vs State: [2018]

1. Chapchar Kut festival: Mizoram


2. Khongjom Parba ballad: Manipur
3. Thang-Ta dance: Sikkim
Which of the pairs given above is/are correct?
A. 1 only
B. 1 and 2
C. 3 only
D. 2 and 3
Ans. B
Chapchar Kut
 is the biggest festival of the Mizos and is celebrated in March after completion of jhum
operation. It is a spring festival.
Khongjom Parba
 is a style of ballad singing from Manipur using Dholak (drum) which depicts stories of the
heroic battle fought by Manipuris against the mighty British Empire in 1891. It is one
of the most popular musical art forms of Manipur inciting the spirit of patriotism and
nationalism among the people at one time.
Thang-Ta
 is a martial dance of Manipur. Manipur dance has a large repertoire, however, the most
popular forms are the Ras, the Sankirtana and the Thang-Ta. Thang-ta dance of Manipur

https://t.me/prelimbits
301

evolved from the martial arts drills promoted by the kings of Manipur.

200. Consider the following pairs Craft vs Heritage of [2018]

1. Puthukkuli shawls: Tamil Nadu


2. Sujni embroidery: Maharashtra
3. Uppada Jamdani saris: Karnataka
Which of the pairs given above is/are correct?
A. 1 only
B. 1 and 2
C. 3 only
D. 2 and 3
Ans. A
Puthukkuli shawls – Tamil Nadu
 Puthukkuli shawls are made by Todas of Nilgiri hills in Tamil Nadu.
 Locally called Pugur, meaning flower, the fine and intricate Toda embroidery is done by
tribal men and women on shawls.
 The shawl adorned with the Toda embroidery is called Poothkuli
Sujni embroidery – Bihar
 Sujni embroider is a form of embroidery originating from the Bhusura village of Bihar.
 It was given a GI tag.
Uppada jamdani sarees – Andhra Pradesh
 Uppada Silk sarees or Jamdani Sari is a silk sari style woven in Uppada of East Godavari
district of Andhra Pradesh.
 It was also given a GI tag.

201. With reference to Indian history, who among the following is a future Buddha, yet to
come to save the world? [2018]

A. Avalokiteshvara
B. Lokesvara
C. Maitreya
D. Padmapani
Ans. C
PROMINENT BODDHISATTVAS UNDER BUDDHISM

Avalokitesvara (Buddha‟s Compassion)

https://t.me/prelimbits
302

 One of Buddha's three protector deities, also known as Padmapani and is described as
carrying a Lotus Flower.
 His picture, which can be found in the Ajanta Caves, is the most well-known of all the
Bodhisattvas.
 The compassionate bodhisattva listens to the world's pleas and responds in skillful ways.
 Under the name Lokesvara, he appears informally in Cambodian Theravada Buddhism. He
is also shown as a woman, and His Holiness The Dalai Lama is thought to be his incarnation.

Vajrapani ( Buddha‟s Power)

 Vajrapani is one of three guardian deities ( others are Manjusri and Avlokiteshwara.
Vajrapani ) that surround Buddha and is represented in the Ajanta Caves.
 He is said to express all of Buddha's powers as well as the powers of the five tathagatas-
Vairocana, Akshobhya, Amitabha, Ratnasambhava, and Amoghasiddhi.

Manjusri (Buddha‟s Wisdom)

 Manjusri is one of three protecting deities that surround Buddha and is also represented in
the Ajanta Caves.
 He is a man Bodhisattva with a sword in his hand who is linked with Buddha's Wisdom.

Finally, Avalokitesvara embodies Buddha's compassion, Vajrapani embodies Buddha's might, and
Manjusri ponders Buddha's knowledge.

Samantabhadra (Buddha‟s Aspirations)

 The meaning of Samantabhadra is ‗Universal Worthy‘ and he is associated with practice


and meditation. He made ten vows to fulfil his journey of becoming a Buddha.

https://t.me/prelimbits
303

 In Buddhism, he is part of the Shakyamuni triad, which also includes the Buddha and
Manjusri.
Ksitigarbha (Buddha‟s Merit)

 He is represented as a Buddhist monk who vowed not to attain Buddhahood until hell
was totally empty.
Maitreya (Buddha‟s Activity)

 Aka. Ajita Boddhisattva


 A future Buddha who will arrive on Earth, attain perfect enlightenment, and teach the pure
dharma.
 Considered as the successor of Gautam Buddha
 Laughing Buddha is supposed to be an incarnation of Maitreya.
Akasagarbha (Buddha‟s Blessings)

 He is boundless as space. He was the twin brother of Ksitigarbha. He manifests as a


blessing.
Sarvanivarana-Vishkambhin (Buddha‟s Qualities):

 He is invoked to remove or eliminate all the obstacles to insure a successful meditation.


Tara

 Tara is a Vajrayana Buddhist deity who embodies the values of success in labor and
accomplishments.
Vasudhara

 He is one who represents riches, prosperity, and abundance. In Nepal, he is quite popular.
Skanda

 He is the protector of viharas and Buddhist teachings.


Sitatapatra

 She is adored in both Mahayana and Vajrayana traditions as protection against supernatural
peril.

Dipankara
 Dipankara was a Buddha from the past who had attained enlightenment before Gautam
Buddha.
 Dipankara was a past Buddha, Gautam was the most recent Buddha, and Maitreya is the
future Buddha, according to Buddhism.
 He is revered by Buddhists of all three sub-sects: Mahayana, Vajrayana, and even Theravada.

https://t.me/prelimbits
304

202. With reference to the religious practices in India, the "Sthanakvasi" sect belongs to
[2018 ]

A. Buddhism
B. Jainism
C. Vaishnavism
D. Shaivism
Ans. B
DIFFERENT SCHOOLS OF JAINISM

Digambara Shvetambara

Literally means ‗sky clad‟. Digambaras stress Literally means ‗white clad‘. Shvetambaras assert
the practice of nudity as an absolute pre- that the practice of complete nudity is not
requisite to the mendicant‘s path and to the essential to attain liberation.
attainment of salvation.

Magadha Famine (200 Yrs af. death of Some stay back at Magadha af. the famine under
Mahavira), many followers went south under the leadership of Sthalabahu.
the leadership of Bhadrabahu lived in the old
traditional way.
[Mahavira asked to DISCARD CLOTH
completely]

Women lacks the body and rigid will Women are capable in the present lifetime of the
necessary to attain moksha she has to be same spiritual accomplishments
reborn as a man before such an Eg. 19 th Tirthankara is a female named Malli,
attainment is possible who is the only female Tirthankara.

On attaining enlightenment, an omniscient is Even the omniscient requires food.


freed from the needs of ordinary existence
such as, hunger, thirst, sleep, disease, or
fear.

Mahavira never married and renounced the Married Princess Yashoda at a fairly young age
world while his parents were alive. and had a daughter from her named
Priyadarshana. It was only after his parents‘ death
that he became an ascetic.

Idols of Tirthankaras as nude, unadorned and wearing a loin-cloth, bedecked with jewels and
with downcast eyes in the contemplative with glass eyes inserted in the marble.

https://t.me/prelimbits
305

mood.

Canonical Literature = original and Believe in the validity and sacredness of canonical
genuine texts were lost long ago. literature, that is, the 12 angas and sutras, as they
They also refuse to accept the exist now.
achievements of the first council which met Tattvartha Sutra is the probably the most
under the leadership of Acharya authoritative text in Jainism, accepted by
Sthulabhadra and consequently the recasting both Svetambara and Digambara.
of the angas.

They eat food only once in a day. eat more than once in a day.

Must Renounce All Possessions, including allowed to have fourteen possessions including
clothes and is allowed only three possessions: White clothes, Rajoharana – broom made up of
Picchi (a small peacock-feather broom for soft white wool to remove small insects from
brushing insects away) and a Kamandalu (a their path.
wooden water-pot for toilet hygiene) and
Shastra – scriptures.

Rajoharaṇa
Picchi and Kamandalu

Tirthankara Mallinatha: Digambara Jains Svetambara Jains believe that Mallinatha was a
believe that Mallinatha was a male. female by the name Malli.

Birth and Death of Mahavira Birth of Mahavira - 599 BC


Birth of Mahavira - 582 BC Death of Mahavira - 527 BC
Death of Mahavira - 510 BC

Major Sub-Sects: Mula Sangh, Bisapantha, Major Sub-Sects: Murtipujaka, Sthanakvasi,


Terapantha, Taranpantha or Samaiyapantha. Terapanthi
Minor Sub-Sets: Gumanapantha, Totapantha.

https://t.me/prelimbits
306

203. Building 'Kalyana Mandapas' was a notable feature in the temple construction in the
kingdom of [2019]

A. Chalukya
B. Chandela
C. Rashtrakuta
D. Vijayanagara
Ans. D
THE VIJAYANAGAR KINGDOM (C.1336–1672 CE)

 After the 13th century (around the reign of Firoze Shah Tughlaq), due to internal
weaknesses, the process of disintegration of the Delhi Sultanate started.
 It was during this period that some of the provincial rulers declared their independence
from the rule of the Sultanate and a large number of regional states arose.
 The establishment of Vijayanagar and the Bahmani kingdom were a result of the
assertion of power by provincial officers, like Harihara and Bukka, and Alauddin Hasan
Bahman Shah respectively.
 Similarly, the Sultanates in Gujarat, Malwa, Bengal and Jaunpur near Varanasi, emerged as
powerful provincial kingdoms.
 The Vijayanagar kingdom constituted four dynasties —
o Sangama (c.1336–1485 CE),
o Saluva (c.1485–1503 CE),
o Tuluva (c.1503–1570 CE) and
o Aravidu (till the end of the 17th century).
 Source:
o Krishnadevaraya‟s Amukthamalyada,
o Gangadevi‟s Maduravijayam, and

https://t.me/prelimbits
307

o Allasani Peddanna‟s Manucharitam are some of the indigenous literature of this


period.
o Many foreign travelers valuable accounts includes the Moroccan Ibn Batuta,
Venetian Nicolo de Conti, Persian Abdur Razzak, Portuguese Fernao Nuniz and
the Portuguese Domingo Paes.
o The copper-plate inscriptions such as the Srirangam copper-plates of Devaraya II
provide the genealogy and achievements of Vijayanagar rulers.
o The Hampi ruins and other monuments & coins issued by the Vijayanagar rulers
 In the south, Vijayanagara’s main rivals were the Sultans of Madurai and the struggle
 By c.1377 CE, the Sultanate of Madurai was wiped out and the Vijaynagara kingdom
comprised the whole of south India up to Rameshwaram, including the Tamil country as
well as that of the Cheras (Kerala).
 In the north, they had continuous hostility with the Bahmani Sultanate (this conflict will
be dealt in detail later)

 Vijayanagra Capital was Hampi.


Sangama (c.1336–1485 CE)

Harihara and Bukka

 Founder : Harihara and Bukka (c.1336–1377 CE)


 Founded a new city of Vidyanagar or Vijayanagara (meaning city of victory) on the
south bank of river Tungabhadra. (Aaked in CSE 2015)
Deva Raya I (c.1406–1422 CE)

 Very capable ruler noted for his military exploits and his support to irrigation works in his
kingdom.
 He constructed a dam across river Tungabhadra to bring canals into the city to relieve
the shortage of water. He also built a dam on the river Haridra for irrigation purposes.
 Continually at war with the Velamas of Telangana, the Bahmani Sultan of Gulbarga, the
Reddis of Kondavidu, and the traditional rivals of Vijayanagara, the Gajapatis of Kalinga.
 He modernised the Vijayanagar army by improving the cavalry, employing Turkic
archers, and procuring horses from Arabia and Persia.
 He maintained a secular attitude in administrative matters. He had 10,000 Muslims in his
army, being the first Vijaynagar king to do so.
 Italian traveller, Nicolo Conti (in c.1420), and Russian merchant Nikitin (author of Voyage
to India), visited during his reign.
 Deva Raya I was a patron of Kannada literature and architecture. There was a pearl hall
where he honoured men of eminence.

https://t.me/prelimbits
308

 Madhura, a noted Jain poet, was in his court (and also in the court of his father King
Harihara II).

o He wrote the Dharmanathapurana on the life of the 15th Jain Tirthankar in


Kannada, as well as a poem in eulogy of Gommateshvara of Shravanabelagola.

 The noted Hazare Rama temple was constructed during his rule.

Deva Raya II (1425–46 CE)

 Greatest ruler of the Sangama dynasty rulers who was an able administrator, an
ambitious warrior, and a man of letters.
 His work:
o Kannada language (Sobagina Sone and Amaruka)
o Sanskrit language (Mahanataka Sudhanidhi)
o Commentary on the Brahmasutra.

 Noted poets of in his court


o Chamarasa and Kumara Vyasa
o Sanskrit poet Gunda Dimdima
o Kavisarvabhauma (Emperor among Poets)
o Srinatha author of Haravilasam
 Noted South Indian mathematician, Parameshvara also lived in his kingdom.

 He acquired the title of Gajabetegara (Hunter of Elephant).


 Persian traveller Abdur Razzak visited during his reign.
 He built Hampi's Vittala Temple

Vittala Temple

Architecture

 The chief characteristics of the Vijayanagara architecture were the constructions of tall
Raya Gopurams or gateways, the Kalyana mandapam with carved pillars in the
temple premises, the Garbhagriha, and the Amman shrine.
 The horse was the most common animal found on the pillars where sculptures were
carved.

https://t.me/prelimbits
309

204. With reference to forced labor (Vishti) in India during the Gupta period, which one of
the following statements is correct? [2019]

A. It was considered a source of income for the State, a sort of tax paid by the people.
B. It was totally absent in the Madhya Pradesh and Kathiawar regions of the Gupta Empire.
C. The forced laborer is entitled to weekly wages.
D. The eldest son of the laborer was sent as the forced laborer.
Ans. A
GUPTA PERIOD : ECONOMY

 The Gupta period witnessed an increase in land taxes but a decrease in trade and
commerce taxes (such as shulka or tolls).

 The period from 4th century to 8th century was a period of agricultural expansion.
Due to the practice of granting lands to brahmanas (brahmadeyas, agraharas or shasanas)
and to some other officers, vast areas of virgin land were brought under cultivation and
major improvements were made in the existing methods of production.
 The king collected taxes varying from one-fourth to one-sixth of the produce.

 Two new agricultural taxes that appear in Gupta inscriptions are uparikara (probably a
tax imposed on temporary tenants) and udranga (its exact nature is not clear, but might
be water tax or a sort of police tax).

 There is also mention of vata-bhuta → cesses for the maintenance of rites and
halirakara, probably plough tax.

 In addition to these taxes, peasants were also subjected to vishti (forced labour) for
serving the royal army and officials. It was considered a source of income for the State, a
sort of tax paid by the people. It was more prevalent in Madhya Pradesh and Kathiyawar.

 The Gupta and post-Gupta period witnessed a Comparative Decline in the


country‘s Trade & Commerce.
o Till 550 CE, India continued to have some trade with the Eastern Roman empire,
to which it exported silk, and spices. Around the sixth century, the Romans
learnt the art of rearing silk worms, which adversely affected India‟s
export foreign trade.
o The Mandasor inscription is testimony to the fact that a guild of silk weavers left
their original home at Lata in western Gujarat and migrated to Mandasor, where
they gave up their original occupation and took to other professions.

o The disruption of the north-western trade route by the Huns was another
factor for this decline.

https://t.me/prelimbits
310

o India tried to make up for the loss by carrying on trade with South-East Asian
countries, but it did not help revive the economy substantially.

o The loss in trade lessened the inflow of gold and silver into the country. It is
confirmed by a general scarcity of gold coins after the Guptas.

o In ancient India, the Guptas issued a large number of gold coins called dinaras
though the gold content is not as pure as Kushan ones.

o The gold coins of each successive Gupta ruler, after Chandragupta II, contain less
gold and more alloy. After the conquest of the western satraps of Gujarat, they
issued a large number of silver coins (rupyakas) mainly for local exchange and very
few copper coins.
Shrenis or Trade Guilds
 The Ramayana and many plays from the Gupta period and Tamil Sangam literature write in
detail about the trade guilds or shrenis.
 These were professional bodies of jewelers, weavers, ivory carvers, or even salt-makers who
came together to control quality production, create sound business ethics, maintain fair
wages and prices, sometimes operated as a cooperative, and controlled the entry of
newcomers by laying down high standards of craftsmanship and enforcing rules regarding
apprenticeship.
 Each guild had its own chief, assisted by others.
o These functionaries were selected with great care.
o Guild members were even entitled to impeach and punish a chief found guilty of
misconduct.
 The shrenis were not necessarily restricted to a locality and were known to move from one
town to another, over a period of time.
 Occasionally, shrenis (of merchants and artisans) came together in a joint organization,
called the nigama, or the equivalent of a chamber of commerce and industry.
o Some nigamas also included a class of exporters, who transported the specialties of a
town over long distances and sold them at higher margins of profit than those they
could obtain locally.
 By all accounts, the shrenis were very sound and stable institutions and enjoyed
considerable moral and social prestige not only among their own members but in society at
large.

MANIGRAMAM
 Manigiramam, or manigramam, typically refers to a medieval merchant guild, organised
by itinerant ethnic Indian traders, primarily active in south India.

 Along with the Ainurruvar (the Ayyavole Five Hundred) and the Anjuvannam
(the anjuman), the manigiramam played a major role in the commercial activities of the
https://t.me/prelimbits
311

region.
 Unlike the anjuvannam which was confined to the port-towns of south India, the
Manigramam is found both in the port-towns and in the hinter-land trade centres.

 A body of merchants known as "the Vaniggrama" or "Vaniyagrama" attest its


presence in north India as early as first century BC
o They appear in a Karle inscription (first century BCE), a charter of king
Vishnusena from Kathiawad (6th century CE) and in a Sanjeli charter of king
Toramana (6th century CE).
 Records of activities of manigramam in South India are visible from the 5th century CE.
The first reference to manigiramam is found in two copper plate grants from south
Karnataka (from Melekote, Tumkur and from Hassan district), both datable to the 5th
century CE. The Melekote charter records a grant of land to a Buddhist shrine.
 The Quilon Syrian copper plates, of 9th century CE, also attests the presence of the
manigiramam representatives.
 It seems that by the 9th century the manigramam had become interested in ocean
trade too and collaborated with the anjuvannam (the anjuman) on the western coast.
 A Tamil inscription from Takua Pa (9th century CE) in Thailand refers to a manigiramam
guild.
 From the early 10th century the ainurruvar (the Ayyavole Five Hundred) expanded as the
superior guild of merchants of south India. The manigiramam and anjuvannam were
incorporated later into the ainurruvar.
 The Ainurruvar, in and after the 12th century CE, acted as an "umbrella organization" to
cover all other merchant guilds

Land measurement during the Gupta period

 There were various texts and inscriptions used for land measurement during the Gupta
period.
 The terms for short distances were-
o The angula (approx ¾ inch).
o The hasta (cubit) was the standardized distance between the tip of the elbow and the
middle finger (18 inches).
 The terms for large distances were-
o Adhavapa (3/8–1/2 acre)
o Dronavapa (1½–2 acres)
o Kulyavapa (12–16 acres)
 The 'Kulyavapa ' was the area required to sow one kulya of grain and the 'Dronavapa' was
the area required to sow one Drona of grain.
 Such a large number of land measure terms indicates that there was no single standard set

https://t.me/prelimbits
312

of measurement and that different measures were current in different regions.

205. Consider the following : [2019]

1. Deification of the Buddha


2. Treading the path of Bodhisattvas
3. Image worship and rituals
Which of the above is/ are the feature/ features of Mahayana Buddhism?
A. 1 only
B. 1 and 2 only
C. 2 and 3 only
D. 1, 2 and 3
Ans. D
SCHOOLS OF BUDDHISM

 Major Schools = Mahayana & Hinayan


 Other Schools = Theravada, Vajrayana & Zen.
Mahayana Buddhism

 The term Mahayana is a Sanskrit word which literally means Great Vehicle

 Sought salvation through grace & help of Buddha & Bodhisattvas.


 It originated in northern India and Kashmir and then spread east into Central Asia, East
Asia and some areas of Southeast Asia.
 Believed that Buddha will be born again.
 Worship Buddha in idol form== Deification
 Language- Sanskrit.
 Sub-schools : Chittmatra & Madhyamaka
 ZEN is a subschool of Mahayana which is prevalent in china, Korea and related to Taoism.
Buddhist schools embedded in China, Korea, Tibet and Japan belong to the Mahayana
tradition.
Hinayana Buddhism

 Considered Lesser vehicle . Also known as Abandoned Vehicle or Defective vehicle. It


believes in the original teaching of Buddha or Doctrine of elders.
 Sought individual salvation through self-discipline & meditation.
 Buddha will never be reborn.

 Did not believe in Idol worship.

https://t.me/prelimbits
313

 Language- Pali language.


 Buddha was an intellectual, not a god
 Encompasses 18 subschools.
 Most important – Sarvastivada, Theravadin & Sautantrika.
Theravada
 Theravada Buddhism is older and the more conservative of the two main divisions of
Buddhism and is often referred to as the „traditions of the elder‟
 Theravada is a Hinayana sect.
 Theravada Buddhists strive to become Arhats and gain freedom from the cycle of
samsara.
 Theravada Buddhism developed in Sri Lanka and subsequently spread to the rest of
Southeast Asia. It is the dominant form of religion in Cambodia, Laos, Myanmar, Sri
Lanka, and Thailand
Vajrayana
 Vajrayana means “The Vehicle of the Thunderbolt”, also known as tantric Buddhism.
 This Buddhist school developed in India around 900 CE.
 Believed in acquiring magical power for liberation.
 Developed in Tibet and believes in the worship of female deities - Taras.
 It became popular in Eastern India, particularly Bengal and Bihar
Zen
 It is a school of Mahayana Buddhism that originated in China during the Tang dynasty as
the Chan school of Chinese Buddhism in and later developed into various schools.
 It spread to Japan in 7th century C.E.
 Meditation is the most distinctive feature of this Buddhist tradition

206. In which of the following relief sculpture inscriptions is 'Ranyo Ashoka' (King
Ashoka) mentioned along with the stone portrait of Ashoka? [2019]

A. Kanganahalli
B. Sanchi
C. Shahbazgarhi
D. Sohgaura
Ans. A
KANGANAHALLI INSCRIPTIONS

 Kanganahalli, situated about 3 km from Sannati, is an important Buddhist site where an

https://t.me/prelimbits
314

ancient Mahastupa was built. It is on the left bank of the Bhima river in Chitapur taluk,
Kalaburagi district in Karnataka.
 145 short inscriptions were discovered from the excavations site, dating between 1st
century BCE to 1st century CE. The very important discovery was the sculpture of Maurya
emperor Ashoka with the label "Rayo Asoka". in Brahmi script.

Ashoka with his Queen, at Sannati (Kanaganahalli Stupa), 1st-3rd century CE.
SOHGAURA COPPER PLATE INSCRIPTION
 The Sohgaura copper plate inscription is an Indian copper plate inscription written in
Prakrit in the Brahmi script.
 It was discovered in Sohgaura, a village on the banks of the Rapti River, about 20 km
south-east of Gorakhpur, in the Gorakhpur District, Uttar Pradesh
 The inscription is sometimes presented as pre-Ashokan, even pre-Mauryan, but the writing
of the plate, especially the configuration of akshara would rather suggest a date after
Ashoka

 The text mentions the establishment of two grain depots (Kosthagara) to fight
against famine.
SHAHBAZ GARHI
 Shahbaz Garhi, or Shahbazgarhi, is a village and historic site located in Mardan District of
Khyber Pakhtunkhwa, Pakistan.
 In old times all these facilities made it attractive for the army and travelers to dig in their
tents here, stay for few days and organize their further strategy. The historic Stones of
Ashoka (commonly known to the native people by the name of Hkule Gutt),
 The town is the location of ancient rock-inscriptions that are cut into two large rock
boulders and written in the Kharosthi script.

 They were constructed during the 3rd Century BC (272-231 BC), during the
reign of Ashoka, the famous Mauryan emperor,
 The rock edicts were added to the UNESCO World Heritage Tentative List on 30 January
2004 in the Cultural category

https://t.me/prelimbits
315

207. Which one of the following is not a Harappan Site? [2019]

A. Chanhudaro
B. Kot Diji
C. Sohagaura
D. Desalpur
Ans. C
Full Doc here : https://t.me/prelimbits/5417

208. With reference to the cultural history of India consider the following pairs: [2020]

1. Parivrajaka - Renunciant and Wanderer


2. Shramana - Priest with a high status
3. Upasaka - Lay follower of Buddhism
Which of the pairs given above are correctly matched?
A. 1 and 2 only
B. 1 and 3 only
C. 2 and 3 only
D. 1, 2 and 3
Ans. B
BUDDHISM : KEY TERM / CONCEPT

Key Term / Concept Meaning / Purpose

Chaityas Prayer halls of monks

Viharas Monasteries

Pravrajya Ceremony to mark a person‟s going forth from home into


homelessness.

Upasampada Ordination ceremony when the novice [new] becomes a full-

https://t.me/prelimbits
316

fledged member of the monastic community.

Uposatha Ceremony held on full moon and new moon.

Parajika Literally meaning defeat, it included four most serious


offences involving expulsion from the Sangha (sexual
intercourse, taking what is not given, killing someone,
and making false claim of spiritual attainment).

Pavarana Pavarana is a Buddhist holy day celebrated on Aashvin full


moon of the lunar month. It marks the end of the rainy
season, sometimes called ‗BUDDHIST LENT‘. On this day, each
monk must come before the community of monks (Sangha)
and atone for an offense he may have committed during the
Vassa (rainy season).

Upasakas & Upasikas Male followers & Female followers who have taken refuge in
the Buddha, Dhamma and Sangha, but who have not taken
monastic vows.

Boddhisattva Enlightened beings who have renounced their own salvation


and put off entering paradise in order to help others attain
enlightenment. They devote all their power and energy to save
suffering beings in this world. As deities of compassion,
Bodhisattvas are typically represented with precious jewellery,
elegant garments, and graceful postures.
Maitreya
 Maitreya will be successor of Gautama Buddha. He is
also known as Ajita Boddhisattva.
 He holds a ―water phial‖ in his left hand.
 Earliest mention of Metteyya is in the Digha Nikaya 26 of
the Pali Canon.
 It is said that he will arrive when oceans will decrease in
size (that is why keeps a Kumbha or philial in his hand)
and will rule the Ketumati Pure Land (Varanasi).

Bhikkhu Sangha & Sangha of monks & Nuns


Bhikkhuni Sangha

Theragatha & Therigatha Verses of Elder Monks & Nuns


Therigatha
 Often translated as Verses of the Elder Nuns.

https://t.me/prelimbits
317

 Is a Buddhist text, a collection of short poems of early


enlightened women who were elder nuns (having
experienced 10 Vassa or monsoon periods).
 In the Pali Canon, the Therigatha is classified as part of
the Khuddaka Nikaya, the collection of short books in
the Sutta Pitaka. It consists of 73 poems organized into
16 chapters. It is the companion text to the
Theragatha, verses attributed to senior monks.

Paribbajaka/Parivrajaka Wanderer

Shakra God Indra

Arhats Liberated beings

Sramana  Sramana means "one who labours, toils, or exerts


themselves (for some higher or religious purpose)" or
"seeker, one who performs acts of austerity, ascetic".
 During its development, the term came to refer to several
non-Brahmanical ascetic religions parallel to but
separate from the Vedic religion.
 The Sramana tradition includes primarily Jainism,
Buddhism, and others such as the Ajivika.
 The Sramana religions became popular in the same
circles of mendicants from greater Magadha that led to
the development of spiritual practices, as well as the
popular concepts in all major Indian religions such as
saṃsāra (the cycle of birth and death) and moksha
(liberation from that cycle).
 The Sramana traditions have a diverse range of beliefs,
ranging from accepting or denying the concept of soul,
fatalism to free will, idealization of extreme
asceticism to that of family life, renunciation, strict
ahimsa (non-violence) and vegetarianism to
permissibility of violence and meat-eating.

209. With reference to the period of the Gupta dynasty in ancient India, the towns
Ghantasala, Kadura, and Chaul were well known as [2020]

A. ports handling foreign trade


B. capital of powerful kingdoms

https://t.me/prelimbits
318

C. places of exquisite stone art and architecture


D. important Buddhist pilgrimage centers
Ans. A
PORTS FOR FOREIGN TRADE DURING ANCIENT TIMES

210. With reference to the history of India, consider the following pairs: [2020-I]

Famous Place Present State

1. Bhilsa Madhya Pradesh

2. Dwarasamudra Maharashtra

https://t.me/prelimbits
319

3. Girnagar Gujarat

4. Sthanesvara Uttar Pradesh

Which of the pairs given above are correctly matched?


A. 1 and 3 only
B. 1 and 4 only
C. 2 and 3 only
D. 2 and 4 only
Ans. A
 Bliilsa- Madhya Pradesh. Known as Vidisha, main centre of the Sunga dynasty &Udaigiri
caves in Madhya Pradesh (not to be confused with Udaigiri-Odisha)
 Girinagar- Gujarat
 Stlianeshwara- located in Harayana (not in Uttarpradesh) Main seat of the
Pushyabhutidyansty- Harshavardhan (606-647CE) originally belonged to Sthaneshwara, who
later on shifted the capital at Kannauj
 Dwarsamudra- Hoysala dynasty used to rule from Dwarsamudra- also known as Halebid -
located in Karnataka.

211. Who among the following rulers advised his subjects through this inscription?:
“Whosoever praises his religious sect or blames other sects out of excessive devotion
to his own sect, with the view of glorifying his own sect, he rather injures his own sect
very severely,” [2020]

A. Ashoka
B. Samudragupta
C. Harshavardhana
D. Krishna Deva Raya
Ans. A
 Ashoka has been credited with the propagation of the tenets of the Buddhist sect.
 Ashoka's dhamma (way of life) did not involve any worship to God or sacrifices, and he
thought that his duty towards his subjects was like that of a father towards his son.
 Ashoka's edicts and inscriptions contained instructions on religious matters especially
Buddhism and rules for the conduct of Bhikshuks.
 Ashoka's inscriptions can be classified into Pillar Edicts, Major Rock Edicts, and Minor
Rock Edicts.
 James Prinsep was the first person to decipher Ashoka‟s edicts.
 The excerpt from the12th Major Rock Edict reads:
https://t.me/prelimbits
320

o "Whosoever praises his religious sector blames other sects out of excessive
devotion to his own sect, with the view of glorifying his own sect, he rather
injures his own sect very severely".

o Growth in essentials can be done in different ways, but all of them have as their root
restraint in speech, that is, not praising one's
own religion, or condemning the religion of others without good cause.
o And if there is cause for criticism, it should be done in a mild way.
o But it is better to honour other religions for this reason.
o By so doing, one's own religion benefits, and so do other religions, while doing
otherwise harms one's own religion and the religions of others.
 The inscriptions of other major rock edicts read-
o First- Prohibits animal slaughter
o Second- Care for man and animals
o Third- Generosity to Brahmins
o Fourth- Sound of Dhamma
o Fifth- Slave right
o Sixth- Welfare measure
o Seventh- Tolerance for all religions
o Eighth- Ashoka's first visit to Bodh Gaya
o Ninth- Condemns popular ceremonies
o Tenth- Disapproves of the individual's desire for glory
o Eleventh- Elaborates on Dhamma
o Thirteenth-Ashoka's victory over Kalinga
o Fourteenth- Engraving of inscriptions installed in various parts of the country.

212. With reference to the history of India, the terms “kulyavapa” and “dronavapa” denote
[2020]

A. measurement of land
B. coins of different monetary value
C. classification of urban land
D. religious rituals
Ans. A
LAND MEASUREMENT DURING THE GUPTA PERIOD

 There were various texts and inscriptions used for land measurement during the Gupta

https://t.me/prelimbits
321

period.
 The terms for short distances were-
o The angula (approx ¾ inch).
o The hasta (cubit) was the standardized distance between the tip of the elbow and
the middle finger (18 inches).
 The terms for large distances were-
o Adhavapa (3/8–1/2 acre)
o Dronavapa (1½–2 acres)
o Kulyavapa (12–16 acres)
 The 'Kulyavapa ' was the area required to sow one kulya of grain and the 'Dronavapa' was
the area required to sow one Drona of grain.
 Such a large number of land measure terms indicates that there was no single standard set
of measurement and that different measures were current in different regions.

213. Which of the following phrases defines the nature of the „Hundi‟ generally referred
to in the sources of the post-Harsha period? [2020]

A. An advisory issued by the king to his subordinates


B. A diary to be maintained for daily accounts
C. A bill of exchange
D. An order from the feudal lord to his subordinates
Ans. C
 Hundi is a financial instrument that developed in Medieval India for use in trade and
credit transactions.
 They were used →
o As remittance instruments (to transfer funds from one place to another)
o As credit instruments (to borrow money)
o trade transactions (as bills of exchange).

214. Consider the following events in the history of India: [2020]

1. Rise of Pratiharas under King Bhoja


2. Establishment of Pallava power under Mahendravarman - I
3. Establishment of Chola Parantaka - I
4. Pala dynasty founded by Gopala
What is the correct chronological order of the above events, starting from the earliest time?

https://t.me/prelimbits
322

A. 2-1-4-3
B. 3-1-4-2
C. 2-4-1-3
D. 3-4-1-2
Ans. C
 Mahendra Varrnan-1 (590- 630 CE)
 Gopala (750 CE)
 King Bhoja- Pratihara( 836-885 CE)
 Prantaka-1 Chola ( 907-953 CE)
MAHENDRAVARMAN I (590−630 CE)
 During his reign, conflict between the Chalukyas of Badami and the Pallavas heightened and
he was defeated by Pulekshin II at Pullalur (near Kanchi) who annexed the northern part
of the Pallava kingdom.
 Was a great patron of arts, was himself a poet and musician.
 He wrote the Mattavilasa Prahasanna and initiated the construction of the famous
cave temple at Mahabalipuram.
 He bore many titles, Vichitrachitta is one of them.
 Was earlier a Jaina but later took up Shaivism under the influence of Appar.
GOPALA (AROUND 750 CE)
 As per Khalimpur copper inscription of Dharampala, he founded Pala dynasty, replacing
later Guptas of Magadha and Khadga dynasty.
 Built famous Odantpuri monastery at Bihar, 2nd largest after Nalanda university
BHOJA I/ MIHIR BHOJA (836 885)
 Greatest ruler of Pratiharas, got victories over Palas and Rashtrakutas.
 Capital at Kannauj, which also known as Mahodaya.
 Barah copper plate inscription, there is mention his military camp i.e., skandhavara at
Mahodaya.
 Called King Baura by Al-Masudi & Abu suleman called him king Juzr. According to Arab
travellers, the Pratihara rulers had the best cavalry in India.
 Devotee of Vishnu, adopted title Adivaraha.
 He made Teli mandir Gwalior Dedicated to Shiva, Vishnu and Matrikas,
Parantaka Chola I
 Parantaka Chola I (873 CE–955 CE) was a Chola emperor who ruled for forty-eight years,
annexing Pandya by defeating Rajasimhan II
 The best part of his reign was marked by increasing success and prosperity.
https://t.me/prelimbits
323

215. With reference to the religious history of India, consider the following statements:
[2020]

1. Sthaviravadins belong to Mahayana Buddhism.


2. Lokottaravadin sect was an offshoot of the Mahasanghika sect of Buddhism.
3. The deification of Buddha by Mahasangliikas fostered Mahayana Buddhism.
Which of the statements given above are correct?
A. 1 and 2 only
B. 2 and 3 only
C. 3 only
D. 1, 2 and 3
Ans. B
Fourth Council convened in Kashmir under the patronage of Kanishka →
 It resulted into division of all the Buddhists into two major sects
 Sarvastivadins (popular in Kashmir and Mathura regions) and Mahasanghikas together
forming the Mahayanists (followers of the Greater Vehicle),
 Sthaviravadins and others forming the Hinayanists (followers of the Lesser Vehicle); Thus,
Sthaviravadins is not part of Mahayana.
 The Mahasanghikas gave rise to the sects → 1. Ekavyaharikas 2. Lokottaravadins 3.
Kukkutikas 4. Bahusrutiyas 5. Prajnapativadins 6. Chaitya-sailas 7. Apara-sailas 8. Uttara-
sailas.

216. With reference to the scholars/litterateurs of ancient India, consider the following
statements: [2020]

1. Panini is associated with Pushyamitra Shunga


2. Amarasimha is associated with Harshavardhana
3. Kalidasa is associated with Chandra Gupta-II
Which of the statements given above is/are correct?
A. 1 and 2 only
B. 2 and 3 only
C. 3 only
D. 1, 2 and 3
Ans. C
 Amarsimha belonged to the Court of Chandra Gupta- II (he was one of the navratna).
Thus, by elimination we are left with correct answer C.

https://t.me/prelimbits
324

 Kalidasa belonged to the Court of Chandra Gupta- II (he was one of the Navratna).
 Panini belonged to the 6th - 4th Century BCE Period. He wrote Astdhvayi, where as
Pushyamitra Sunga belonged to the Post Mauryan Period
AMARASIMHA

 Sanskrit Grammarian and Poet


 One of the nine gems of Chandragupta Vikramaditya (Aka. Chandragupta II)
 Contribution: Amara-Kosha
PANINI

 Paṇini was a Sanskrit philologist, grammarian, and revered scholar in ancient India,
variously dated between the 6th and 4th century BCE.
 Panini has been considered the "First Descriptive Linguist", and even labelled as ―The
Father of Linguistics‖
 Paṇini is known for his text Astadhyayi, a sutra-style treatise on Sanskrit grammar
o Which is the foundational text of the Vyakaraṇa branch of the Vedanga, the
auxiliary scholarly disciplines of the Vedic period
 His aphoristic text attracted numerous bhashya (commentaries), of which Patanjali's
Mahabhaṣya is the most famous.
o It is a commentary on selected rules of Sanskrit grammar from Paṇini's treatise,
the Astadhyayi, as well as Katyayana's Varttika-sutra, an elaboration of Paṇini's
grammar. It is dated to the 2nd century BCE
 Paṇini's comprehensive and scientific theory of grammar is conventionally taken to mark
the start of Classical Sanskrit.
 He probably lived between 400 and 350 BCE. In an inscription of Siladitya VII of Valabhi,
he is called Śalāturiya, which means "man from Salatura". This means Panini lived in Salatura
of ancient Gandhara (present day north-west Pakistan),
o According to the memoirs of 7th-century Chinese scholar Hiuen Tsang, there was a
town called Suoluoduluo on the Indus where Paṇini was born, and he composed the
Qingming-lun (Sanskrit: Vyakaraṇa)

217. With reference to the cultural history of India, which one of the following is the
correct description of the term „paramitas‟? [2020]

A. The earliest Dharmashastra texts were written in aphoristic (sutra) style.


B. Philosophical schools that did not accept the authority of Vedas.
C. Perfections whose attainment led to the Bodhisattva path
D. Powerful merchant guilds of early medieval South India.
Ans. C

https://t.me/prelimbits
325

PARAMITAS OR PATH TO PERFECTION FOR BECOMING A BODHISATTVA

 A Bodhisattva practices paramita to become a Buddha by developing goodness to others


without any trace of selfish behaviour.
 Paramita, or perfection in Buddhism is associated with noble qualities that are found in
Buddha-like enlightened beings.
 These qualities are:
o Daan parami: generosity, giving of oneself
o Sheel parami: virtue, morality, proper conduct
o Nishkaam parami: renunciation
o Panna parami: wisdom, discernment
o Virya parami: energy, diligence, vigour, effort
o Shanti parami: patience, tolerance, forbearance, acceptance, endurance
o Sacha parami: truthfulness, honesty
o Adhiṭṭhana parami: determination, resolution
o Metta parami: goodwill, friendliness, loving-kindness
o Upekkhā pāramī: equanimity, serenity

218. Which one of the following ancient towns is well known for its elaborate system of
water harvesting and management by building a series of dams and channelising
water into connected reservoirs? [2021]

A. Dholavira
B. Kalibangan
C. Rakhigarhi
D. Ropar
Ans. A
Dholavira (Luni) Guj
 Dholavira is a Harappan City in the Rann of Kutch in the state of Gujarat in western India.
 Dholavira‘s location is on the Tropic of Cancer. It is one of the five
largest Harappan sites and most prominent archaeological sites in
India
 It is located on Khadir Bet Island in the Kachchh Desert Wildlife
Sanctuary in the Great Rann of Kachchh.
 Dholavira is the first site of the ancient Indus Valley Civilisation
(IVC) in India to get the UNESCO World Heritage tag.
 It was discovered in 1968 by archaeologist Jagat Pati Joshi.
https://t.me/prelimbits
326

 It was a commercial and manufacturing hub for about 1,500 years before
 its decline and eventual ruin in 1500 BC.
 Distinct features:
o Cascading series of water reservoirs.
o Fortified citadel, a middle town and a lower town
o Outer fortification.
o Two multi-purpose grounds, one of which was used for festivities and other as a
marketplace.
o Nine gates with unique designs.
o Funerary architecture featuring tumulus — hemispherical structures like the
Buddhist Stupas.
o Multi-layered defensive mechanisms, extensive use of stone in construction and
special burial structures.
o Longest inscription comprising ten large-sized signs of the Harappan script. It
was embedded on a three-meter-long board

219. From the decline of Guptas until the rise of Harshavardhana in the early seventh
century, which of the following kingdoms were holding power in Northern India?

1. The Guptas of Magadha


2. The Paramaras of Malwa
3. The Pushyabhutis of Thanesar
4. The Maukharis of Kanauj
5. The Yadavas of Devagiri
6. The Maitrakas of Valabhi
Select the correct answer using the code given below:
A. 1, 2 and 5
B. 1, 3, 4 and 6
C. 2, 3 and 4
D. 5 and 6
Ans. B
 After the decline of the Guptas, the Later Guptas succeeded as the rulers of Magadha
during the early seventh century. They ruled parts of the northern Including Magadha
(current day Bihar). Hence statement 1 is correct.
 The Paramara dynasty ruled the western and central parts of India including Malwa,
between the 9th and 14th century AD, before being captured by Delhi Sultanate
https://t.me/prelimbits
327

rulers. Hence statement 1 is not correct.


 The Pushyabhutis of Thanesar (current day Haryana) ruled various parts of Northern
India during the 6th and 7th century AD. Hence statement 3 is correct.
 The Maukharis dynasty ruled over parts of Uttar Pradesh and Magadha including
Kannauj. The dynasty ruled till around 606 AD when it was reconquered by the Later
Guptas. Hence statement 4 is correct.
 The Yadavas of Devagiri ruled from around 1187 to 1317 AD in western parts of
India. Hence statement 5 is not correct.
 The Maitrakas of Valabhi ruled western and northern parts of current-day Gujarat
during the late 5th century to late 8th century AD. Hence statement 6 is correct.

220. With reference to the history of ancient India, which of the following statements
is/are correct?

1. Mitakshara was the civil law for upper castes and Dayabhaga was the civil law for lower
castes.
2. In the Mitakshara system, the sons can claim the right to the property during the lifetime of
the father, whereas in the Dayabhaga system, it is only after the death of the father that the
sons can claim the right to the property.
3. The Mitakshara system deals with matters related to the property held by male members
only of a family, whereas the Dayabhaga system deals with the matters related to the
property held by both male and female members of a family.
Select the correct answer using the code given below:
A. 1 and 2
B. 2 only
C. 1 and 3
D. 3 only
Ans. B
Dayabhaga By Jimutavahana

 The Dayabhaga is a Hindu law treatise written by Jimutavahana which primarily focuses
on inheritance procedure.

 The Dayabhaga was the strongest authority in Modern British Indian courts in the
Bengal region of India, although this has changed due to the passage of the Hindu
Succession Act of 1956 and subsequent revisions to the act.

 Based on Jimutavahana criticisms of the Mitakṣara, it is thought that his work is


precluded by the Mitākṣarā. This has led many scholars to conclude that the Mitākṣarā
represents the orthodox doctrine of Hindu law, while the Dāyabhāga represents the
reformed version.
https://t.me/prelimbits
328

 The central difference between the texts is based upon when one becomes the owner of
property.

o The Dāyabhāga does not give the sons a right to their father's ancestral property
until after his death,

o unlike Mitākṣarā, which gives the sons the right to ancestral property upon their
birth.

Jimutavahana

 Jimutavahan (c. 12th century) was an Indian Sanskrit scholar and writer of legal and
religious treatises on Vaishnavism of early medieval period.

 He was the earliest writer on smriti (law) from Bengal whose texts are extant.

 Major works :

o Kalaviveka → exhaustive analysis of the auspicious kala (timings) for the


performance of religious rites and ceremonies. This text also contains discussions on
solar and lunar months.

o Vyvahāra-mātrikā or Nyayaratna-mātrikā or Nyayamātrikā → Dealt with


vyavahāra (judicial procedure). The text is divided into five sections,
Vyvaharamukha, Bhashapada, Uttarapada, Kriyapada and Nirnayapada.

o His magnum opus Dāyabhāga has dealt with the laws of inheritance based on
Manusmriti.

 In Bengal (and post-independence West Bengal and Tripura) and Assam,


Dāyabhāga was the principal guide for laws on inheritance till the enactment
of the Hindu Succession Act, 1956.

 This treatise differs in some aspects from Mitakshara, which was prevalent in
other parts of India based on Yajnavalka Smriti.

 The right of a widow without any male issue to inherit the properties of her
deceased husband is recognized in Dāyabhāga.

MITAKSHARA BY VIJNANESVARA

 The Mitakshara is a vivṛti (legal commentary) on the Yajnavalkya Smriti best known for
its theory of "inheritance by birth."
 It was written by Vijnanesvara, a scholar in the Western Chalukya [Kalyani] court in the
late eleventh and early twelfth century during the reign of Vikramaditya VI
 Along with the Dāyabhāga, it was considered one of the main authorities on Hindu Law
from the time the British began administering laws in India.
 The entire Mitakshara, along with the text of the Yajnavalkya Smriti, is approximately 492
closely printed pages.

https://t.me/prelimbits
329

221. With reference to the history of ancient India, Bhavabhuti, Hastimalla and
Kshemeshvara were famous [2021]

A. Jain monks
B. playwrights
C. temple architects
D. philosophers
Ans. B
BHAVABHUTI

 8th-century Sanskrit poet & Play writer


 Patron: Yashovarman of Kannauj
 Yasovarman Malatimadhava, Mahaviracharita
 Works
o Mahaviracharita → depicting the early life of Rama
o Malatimadhava a play based on the romance of Malati and Madhava
o Uttararamacarita (The story of Rama's later life) → coronation, the abandonment
of Sita, and their reunion
HASTIMALLA (13th century)
 Wrote 8 plays including Vikrant Kaurava and Subhadra Harana.
 He was a noted Kannada poet and playwright in the Hoysala Empire.
KSHEMESHVARA (990 – c. 1070 CE)
 Was an 11th-century Sanskrit poet from Kashmir in India.
 One of the important work attributed to him is Dasavatar Charita.

222. Consider the following pairs: [2021]

Historical place Well-known for

1. Burzahom Rock-cut shrines

2. Chandraketugarh Terracotta art

3. Ganeshwar Copper artefacts

Which of the pairs given above is/are correctly matched?


A. 1 only
B. 1 and 2

https://t.me/prelimbits
330

C. 3 only
D. 2 and 3
Ans. D
Burzahom, Kashmir

 At Burzahom, Kashmir valley, tools made of bones and stones and tools representing
the ritualistic practices were found. It represents the natural setting of the Neolithic
people.

 Several terracotta artefacts have been unearthed at Chandraketugarh (West Bengal)


including erotic art on the plaques.

 At the sites of Ganeshwar (Jodhpur) near the Khetri copper mines, several copper
artefacts were excavated viz. arrowheads, spearheads, bangles, pottery and chisels etc.
o Ganeshwar had mainly supplied copper objects to Harappa. The archaeological site
was named after the present-day village, Gilund, and is locally known as Modiya
Magari which means ―bald habitation mound‖.

223. Which one of the following statements is correct? [2021]

A. Ajanta Caves lie in the gorge of the Waghora river.


B. Sanchi Stupa lies in the gorge of the Chambal river.
C. Pandu – lena cave shrines lie in the gorge of the Narmada river.
D. Amaravati Stupa lies in the gorge of the Godavari river.
Ans. A

 The Ajanta caves are cut into the side of a cliff that is on the south side of a U-shaped
gorge on the small river Waghora.

 Sanchi Stupa lies in an upland plateau region, on a flat-topped sandstone hill, just west
of the Betwa River and about 5 miles (8 km) southwest of Vidisha.

 The Buddhist monuments Pandavleni Caves also known by the names Pandu Lena
caves and Trirashmi caves are a group of 24 rock-cut caves.
o It is located high up on the north face of Trivashmi hill 8 km south of Nasik city.
o The donative inscriptions indicate that these excavations are belonging to the
Satavahana and Kshatrapa periods.
o The nearest river is the Godavari.

 Amravati stupa was founded by an emissary of Emperor Ashoka and its construction
got completed around 2000 years ago.
o Amravati Stupa illustrates Lord Buddha in a human form and He is shown
subduing an elephant.

https://t.me/prelimbits
331

o It is taller than the Sanchi stupa and has 95 ft high platforms, extending in the four
cardinal directions, along with a huge circular dome.
o It is situated near river Krishna.

224. According to Kautilya‟s Arthashastra, which of the following are correct? [2022]

1. A person could be a slave as a result of a judicial punishment.


2. If a female slave bore her master a son, she was legally free.
3. If a son born to a female slave was fathered by her master, the son was entitled to the legal
status of the master‘s son.
Which of the statements given above are correct?
A. 1 and 2 only
B. 2 and 3 only
C. 1 and 3 only
D. 1, 2 and 3
Ans. D

 The Arthashastra states that a man could be a slave either by birth, by voluntarily
selling himself, by being captured in war, or as a result of judicial punishment.

 Slavery was a recognized institution and the legal relationship between master and
slave was clearly defined e.g. if a female slave bore her master a son,
o She was legally free, and
o the son was entitled to the legal status of the master's son.
 When a female slave gives birth to a child of her master, both the child and its mother shall
be considered free at once. If the mother is to be in bondage for sustenance, her brother
and sister will be set free.
 Selling or pawning the life of a male or female slave once freed shall be punished with a fine
of 12 panas, except those who enslave themselves.
 Mauryan empire was a vast territorial entity. Various levels of administration were
required to govern it well.
 The Arthashastra, Greek accounts and Ashokan inscriptions give us a good idea about
the administrative system.
 According to the Arthshastra of Kautilya, a striking development of the Mauryan
period was the employment of slaves in agricultural operations.
 Domestic slaves were found in India from Vedic times onwards. However, ancient Indian
society was not a slave society.
 Kautilya‘s Arthshastra contains 15 adhikaranas or books.

https://t.me/prelimbits
332

o The first five deal with ‗tantra‘ i.e. internal administration of the state.
o The next eight deal with ‗avapa‘ or its relations with neighbouring states.
o The last two are miscellaneous in character.
 Arthshastra deals with problems of administration as well as laws of property and crime.
 Arthshastra argues the case for a fully-centralised government for the empire.
 It also mentions female bodyguards and spies that Mauryan rulers maintained. These
female bodyguards were skilled in archery.

225. With reference to Indian history, consider the following pairs:

Historical person Jaina scholar

1. Aryadeva Jaina scholar

2. Dignaga Buddhist scholar

3. Nathamuni Vaishnava scholar

How many pairs given above are correctly matched ?


A. None of the pairs
B. Only one pair
C. Only two pairs
D. All three pairs
Ans. C

Historical person Known as

Aryadeva  Aryadeva, was a Mahayana Buddhist monk, a disciple of


Nagarjuna and a Madhyamaka philosopher.

Dignaga  Dignaga was an Indian Buddhist scholar and one of the


Buddhist founders of Indian logic (hetu vidya).
 Dignaga‘s work laid the groundwork for the development of
deductive logic in India and created the first system of Buddhist
logic and epistemology (Pramana).

Nathamuni  Sri Ranganathamuni, popularly known as Sriman Nathamuni, was


a Vaishnava theologian who collected and compiled the Nalayira
Divya Prabandham.

https://t.me/prelimbits
333

226. With reference to Indian history, consider the following texts: [2022]

1. Nettipakarana
2. Parishishtaparvan
3. Avadanashataka
4. Trishashtilakshana Mahapurana
Which of the above are Jaina texts
A. 1,2 and 3
B. 2 and 4 only
C. 1,3 and 4
D. 2, 3 and 4
Ans. B

 Nettipakaraṇa is related to Buddhism‟s Pali Canon. Hence, option 1 is incorrect.


o The Nettipakaraṇa is a mythological Buddhist scripture, sometimes included in the
Khuddaka Nikaya of Theravada Buddhism's Pali Canon.
o The main theme of this text is Buddhist Hermeneutics through a systematization of
the Buddha's teachings.

 Parishishtaparvan was written by Hemachandra which describes the establishing


Chandragupta‟s connections with Jainism.
o According to the Parishishtaparvan, Jayasimha spotted Hemachandra while passing
through the streets of his capital.
o The king was impressed with an impromptu verse uttered by the young monk
o Acharya Hemachandra was an Indian Jain scholar, poet, mathematician, and
polymath who wrote on grammar, philosophy, prosody, mathematics, and
contemporary history.
o Noted as a prodigy by his contemporaries, he gained the title kalikalasarvajna,
"the knower of all knowledge in his times".

 The Avadanashataka or ―Century of Noble Deeds‖ is an anthology in Sanskrit which


contains some collection of Buddhist narratives compiled from the second to fifth centuries
CE.

 Trishashthilkshana Mahapurana is a major Jain text composed largely by Acharya


Jinasena during the rule of Rashtrakuta.

227. Consider the following pairs: [2022]

Site of Ashoka‟s major rock edicts Location in the State of

https://t.me/prelimbits
334

1. Dhauli Odisha

2. Erragudi Andhra Pradesh

3. Jaugada Madhya Pradesh

4. Kalsi Karnataka

How many pairs given above are correctly matched?


A. Only one pair
B. Only two pairs
C. Only three pairs
D. All four pairs
Ans. B
DHAULI

 Dhauli Hill or Dhauligiri is located on the left banks of river Daya, a tributary of
Mahanadi, and situated about 10 km from Bhubaneswar in Odisha.
 It is known for "Dhauli Santi Stupa", a peace pagoda monument which witnesses the
great Kalinga War.
 Dhauli is an important early historical urban center of Orissa. Archaeological remains found
at Dhauli trace its antiquity to the 3rd century BC, especially from the time of Ashoka.
Dhauli is of historical importance as one of the famous inscriptions of Emperor Ashoka is
located here.
 It is believed to be the location of the famous Kalinga war, where the banks of river Daya
would have provided a natural boundary line between the two armies. Kalinga war holds a
very important position in the Indian history, as after witnessing the bloodshed and
massacre Ashoka laid down his arms to follow the path of Dharma-Vijaya (religious victory).
ERRAGUADI
 Situated in Kurnool district, Andhra Pradesh.
 Both major edicts and minor edicts of Ashoka giving the principles of Dhamma have
been excavated here.
 No remains of a town have been discovered in the area, but it is possible that it was an
important frontier town during Ashoka‟s reign, with a route leading through it to South
Indian kingdoms.
JAUGADA
 Jaugada is a ruined fortress in the Ganjam district in Odisha.
 Jaugada lies 35 km north-west of Berhampur and 160 km south-west of Bhubaneshwar.
 At Jaugada like Dhauli we find a Major Rock Edict of emperor Ashoka.

https://t.me/prelimbits
335

KALSI
 Kalsi is a small town located in Uttarakhand between Chakrata and Dehradun near
Dakpatthar on the banks of Yamuna river.
 Kalsi town is known for its heritage importance.
 Famous Mauryan Emperor, Ashoka's inscriptions as rock edicts is present in Kalsi. It's
importance rises as it is the only Ashoka's Rock edict located in North India.
 The language used in these edicts is Pali.

228. Which one of the following statements about Sangam literature in ancient South
India is correct?

A. Sangam poems are devoid of any reference to material culture.


B. The social classification of Varna was known to Sangam poets.
C. Sangam poems have no reference to warrior ethic.
D. Sangam literature refers to magical forces as irrational.
Ans. B
SANGAN LITERATURE
 Sangam literature is the name given to the earliest available Tamil literature.
 The collection of poems known as Sangam literature was produced over six centuries from
about 300 BC to AD 300 by Tamils from diverse social backgrounds, although most of the
work is believed to have been composed between 100 CE and 250 CE.
 The word ‗Sangam‘ literally means association. Here, it implies an association of Tamil poets
that flourished in ancient southern India.
 The three chief Tamil kingdoms of this period were the Cheras, the Cholas, and the
Pandyas.
 In Sangam society, It was during the stimulus of contact with the elements of material
culture brought from the north to the extreme end of the peninsula by traders, conquerors
and Jaina, Buddhists, and some Brahmana missionaries, that they came to have social
classes, and they also came to practice wet paddy cultivation and founded numerous
villages and towns.
 Sangam poems contain several incidental references to material culture, often as part of the
poem‘s setting or in similes and allusions. There are references to farming (rice and barley
are mentioned), cattle rearing, and fishing. There are also several references to iron.
Kuruntokai 16 refers to iron-tipped arrows. Akananuru 72 compares a bear digging out the
comb from a termite mound, the front of which is swarming with fireflies glimmering like
sparks from beaten metal, to a blacksmith forging iron. Hence statement 1 is incorrect.
 The Brahmanas came there much later from northern India. But in ancient times, they
followed neither the Varna system nor the Ashram system.

https://t.me/prelimbits
336

 Broadly speaking, there were chiefly two classes of people in the early Tamil society – those
who tilled the land themselves and those who got it tilled by others. The latter were
wealthier and this very fact introduced inequalities in the social system. Gradually, the Varna
System also started.
 The social classification of varna was known to Sangam poets. There is mention of the
Arashar (kings), Vaishiyar (traders), and Velalar (farmers). The Brahmanas are also
mentioned, some of them closely associated with the courts of kings and patronized by
ruling elites. However, the four-fold varna classification had little application to ancient
Tamil society. The jati system was not a feature of this society either. Hence statement 2 is
correct
 Sangam literature which combines idealism with realism and classic grace with indigenous
industry and strength is rightly regarded as constituting the Augustan age of Tamil
literature. It deals with a secular matter relating to public and social activity like government,
war, charity, trade, worship, agriculture, etc.
 Sangam Poems are pervaded with a warrior ethic. Puram in Sangam literature is the War
Poems that deal with the outer life of people. Speak of public celebration of the feats of the
heroes even the death of heroes in wars. Hence statement 3 is incorrect.
 Manimekalai is a Sangam epic composed by Sattanar around 1400 years ago. It describes
the story of the daughter of Kovalan and Madhavi. In this epic Magical incidences have
been mentioned-"One angel helps her magically disappear to an island while the prince
tries to chase her, grants her powers to change forms and appear as someone else. On the
island, she receives a magic begging bowl, which always gets filled".
 Sangam literature reflects a belief in sacred or magical forces called ananku that were
supposed to inhabit various objects. The job of carrying out rites and rituals to control the
ananku was that of groups such as the pariyans, tutiyans, panas and velans. They were
associated with ritualistic singing, dancing, and trances, and with lighting the cremation fire
and worshipping memorial stones. Hence statement 4 is incorrect.

229. The Prime Minister recently inaugurated the new Circuit House near Somnath
Temple Veraval. Which of the following statements are correct regarding Somnath
Temple? [2022]

1. Somnath Temple is one of the Jyotirlinga shrines.


2. A description of Somnath Temple was given by Al-Biruni.
3. Pran Pratishtha of Somnath Temple (installation of the present day temple) was done by
President S. Radhakrishnan.
Select the correct answer using the code given below:
A. 1 and 2 only
B. 2 and 3 only
C. 1 and 3 only

https://t.me/prelimbits
337

D. 1,2 and 3
Ans. A
SOMNATH TEMPLE [GUJARAT]

 It is lso known as Deo Patan.


 It is one of the 12 jyotirlinga shrines of Lord Shiva.
 Constructed in Chalukyan style.
 It was mentioned by the Arab traveler Al-Biruni in his travelogue, influenced by which
Mahmud Ghaznavi attacked the Somnath temple in 1024 with his five thousand soldiers
and looted its property, and completely destroyed the temple.
 Somnath was destroyed when Delhi Sultanate occupied Gujarat in 1299. Afzal Khan,
the commander of Ala-ud-din Khilji, plundered it.
 In 1394 it was destroyed again.
 In 1706, Mughal ruler Aurangzeb again demolished the temple.
 1947 – Sardar Vallabh Bhai Patel ordered the reconstruction of the temple that completed
in 1951.

 The first President of India Dr Rajendra Prasad installed the Jyotirling in the new temple
on May 11, 1951.
 He was criticised by the then Prime Minister for displaying personal faith in public while
being on the highest constitutional post of a secular nation.

List of 12 Jyotirling Temples in India


 Somnath Temple, Gujarat
 Mallikarjuna Temple, Andhra Pradesh
 Mahakaleshwar Temple, Madhya Pradesh
 Omkareshwar Temple and Mamleshwar Temple, Madhya Pradesh
 Baidyanath Dham, Jharkhand
 Bhimashankar Temple, Maharashtra
 Ramanathaswamy Temple, Rameshwaram
 Nageshwar Temple, Gujarat
 Kashi Vishwanath Temple, Uttar pradesh
 Trimbakeshwar Temple, Maharashtra
 Kedarnath Temple, Uttarakhand
 Grishneshwar Temple, Maharashtra
Al-Beruni
 Alberuni was a renowned philosopher, a mathematician as well as a historian.
https://t.me/prelimbits
338

 He came in contact with Mahmud when he had invaded Khiva and he was presented to
him as prisoner.
 In his seminal piece Tahqiq-i-Hind, he portrays the social, political, religious and economic
condition of the then India.

https://t.me/prelimbits

You might also like